Download as pdf or txt
Download as pdf or txt
You are on page 1of 216

TARGET PT 2020 3

TEST
DAY - 31

Time Allowed: 30 mins Maximum Marks: 50

1. With reference to the ecological (c) Both 1 and 2


pyramids, consider the following
(d) Neither 1 nor 2
statements:
1. Unlike in the grazing food chain, the 4. Consider the following statements
pyramid of biomass in the aquatic food regarding biosphere on the Earth.
chain is inverted. 1. It is exclusively present on the Earth.
2. In the forest ecosystem, the pyramid of 2. It includes the lithosphere and
numbers is always upright. hydrosphere only.
3. Since the sun is not a primary source 3. It includes only living organisms on
of energy in a detritus food chain, the earth.
therefore the pyramid of energy is
inverted here. Which of the above statements is/are
correct?
Which of the above statement is/are
incorrect? (a) 1 and 2 only

(a) 1 only (b) 2 and 3 only

(b) 1 and 2 only (c) 1, 2, and 3

(c) 2 and 3 only (d) 1 only

(d) 1, 2 and 3 5. Which of the following statements is/


are incorrect?
2. Which of the following is the author of
1. Standing crop is the number of
“Small Is Beautiful”?
total living organisms present in an
(a) Rachel Carson ecosystem.
(b) E. F. Schumacher 2. Standing state is the amount of biomass
present in the ecosystem.
(c) Leopold Kohr
(d) Alan Weisman Select the correct answer using the code
given below:
3. Consider the following statements
(a) 1 only
regarding Tundra ecosystems:
1. Tundra ecosystems are exclusively (b) 2 only
found in the Arctic region. (c) Both 1 and 2
2. Mammals of the tundra region have a (d) Neither 1 nor 2
large body size, small tail, and small
ear. 6. Which of the following statements is/
are correct?
Select the correct answer using the code
given below: 1. A food chain is a subset of the food
web.
(a) 1 only
2. The presence of complex food webs
(b) 2 only leads to instability in the ecosystem.
4 TARGET PT 2020

Select the correct answer using the code 10. If an elephant walks on the grass,
given below: many insects along with grasses are
(a) 1 only killed. Which type of biotic interaction
is this?
(b) 2 only
(a) Commensalism
(c) Both 1 and 2
(b) Amensalism
(d) Neither 1 nor 2
(c) Predation
7. Consider the following pairs of the
aquatic organisms and their features: (d) Neutralism
1. Periphyton: Attached to roots 11. Consider the following statements:
2. Plankton: Limited locomotory powers 1. In bioaccumulation, there is an increase
3. Benthos: Lives in the bottom of the in the concentration of a pollutant from
water mass one link in a food chain to another.
Which of the above pairs is/are correctly 2. Biomagnification occurs if the pollutant
matched? is fat-insoluble.
(a) 3 only Which of the above statements is/are
(b) 2 and 3 only correct?
(c) 1 and 3 only (a) 1 only
(d) 1 and 2 only (b) 2 only
8. Which of the following statements is/ (c) Both 1 and 2
are correct? (d) Neither 1 nor 2
1. Unlike in the photic zone where both
photosynthesis and respiration occur, 12. Consider the following statements
only respiration takes place in the regarding Nutrient Cycle:
aphotic zone.
1. Unlike energy, the nutrient of an
2. The profundal zone lies between photic ecosystem flow in a circular manner.
and the aphotic zones.
2. Most gaseous cycles are generally
Select the correct answer using the code considered as imperfect cycles.
given below:
Which of the above statements is/are correct?
(a) 1 only
(a) 1 only
(b) 2 only
(c) Both 1 and 2 (b) 2 only

(d) Neither 1 nor 2 (c) Both 1 and 2


(d) Neither 1 nor 2
9. Consider the following statements
regarding oxygen in the aquatic system: 13. Which of the following is correct
1. The concentration of oxygen in the regarding the term species?
aquatic system is higher than that on
the terrestrial ecosystems (a) It includes various genera that share a
few similarities.
2. The amount of dissolved oxygen present
in water is influenced by temperature. (b) It is a group of similar populations of
organisms whose members are capable
Which of the above statements is/are incorrect?
of interbreeding, and to produce fertile
(a) 1 only offspring.
(b) 2 only (c) A group of similar genus forms a
(c) Both 1 and 2 species.
(d) Neither 1 nor 2 (d) All of the above
TARGET PT 2020 5

14. Which of the following statements is/ 1. During the process of decomposition
are correct? decomposers break down complex
1. The term “Ecosystem” was coined by inorganic substances into organic
Arthur Tansley. matter.

2. Ernst Haeckel was the first to coin the 2. Catabolism is a process where bacterial
term “Ecology”. and fungal enzymes degrade detritus
into simpler inorganic substances.
3. Charles Sutherland Elton coined the
term “Ecological niche”. 3. The rate of decomposition is controlled
by biological composition of detritus
Select the correct answer using the code and climatic factors.
given below:
Which of the following statements is/are
(a) 2 only incorrect?
(b) 3 only (a) 1 and 3 only
(c) 1 and 2 only (b) 1 only
(d) 1, 2, and 3 (c) 2 and 3 only

15. Which of the following can be factor(s) (d) 1 and 2 only


of the environment?
18. Which of the following statements is
1. Edaphic factors incorrect regarding the Energy Flow
2. Animals in an ecosystem?
3. Temperature (a) All animals depend on plants (directly
or indirectly) for their food needs.
Select the correct answer using the code
given below: (b) Sun is the only source of energy for all
ecosystems on Earth.
(a) 2 only
(c) The detritus food chain (DFC) begins
(b) 3 only with dead organic matter.
(c) 1 and 2 only (d) In an aquatic ecosystem, grazing food
(d) 1, 2, and 3 chain (GFC) is the major conduit for
energy flow.
16. Consider the following statements
regarding the ecosystem: 19. Consider the following statements
regarding the Ecological Pyramids:
1. Vertical distribution of different
species occupying different levels is 1. The pyramid of biomass in sea is
called stratification. generally inverted.

2. In a pond ecosystem, the producers are 2. The amount of energy available by


represented by phytoplankton whereas the time the top of the pyramid is
the consumers are represented by the reached is not enough to sustain many
zooplankton. individuals.
3. The base of each pyramid represents
Which of the following statements is/are the producers or the first trophic level
correct? while the apex represents tertiary or
(a) 1 only top level consumer.

(b) 2 only Which of the following statements is/are


correct?
(c) Both 1 and 2
(a) 1 and 2 only
(d) Neither 1 nor 2
(b) 1 and 3 only
17. Regarding the process of
(c) 2 and 3 only
Decomposition, consider the
following: (d) 1, 2 and 3
6 TARGET PT 2020

20. Consider the following statements: 23. “Hera mission” which was currently
1. The community that is in near in news is been launched by which
equilibrium with the environment and organization?
that is called a climax community.
(a) ESA
2. Hydrarch succession takes place
in wetter areas while the xerarch (b) NASA
succession takes place in dry areas.
(c) ISRO
Which of the following statements is/are
correct regarding Ecological Succession? (d) Canadian Space Agency
(a) 1 only
24. With reference to Data Protection
(b) 2 only Bill 2019, consider the following
(c) Both 1 and 2 statements
(d) Neither 1 nor 2 1. Bill governs the processing of personal
21. Consider the following statements data by Government only.
about Indian national Congress: 2. A data fiduciary is an entity or
1. The theme of 107th Indian Science individual who decides the means and
Congress is “Science and Technology:
purpose of processing personal data.
Rural Development”.
2. The Indian Science Congress Which of the following statement is/are
Association (ISCA) owes its origin to
correct?
the foresight and initiative of Professor
J. L. Simonsen and scientist Homi J. (a) 1 only
Bhaba.
(b) 2 only
3. The 107th Indian Science Congress was
inaugurated by PM Narendra Modi at (c) Both 1 and 2
New Delhi.
(d) Neither 1 nor 2
Which of the above statement(s) is/are
incorrect? 25. Consider the following statements
(a) 1 only regarding National Clean Air
(b) 1 and 2 only Programme
(c) 1, 2 and 3 1. The government is targeting to achieve
(d) 2 and 3 only 20 per cent to 30 per cent reduction in
Particulate Matter PM10 and PM2.5
22. Consider the following statements concentrations by 2024.
about The India State of Forest Report
2019- 2. Base year would be 2011-12
1. Total tree and forest cover in the 3. An all India standard plan has been
country has decreased in the last two
prepared and approved for ground
years.
implementation.
2. There is an increase in the carbon stock
of the country
Which of the following statement is/are
Which of the above statements is/are correct?
correct?
(a) 1 only
(a) 1 only
(b) 2 and 3 only
(b) 2 only
(c) Both 1 and 2 (c) 1 and 3 only

(d) Neither 1 nor 2 (d) 1, 2 and 3


TARGET PT 2020 7

ANSWER HINTS
DAY - 31

1. Correct Option: (c)  Tundra means a “barren land” since they


are found where environmental conditions
Explanation: are very severe.
Ecological pyramids  There are two types of tundra viz. arctic
and alpine.
 In the grazing food chain (many plants-
many herbivores-fewer carnivores) the Arctic Tundra
pyramid of biomass is upright (decreasing
 Arctic tundra extends as a continuous belt
at each trophic level) below the polar ice cap and above the tree
 In the aquatic food chain of line in the northern hemisphere. It occupies
Phytoplankton (low biomass)- the northern fringe of Canada, Alaska,
zooplankton(more biomass)- European Russia, Siberia and island group
predator(more biomass), the pyramid of Arctic Ocean. On the south pole, tundra
is very small since most of it is covered by
of biomass is inverted.
the ocean.
 Generally, the pyramid of number is
 Flora and fauna: Typical vegetation of arctic
upright in the forest ecosystem but, tundra is cotton grass, sedges, dwarf heath,
there are exceptions. For instance, willows, birches, and lichens. Animals of
in a food chain starting from one tundra are reindeer, musk ox, arctic hare,
tree( one Tree-many herbivores-few caribous, lemmings, and squirrel.
carnivores), the pyramid is neither
 Most of them have a long life e.g. arctic
upright nor inverted.
willow has a life span of 150 to 300 years.
 Pyramid of energy in any food chain of They are protected from chillness by the
any ecosystem is always upright. presence of thick cuticle and epidermal
hair.
2. Correct Option: (b)
Alpine Tundra
Explanation:  Alpine tundra occurs at high mountains
above the with respect to Arctic mountains
Small Is Beautiful
are found at all latitudes, therefore, alpine
 It is a collection of essays by German-born tundra shows day and night temperature
British statistician and economist E. F. variations.
Schumacher.  Mammals of the tundra region have
 Rachel Carson wrote the Silent Spring. a large body size, small tail and small
ear to avoid the loss of heat from the
surface. The body is covered with fur
3. Correct Option: (b)
for insulation. Insects have short life
Explanation: cycles that are completed during a
favorable period of the year.
Tundra
4. Correct Option: (d)
 Tundra ecosystems are treeless regions
found in the Arctic and on the tops of Explanation:
mountains, where the climate is cold and
windy, and rainfall is scant. Tundra lands Biosphere
are covered with snow for much of the year,  The biosphere is the layer of the planet
but summer brings bursts of wildflowers. Earth only where life exists.
8 TARGET PT 2020

 It is the intersection of lithosphere, in an ecosystem and therefore increases


hydrosphere, and atmosphere. their chance of survival.
 The biosphere includes all living  For example, grasses may serve food for
organisms on earth, together with rabbits or grasshoppers or goats or cows.
the dead organic matter produced by Similarly, a herbivore may be a food source
them. for many carnivorous species.

5. Correct Option: (b)  The presence of complex food


webs increases the stability of the
Explanation: ecosystem.
Standing state and Standing crop  More complex food webs improve the
adaptability and competitiveness of the
 Each trophic level has a certain mass
organisms. Also, food availability and
of living material at a particular time
preferences of food of the organisms may
called the standing crop. The standing
shift seasonally. E.g. we eat watermelon in
crop is measured as the mass of living
summer and peaches in the winter.
organisms (biomass) or the number in
a unit area. There is no circulation of the  Thus, there are interconnected networks of
standing crops. feeding relationships that take the form of
 Standing state is the amount of food webs.
inorganic nutrients found in an
ecosystem. It represents part of non-living 7. Correct Option: (b)
matter. It circulates between living and Explanation:
non-living components of the ecosystem. It
is being regularly depleted and replenished Aquatic Organisms
by the 1 living matters.
 Periphyton: These are organisms that
6. Correct Option: (a) remain attached to stems and leaves
of rooted plants or substances emerging
Explanation above the bottom mud such as sessile algae
and their associated group of animals.
Food Chain and web
 Plankton: This group includes
 A food chain is a linear sequence of
both microscopic plants like algae
organisms through which nutrients and
(phytoplankton) and animals like
energy pass as one organism eats another.
crustaceans and protozoans (zooplankton)
 Organisms in the ecosystem are related to found in all aquatic ecosystems, except
each other through feeding mechanisms or certain swift-moving waters. The
trophic levels, i.e. one organism becomes locomotory power of the planktons
food for the other. is limited so that their distribution is
 A food chain starts with producers and ends controlled, largely, by currents in the
with top carnivores. In this, each organism aquatic ecosystems.
occupies a different trophic level.  Benthos: The benthic organisms are those
Food Web found living at the bottom of the water
mass. Practically every aquatic ecosystem
 A food chain represents only one part contains well-developed benthos.
of the food or energy flow through
an ecosystem and implies a simple, 8. Correct Option: (a)
isolated relationship, which seldom
occurs in the ecosystems. Explanation:
 “A food web illustrates, all possible Photic zone
transfers of energy and nutrients among
the organisms in an ecosystem, whereas a  It is the upper layer of the aquatic
food chain traces only one pathway of the ecosystems, up to which light penetrates
food”. and within which photosynthetic activity
is confined.
 If any of the intermediate food chains are
removed, the succeeding links of the chain  The depth of this zone depends on the
will be affected largely. transparency of water.
 The food web provides more than one  Both photosynthesis and respiration
alternative for food to most of the organisms activity takes place.
TARGET PT 2020 9

 Dissolved oxygen escapes the water body


Littoral
zone
Limnetic through the air-water interface and
zone
through respiration of organisms (fish,
decomposers, zooplanktons, etc).
Photic zo
ne  The amount of dissolved oxygen
Profundal retained in water is also influenced
(aphotic)
Bentic zone by temperature. Oxygen is less soluble
zone
in warm water. Warm water also enhances
decomposer activity. Therefore, increasing
the temperature of a water body increases
the rate at which oxygen is depleted from
water.
 The photic (or “euphotic”) zone is the
lighted and usually well-mixed portion  When the dissolved oxygen level falls
that extends from the lake surface down to below 3-5 ppm, many aquatic organisms
where the light level is 1% of that at the are likely to die.
surface.
10. Correct Option: (b)
Aphotic zone
Explanation:
 The lower layers of the aquatic ecosystems,
where light penetration and plant growth Types of biotic interaction
are restricted forms the aphotic zone.
 Mutualism: When both species benefit.
 Only respiration activity takes place. Example- In Lichens, Algae provides food
 The aphotic zone is positioned below the by photosynthesis and Fungi provides
littoral and photic zones to the bottom of shelter, thus both get benefits.
the lake where light levels are too low for
 Commensalism: When one species benefits,
photosynthesis. Respiration occurs at all
the other is unaffected. Example- cow dung
depths so the aphotic zone is a region of
provides food and shelter to dung beetles.
oxygen consumption.
The beetles have no effect on the cows.
 This deep, unlit region is also known
as the profundal zone.  Amensalism: One species is harmed,
the other is unaffected. Example-as in
9. Correct Option: (a) the above question.

Explanation:  Predation and parasitism: one species


benefits, the other is harmed. Example-
Dissolved Oxygen in Aquatic Ecosystems Predator kills and eats prey; Virus(Parasite)
causes harm to the host
 In aquatic ecosystems, oxygen is dissolved
in water, where its concentration varies  Competition: both species are harmed by
constantly depending on factors that the interaction. Example-Two species live
influence the input and output of oxygen in in the same niche are in competition.
the water.
 Neutralism: There is no net benefit or
 In freshwater, the average harm to either species. Example- Two same
concentration of dissolved oxygen species inhabiting the same space(large)
is 0.0010 percent (also expressed as and using the same resources(large).
10 parts per million or 10 ppm) by
weight, which is 150 times lower than 11. Correct Option: (d)
the concentration of oxygen in an
equivalent volume of air. Thus, the Explanation:
concentration of oxygen is less in the
aquatic ecosystems than that in the Pollutants in the trophic level
terrestrial ecosystems.  Bioaccumulation: In bioaccumulation,
 Oxygen enters the aquatic ecosystem there is an increase in the concentration of a
through the air-water interface and by the pollutant from the environment to the first
photosynthetic activities of aquatic plants. organism in a food chain. Thus, It refers to
how pollutants enter a food chain.
 Therefore, the quantity of dissolved oxygen
present in an ecosystem depends on the  Biomagnification: in this, there is an
rate at which the aforesaid two processes increase in the concentration of a
occur. pollutant from one trophic level to another.
10 TARGET PT 2020

 Condition for bioaccumulation - High  An ecosystem is a community of organisms,


level of pollutants in the environment. their environment, and their interactions as
a system. It was first coined by Arthur
 Condition for biomagnification-
Tansley.
Pollutants must be fat-soluble, long-
lived, biologically active, etc.  Ecological niche was coined by the
naturalist Roswell Hill Johnson. but
12. Correct Option: (d) Joseph Grinnell was probably the first
to use it in a research program in 1917.
Explanation: It was further described by Charles
Sutherland Elton.
Nutrient Cycling
 The nutrient cycle is a concept that 15. Correct Option: (d)
describes how nutrients move from Explanation:
the physical environment to the living
organisms, and subsequently recycled Environmental factors
back to the physical environment.
This movement of nutrients from the  Environment is the natural component
in which biotic (living) and abiotic (non-
environment into plants and animals and
living) factors interact among themselves
again back to the environment is essential
and with each other. In a biological sense,
for life and it is the vital function of the
the environment constitutes the physical
ecology of any region.
(nutrients, water, air) and biological factors
 Based on the replacement period a nutrient (biomolecules, organisms) along with their
cycle is referred to as a Perfect or Imperfect chemical interactions (chemical cycles –
cycle. carbon cycle, nitrogen cycle etc.) that affect
an organism or a group of organisms.
 A perfect nutrient cycle is one in which
nutrients are replaced as fast as they  All these environmental factors can
are utilized. Most gaseous cycles are be broadly classified into the following
generally considered as perfect divisions:
cycles. ! Climatic or Aerial factors: they include
 In contrast, sedimentary cycles are light, temperature, Water, Rainfall,
considered relatively imperfect, as some Humidity, Atmospheric gases (wind),
nutrients are lost from the cycle and get etc.
locked into sediments and so become ! Topographic or Physiographic factors:
unavailable for immediate cycling. Altitude, slopes, etc.
! Edaphic factors: Soil
13. Correct Option: (b)
! Biotic factors: Plants, animals,
Explanation: microbes, etc.
Species ! Limiting Factors: time, nutrients, etc.
 A species is defined as; “a group of similar
16. Correct Option: (c)
populations of organisms whose
members are capable of interbreeding, Explanation:
and to produce fertile offspring
(children)”. A tiger, a lion, a lotus, etc., Ecosystem
are some examples of different species.  Interaction of biotic and abiotic components
 Speciation is the process by which new results in a physical structure that is
species are formed and evolution is the characteristic for each type of ecosystem.
mechanism by which speciation is brought  Identification and enumeration of plant
about. and animal species of an ecosystem gives
its species composition.
14. Correct Option: (c)
 Vertical distribution of different
Explanation: species occupying different levels is
called stratification. For example, trees
Founders of the ecological studies occupy top vertical strata or layer of a
forest, shrubs the second and herbs and
 Ecology was first coined and defined in
grasses occupy the bottom layers.
1869 by Ernst Haeckel as the ‘study of
the interaction of organisms with their  A pond is a shallow water body in which all
environment. the four basic components of an ecosystem
TARGET PT 2020 11

(Productivity; Decomposition; Energy flow; horizon and get precipitated as unavailable


and Nutrient cycling) are well exhibited. salts.
 The abiotic component is the water with  Bacterial and fungal enzymes degrade
all the dissolved inorganic and organic detritus into simpler inorganic
substances and the rich soil deposit at the substances. This process is called as
bottom of the pond. catabolism. It is important to note that all
the above steps in decomposition operate
 The solar input, the cycle of temperature, simultaneously on the detritus.
day-length and other climatic conditions
regulate the rate of function of the entire  Humification and mineralisation
pond. occur during decomposition in the soil.
Humification leads to accumulation of a
 The autotrophic components include dark coloured amorphous substance called
the phytoplankton, some algae and the humus that is highly resistant to microbial
floating, submerged and marginal plants action and undergoes decomposition at an
found at the edges. extremely slow rate.
 The consumers are represented by  Being colloidal in nature it serves as a
the zooplankton, the free swimming and reservoir of nutrients. The humus is further
bottom dwelling forms. degraded by some microbes and release of
 The decomposers are the fungi, bacteria inorganic nutrients occur by the process
and flagellates especially abundant in the known as mineralisation.
bottom of the pond.  Decomposition is largely an oxygen-
 This system performs all the functions of any requiring process. The rate of
ecosystem and of the biosphere as a whole, decomposition is controlled by
chemical composition of detritus and
i.e., conversion of inorganic into organic
climatic factors.
material with the help of the radiant energy
of the sun by the autotrophs; consumption  In a particular climatic condition,
of the autotrophs by heterotrophs; decomposition rate is slower if detritus
decomposition and mineralisation of the is rich in lignin and chitin, and quicker,
dead matter to release them back for reuse if detritus is rich in nitrogen and water-
by the autotrophs, these event are repeated soluble substances like sugars.
over and over again.
 Temperature and soil moisture are the most
 There is unidirectional movement of important climatic factors that regulate
energy towards the higher trophic levels decomposition through their effects on the
and its dissipation and loss as heat to the activities of soil microbes.
environment.  Warm and moist environment favour
decomposition whereas low temperature
17. Correct Option: (a) and anaerobiosis inhibit decomposition
Explanation: resulting in build up of organic materials.

Decomposition 18. Correct Option: (a)


 Decomposers break down complex organic Explanation:
matter into inorganic substances like
carbon dioxide, water and nutrients and Energy Flow
the process is called decomposition.  All animals depend on plants (directly
 Dead plant remains such as leaves, bark, or indirectly) for their food needs.
flowers and dead remains of animals, They are hence called consumers and
including fecal matter, constitute also heterotrophs. If they feed on the
detritus, which is the raw material for producers, the plants, they are called
decomposition. primary consumers, and if the animals
eat other animals which in turn eat the
 The important steps in the process plants (or their produce) they are called
of decomposition are fragmentation, secondary consumers. Likewise, you could
leaching, catabolism, humification have tertiary consumers too. Obviously
and mineralisation. Detritivores (e.g., the primary consumers will be herbivores.
earthworm) break down detritus into Some common herbivores are insects, birds
smaller particles. This process is called and mammals in terrestrial ecosystem and
fragmentation. molluscs in aquatic ecosystem.
 By the process of leaching, watersoluble  Except for the deep sea hydro-thermal
inorganic nutrients go down into the soil ecosystem, sun is the only source of
12 TARGET PT 2020

energy for all ecosystems on Earth. Of  Since only 10% of the energy is transferred
the incident solar radiation less than 50 from one trophic level to the next, fewer
per cent of it is photosynthetically active and fewer numbers of individuals can
radiation (PAR). We know that plants be sustained as we go up the ecological
and photosynthetic and chemosynthetic
pyramid.
bacteria (autotrophs), fix suns’ radiant
energy to make food from simple inorganic
materials. Plants capture only 2-10 per 20. Correct Option: (c)
cent of the PAR and this small amount of Explanation:
energy sustains the entire living world.
 The detritus food chain (DFC) begins Ecological Succession
with dead organic matter. It is made  An important characteristic of all
up of decomposers which are heterotrophic
communities is that composition and
organisms, mainly fungi and bacteria.
structure constantly change in response to
They meet their energy and nutrient
requirements by degrading dead organic the changing environmental conditions.
matter or detritus. These are also known  This change is orderly and sequential,
as saprotrophs (sapro: to decompose). parallel with the changes in the physical
 In an aquatic ecosystem, GFC is the environment. These changes lead
major conduit for energy flow. As finally to a community that is in near
against this, in a terrestrial ecosystem, equilibrium with the environment and
a much larger fraction of energy flows that is called a climax community.
through the detritus food chain than
through the GFC. Detritus food chain may  The gradual and fairly predictable change
be connected with the grazing food chain in the species composition of a given area is
at some levels: some of the organisms of called ecological succession.
DFC are prey to the GFC animals, and in
a natural ecosystem, some animals like  During succession some species colonise an
cockroaches, crows, etc., are omnivores. area and their populations become more
These natural interconnection of food numerous, whereas populations of other
chains make it a food web. species decline and even disappear.

19. Correct Option: (d)  Based on the nature of the habitat –


whether it is water (or very wet areas) or it
Explanation: is on very dry areas – succession of plants
is called hydrach or xerarch, respectively.
Ecological Pyramids
 The pyramid of biomass in sea is  Hydrarch succession takes place
generally inverted because the biomass in wetter areas and the successional
of fishes far exceeds that of phytoplankton. series progress from hydric to the mesic
conditions.
 The base of a pyramid is broad and it
narrows down at the apex. One gets  As against this, xerarch succession takes
a similar shape, whether you express place in dry areas and the series progress
the food or energy relationship between from xeric to mesic conditions. Hence, both
organisms at different trophic level. hydrarch and xerach successions lead to
Thus, relationship is expressed in terms
medium water conditions (mesic) – neither
of number, biomass or energy. The base
of each pyramid represents the producers too dry (xeric) nor too wet (hydric).
or the first trophic level while the apex
represents tertiary or top level consumer. 21. Correct Answer: (D)
The three ecological pyramids that are
Explanation:
usually studied are (a) pyramid of number;
(b) pyramid of biomass and (c) pyramid of  1st statements is correct
energy.
 2nd and 3rd statements are incorrect.
 Pyramid of energy is always upright, The Indian Science Congress Association
can never be inverted, because when
(ISCA) owes its origin to the foresight
energy flows from a particular trophic level
and initiative of two British Chemists,
to the next trophic level, some energy is
always lost as heat at each step. Each bar namely, Professor J. L. Simonsen and
in the energy pyramid indicates the amount Professor P.S. MacMahon. The 107th Indian
of energy present at each trophic level in a Science Congress was inaugurated by PM
given time or annually per unit area. Narendra Modi at Bangalore.
TARGET PT 2020 13

22. Correct answer: (b)


Facts about Science Status in the
world Explanation
 India now stands at 3rd position globally, Only statement (2) is correct.
in the number of Peer-reviewed Science
and Engineering Publications.  According to this report, the forest area in
the country has increased by 5,188 square
 India has improved its ranking at
km in the last two years.
Innovation Index to 52.
 Government programmes have created  There is an increase of 42.6 million tonnes
more incubators in the last 5 years. in the carbon stock of the country as
compared to the last assessment of 2017.
 Technology is being harnessed at a large
scale to achieve the objective of Good Supplementary notes
Governance.
Highlights of The India State of Forest
Supplementary Notes Report 2019
 Recently, PM inaugurates the 107th Indian  The report mentions that green area covers
Science Congress at University of about 25% of the total geographic area of
Agricultural Sciences, Bengaluru. the country.
 The theme of 107th Indian Science  India State of Forest Report 2019 has been
Congress is “Science and Technology: prepared by Forest Survey of India (FSI). It
Rural Development”.
is a biannual report.
 The Congress is to be hosted by Department
 The report is prepared after mapping of
of Science and Technology.
forests and trees through satellite.
 The event acts as a common platform for
researchers, scientists and academicians.  According to the report, the forest area has
Around 28 plenary sessions are to be increased by 3976 kilometers and the area
held during the event. It includes leading of trees by 1212 square kilometers in two
technologies such as Climate Smart years.
Agriculture for food security, challenges in  According to the report, the three states
cancer drugs, nano materials for energy,
where forest area has grown the most
solutions for oil and gas, environment and
include Karnataka (1025 km), Andhra
health care. For the first time, Farmers
Pradesh (990 km) and Kerala (823 km).
Science Congress is to be held on the
sidelines of Indian Science  It has also mentioned in the report
 Farmers Science Congress: The that the forest area of the northeastern
congress will cover themes of innovation on states has decreased except Assam.
integrated agriculture. It will also focus on A marginal increase in forest cover was
themes of doubling farmers’ income, farmer recorded in Himachal, Uttarakhand, Uttar
empowerment, conservation, bio diversity, Pradesh, Delhi, and Bihar.
etc. The experts from ICAR (Indian
 The report shows that the forest area has
council of Agricultural Research) and UAS
also increased in the infertile regions of
(University of Agricultural Sciences) will
also participate at the event along with the Rajasthan.
farmers whose innovative methodologies  Jammu and Kashmir grew 371 sq. km of
have brought in huge harvests. forest cover while Himachal Pradesh grew
 Children Science Congress: The 334-sq. km of forest area.
Children Science Congress also called the • The report highlights that all hill districts
“Rashtriya Kishre Vaiguanik Sammelana” of India has 2,84,006 sq. km of forest area
will also be held alongside 107th India which is 40.30% of the total geographical
Indian Science Congress. The aim of the
area of all hill districts of the country.
congress is to motivate students and
increase their participation in scientific
and technological fields. Apart from these,
23. Correct option: (a)
women science congress will also be held Explanation
showcasing women achievements. Also, VC
Science congress will be held to address the  European ministers in charge of the ESA
need of higher education institutions. The space agency have approved Hera, a mission
event will also hold Science Communicators that will test whether deflection could save
Meet. humanity from a rogue asteroid.
14 TARGET PT 2020

Supplementary notes Supplementary notes

Hera Mission Data Protection Bill 2019


 Scientists are studying asteroids and trying  The Bill seeks to provide for protection of
to find ways to deflect them from a collision personal data of individuals, and establishes
course with Earth. a Data Protection Authority for the same.
 The Personal Data Protection bill, drafted by
 One such project is the Asteroid Impact
a panel headed by a former Supreme Court
and Deflection Assessment (AIDA),
judge and submitted to the government last
which includes NASA’s Double Asteroid year, is key for how firms including global
Redirection Test (DART) mission and the tech giants Amazon, Facebook, Alphabet’s
European Space Agency’s (ESA) Hera. Google and others process, store and
transfer Indian consumers’ data.
Additional information
 Broad guidelines on collection, storage
 ESA and partner NASA will send a pair and processing of personal data, consent of
of spacecraft to a double-asteroid system individuals, penalties and compensation,
called Didymos. code of conduct and an enforcement model
is likely to be a part of the law.
 NASA will first crash its DART probe into
the smaller asteroid (Didymoon). Hera will  Personal data is data which pertains to
arrive later to map the impact crater and characteristics, traits or attributes of
identity, which can be used to identify an
measure the asteroid’s mass.
individual.
 The moon orbiting Didymos, called
 The Bill categorises certain personal data
‘Didymoon’ — almost the size of the Giza as sensitive personal data. This includes
Pyramid in Egypt, measuring just 160 financial data, biometric data, caste,
metres in diametre — will be the smallest religious or political beliefs, or any other
asteroid ever explored. category of data specified by the government,
in consultation with the Authority and the
 Hera will carry two CubeSats that can fly
concerned sectoral regulator.
extremely close to the asteroid’s surface
before touching down. About data fiduciary-
 Those briefcase-sized spacecraft will act  A data fiduciary is an entity or individual
like drones, capturing vital data about the who decides the means and purpose of
impact crater and giving scientists data processing personal data. Such processing
including the mass of the asteroid that will will be subject to certain purpose, collection
help them deduce its composition. and storage limitations.
 For instance, personal data can be
 While an asteroid collision is a pretty
processed only for specific, clear and lawful
unlikely event, it’s actually preventable,
purpose. Additionally, all data fiduciaries
unlike an earthquake or volcanic must undertake certain transparency and
explosion. accountability measures such as:
 However, Hera mission won’t be the first to ! implementing security safeguards
reach Didymos. The National Aeronautics (such as data encryption and
and Space Administration (NASA) in preventing misuse of data), and
USA plan to launch the Double Asteroid ! Instituting grievance redressal
Redirection Test (DART) between 2020 and mechanisms to address complaints of
2021, which will target Didymoon as part of individuals. They must also institute
its planetary defence programme. mechanisms for age verification and
parental consent when processing
24. Correct option: (b) sensitive personal data of children.
Explanation
25. Correct answer: (a)
 Statement 1 is incorrect: The Bill governs
Explanation
the processing of personal data by:
 Statement 2 is incorrect: The government
 government, is targeting to achieve 20 per cent to 30 per
 companies incorporated in India, and cent reduction in Particulate Matter PM10
and PM2.5 concentrations by 2024 keeping
 foreign companies dealing with personal 2017 as the base year for the comparison of
data of individuals in India concentration.
TARGET PT 2020 15

 Statement 3 is incorrect: The city-  The city-specific Action Plans have


specific Action Plans have been prepared been prepared and approved for ground
and approved for ground implementation
implementation for all 102 non-attainment
for all 102 non-attainment cities.
cities.
Supplementary notes
 India is committed to clean environment
National Clean Air Programme and pollution free air and water. In fact, it
 The government is targeting to achieve is mandated in our constitution.
20 per cent to 30 per cent reduction in
Particulate Matter PM10 and PM2.5  The Air (Prevention and Control of
concentrations by 2024 keeping 2017 Pollution) Act, 1981, was enacted under
as the base year for the comparison of Art. 253 of the Constitution to implement
concentration.
the decisions taken at the United Nations
 Under NCAP, 102 non-attainment cities
Conference on Human Environment held
have been identified based on ambient air
quality data for the period 2011 - 2015 and at Stockholm in June 1972, in which India
WHO report 2014 - 2018. participated.

**********
16 TARGET PT 2020

TEST
DAY - 32

Time Allowed: 30 mins Maximum Marks: 50

1. Which of the following statements is/ 1. Karnataka


are correct?
2. Tamil Nadu
1. Soil Organic Carbon (SOC) represents
the largest pool of carbon stock in the 3. Andhra Pradesh
forests of India. 4. Jammu and Kashmir
2. Despite Madhya Pradesh has the
largest area under forest, Arunachal Select the correct option using the codes
Pradesh has the largest carbon stock given below:
in India.
(a) 1-2-3-4
Select the correct option using the codes
(b) 2-1-3-4
given below:
(c) 3-1-2-4
(a) 1 only
(b) 2 only (d) 4-3-2-1

(c) Both 1 and 2 4. Deserts are formed in regions with


(d) Neither 1 nor 2 less than 25 cm of annual rainfall,
or sometimes in hot regions, where
2. Consider the following statements
there is more rainfall, but unevenly
regarding features of a biome:
distributed in the annual cycle. In
1. It is the world’s largest land biome.
this context consider the following
2. It is characterized by a long winter and
statements:
short summer.
1. The climate of these biomes is modified
3. It is considered the richest source
of softwood for use in building by altitude and latitude.
construction. 2. The perennial plant like creosote bush,
4. Spruce and Larch are amongst the cactus are scattered throughout the
major species in this biome. desert biomes.
Which of the following biomes is being 3. The productivity of a given desert
described above: is almost directly dependent on the
(a) Tundra rainfall.
(b) Taiga Which of the above statements are correct?
(c) Temperate Grassland
(a) 1 and 2 only
(d) Tropical forest
(b) 1 and 3 only
3. Arrange the following states into (c) 2 and 3 only
decreasing order of tree diversity in
India: (d) 1, 2 and 3
TARGET PT 2020 17

5. Which of the following are the findings 1. Peru


of the “India State of Forest Report 2. Ecuador
(ISFR) 2019”?
3. Argentina
1. There has been a decrease of forest
cover in the North-East region except 4. France
Assam and Tripura. 5. Bolivia
2. There has been a decrease of forest
Select the correct option using the codes
cover in the tribal districts of India.
given below:
3. Karnataka has shown the maximum
increase in the forest cover area since (a) 3 and 5 only
the last survey. (b) 1, 2, and 3 only
Select the correct option using the codes (c) 1, 2, 4, and 5 only
given below: (d) 1, 2, 3, 4, and 5
(a) 1 and 3 only
9. Consider the following statements
(b) 2 and 3 only
regarding the tropical seasonal
(c) 1, 2, and 3 forests:
(d) 1 and 2 only 1. They are known as monsoon forests.
2. They are present only in South Asia.
6. Which of the following pairs regarding
grasslands of India and their location Which of the above statements is/are
are correctly matched? incorrect?
1. Banni: Gujarat
(a) 1 only
2. Shola: Tamil Nadu
(b) 2 only
3. Vidi: Uttarakhand
(c) Both 1 and 2
4. Chuar: Kerala
(d) Neither 1 nor 2
Select the correct option using the codes
given below: 10. Cold and hot deserts With reference
(a) 1 and 2 only to the Montane Wet temperate forests,
consider the following statements:
(b) 1 and 4 only
1. These are found in the Kerala and
(c) 3 and 4 only Arunachal Pradesh.
(d) 2 and 3 only 2. They receive a minimum rainfall of
2000 mm in Arunachal Pradesh.
7. Consider the following features
regarding a grassland: 3. Oak and Champa trees are found in
these forests.
1. Heavy rainfall and long drought
2. Scattered trees Which of the above statements are correct?
3. Presence of Elephant Grass (a) 1 and 2 only

Which of the following grasslands is being (b) 2 and 3 only


described above? (c) 1 and 3 only
(a) Savanna (d) 1, 2 and 3
(b) Prairies
11. Which of the following are not features
(c) Alpine Meadows of tropical rainforests?
(d) Desert shrubs 1. Soils are rich in nutrients.

8. In which of the following countries 2. Woods are good for export.


Amazon forest is present? 3. Evenly distributed rainfall
18 TARGET PT 2020

Select the correct option using the codes 15. Consider the following pairs of forest
given below: types and corresponding states in
(a) 1 and 2 only India:
1. Tropical Wet evergreen forests:
(b) 1 and 3 only
Gujarat
(c) 2 and 3 only 2. Tropical Semi-evergreen forests:
(d) None of the above Maharashtra
3. Tropical thorn forests: Delhi
12. Match the following Lists:
Which of the above pairs is/are correctly
Grasslands Countries
matched?
A. Canterbury Plains 1. Uruguay (a) 3 only
B. Tussock Grasslands 2. Ukraine (b) 1 and 2 only

C. Campos 3. Australia (c) 2 and 3 only

D. Steppes 4. New Zealand (d) 1, 2, and 3

Select the correct option from the codes 16. Consider the following statements
given below the lists: regarding Biome:
1. Two biomes can never be alike.
A B C D
2. A biome is not an ecosystem.
(a) 2 3 4 1
3. Boundaries between two biomes are
(b) 3 4 1 2 always sharply defined.
(c) 4 2 1 3
Which of the following statements is/are
(d) 4 3 1 2 correct?

13. Consider the following statements: (a) 1 and 3 only


1. They are found between 250N and 250S (b) 1 and 2 only
latitudes. (c) 2 only
2. Presence of these can save lives and
(d) 2 and 3 only
property during natural hazards like
cyclones, storm surges, and erosion. 17. Consider the following statements:
3. They are adapted to the low oxygen 1. It means a “barren land” since they are
(anoxic) conditions. found where environmental conditions
are very severe.
About which of the following plant species
these statements are? 2. Typical vegetation is cotton grass,
sedges, dwarf heath, willows, birches
(a) Corals
and lichens.
(b) Mangroves
3. On the South Pole, it is very small since
(c) Cactus most of it is covered by ocean.
(d) Seagrass 4. Mammals of the this region have large
body size, small tail and small ear.
14. Which of the following organizations
maintains the World Database on Which type of biome has been defined in
Protected Areas? the above statements?
(a) IUCN (a) Desert
(b) UN Environment (b) Forest
(c) UNESCO (c) Taiga
(d) None of the above (d) Tundra
TARGET PT 2020 19

18. Which of the following state has not 2. People can get EVD through direct
declared a State Butterfly yet? contact with an infected animal (bat or
nonhuman primate) or a sick or dead
(a) Tamil Nadu
person infected with Ebola virus.
(b) Karnataka
Choose the correct option from the
(c) Kerala following
(d) Andhra Pradesh (a) 1 only
19. Consider the following statements (b) 2 only
regarding “Ansupa Lake” which has (c) Both 1 and 2
been recently in news.
(d) Neither 1 nor 2
1. Ansupa Lake is one of the largest fresh
water lake of Odisha 22. National Green Corps ‘Eco Club’
2. It was created by Mahanadi and got a Programme that was in news recently
shape like the hoof of a horse. aims at

3. The lake is surrounded with high (a) To provide opportunities for children
hills. to understand the environment and
environmental problems through
Which of the statements given above is/are school eco-clubs.
correct? (b) To make polluting industries conscious
(a) 1 and 3 only of the importance of the environment
for the existence of life.
(b) 2 only
(c) To conserve tiger population
(c) 2 and 3 only
(d) To protect the endangered wildlife
(d) 1,2 and 3
23. What are the various problems that
20. Consider the following statements. currently ail Indian oil and gas
1. This animal has been accorded sector?
protection from trade for the first 1. Declining domestic crude production
time. 2. Large crude import bills
2. There is currently only one recognized 3. Inadequate transmission & distribution
species, with nine sub-species of this infrastructure
particular animal.
4. Large share of MNCs
3. They have been listed as ‘vulnerable’
on the International Union for Choose the correct option
Conservation of Species Red List, (a) 1, 2 and 3
with some sub-species classified as
‘endangered’’ or ‘critically endangered’. (b) 1 and 2
(c) 1, 2, 3 and 4
The above statements are related to which
one of the following animal? (d) 3 and 4

(a) Giraffe 24. Consider the following statements


(b) Rhino regarding Indian Railway Management
Service (IRMS):
(c) Monkey
1. It has unified the existing eight Group
(d) Tardigrade A services of the Railways into a
Central Service.
21. With reference to Ebola Virus 2. Unification of services had been
Disease (EVD), consider the following recommended by various committees
statements for reforming Railways including - the
1. It was first discovered in Uganda. Prakash Tandon Committee (1994)
20 TARGET PT 2020

3. Decisions regarding IRMS have been 2. The index documents the progress
taken by the Railway Ministry. made by India’s States and Union
Territories towards implementing the
Which of the following statements is/are 2030 SDG targets.
correct? 3. It has been developed in collaboration
(a) 1 and 2 only with the Ministry of Statistics and
Programme Implementation (MoSPI),
(b) 2 only the United Nations in India, and the
(c) 1 and 3 only Global Green Growth Institute.

(d) 1, 2 and 3 Which of the following statements is/are


correct?
25. Consider the following statements (a) 1 and 2 only
regarding SDG India Index,
(b) 2 only
1. 2019 will be the second edition of the
Sustainable Development Goals (SDG) (c) 1 and 3 only
India Index. (d) 1, 2 and 3

**********
TARGET PT 2020 21

ANSWER HINTS
DAY - 32

1. Correct Option: (c) the litter derived from conifer needle (leaf)
is decomposed very slowly and is not rich
Explanation: in nutrients.
Carbon Stock in India  These soils are acidic and are mineral
deficient.
 Under the current assessment the total
carbon stock in the country’s forest is  There are four major species in the
estimated 7,124.6 million tonnes and there coniferous forests.
an increase of 42.6 million tonnes in the
carbon stock of country as compared to ! Pine, e.g. white pine, red pine, Scots
the last assessment of 2017. The annual pine, Jackpine, lodgepole pine.
increase in the carbon stock is 21.3 million ! Fir, e.g. Douglas fir and balsam fir.
tonnes, which is 78.2 million tonnes CO2
eq. ! Spruce.
 Soil Organic Carbon (SOC) represents ! Larch.
the largest pool of carbon stock in  The coniferous forest belts of Eurasia and
forests, which has been estimated North America are the richest sources
at 4,004 million tonnes. The SOC of softwood for use in building
contributes 56% to the total forest construction, furniture, matches, paper
carbon stock of the country. and pulp, rayon and other branches of the
 Arunachal Pradesh has the largest chemical industry. The world’s greatest
carbon stock (1051 million tonnes) in softwood producers are U.S.S.R., U.S.A.,
India. Canada, and the Fennoscandian countries
(Finland, Norway, and Sweden).
 Madhya Pradesh has the second-largest
carbon stock despite it has the largest 3. Correct Option: (a)
forest cover. This is because Arunachal
Pradesh has tropical wet evergreen and Explanation:
semi-evergreen forests of high canopy and
tree diversity. Tree diversity in India
 According to the “India State of Forest
2. Correct Option: (b) Report (ISFR) 2019, Karnataka has
Explanation: the highest tree richness, followed by
Tamil Nadu and Andhra Pradesh.
Coniferous forest (Boreal Forest/Taiga)  Low tree richness has been noticed
 The taiga or boreal forest is the world’s in Jammu and Kashmir, Punjab, and
largest land biome. It is found in North Haryana.
America, Eurasia, northern Mongolia, and
northern Japan. 4. Correct Option: (d)
 Cold regions with high rainfall, strong Explanation:
seasonal climates with long winters and
short summers are characterized by Desert Ecosystem
boreal coniferous forest.
 Deserts are formed in regions with less
 Boreal forest soils are characterized by than 25 cm of annual rainfall, or sometimes
thin podzols and are rather poor. Both in hot regions where there is more rainfall
because the weathering of rocks proceeds but unevenly distributed in the annual
slowly in cold environments and because cycle.
22 TARGET PT 2020

 Lack of rain in the mid-latitude is often to the biodiversity in forest areas, both of
due to stable high-pressure zones; deserts faunal and floral species.
in temperate regions often lie in “rain
 Due to the importance of wetlands, FSI
shadows”, that is, where high mountains
block off moisture from the seas. has carried out an exercise at the national
level to identify wetlands of more than 1 ha
 The climate of these biomes is modified within RFA.
by altitude and latitude. At high
altitudes and at a greater distance  There are 62,466 wetlands covering 3.8%
from the equator the deserts are cold of the area within the RFA/GW of the
and hot near equator and tropics. country.

 The perennial plant like creosote  The total forest cover in the tribal districts
bush, cactus are scattered throughout is 4,22,351 sq km, which is 37.54% of the
the desert biomes. geographical area of these districts.
 In shallow depressed areas with salt  There has been a decrease of 741 sq km
deposits greasewood, sapwood and salt of forest cover within the Recorded
grasses are common. Forest Area/ Green Wash (RFA/GW) in
the tribal districts and an increase of
 Because water is the dominant
1,922 sq km outside.
limiting factor, the productivity
of a given desert is almost directly  Total forest cover in the North-Eastern
dependent on the rainfall. Where soils region is 1,70,541 sq km, which is 65.05%
are suitable, irrigation can convert deserts of its geographical area. There has been a
into productive agricultural land. decrease of forest cover to the extent of 765
 As the large volume of water passes sq km (0.45%) in the region. Except for
through the irrigation system, salts may be Assam and Tripura, all the States in
left behind that will gradually accumulate the region show a decrease in forest
over the years until they become limiting cover.
unless means of avoiding this difficulty are
devised. 6. Correct Option: (a)
Explanation:
5. Correct Option: (c)
Explanation: Grasslands of India
 Grasslands occupy nearly 24 percent of the
Major findings of the India State of Forest geographical area in India.
Report (ISFR) 2019
 The major types of grasslands in India are
 Area-wise Madhya Pradesh has the largest
as follows:
forest cover in the country followed by
Arunachal Pradesh, Chhattisgarh, Odisha ! Alpine moist meadows of the Greater
and Maharashtra. Himalayas;
 In terms of forest cover as a percentage of ! Alpine arid pastures or steppe
total geographical area, the top five States formations of the trans-Himalayas;
are Mizoram (85.41%), Arunachal Pradesh
(79.63%), Meghalaya (76.33%), Manipur ! Chaurs of the Himalayan
(75.46%) and Nagaland (75.31%). foothills;

 The Mangrove ecosystems are unique ! Terai grasslands on the Gangetic and
& rich in biodiversity and they provide the Brahmaputra floodplains;
numerous ecological services. ! Phumdis or floating grasslands of
 Mangrove cover has been separately Manipur;
reported in the ISFR 2019 and the total ! Banni and Vidis of Gujarat;
mangrove cover in the country is 4,975 sq
km. An increase of 54 sq Km in mangrove ! Savannas of western and peninsular
cover has been observed as compared to India;
the previous assessment of 2017. The ! Plateau and valley grasslands in the
top three states showing mangrove cover Satpuras and Maikal hills;
increase are Gujarat (37 sq km) followed
by Maharashtra (16 sq km) and Odisha (8 ! Dry grasslands of the Andhra Pradesh
sq km). and Tamil Nadu plains and
 Wetlands within forest areas form ! Shola grasslands of the Western
important ecosystems and add richness Ghats including Tamil Nadu.
TARGET PT 2020 23

7. Correct Option: (a) 9. Correct Option: (b)


Explanation: Explanation:
Savanna Tropical Seasonal Forests
 Savanna is grassland with scattered  Tropical seasonal forests also are
individual trees. Savannas of one sort known as monsoon forests occur in
or another cover almost half the surface of regions where total annual rainfall is very
Africa and large areas of Australia, South high but segregated into pronounced wet
America, and India. and dry periods.
 Savanna has both a dry and a rainy season.  This kind of forest is found in South
Savannas are always found in warm or hot East Asia, central and south America,
climates where the annual rainfall is from northern Australia, western Africa
about 50.8 to 127 cm (20-50 inches) per
and tropical islands of the Pacific as
year. It is crucial that the rainfall is
well as in India.
concentrated in six or eight months of
the year, followed by a long period of
drought when fires can occur. If the rain
10. Correct Option: (d)
were well distributed throughout the year, Explanation:
many such areas would become tropical
forest. Montane Wet Temperate Forests
 The savanna landscape is typified by  In the North, Montane wet temperate
tall grass and short trees. It is rather forests are found in the region to the
misleading to call the savanna ‘tropical east of Nepal into Arunachal Pradesh,
grassland’, because trees are always receiving a minimum rainfall of 2000
present with the luxuriant tall grass. The mm.
terms ‘parkland’ or ‘bush-veld’ perhaps
describe the land¬scape better.  In the North, there are three layers of
forests: the higher layer has mainly
 In true savanna lands, the grass is tall coniferous, the middle layer has deciduous
and coarse, growing 6 to 12 feet high.
trees such as the oak and the lowest
The elephant grass may attain a height
layer is covered by rhododendron and
of even 15 feet. The grass tends to grow
Champa.
in compact tufts and has long roots that
reach down in search of water. It appears  In the South, it is found in parts of
greenish and well-nourished in the rainy the Niligiri Hills, the higher reaches
season but turns yellow and dies down of Kerala. The forests in the northern
in the dry season that follows. The grass region are denser than in the South.
lies dormant throughout the long, rainless Rhododendrons and a variety of ground
period and springs up again in the next flora can be found here
rainy season. In between the tall grass are
scattered short trees and low bushes. As 11. Correct Option: (a)
the rainfall diminishes towards the deserts
the savanna merges into thorny scrub. Explanation:

8. Correct Option: (c) Tropical rainforests

Explanation:  Tropical forests are characterized by the


greatest diversity of species. They occur
Amazon rainforest near the equator, within the area bounded
by latitudes 23.5 degrees N and 23.5 degrees
 The Amazon rainforest is an evergreen
tropical rainforest that covers most of the S. One of the major characteristics of
Amazon basin of South America. tropical forests is their distinct seasonality:
winter is absent, and only two seasons
 This region includes territory belonging to are present (rainy and dry). The length of
nine nations. daylight is 12 hours and varies little.
 The majority of the forest is contained  Precipitation is evenly distributed
within Brazil, with 60% of the throughout the year, with annual
rainforest, followed by Peru with 13%, rainfall exceeding 200 cm.
Colombia with 10%, and with minor
amounts in Venezuela, Ecuador,  Soil is nutrient-poor and acidic.
Bolivia, Guyana, Suriname and France Decomposition is rapid and soils are
(French Guiana). subject to heavy leaching.
24 TARGET PT 2020

 It has great potential in timber  Veldts: These are found in Southern African
resources, but commercial extraction Countries such as Zimbabwe, South Africa,
is difficult because the trees do not etc.
occur in homogenous stands, there  Canterbury Plains: These are found in
are no frozen surfaces to facilitate New Zealand.
logging and the tropical hardwoods
are sometimes too heavy to float in the  Campos: These are found in Brazil and
Uruguay.
rivers, even if these flow in the desired
directions. Hence, timbers have less  Tussock Grasslands: These are found
potential for the export. in Australia and New Zealand.

 The equatorial vegetation comprises a  Pampas is found in Argentina.


multitude of evergreen trees that yield
tropical hardwood, e.g. mahogany, ebony, 13. Correct Option: (b)
greenheart, cabinet woods, and dyewoods. Explanation:
There are smaller palm trees, climbing
plants like the lianas or rattan which may Mangroves.
be hundreds of feet long and epiphytic and  A mangrove is a shrub or small tree that
parasitic plants that live on other plants. grows in coastal saline or brackish water.
Under the trees grow a wide variety of
 They occur worldwide in the tropics and
ferns, orchids, and lalang.
subtropics, mainly between latitudes 25°
 Unlike the temperate forests, where only N and 25° S.
a few species occur in a particular area,  They are salt-tolerant trees, also called
the trees of the tropical rain forests are not halophytes, and are adapted to live in
found in pure stands of a single species. It harsh coastal conditions.
has been estimated that in the Malaysian
 Mangroves contain a complex salt filtration
jungle as many as 200 species of trees may
system and complex root system to cope
be found in an acre of forest.
with salt water immersion and wave
action.
 They are adapted to the low oxygen
(anoxic) conditions of waterlogged mud.
 Presence of Mangrove ecosystems
on the coastline can save lives and
property during natural hazards like
cyclones, storm surges, and erosion,
as well.
 The deltas of the Ganga, the Mahanadi, the
Krishna, the Godavari, and the Kaveri are
covered with mangrove vegetation.
12. Correct Option: (d)
Explanation: 14. Correct Option: (a)

Temperate grasslands
Explanation:

 All the above grasslands are the example World Database on Protected Areas
of Temperate grassland. These are areas of  The World Database on Protected Areas
open grassy plains that are very sparsely (WDPA) is the most comprehensive
populated with trees. global database on terrestrial and marine
protected areas.
 There are various names of the grassland
according to the regions.  It is a joint project between the United
Nations Environment Programme
 Steppe: These are the grassland in Central (UNEP) and the International Union for
Asian countries such as Russia, Ukraine, Conservation of Nature (IUCN), managed
Mongolia, Kazakhstan, China, etc. by UNEP World Conservation
 Prairies: These are the temperate Monitoring Centre (UNEP-WCMC).
grasslands of North America(the USA and  The headquarter of IUCN is at Gland,
Canada). Switzerland.
TARGET PT 2020 25

15. Correct Option: (c) tundra shows day and night temperature
variations.
Explanation:
 Flora and fauna: Typical vegetation of
Forest types of India arctic tundra is cotton grass, sedges,
dwarf heath, willows, birches and
 Tropical wet evergreen forests are lichens. Animals of tundra are reindeer,
present in Arunachal Pradesh, Assam, musk ox, arctic hare, caribous, lemmings
Nagaland, Goa, Karnataka, Kerala, and squirrel.
Tamil Nadu, and A&N Islands.
 Most of them have long life e.g. arctic
 Tropical semi-evergreen forests are willow has a life span of 150 to 300 years.
present in 15 states/UTs in India
 They are protected from chillness by the
including Maharashtra. presence of thick cuticle and epidermal
 Tropical thorn forest is present in 11 hair. Mammals of the tundra region
states/UTs in India including Delhi. have large body size, small tail and
small ear to avoid the loss of heat from
the surface.
16. Correct Option: (b)
18. Correct Option: (d)
Explanation:
Explanation:
Biome
 Option (d) is correct: Tamil Nadu
 Boundaries between biomes are not
Becomes Fifth Indian State to Declare
always sharply defined. For instance,
a State Butterfly, after Maharashtra
there are sometimes transition zones
between grassland and forest biomes. (Blue Mormon), Uttarakhand (Common
Coasts and wetlands are transition zones peacock), Karnataka (Southern birdwings)
between terrestrial and aquatic biomes. and Kerala (Malabar banded peacock) to
bestow a state emblem status to one of its
 A biome is NOT an ecosystem, although colorful insects.
in a way it can look like a massive
ecosystem. If we take a closer look, we will Supplementary notes:
notice that plants or animals in any of the
biomes have special adaptations that make  Tamil Nadu Becomes Fifth Indian State to
it possible for them to exist in that area. Declare a State Butterfly.

 No two biomes are alike. The climate  Tamil Nadu has recently declared Tamil
determines the boundaries of a biome and Yeoman (Cirrochroa thais) as its state
abundance of plants and animals found butterfly to symbolize its rich natural
in each one of them. The most important and cultural heritage, in a move aimed
climatic factors are temperature and at boosting the conservation efforts of the
precipitation. attractive insects.
 Locally known as Tamil Maravan meaning
17. Correct Option: (d)
‘Tamilian Warrior’, the canopy butterfly,
Explanation: belongs to the family of brush-footed
butterflies or the Nymphalid.
Tundra Biome
 This is the latest addition to Tamil Nadu’s
 Tundra means a “barren land” since existing symbols from the natural world –
they are found where environmental
palmyra as the state tree, gloriosa lily as
conditions are very severe. There are
the state flower, emerald dove as the state
two types of tundra- arctic and alpine.
bird, and jackfruit as the state fruit and
 Distribution: Arctic tundra extends as Nilgiri tahr as the state animal.
a continuous belt below the polar ice cap
and above the tree line in the northern  This butterfly species is endemic to Western
hemisphere. It occupies the northern fringe Ghats.
of Canada, Alaska, European Russia,  Once the species is declared as a state
Siberia and island group of Arctic Ocean. butterfly, this will help channelizing
On the South Pole, tundra is very small government funds towards a particular
since most of it is covered by ocean. environmental cause.
Alpine tundra occurs at high mountains
above the with respect to Arctic mountains  Tamil Nadu has a total of 32 species of
are found at all latitudes therefore alpine butterflies endemic to the state.
26 TARGET PT 2020

 It has become the fifth India state after Appendix II of CITES.


Maharashtra (Blue Mormon), Uttarakhand
 Giraffes, those tall, stately and graceful
(Common peacock), Karnataka (Southern
animals of Africa’s savannahs, have been
birdwings) and Kerala (Malabar banded accorded protection from unregulated trade
peacock) to bestow a state emblem status as the world finally woke up to their ‘silent
to one of its colorful insects. extinction’.
 Both southern bird wings, which is the  The Appendix II listing was proposed by
largest butterfly species found in India, Central African Republic, Chad, Kenya,
and Malabar banded peacocks are, like the Mali, Niger and Senegal.
Tamil Yeoman, endemic to the Western
Ghats as well.  “Appendix II includes species not
necessarily threatened with extinction, but
19. Correct Option: (d) in which trade must be controlled in order
to avoid utilization incompatible with their
Explanation: survival,”

All the above statements are correct  Giraffes once ranged over much of the semi-
arid savannah and savannah woodlands of
Supplementary notes: Africa. But their numbers have plummeted
dramatically — by up to 40 per cent over
Ansupa Lake the last 30 years — due to threats including
 The Odisha Wetland Authority has international trade in their parts, as well
approved implementation of an integrated as habitat loss, civil unrest and illegal
management plan for Chilika, country’s hunting.
largest brackish water lagoon, and Ansupa,  While giraffes fall prey to poaching for bush
State’s largest freshwater lake. meat, bones, skin and tail hair, there is also
 The five-year management of lakes is a significant amount of international trade
intended at strengthening livelihood of in their bone carvings and trophies.
thousands of fishermen relying on the two
 There is currently only one recognized
water-bodies. Besides, tourism promotion
species of giraffe, with nine sub-species.
and conservation of ecology will be taken
They have been listed as ‘vulnerable’ on
up.
the International Union for Conservation
 Ansupa Lake is one of the largest fresh of Species Red List since 2016, with some
water lake of Odisha situated in Banki, sub-species classified as ‘endangered’’ or
Cuttack. ‘critically endangered’.
 It was created by Mahanadi and got a  Five of the nine sub-species have only a
shape like the hoof of a horse. small wild population, while four have
 It spreads over a vast area of 141 hectare, a decreasing population trend. All are
and surrounded by Saranda Hills in its affected by trade.
length.  While the Appendix II listing will not stop
 The lake is surrounded with high all trade in giraffe parts, it will ensure this
hills. One can have a view of high hills is not contributing to further population
around the lake. declines and provide global scale data that
could not otherwise be obtained.
20. Correct Option: (a)
21. Correct option: (b)
Explanation:
Explanation
Option (a) is correct.
 Statement 1 is incorrect: Ebola virus
Supplementary notes: was first discovered in 1976 near the
Ebola River in what is now the Democratic
Giraffe Republic of Congo.
 The giraffe has been placed in Appendix
Supplementary notes
II of CITES, which places prohibitions
on uncontrolled trade in a species. Ebola epidemic
The Conference of Parties (CoP) to the
Convention on International Trade in  Health officials in Eastern Congo have
Endangered Species or CITES in Geneva documented the first relapse of the
passed a resolution to place the giraffe in current Ebola epidemic.
TARGET PT 2020 27

 The Ebola outbreak in Democratic  National Green Corps is a programme


Republic of Congo has so far infected started in 2001-02 by the Ministry of
more than 3,300 people and killed more Environment Forests and Climate Change.
than 2,200 since the middle of last year,  Aim: To provide opportunities for children
making it the second-worst on record. to understand the environment and
 According to WHO, this recent relapse environmental problems through school
happened in the Aloya district where eco-clubs.
a motorcycle taxi operator is said to have  The programme is a sub part of Environment
come into contact with 33 people potentially Education Awareness and Training (EEAT)
the cause of 11 new confirmed Ebola cases which is a central sector scheme of the
in the past week. Ministry of the Environment continuing
since 1983-84.
About Ebola
Objectives of the Programme
 Ebola Virus Disease (EVD) is a rare and
deadly disease in people and nonhuman  To impart knowledge to school children
primates. The viruses that cause EVD are through hands-on experience, about their
located mainly in sub-Saharan Africa. immediate environment, interactions
within it and the problems therein.
 People can get EVD through direct
contact with an infected animal (bat  To develop requisite skills of observation,
or nonhuman primate) or a sick or dead experimentation, survey, recording,
person infected with Ebola virus. analysis and reasoning for conserving the
environment through various activities.
 The US Food and Drug Administration
(FDA) has not yet approved a vaccine  To inculcate the proper attitude towards
or treatment for Ebola virus infection. It the environment and its conservation
is caused by an infection with a group of through community interactions.
viruses within the genus Ebolavirus:  To sensitize children to issues related to
! Ebola virus (species Zaire ebolavirus) environment and development through
field visits and demonstrations.
! Sudan virus (species Sudan
ebolavirus)  To promote logical and independent
thinking among children so that they are
! Taï Forest virus (species Taï Forest able to make the right choices in a spirit of
ebolavirus, formerly Côte d’Ivoire scientific inquiry.
ebolavirus)
 To motivate and stimulate young minds by
! Bundibugyo virus (species Bundibugyo involving them in action projects related to
ebolavirus) environmental conservation.
! Reston virus (species Reston
ebolavirus) Implementation

! Bombali virus (species Bombali  The scheme is being operated through Eco-
ebolavirus) clubs of 50-60 students having an interest
in environment related issues, formed in
 Of these, only four (Ebola, Sudan, Taï member schools.
Forest, and Bundibugyo viruses) are
known to cause disease in people.  Eco clubs are supervised by a Teacher In-
charge who is selected from among the
 Ebola virus was first discovered in 1976 teachers of the member schools on the basis
near the Ebola River in what is now of his/her interest in environment related
the Democratic Republic of Congo. issues.
 It kills up to 90% of people who are  There is District Implementation and
infected. Monitoring Committee to supervise,
organise training for In-charge teachers and
22. Correct Answer: (a) monitor periodically the implementation of
Explanation: Option (A) is correct. scheme at the District level.
 There is a State Steering Committee for
Supplementary Notes:
guidance, direction and to oversee the
 Recently, the Ministry of Environment, implementation of the scheme.
Forest and Climate Change (MoEFCC)
 The State Nodal Agency coordinates the
organized an annual meeting of state
implementation of the scheme in the State
nodal agencies implementing the ‘Eco club’
and organize related activities like training
programme for the first time.
to Master Trainers.
28 TARGET PT 2020

 The National Steering Committee will give  Re-organisation of Railway Board on


overall direction to the programme and functional lines headed by CRB with four
ensure linkages at all levels. Members and some Independent Members
 The existing service of Indian Railway
23. Correct Answer: (a) Medical Service (IRMS) to be consequently
 Explanation: 4thstatement is incorrect. renamed as Indian Railway Health Service
Despite being one of the largest consumers (IRHS)
of energy, India has a low share of MNCs in
 Railways has an ambitious programme
the domestic market.
to modernise and provide the highest
Supplementary Notes standards of safety, speed and services to
the passengers with a proposed investment
Key Problems of Oil and Gas Sector of Rs. 50 lakh crore over the next 12 years.

 Declining domestic crude  Unlike Railway systems the world over,


production: Most of the producing fields which have been corporatized, Indian
(in Cambay, Assam-Arakan and Mumbai Railways is managed by the Government
Offshore) are maturing or have already directly. It is organised into various
matured. Due to inadequate new oil and gas departments such as Traffic, Civil,
discoveries and subsequent development, Mechanical, Electrical, Signal & Telecom,
India is witnessing a decline in crude Stores, Personnel, and Accounts etc.
production.  These departments are vertically separated
 Large crude import bills: In 2018-19, from top to bottom and are headed by a
India imported more than 80% of its crude Secretary level officer (Member] in the
consumption and spent in excess of $110 Railway Board. This organization of the
billion. department runs deep down to the grassroot
level of the Railways.
 Inadequate transmission & distribution
infrastructure: India needs to invest  Unification of services will end this
heavily in midstream and downstream sector ‘departmentalism’, promote smooth working
of Railways, expedite decision making,
to overcome infrastructure constraints in
create a coherent vision for organisation
LNG, gas pipelines and CGD.
and promote rational decision making.
 Technology constraints: The country
 Unification of services has been
needs investment in exploring and
recommended by various committees for
developing Category-2 and Category-3
reforming Railways including - the Prakash
basins. However, Indian operators do
Tandon Committee (1994), Rakesh Mohan
not have the requisite technology and
Committee (2001), Sam Pitroda Committee
experience in this area.
(2012) and Bibek Debroy Committee
 Low share of MNCs: Despite being one (2015).
of the largest consumers of energy, India
 It is now proposed to create a unified
has a low share of MNCs in the domestic
Group A’ service called “Indian Railways
market.
Management Service” (IRMS) from the
 Environmentally friendly fuels: India next recruitment cycle. Creation of the new
continues to rely heavily on coal and service will be done in consultation with
petroleum products to meet its energy DoPT and UPSC to facilitate recruitment
needs. in the next recruitment year. It will
enable Railways to recruit engineers/non-
24. Correct option: (a) engineers as per need, and offer equality
of opportunity to both categories in career
Explanation progression.
 Statement 3 is incorrect: The Union  The modalities and unification of the
Cabinet chaired by Prime Minister has services will be worked out by the Ministry
approved a transformational organisational of Railways in consultation with DoPT with
restructuring of the Indian Railways. the approval of Alternate Mechanism to be
appointed by Cabinet in order to ensure
The reforms include: fairness and transparency. The process
 Unification of the existing eight Group shall be completed within a year.
A services of the Railways into a Central  Railway board will no longer be organised
Service called Indian Railway Management on departmental lines, and replaced with
Service (/RMS) a leaner structure organised on functional
TARGET PT 2020 29

lines. It will have a Chairman, who will Supplementary notes


act as ‘Chief Executive Officer (CEO)’
 NITI Aayog has the mandate of overseeing
along with 4 Members responsible for
the adoption and monitoring of SDGs in the
Infrastructure, Operations & Business country, at the national and sub-national
Development, Rolling Stock and Finance level.
respectively.
 The SDG India Index, whose first edition
was launched in December 2018, was the
25. Correct option: (d)
first tool developed by any large country
Explanation to monitor the progress towards achieving
SDGs at the sub-national level.
 All three Statements are correct:
NITI Aayog will launch the second edition  The SDG India Index and Dashboard 2019
of the Sustainable Development Goals tracks the progress of and ranks all States
(SDG) India Index, which documents the and UTs on 100 indicators drawn from
progress made by India’s States and Union MoSPI’s National Indicator Framework,
Territories towards implementing the 2030 comprising 306 indicators.
SDG targets, on 30 December 2019 at NITI  It indicates where the country and its
Aayog, New Delhi. States and UTs currently are on SDG
 The SDG India Index and Dashboard 2019– implementation, and charts the distance to
20 have been developed in collaboration be travelled to reach the SDG targets.
with the Ministry of Statistics and  The Index covers 16 out of 17 SDGs and
Programme Implementation (MoSPI), the a qualitative assessment on Goal 17. This
United Nations in India, and the Global marks an improvement over the 2018
Green Growth Institute. Index, which covered only 13 goals.

**********
30 TARGET PT 2020

TEST
DAY - 33

Time Allowed: 30 mins Maximum Marks: 50

1. Consider the following statements 2. Soft corals do not always have


regarding Seaweeds: zooxanthellae.
1. They are small rooted well-developed 3. the Mesoamerican Reef is the second-
plants of high economic and medicinal largest reef system in the world.
values.
Which of the above statements is/are
2. They grow only on the surface of the
seawaters. correct?

3. In India, only the Andaman and (a) 1 only


Nicobar islands have the seaweeds. (b) 1 and 2 only
Which of the above statements are (c) 2 and 3 only
incorrect?
(d) 1, 2, and 3
(a) 1 and 3 only
4. What are the features of the
(b) 2 and 3 only
Seagrasses?
(c) 1 and 2 only
1. Seagrasses are not grass but type of
(d) 1, 2 and 3 seaweeds.

2. Consider the following statements 2. They are found in the shallow salty
regarding phytoplanktons diversity: and brackish waters.

1. Phytoplanktons are also known as 3. They are one of the most productive
microalgae. ecosystems in the world.
2. Dinoflagellates and Diatoms are the Select the correct option using the codes
two main classes of phytoplankton. given below:
3. While the bodies of the Dinoflagellates
(a) 2 only
are covered with complex shells, the
Diatoms are shell-less organism. (b) 1 and 3 only

Which of the above statements is/are (c) 2 and 3 only


correct? (d) 1, 2, and 3
(a) 1 only
5. Which of the following countries are
(b) 2 and 3 only included in the “Coral Triangle”?
(c) 1 and 2 only 1. Australia
(d) 1, 2, and 3 2. New Zealand
3. Indonesia
3. Consider the following statements:
1. The hard coral reefs are entirely built 4. Papua New Guinea
by the polyps. 5. Solomon Islands
TARGET PT 2020 31

Select the correct option using the codes Ramsar sites: States
given below: 1. Nawabganj: Uttar Pradesh
(a) 1, 2, and 4 only 2. Sarsai Nawar: Maharashtra
(b) 3, 4, and 5 only 3. Keshopur-Miani: Punjab

(c) 1 and 2 only Select the correct option from the codes
given below:
(d) 1, 2, 3, 4, and 5
(a) 1 only
6. The largest freshwater lake of
(b) 1 and 3 only
Northeast India is in which state?
(c) 2 and 3 only
(a) Assam
(d) 1, 2, and 3
(b) Manipur
(c) Mizoram 10. Arrange the above Ramsar sites in
North-South direction:
(d) Tripura
A. Pong Dam
7. Consider the following statements
B. Harike
regarding ‘Neuston’:
1. These are found living at the surface of C. Loktak lake
the ocean.
D. Nalsarovar
2. They are just like reefs on the seafloor.
Select the correct option from the codes
Select the correct answer using the code given below:
given below: (a) A-B-C-D
(a) 1 only (b) C-A-B-D
(b) 2 only (c) C-A-D-B
(c) Both 1 and 2 (d) B-C-D-A
(d) Neither 1 nor 2
11. Which of the following statements
regarding coral reefs is/are correct?
8. Which of the following statements
regarding Fjord is/are incorrect? 1. Not all Coral reefs require
photosynthesis to survive.
1. It is a type of estuary created by the
glaciers. 2. Over half of all the known coral species
are found in the deep, dark ocean.
2. Fjords tend to have more input of
freshwater than the saltwater. Select the correct option from the codes
given below:
Select the correct option from the codes
(a) 1 only
given below:
(b) 2 only
(a) 1 only
(c) Both 1 and 2
(b) 2 only
(d) Neither 1 nor 2
(c) Both 1 and 2
12. Consider the following statements
(d) Neither 1 nor 2
regarding a lake:
9. Recently, ten more wetlands from 1. It is the first lake in India, declared as
India get the Ramsar site tag. In this the Biodiversity Heritage Site.
regard, which of the following pairs is/ 2. It is in Sangareddy district of
are correctly matched? Telangana.
32 TARGET PT 2020

Which of the following lakes has been 1. The depth of the photic zone depends
described above? on the transparency of water.
(a) Pulicat Lake 2. Only respiration activity takes place in
the aphotic zone.
(b) Kolleru Lake
3. No photosynthesis activity takes place
(c) Ameenpur lake
in the aphotic zone.
(d) Fox Sagar Lake
Which of the following statements is/are
13. Which of the following is not an example correct?
of the Lentic ecosystem?
(a) 1, 2 and 3
(a) Pond
(b) 1 only
(b) Lake
(c) 3 only
(c) Swamps
(d) 1 and 3 only
(d) Rivers
17. What is the possible implication of
14. In aquatic ecosystems, oxygen adding fertilizer to aquatic systems?
is dissolved in water, where its
concentration varies constantly (a) Increased growth of phytoplankton and
depending on factors that influence hypoxic conditions
the input and output of oxygen in the (b) Increased growth of phytoplankton and
water. In this context, consider the hyperoxic conditions
following statements:
(c) Decreased growth of phytoplankton
1. Oxygen enters the aquatic ecosystem
and hypoxic conditions
through the air-water interface.
2. The amount of dissolved oxygen (d) Decreased growth of phytoplankton
present in water is not influenced by and hyperoxic conditions
temperature.
18. What is Red Tide?
Which of the above statements is/are (a) It is a process in which algal blooms
incorrect? produce toxins that affect aquatic
(a) 1 only organisms and humans.
(b) 2 only (b) The red color of ocean tides due to the
reflection of the sunlight
(c) Both 1 and 2
(c) It is a phenomenon where phytoplankton
(d) Neither 1 nor 2
species contain pigments and “bloom”
15. Which of the following statements is/ such that the human eye perceives the
are correct regarding the Neuston? water to be discolored
1. These are found living at the bottom of (d) None of the above.
the sea-level.
19. What are the reasons behind the
2. They obtain their food within the water
removal of the nutrients from a lake?
like back-swimmers.
1. Deepwater abstraction
Select the correct answer using the code
2. Harvest of fishes
given below:
3. Removal of sludge
(a) 1 only
(b) 2 only Select the correct answer using the code
given below:
(c) Both 1 and 2
(a) 1 and 2 only
(d) Neither 1 nor 2
(b) 2 and 3 only
16. Consider the following statements
(c) 1 and 3 only
regarding various zones in an aquatic
ecosystem: (d) 1, 2 and 3
TARGET PT 2020 33

20. Which of the following pair are (c) 1 and 3 only


incorrectly matched regarding the types
(d) 1, 2 and 3
of lakes on the basis of nutrient content
in India? 23. Consider the following statements
1. Oligotrophic: highly nutrient-rich about International Financial Services
2. Mesotrophic: moderate nutrients Centres Authority
3. Eutrophic: very low nutrients 1. An international financial services
centre caters to customers outside
Select the correct answer using the code the jurisdiction of domestic economy,
given below: dealing with flows of finance, financial
(a) 1 and 2 only products and services across borders.

(b) 2 and 3 only 2. It was set up in 2018 with headquarter


in Mumbai.
(c) 1 and 3 only
Which of the above statement(s) is/are
(d) 1, 2 and 3
correct?
21. With reference to World Malaria (a) 1 only
Report 2019, consider the following
statements (b) 2 only
1. It provides a comprehensive update on (c) Both 1 and 2
global and regional malaria data and (d) None
trends.
2. Africa and India saw the maximum 24. In which of the following case Supreme
dip in malaria cases between 2017 and Court held that creamy layer principle
2018. should be applied to the SC and ST
communities?
Which of the following statement is/are
correct? (a) M. Nagaraj v. Union of India (2006)
(a) 1 only (b) Jarnail Singh v. Lachhmi Narain
Gupta case of 2018
(b) 2 only
(c) Keshavananda Bharti Case (1973)
(c) Both 1 and 2
(d) Indra Sawhney v. Union of India
(d) Neither 1 nor 2
(1992)
22. Recently, the Central Water
25. Consider the following statements
Commission (CWC) has reported
that the samples from two-thirds of about Human Development Report
the water quality stations spanning 2019
India’s major rivers have a heavy metal 1. The annual HDI 2019 report, ranked
contamination problem. Consider the India at the 129th position, one rank
following statements regarding it above last year’s ranking, out of a total
1. The exercise was limited to surface 189 countries.
water and did not cover groundwater 2. India remains the home to 28 percent
contamination. of global poor
2. The most common heavy metal found 3. HDI was created by Indian Economist
was iron. Amartya Sen.
3. Arsenic and zinc are the two toxic
metals whose concentration was always Which of the above statement(s) is/are
obtained within the limits. correct?
(a) 1 only
Which of the following statement is/are
correct? (b) 2 only
(a) 1 only (c) 1 and 2 only
(b) 2 and 3 only (d) 1, 2 and 3
34 TARGET PT 2020

ANSWER HINTS
DAY - 33

1. Correct Option: (d) get enough sunlight and its growth will be
poor.
Explanation:
 Seaweeds provide a new renewable
Seaweeds source of food, energy, chemicals,
and medicines. It also provides a
 The Seaweeds are a group of
valuable source of raw material for
photoautotrophic, macrophytic algae, a
industries like health food, medicines,
primitive type of plants lacking true
pharmaceuticals, textiles, fertilizers,
roots, stems, and leaves.
animal feed etc.
 The word seaweed gives the wrong
 Seaweed is chock-full of vitamins,
impression that it is a useless plant.
minerals, and fiber, and can be tasty.
Seaweeds are wonder plants of the sea and
For at least 1,500 years, the Japanese have
highly useful plants.
enrobed a mixture of raw fish, sticky rice,
 Seaweeds grow in the shallow and other ingredients in a seaweed called
waters. Root system and conducting nori. The delectable result is a sushi roll.
tissues like land plants are absent in In fact, seaweeds are called medical food of
seaweeds. the 21st century.
 Many seaweeds contain anti-inflammatory
and anti-microbial agents. Their known
medicinal effects have been legion for
thousands of years; the ancient Romans
used them to treat wounds, burns, and
rashes. Anecdotal evidence also suggests
that the ancient Egyptians may have used
them as a treatment for breast cancer.
 Certain seaweeds do, in fact, possess
powerful cancer-fighting agents that
researchers hope will eventually prove
 Some seaweeds are microscopic, such as the effective in the treatment of malignant
phytoplankton that live suspended in the tumors and leukemia in people. While
water column and provide the base for most dietary soy was long credited for the low
marine food chains. Some are enormous, rate of cancer in Japan, this indicator of
like the giant kelp that grows in abundant robust health is now attributed to dietary
seaweed.
“forests” and tower-like underwater
redwoods from their roots at the bottom  These versatile marine plants and algae
of the sea. Most are medium-sized, come have also contributed to economic growth.
in colors of red, green, brown, and black, Among their many uses in manufacturing,
and randomly wash up on beaches and they are effective binding agents
shorelines just about everywhere. (emulsifiers) in such commercial goods
as toothpaste and fruit jelly, and popular
 Seaweeds grow in the shallow waters.
softeners (emollients) in organic cosmetics
Seaweed planted in shallow water near
and skin-care products.
the surface (30 to 50 centimeters) receives
plenty of sunlight and its growth will be  Seaweeds grow abundantly along the
good. Seaweed planted in deep water (more Tamil Nadu and Gujarat coasts and
than 1 meter from the sea surface) does not around Lakshadweep and Andaman
TARGET PT 2020 35

and Nicobar islands. There are also for photosynthesis to take place. This
rich seaweed beds around Mumbai, photosynthetic rate decreases as the depth
Ratnagiri, Goa, Karwar, Varkala, in the aquatic system increase owing to a
Vizhinjam and Pulicat in Tamil Nadu reduction in light intensity.
and Chilka in Orissa.
 The photosynthetic rate increases with an
increase in temperature but diminishes
2. Correct Option: (c) sharply after a point is reached as the
Explanation: protoplasmic activity cannot be maintained
at temperatures above 40-degree
Phytoplanktons centigrade. Thus, temperature together
with illumination influences the seasonal
 Phytoplankton, also known as microalgae,
variation of phytoplankton production
are similar to terrestrial plants in that they more specifically on temperate latitudes.
contain chlorophyll and require sunlight in Phytoplanktons produce more than 60% of
order to live and grow. oxygen produced from all the plants.
 Most phytoplankton are buoyant and  Phytoplankton, like land plants, require
float in the upper part of the ocean, where nutrients such as nitrate, phosphate,
sunlight penetrates the water. silicate, iron, and calcium at various levels
 Phytoplankton also requires inorganic depending on the species.
nutrients such as nitrates, phosphates, and  Some phytoplankton can fix nitrogen
sulfur which they convert into proteins, and can grow in areas where nitrate
fats, and carbohydrates. concentrations are low.
 The two main classes of phytoplankton  They also require trace amounts of iron
are dinoflagellates and diatoms. which limits phytoplankton growth in
Dinoflagellates use a whip-like tail, or large areas of the ocean because iron
flagella, to move through the water and concentrations are very low.
their bodies are covered with complex
shells. Diatoms also have shells, but  Other factors influencing phytoplankton
they are made of a different substance growth rates include water temperature
and their structure is rigid and made and salinity, water depth, wind, and what
of interlocking parts. Diatoms do not kinds of predators are grazing on them.
rely on flagella to move through the water
and instead rely on ocean currents to travel 3. Correct Option: (c)
through the water. Explanation:
 In a balanced ecosystem, phytoplankton
provide food for a wide range of sea Coral reefs
creatures including whales, shrimp, snails,  Coral reefs only occupy 0.1% of the area
and jellyfish. When too many nutrients of the ocean but they support 25% of all
are available, phytoplankton may grow marine species on the planet. Hundreds
out of control and form harmful algal of millions of people rely on coral reefs for
blooms (HABs). These blooms can produce essential nutrition, livelihoods, protection
extremely toxic compounds that have from life-threatening storms and crucial
harmful effects on fish, shellfish, mammals, economic opportunity.
birds, and even people.

Table coral (Acropora cytherea)


 Coral reefs are built by coral polyps as they
secrete layers of calcium carbonate beneath
 They are limited to the uppermost layers of their bodies. The corals that build reefs are
the ocean where light intensity is sufficient known as “hard” or “reef-building” corals.
36 TARGET PT 2020

Soft corals, such as sea fans and sea whips, 4. Correct Option: (c)
do not produce reefs; they are flexible
organisms that sometimes resemble plants Explanation:
or trees. Seagrasses
 Soft corals do not have stony skeletons  Seagrasses are found in shallow salty
and do not always have zooxanthellae. and brackish waters in many parts of
They can be found in both tropical the world, from the tropics to the Arctic
seas and in cooler, darker parts of the Circle.
ocean.
 Seagrasses are so-named because most
 The coral polyps that build the reef survive species have long green, grass-like leaves.
by forming a symbiotic relationship with
microscopic algae called zooxanthellae.  They are often confused with
The polyps offer the algae shelter while seaweeds(algae), but are actually
the zooxanthellae create energy—through more closely related to the flowering
photosynthesis—that the corals use as food. plants that we see on land.
In a sense, the coral polyps are “farming”
the algae. The waste products of the polyps
also serve as food for the zooxanthellae.
Corals are also predators; they extend
their tentacles at night and capture tiny
organisms (zooplankton, small fish, or
other potential food item) that happen to
be floating by with stinging cells called
nematocysts. The captured prey is then
moved into the polyps’ mouths and digested
in their stomachs.
 Other types of animals and plants also
contribute to the structure of coral
reefs. Many types of algae, seaweed,
sponges, sediment, and even mollusks Sea weeds vs Seagrasses
like giant clams and oysters add to
the architecture of coral reefs. When  Like land plants, seagrass produce
these organisms die, they also serve as oxygen. The depth at which seagrass
foundations for new corals. are found is limited by water clarity,
which determines the amount of light
 Stretching nearly 1,000km from the reaching the plant. Light is required
northern tip of Mexico’s Yucatan Peninsula for the plants to make food through
and the Caribbean coasts of Belize and photosynthesis.
Guatemala to the Bay Islands off northern
Honduras, the Mesoamerican Reef is  Seagrasses have roots, stems and
the largest reef system in the Americas leaves, and produce flowers and seeds.
and only second in the world after the They evolved around 100 million years
Great Barrier Reef, the largest living ago, and today there are approximately 72
thing on the planet. different seagrass species that belong to
four major groups.
 Therefore, seagrasses are neither
grasses nor seaweeds.

Mesoamerican Reef (Jewel of the


Caribbean)
TARGET PT 2020 37

Well-developed Seagrasses 7. Correct Option: (c)


 Seagrasses can form dense underwater Explanation:
meadows, some of which are large enough
to be seen from space. Neuston
 Although they often receive little  The term neuston refers to the
attention, they are one of the most assemblage of organisms associated
productive ecosystems in the world. with the surface film of lakes, oceans,
Seagrasses provide shelter and food and slow-moving portions of streams.
to an incredibly diverse community It generally includes species living
of animals, from tiny invertebrates just underneath the water surface
to large fish, crabs, turtles, marine (hyponeuston), individuals that are
mammals and birds. above but immersed in the water
 Seagrasses provide many important (epineuston), and taxa that travel over
services to people as well, but many the surface on hydrophobic structures
seagrasses meadows have been lost (superneuston or, more properly, a
because of human activities. form of epineuston).
 Thus, these are unattached organisms
5. Correct Option: (b) that live at the air-water interface
such as floating plants, etc.
Explanation:
Coral Triangle
 The Coral Triangle, the global center of
marine biodiversity, is a 6 million km2
area spanning Indonesia, Malaysia,
the Philippines, Papua New Guinea,
Timor Leste and the Solomon Islands.
 Within this nursery of the seas live 76% of
the world’s coral species, 6 of the world’s  The neustonic food web is primarily
7 marine turtle species, and at least 2,228 supported by a thin bacterial film on the
reef fish species. upper surface of the water, a concentration
of phytoplankton near the surface, and
allochthonous inputs from trapped
6. Correct Option: (b) terrestrial and aquatic organisms.
 Just like reefs on the seafloor, this
Explanation:
ecosystem does not stand apart from
Loktak lake the open ocean around it. The neuston
is a nursery for multiple species of
 Loktak lake, in Manipur, is the largest larval fish and a hunting ground for
freshwater lake in Northeast India. paper nautilus octopuses. It supports
 It is known for its circular floating swamps, sunfish, leatherback turtles, and
called phumdis in the local language. diverse ocean grazers, which frequent
these islands, relying on them as a
food source.

8. Correct Option: (d)


Explanation:
Fjords
 Fjords (pronounced fee-YORDS) are typically
long, narrow valleys with steep sides that
are created by advancing glaciers.
 The glaciers leave deep channels carved
Phumdis on the Loktak lake into the earth with a shallow barrier, or
narrow sill, near the ocean.
 The Keibul Lamjao National Park located
at the southwestern part of the lake is a  When the glaciers retreat, seawater
habitat of brow-antlered deer, Sangai, floods the deeply incised valleys, creating
which is also the state animal of Manipur. estuaries.
38 TARGET PT 2020

 Fjords tend to have a moderately high  Harike Lake is in Punjab.


input of freshwater. In comparison, very
 Loktak lake is in Manipur Park.
little seawater flows into the fjord because
of the sill.  Nalsarovar is in Gujarat.
 In addition, the sill prevents deep waters in
the fjord from mixing with deep waters of 11. Correct Option: (c)
the sea. This poor water exchange results
in stagnant, anoxic (low oxygen) water that Explanation:
builds upon the bottom of the fjord.
Deep-water corals
 Not surprisingly, fjords are found in areas
 There are both warm (tropical) corals
that were once covered with glaciers. and cold-water corals. Corals are not just
 Glacier Bay in Alaska and the Georgia warm-water creatures. They also live at
Basin region of Puget Sound in Washington depths of 40m down to 2,000m, in water
State are good examples of fjords. temperatures as low as 40C.
 Fjords are also found throughout Canada, Coral reefs
Chile, New Zealand, Greenland, Norway, Warm Water
Cold water
Siberia, and Scotland.

PC: NOAAA, USA

9. Correct Option: (b)


Explanation:
 In fact, more than half of coral species
New Ramsar Sites in India
are found in the cold water.
 In a major recognition towards the
 Cold-water corals are formed under the
Government of India’s effort towards deep and dark sea where sunlight does not
conservation, restoration, and rejuvenation reach.
of its wetlands, Ramsar has declared 10
more wetland sites from India as sites of  Unlike tropical corals, cold-water corals
international importance. The Ramsar don’t have symbiotic photosynthetic algae
Convention signed on February 2, 1971, is viz. Zooxanthellae, so they don’t need
one of the oldest inter-governmental accord photosynthesis to survive.
signed by member countries to preserve  In fact, they feed solely by capturing food
the ecological character of their wetlands particles from the surrounding water
of international importance.
 Deep-sea coral reefs are made up of only a
 With this, the numbers of Ramsar sites few coral species but they provide a home
in India are now 37 and the surface area for many other animals, including sea fans,
covered by these sites is now 1,067,939 sponges, worms, starfish, brittle stars, sea
hectares. urchins, crustaceans, and fishes, etc.
 Maharashtra gets its first Ramsar site  Example- Coral: Lophelia(Rost reef,
(Nandur Madhameshwar), Punjab Norway)
which already had 3 Ramsar sites
adds 3 more (Keshopur-Miani, Beas  Some large cold-water coral reefs are found
Conservation Reserve, Nangal) and in the far North and far South Atlantic
UP with 1 Ramsar site has added 6 more oceans.
(Nawabganj, Parvati Agra, Saman,
Samaspur, Sandi and Sarsai Nawar). 12. Correct Option: (c)
Explanation:
10. Correct Option: (a)
Ameenpur Lake
Explanation:
 “Biodiversity Heritage Sites” (BHS)
Ramsar sites in India are well-defined areas that are unique,
ecologically fragile ecosystems - terrestrial,
 Presently, India has 37 wetlands which
coastal and inland waters and, marine
have been recognized as Ramsar Sites.
having rich biodiversity comprising of any
 Pong Dam Lake is in Himachal one or more of the following components:
Pradesh richness of wild as well as domesticated
TARGET PT 2020 39

species or intra-specific categories, high Ameenpur Lake


endemism, presence of rare and threatened
species, keystone species, species of  Under Section 37 of Biological Diversity
evolutionary significance, wild ancestors Act, 2002 (BDA) the State Government in
of domestic/ cultivated species or their consultation with local bodies may notify
varieties, past pre-eminence of biological
components represented by fossil beds in the official gazette, areas of biodiversity
and having significant cultural, ethical or importance as Biodiversity Heritage Sites
aesthetic values and are important for the (BHS).
maintenance of cultural diversity, with or
without a long history of human association  Ameenpur lake is the first water body
with them. in India that was declared as the BHS.
Further, it is the first biodiversity site to
be approved in an urban area. The lake
dates back to the time of Ibrahim Qutb
Shah, who ruled the kingdom of Golconda
between 1550 and 1580.

 Other BHS are as follows:

Name of the Name of


S. No. Importance of the area
Site the District

1 Nallur Tamarind Bengaluru It is popularly believed to be a relic of the Chola Dynasty


Grove that ruled nearly 800 years ago, is a spectacle of awesome
wonder and a freakish site. This BHS spread over 54 acres
comprising a population of nearly 300 trees, is a picture
of the dynamic pattern of plant diversity. The significant
component of this popular structure is a group of old plants
standing like ageless sentinels, firmly rooted to the ground
with their gigantic trunks, along with large picturesque
crowns spread very high and aloft like open wings.

2 Hogrekan The area has unique Shola vegetation and grassland with
a number of floral species that are unique and have a lot of
medicinal value. Hogrekan is moderately wooded land and
its vegetation is of dry deciduous type and has a link with
Bababudanagiri and Kemmangundi, adjoining Bhadra
Wildlife Sanctuary and Yemmedode Tiger Reserve and
serving as "Wildlife Corridor" between Kudremukha and
Bhadra Wildlife Sanctuary.

3 University of Bengaluru The GKVK campus is considered one of the greenest


Agricultural areas in Bengaluru. Biological diversity of this campus
Sciences, constitutes a critical repository of various forms of flora
GKVK Campus, and fauna (including 13 sp of mammals, 10 sp of reptiles,
Bengaluru 165 sp of birds and 530 sp of plants) which needs to be
protected nurtured to posterity.

4 Ambaraguda Shimoga It is a revenue land located between Sharavathi Wild


Life Sanctuary and Someshwara Wildlife Sanctuary. It
has Shola vegetation which is primitive vegetation in the
Western Ghat and also has grasslands.

5 Glory of Allapalli Gadchiroli It is a reserved forest being preserved as natural forest


having biological, ethical and historical values.

6 Tonglu BHS Darjeeling It is a Medicinal Plant Conservation Areas


under the
Darjeeling Forest
Division
40 TARGET PT 2020

7 Dhotrey BHS Darjeeling It is a Medicinal Plant Conservation Areas


under the
Darjeeling Forest
Division
8 Dialong Village Tamenglong ---

9 Ameenpur lake Sangareddy ---

10 Majuli Majuli It is an island situated in the Brahmaputra River which is


harboring unique Ecological and Cultural Heritage.
11 Gharial Lucknow It is a center established for conservation and rehabilitation
Rehabilitation of critically endangered species of Gharial.
Centre
12 Chilkigarh Kanak Jhargram Chilkigarh Kanak Durga Sacred Grove is a remnant forest
Durga with traditional beliefs and taboos of local inhabitants
and rich in biodiversity covering an area of 55.9 acres in
Jhargram District of West Bengal.

13. Correct Option: (d)  Therefore, the quantity of dissolved oxygen


present in an ecosystem depends on the
Explanation: rate at which the aforesaid two processes
Freshwater ecosystem occur.

 Water on land which is continuously  Dissolved oxygen escapes the water body
cycling and has low salt content is known through the air-water interface and through
as freshwater and respiration of organisms (fish, decomposers,
zooplanktons, etc).
 its study is called limnology.
 The amount of dissolved oxygen
! Static or still water (Lentic) e.g. pond, retained in water is also influenced
lake, bogs, and swamps. by temperature. Oxygen is less soluble
! Running water (Lotic) e.g. springs, in warm water. Warm water also enhances
mountain brooks, streams, and rivers. decomposer activity. Therefore, increasing
the temperature of a water body increases
14. Correct Option: (b) the rate at which oxygen is depleted from
water.
Explanation:
 When the dissolved oxygen level falls below
Dissolved Oxygen in Aquatic Ecosystems 3-5 ppm, many aquatic organisms are likely
 In aquatic ecosystems, oxygen to die.
is dissolved in water, where its
concentration varies constantly
15. Correct Option: (b)
depending on factors that influence Explanation:
the input and output of oxygen in the
water. Neuston

 In freshwater, the average concentration  These are unattached organisms that


of dissolved oxygen is 0.0010 percent (also live at the air-water interface such as
expressed as 10 parts per million or 10 floating plants, etc.
ppm) by weight, which is 150 times lower
 Some organisms spend most of their lives
than the concentration of oxygen in an
on top of the air-water interface such as
equivalent volume of air.
water striders, while others spend most
 Oxygen enters the aquatic ecosystem of their time just beneath the air-water
through the air-water interface and interface and obtain most of their food
by the photosynthetic activities of within the water. E.g., beetles and
aquatic plants. back-swimmers.
TARGET PT 2020 41

16. Correct Option: (a) usually not harmful, and some species
can be harmful or dangerous at low cell
Explanation: concentrations that do not discolor the
 Photic zone water.
! Photic (or “euphotic”) zone is the  They are scientifically referred to as
portion that extends from the lake Harmful Algal Blooms (HABs).
surface down to where the light level
is 1% of that at the surface. The 19. Correct Option: (d)
depth of this zone depends on the
transparency of water. Explanation:

! Photosynthetic activity is confined Removal of the nutrients from a lake


to the photic zone.
 Deepwater abstractions
! Both photosynthesis and respiration
 Flushing with nutrient-poor waters
activity takes place.
 On-site algae removal by filters and
 Aphotic zone
P-adsorbers
! The lower layers of the aquatic
 On-site algae skimming and separator
ecosystems, where light penetration
thickening
and plant growth are restricted form
the aphotic zone (profundal zone).  Artificial mixing / Destratification
! Only respiration activity takes place (permanent or intermittent)
in this zone. The aphotic zone extends  Harvest of fishes and macrophytes
from the end of the photic zones to
bottom of the lake  Sludge removal

17. Correct Option: (a) 20. Correct Option: (c)

Explanation: Explanation:

Aquatic Ecosystem Classification of Lakes

 The foremost implication of the addition  Lakes are also classified on the basis of
of fertilizers to the aquatic ecosystems their water chemistry. Based on the levels
is eutrophication which in turn leads to of salinity, they are known as Freshwater,
increased growth of Phytoplanktons and Brackish or Saline lakes (similar to that of
other algae matter. classification of an aquatic ecosystem).

 When these dense algal blooms eventually  On the basis of their nutrient content,
die, microbial decomposition severely they are categorized as Oligotrophic
depletes dissolved oxygen, creating a hypoxic (very low nutrients), Mesotrophic
(moderate nutrients) and Eutrophic
or anoxic ‘dead zone’ lacking sufficient
(highly nutrient-rich).
oxygen to support most organisms.
 The vast majority of lakes in India are either
18. Correct Option: (c) eutrophic or mesotrophic because of the
nutrients derived from their surroundings
Explanation: or organic wastes entering them.
Red Tide
21. Correct option: (c)
 “Red Tide” is a common name for such a
Explanation
phenomenon where certain phytoplankton
species contain pigments and “bloom” such  Both the statements are correct
that the human eye perceives the water to
Supplementary notes
be discolored.
 Blooms can appear greenish, brown, and World Malaria Report 2019
even reddish-orange depending upon the  Despite a dip in cases, India still one of the
type of organism, the type of water, and the worst-hit countries
concentration of the organisms.
 While Africa and India saw the maximum
 The term “red tide” is thus a misnomer dip in malaria cases between 2017 and
because blooms are not always red, they 2018, they still accounted for 85 per cent
are not associated with tides, they are deaths.
42 TARGET PT 2020

 The World malaria report 2019 provides  The Central Water Commission (CWC)
a comprehensive update on global and has collected a total of 442 surface water
regional malaria data and trends. samples, of which 287 were found to be
 The report tracks investments in polluted by heavy metals.
malaria programmes and research as  The exercise was limited to surface
well as progress across all intervention water and did not cover groundwater
areas: prevention, diagnosis, treatment, contamination.
elimination and surveillance.
 The most common heavy metal found was
 It also includes dedicated chapters on the iron, and above safe limits in 156 samples.
consequences of malaria on maternal,
infant and child health, the “High Burden to  Lead, nickel, chromium, cadmium and
High Impact” approach as well as biological copper were the other metals.
threats to the fight against malaria.  The CWC study covered 67 rivers in 20
 The 2019 report is based on information river basins, and across three seasons.
received from more than 80 countries and  According to the report, Arsenic and zinc are
areas with ongoing malaria transmission.
the two toxic metals whose concentration
 Nineteen countries in sub-Saharan Africa was always obtained within the limits
and India accounted for 85 per cent of the throughout the study period.
global malaria burden in 2018. Globally
 For other metals, contamination levels
228 million malaria cases were reported in
changes with the season.
2018, which is marginally lower than the
number of cases in 2017 (231 million), as  For instance, iron contamination was
per the World Malaria Report 2019 released persistent through most of the Ganga
by the World Health Organization. during monsoon but dipped significantly
 India registered a fall of 2.6 million malarial during the non-monsoon periods.
cases in 2018 as compared to previous
year. The country also has one of the lowest 23. Correct Answer: (a)
funding per person at risk of being inflicted
Explanation:
with malaria at just US$0.2.
 2nd statement is incorrect: International
 Despite being the highest burden country
Financial Services Centres Authority
in the South-East Asia region, India
bill was passed in 2019 by Parliament
showed a reduction in reported cases of 51
and the Authority is yet to be set up.
per cent compared to 2017 and of 60 per
cent compared to 2016. Supplementary Notes
 Although cases continue to decrease in  International Financial Services Centres
the public sector, estimates indicate that Authority Bill, 2019 was introduced in Lok
there are still gaps in reporting from the Sabha by Union Finance Minister.
private sector and those seeking treatment
in India, as in Myanmar and Indonesia.  The Bill provides for the establishment of
The estimated burden of malaria, the an Authority to develop and regulate
WHO report says is 6.7 million while only the financial services market in the
4 million cases were reported in 2018. International Financial Services
Centres in India.
 India, Indonesia and Myanmar accounted
for 58 per cent, 21 per cent and 12 per cent  Key features of the Bill include:
of the total reported deaths in the region, ! It will apply to all International
respectively. Financial Services Centres (IFSCs)
set up under the Special Economic
22. Correct answer: (d) Zones Act, 2005.
Explanation ! It sets up the International
Financial Services Centres
Heavy Metals Contaminating India’s Authority (IFFSCA)
Rivers
! Composition:
 According to the Report, samples taken
from two-thirds of water quality stations
 IFFSCA will consist of nine
members, appointed by the
on major rivers revealed the presence of a
central government.
heavy metal (or in some cases more than
one) beyond limits specified by the Bureau  Members of the Authority will
of Indian Standards. include: (i) the Chairperson, (ii) one
TARGET PT 2020 43

member each to be nominated from Committee will consist of at least two


the Reserve Bank of India (RBI), members of the Authority. It will review
the Securities and Exchange Board whether: (i) the Authority has adhered to
of India (SEBI), the Insurance the provisions of the applicable laws while
Regulatory and Development exercising powers or performing functions,
Authority of India (IRDAI), and (ii) the regulations made by the Authority
the Pension Fund Regulatory and promote transparency and best practices
Development Authority (PFRDA), of governance, and (iii) the Authority
(iii) two members from among is managing risks to its functioning in
officials of the Ministry of Finance, a reasonable manner. The Committee
and (iv) two members to be must submit a report of its findings to the
appointed on the recommendation Authority at least once every year.
of a Search Committee.
 As per the Bill, all transactions of financial
 Members will have a term of three services in IFSCs will be in such foreign
years, subject to reappointment. currency as specified by the Authority, in
consultation with the central government.
 Functions of the Authority:
! To regulate financial products (such 24. Correct Answer: (a)
as securities, deposits or contracts
of insurance), financial services, and Explanation: Option (a) is correct.
financial institutions which have Supplementary Notes
been previously approved by any
appropriate regulator (such as RBI or  The union government has called upon
SEBI), in an IFSC. the Supreme Court to form a seven judge
Bench to reconsider the formulation in M.
! It will follow all processes which are Nagaraj vs Union of India (2006) that
applicable to such financial products, creamy layer should be applied to the SC
financial services, and financial and ST communities.
institutions under their respective
laws.  The Supreme Court in M. Nagaraj v.
Union Of India 2006 case while upholding
! The appropriate regulators are listed the constitutional validity of Art 16(4A)
in a Schedule to the Bill and include held that any such reservation policy in
the RBI, SEBI, IRDAI, and PFRDA. order to be constitutionally valid shall
The central government may amend satisfy the following three constitutional
this schedule through a notification. requirements:
! Regulating any other financial ! The SC and ST community should
products, financial services, or be socially and educationally
financial institutions in an IFSC, backward.
which may be notified by the central
government, and recommending any ! The SC and ST communities are not
other financial products, financial adequately represented in Public
services, or financial institutions to employment.
the central government, which may ! Such reservation policy shall not
be permitted in an IFSC. affect the overall efficiency in the
 The Bill sets up the International administration.
Financial Services Centres Authority
 In Jarnail Singh vs Lachhmi Narain
Fund. The following items will be
Gupta case of 2018, Supreme Court holds
credited to the Fund: (i) all grants, fees
that reservation in promotions does not
and charges received by the Authority,
require the state to collect quantifiable
and (ii) all sums received by the Authority
data on the backwardness of the Scheduled
from various sources, as decided by the
Castes and the Scheduled Tribes.
central government. The Fund will be
used for: (i) salaries, allowances and other  The Court held that creamy layer exclusion
remuneration of members and employees extends to SC/STs and, hence the State
of the Authority, and (ii) expenses incurred cannot grant reservations in promotion
by the Authority. Further, the central to SC/ST individuals who belong to the
government may provide grants to the creamy layer of their community.
Authority for the regulation of IFSCs.
 In May 2019 the Supreme Court upheld
 The IFFSCA will constitute a the Karnataka law that allows reservations
Performance Review Committee in promotions for SCs and STs with
to review its functioning. The Review consequential seniority.
44 TARGET PT 2020

 Centre’s Arguments against the the most backward among the backward
Extension of Creamy layer concept to sections.
SC and ST  Another problem is the question whether
 While the Centre has accepted that the the exclusion of the advanced sections
‘creamy layer’ norm is needed to ensure among SC/ST candidates can be disallowed
that only for promotions. Most of them may not
fall under the ‘creamy layer’ category at the
entry level, but after some years of service
Constitutional Provisions Governing and promotions, they may reach an income
Reservation in India level at which they fall under the ‘creamy
 Part XVI deals with reservation layer’. This may result in the defeat of the
of SC and ST in Central and State object of the Constitution amendments
legislatures. that the court itself had upheld to protect
reservation in promotions as well as
 Article 15(4) and 16(4) of the consequential seniority. Another landmark
Constitution enabled the State and verdict in the history of affirmative action
Central Governments to reserve seats in jurisprudence may be needed to settle
government services for the members of these questions.
the SC and ST.
 The Constitution was amended by 25. Correct Answer: (c)
the Constitution (77th Amendment)
Explanation:
Act, 1995 and a new clause (4A) was
inserted in Article 16 to enable the  3rd statement is incorrect. HDI was
government to provide reservation in created by Pakistani Economist
promotion. Mahbub ul Haq.
 Later, clause (4A) was modified by Supplementary Notes
the Constitution (85th Amendment)
Act, 2001 to provide consequential  Recently, Human Development Report
seniority to SC and ST candidates 2019 says that India is home to 28% of
promoted by giving reservation. world’s poor.

 Constitutional 81st Amendment Act,  The annual HDI 2019 report, ranked India
2000 inserted Article 16 (4 B) which at the 129th position, one rank above
enables the state to fill the unfilled last year’s ranking, out of a total 189
vacancies of a year which are reserved countries.
for SCs/STs in the succeeding year,  India remains the home to 28 percent of
thereby nullifying the ceiling of fifty global poor. About 41 per cent of the world’s
percent reservation on total number of poor live in South Asia.
vacancies of that year.
 Between 1990 and 2018, India’s HDI value
 Article 330 and 332 provides for specific increased by 50 per cent (from 0.431 to
representation through reservation of 0.647), which places it above the average
seats for SCs and STs in the Parliament for countries in the medium human
and in the State Legislative Assemblies development group (0.634) and above the
respectively. average for other South Asian countries
 Article 243D provides reservation (0.642).
of seats for SCs and STs in every  This means that in the last three decades,
Panchayat. life expectancy at birth in India increased
 Article 233T provides reservation by 11.6 years, whereas the average number
of seats for SCs and STs in every of schooling years increased by 3.5 years.
Municipality. Per capita incomes increased 250 times.
 Article 335 of the constitution says  The report finds that despite progress,
that the claims of STs and STs shall be group-based inequalities persist on the
taken into consideration constituently Indian subcontinent, especially affecting
with the maintenance of efficacy of the women and girls.
administration.  While Singapore has the region’s lowest
incidence of intimate partner violence
 only those genuinely backward get against women, the report states that a
reservation benefits, it is justifiably upset staggering 31 per cent of women in South
that this principle has been extended to Asia have experienced intimate partner
Dalits, who have been acknowledged to be violence.
TARGET PT 2020 45

 India is only marginally better than ! Standard of living measured by the


the South Asian average on the Gender gross national income (GNI) per
Development Index (0.829 vs 0.828), and capita.
ranks at a low 122 (of 162) countries on the
2018 Gender Inequality Index. ! Health measured by the life expectancy
 The report states that as the number of at birth.
people coming out of poverty is increasing, ! Education levels calculated by mean
the world is veering towards another type
years of education among the adult
of poverty. The old inequalities were based
on access to health services and education population and the expected years of
whereas the next generation of poverty is schooling for children.
based on technology, education and climate,
 This index makes it possible to follow
according to the report.
changes in development levels over time
 The report ranked countries after analysing and to compare the development levels of
reduction in absolute poverty, gains in life
different countries.
expectancy, education, and access to health
care.  Additional indices have been developed
 India has both types of poverty. Even as to capture other dimensions of human
Indians continue to face a lack of access to development to identify groups falling
healthcare and education, many others are behind in human progress and to monitor
becoming poor based on the new criteria. the distribution of human development.
What is HDI?  In 2010 three indices were launched to
 The underlying principle of the HDI, monitor poverty, inequality and gender
considered path breaking in 1990, (created empowerment across multiple human
by Pakistani economist Mahbub ul Haq) is development dimensions
elegantly simple: National development
should be measured not only by income per ! The Multidimensional Poverty Index
capita, but also by health and education (MPI),
achievements.
! The Inequality-adjusted Human
 The HDI is the composite measure of Development Index (IHDI)
every country’s attainment in three basic
dimensions: ! The Gender Inequality Index (GII).

**********
46 TARGET PT 2020

TEST
DAY - 34

Time Allowed: 30 mins Maximum Marks: 50

1. Consider the following statements with 4. Consider the following statements


reference to the level of biodiversity: regarding the measurement of
1. Ecological diversity refers to the variety biodiversity:
of organisms within an ecosystem. 1. Species richness measures the
2. Species diversity ensures that the proportion of species at a given site.
species can survive drastic changes in
the environmental conditions. 2. Species evenness measures the number
of species found in a community
Which of the above statements is/are
correct? Which of the above statements is/are correct?
(a) 1 only (a) 1 only
(b) 2 only (b) 2 only
(c) Both 1 and 2 (c) Both 1 and 2
(d) Neither 1 nor 2
(d) Neither 1 nor 2
2. Which of the following action is not a
threat to the biodiversity of a region? 5. Consider the following list:

(a) Introduction of a species in a new 1. Predictable environment


ecosystem. 2. The complex relationship amongst
(b) Practicing Jhum cultivation. species
(c) Reintroduction of a lost species in its 3. Niche similarity amongst species
original ecosystem. 4. More insolation
(d) A large but unmanaged ecosystem is
fragmented into smaller ones. Which of the above is/are the reason(s) to
enhance the biodiversity of the Earth?
3. Which of the following are the
methods of ex-situ conservation of (a) 2 and 3 only
biodiversity? (b) 1, 2 and 4 only
1. Seed Banks
(c) 2 only
2. Cryopreservation
(d) 1 and 4 only
3. Sacred groves
4. Tissue culture propagation 6. Meghalaya is popular for the living
root bridge. The roots of which of the
Select the correct option using the codes
given below: following trees is used for this?

(a) 3 and 4 only (a) Banayan trees

(b) 1 and 3 only (b) Rubber trees


(c) 1, 2, and 4 only (c) Bamboo grass
(d) 1, 2, 3 and 4 (d) Mahogany trees
TARGET PT 2020 47

7. Consider the following statements 4. Madar


regarding the mangrove forests: 5. Red sanders
1. They live in the intertidal zone.
Select the correct option using the codes
2. They cannot withstand freezing
given below:
temperatures.
(a) 1, 3, and 4 only
3. Approx. one-fourth of the total
mangroves in India are in Gujarat. (b) 2, 3, and 4 only

Which of the above statements is/are (c) 1 and 5 only


correct? (d) 1,2,3, and 4 only
(a) 1 and 2 only
12. Consider the following statements
(b) 2 and 3 only regarding a plant species of India:
(c) 1 only 1. It is the largest genus of carnivorous
plants.
(d) 1, 2, and 3
2. It is of high medicinal values.
8. Which of the following is the species of 3. It inhabits freshwater wetlands and
the Asian elephant? waterlogged areas.
(a) Elephas maximus
Which of the following species has been
(b) Loxodonta cyclotis described above?
(c) Loxodonta africana (a) Utricularia
(d) All of the above (b) Sarpagandha
9. A new fossil record has shown that (c) Ladies Slipper Orchid
India is the birthplace of Asian bamboo. (d) Aldrovanda
In which of the following states, the
fossils have been found? 13. Which of the following correctly
(a) Mizoram represents Genetic Diversity?
(b) Manipur (a) It is concerned with the variation in
genes within a particular species.
(c) Arunachal Pradesh
(b) It is the variety of living organisms on
(d) Assam earth.
10. Which of the following is correct (c) This refers to the different types
regarding ‘Red Vanda’? of habitats occupied by various
1. It is included in the Schedule-VI of the organisms.
Wildlife Protection Act, 1972 (d) It is the variation of the ecological
2. It is found in Sikkim. relations that a living organism has
with non-living objects such as trees.
Select the correct option using the codes
given below: 14. What do you understand by Community/
Ecosystem diversity?
(a) 1 only
(a) It is a variety of different types of
(b) 2 only habitats in an area.
(c) Both 1 and 2 (b) It is the diversity of living creatures
(d) Neither 1 nor 2 found in a food chain.
(c) It defines the different natural services
11. Which of the following plants are alien
provided by plants for human beings.
invasive species in India?
1. Blue Vanda (d) It is the diversity of a scientifically
planned collection of living trees,
2. Touch-Me-Not shrubs etc. from various parts of the
3. Datura world.
48 TARGET PT 2020

15. Which of the following mentioned is not 20. Identify the Artificial cause of extinction
a Biogeographic province in India? of species from the following:
(a) Assam Plains (a) Climate change
(b) Tibetan Plateau (b) Tectonic activity
(c) Central highlands (c) Introduction of invasive species
(d) Lower Gangetic Plains (d) Increased volcanic activity

16. Identify the Non-flowering plant group 21. With reference to Citizenship
from the following: Amendment Bill, 2019, consider the
following statements
1. Bryophytes
1. Bill seeks to grant citizenship to
2. Gymnosperms
non-Muslim refugees from Pakistan,
3. Angiosperms Bangladesh and Afghanistan only.
Select the correct option from the codes 2. The Act amends the Citizenship Act,
given below: 1955.

(a) 1 only 3. The Act also amends the provisions


on registration of Overseas Citizens of
(b) 1 and 2 only India (OCI).
(c) 2 and 3 only Which of the following statement is/are
(d) 1, 2 and 3 correct?
(a) 1 and 2only
17. Which among the following regions
of India has the highest floral (b) 2 and 3 only
endemism? (c) 1 and 3 only
(a) Eastern Himalaya and north-eastern
(d) 1, 2 and 3
region
(b) North-western Himalaya 22. Consider the following statements
about Anglo-Indians
(c) Andaman & Nicobar Islands
1. Article 334 of the Constitution Provides
(d) Western and Eastern Ghats for nomination of two Anglo-Indians to
Lok Sabha before amendment.
18. Identify the incorrect statement with
respect to Insectivorous Plants: 2. Parliament has passed the Constitution
(126th Amendment) Bill that extends
(a) These are plants that are specialized reservation for SC/STs but has done
in trapping insects. away with the provision for nomination
(b) Brilliant colors, sweet secretions are of Anglo Indians to Lok Sabha and
few attractions used by these plants to some state Assemblies.
lure their victims. Which of the above statement(s) is/are
(c) Some of them even prey upon large correct?
animals as well. (a) 1 only
(d) Leaf traps and Pitfall mechanisms are (b) 2 only
associated with Insectivorous plants.
(c) Both 1 and 2
19. Pitcher Plant is a member of which (d) None
plant family?
(a) Nepenthes 23. What are the major problems that have
crippled the growth in real estates
(b) Asteraceae industry?
(c) Rosaceae 1. Approvals and Procedural difficulties
(d) None of the above 2. Lack of clear land titles
TARGET PT 2020 49

3. Poor lending by Banks Choose the correct answer:


4. Real estates industry is not organized (a) Only 1
Choose the correct option (b) 1 and 2 only
(a) 1, 2 and 3 (c) 2 and 3 only
(b) 1 and 2 (d) 1, 2 and 3
(c) 1 and 3 25. Many cyclonic instances were in news
(d) 1, 2, 3 and 4 recently. Match each of the following
cyclones with their correct place of
24. Recently, IUCN released a report origin.
on Ocean deoxygenation. Which of 1. Cyclone Vayu – Indian Ocean
the following statements is correct
2. Cyclone Pawan – Gulf of Aden
regarding the process of ocean
deoxygenation? 3. Cyclone Idai – Pacific Ocean
1. Warm ocean waters hold less oxygen Choose the correct answer:
and become more buoyant.
(a) Only 1
2. Nutrient pollution causes excessive
growth of algae. (b) 1 and 2 only
3. Deoxygenation favours low-oxygen (c) Only 3
sensitive marine species.
(d) 1, 2 and 3

**********
50 TARGET PT 2020

ANSWER HINTS
DAY - 34

1. Correct Option: (d) of resources, Alien species invasions,


Nutrient loading, Climate change, and
Explanation: environmental pollution.
Levels of biodiversity  Reintroduction of a lost species in its
 There are three levels of biodiversity in original ecosystem would not cause
a region viz. Genetic Diversity, Species any threat.
Diversity, and Ecological Diversity.
3. Correct Option: (c)
 Genetic Diversity is the diversity
at the genetic level, for instance, Explanation:
different strains of the same gene.
Genetic diversity allows species to Modes of Conservation:
adapt to changing environments.  Ex-situ conservation: Conserving
This diversity aims to ensure that biodiversity outside the areas where
some species survive drastic changes they naturally occur is known as ex-situ
and thus carry on desirable genes. conservation. Example- Seed banks,
The genetic diversity gives us beautiful zoological botanical, horticultural and
butterflies, roses, parakeets or coral in recreational gardens, reintroduction
myriad hues, shapes, and sizes. to a habitat, Cryopreservation, Tissue
 Species diversity of an ecosystem refers to culture propagation, and In-vitro
the number of types of species within an fertilization, etc.
ecosystem. It refers to the variety of living  In-situ conservation: Conserving the
organisms on earth. It is the ratio of one animals and plants in their natural
species population over a total number of habitats is known as in-situ conservation.
organisms across all species in the given Example- National parks, Sanctuaries,
biome. Biosphere reserves, Reserved forests,
 Ecological diversity refers to the Protected forests, sacred groves, etc.
variety of organisms between two or
more ecosystems. For instance, western 4. Correct Option: (d)
Ghats are more ecologically diverse than
Explanation:
the Eastern Ghats. As the environment
changes, species best adapted to that Measurement of biodiversity
environment becomes predominant. Thus
the variety or diversity of species in the  Diversity mainly includes two different
ecosystem is influenced by the nature of aspects: species richness and evenness.
the ecosystem.  Species richness is the number of
species. It is the simplest measure of
2. Correct Option: (c) diversity and does not consider differences
Explanation: in species relative abundance.
 Species evenness is the similarity
Threats to the biodiversity
in species relative abundance in a
 According to the Convention on Biological community captures another aspect of
Diversity, there are six major threats to the diversity by determining diversity as a
biodiversity of an ecosystem viz. Habitat standardized index of relative species
loss and Fragmentation, Over-exploitation abundance.
TARGET PT 2020 51

 The relationship between species richness


and evenness can vary with change in key
ecological processes such as competition,
predation, and succession, each of which
can alter proportional diversity through
changes in evenness without any change in
species composition

5. Correct Option: (b)


Explanation:

Biodiversity
 Some important reasons that influence the  It can take 15 to 20 years for the surreal,
pattern of Biodiversity are as follows: strong web of tangled root bridges to
connect the two banks. Unlike traditional
! Speciation which is needed for species constructions, Meghalaya’s root bridges
diversification only grow stronger with time, never
requiring major maintenance or rebuilding;
! A relatively more constant and the strongest root bridges are more than
predictable environment that 100 years old.
promotes niche specialization and
lead to greater species diversity. 7. Correct Option: (d)
! More insolation contributes Explanation:
to higher productivity, in turn,
Mangrove forests
contribute indirectly to greater
diversity.  Mangroves are a group of trees and shrubs
that live in the coastal intertidal zone.
! The more complex ecosystem provides
 Mangrove forests only grow at tropical
more food web i.e. many entry points
and subtropical latitudes near the equator
for any organism. This, in turn, because they cannot withstand freezing
sustains the greater biodiversity. temperatures.
! Nich similarity is not good,  Many mangrove forests can be recognized
promotes high competition and by their dense tangle of prop roots that
leads to loss of biodiversity. make the trees appear to be standing on
stilts above the water. This tangle of roots
allows the trees to handle the daily rise
6. Correct Option: (b)
and fall of tides, which means that most
Explanation: mangroves get flooded at least twice per
day. The roots also slow the movement of
Living root bridges tidal waters, causing sediments to settle
out of the water and build up the muddy
 Living root bridges are a form of tree bottom.
shaping common in the southern part of
 Mangrove forests stabilize the coastline,
the Northeast Indian state of Meghalaya. reducing erosion from storm surges,
 The southern Khasi and Jaintia hills are currents, waves, and tides. The intricate
root system of mangroves also makes
humid and warm, crisscrossed by swift-
these forests attractive to fish and other
flowing rivers and mountain streams. organisms seeking food and shelter from
 On the slopes of these hills, a species of predators.
Indian rubber tree(Ficus elastica) with  The total mangrove cover in the country is
an incredibly strong root system thrives 4,975 sq km.
and flourishes.  An increase of 54 sq Km in mangrove
 The bridges are handmade from the aerial cover has been observed as compared to
the previous assessment of 2017. The
roots of Rubber Trees by the Khasi and
top three states showing mangrove cover
Jaintia peoples of the mountainous terrain increase are Gujarat (37 sq km) followed
along the southern part of the Shillong by Maharashtra (16 sq km) and Odisha
Plateau. (8 sq km).
52 TARGET PT 2020

 The Asian elephant is classified as


Endangered by the International Union for
the Conservation of Nature (IUCN).

 According to the FSI report, West Bengal 9. Correct Option: (d)


has 42.45% of the country’s mangrove
Explanation:
cover, while Gujarat has 23.66%.
Asia’s oldest bamboo
 With over 49,000 plant species reported as
of 2018, India holds about 11.5% of all flora
in the world. Now, a new fossil record has
shown that India is the birthplace of Asian
bamboo, and they were formed about 25
million years ago in the north-eastern part
of the country.

8. Correct Option: (a)


Explanation:
 They were named Bambusiculmus
Asian elephant tirapensis and B. makumensis - as they
 The elephant is Earth’s largest land were found in the Tirap mine of Makum
animal. Coalfield in Assam. These belonged to the
 Three species of elephants are late Oligocene period of about 25 million
recognized; the African bush elephant years ago.
(Loxodonta africana) and forest
elephant (Loxodonta cyclotis) of 10. Correct Option: (a)
sub-Saharan Africa, and the Asian
elephant (Elephas maximus) of South Explanation:
and Southeast Asia.
Red Vanda
 Asian elephants can be identified by
their smaller, rounded ears. (An African  Red vanda is a type of Indian orchid
elephant’s ears resemble the continent species.
of Africa.). They live in forested regions
 It is included in the Schedule-VI of the
of India and throughout Southeast Asia,
Wildlife Protection Act, 1972, whose
including Myanmar, Thailand, Cambodia,
and Laos. cultivation, Collection, extraction, trade,
etc are prohibited.
 The Indian elephant(Elephas maximus
indicus) is a subspecies of the Asian  It is found in Manipur, Assam, Andhra
elephant. Pradesh.
TARGET PT 2020 53

11. Correct Option: (b)

Explanation:

Alien Invasive species in India

 An alien plant also referred to as exotic,


Touch-me-not
introduced, foreign, non-indigenous or non-
native, is one that has been introduced by
humans intentionally or otherwise through
human agency or accidentally from one
region to another. An alien plant that has
escaped from its original ecosystem and is
reproducing on its own in the regional flora
is considered a naturalized species. Those
naturalized aliens that become so successful
as to spread in the flora and displace native
biota or threatens valued environmental,
agricultural or personal resources by the
damage it causes are considered invasive.

 These species threaten native plants and


animals or other aspects of biodiversity.
Datura
 They occur in all groups of plants and
animals, as competitors, predators,
pathogens and parasites, and they have
invaded almost every type of native
ecosystem.

 The effects on biodiversity are enormous


and often irreversible.

 Some of the plant species are Goat


weed(from America), Touch-Me-
Not(from Brazil), Datura(from
America), Madar(from Africa), Water
Hyacinth(from America), Prosopis Madar
juliflora (from Mexico) 4 ‘0’ clock  Red Sanders (or Red Sandalwood) is
plant(Peru), etc. an Indian species, found in South India.
54 TARGET PT 2020

Red Sanders

 Blue vanda or autumn lady’s tresses,


is a native species of orchid found in
Assam and neighboring Khasi hills
with its range extending to China.

13. Correct Option: (a)


Explanation:

Genetic Diversity
 Genetic diversity is concerned with the
variation in genes within a particular
species.
 Genetic diversity allows species to adapt to
changing environments.
 It aims to ensure that some species survive
drastic changes and thus carry on desirable
genes.
Blue vanda
 The survival of individuals ensures the
12. Correct Option: (a) survival of the population.
 The genetic diversity gives us beautiful
Explanation: butterflies, roses, parakeets or coral in
myriad hues, shapes, and sizes.
Utricularia/ Bladderworts

 Bladderworts generally inhabit freshwater 14. Correct Option: (a)


wetlands and waterlogged areas. Some Explanation:
species are associated with moist moss-
Ecosystem diversity
covered rock surfaces and damp soils
during rains.  Community/Ecosystem diversity is the
variety of different types of habitats
 Utricularia in its bladders mouth has in an area.
sensitive bristles or hairs. When an insect  A habitat is the cumulative factor of the
happens to contact these hairs the door climate, vegetation, and geography of a
opens, carrying the insect into the bladder region.
along with a little current of water. The  There are several kinds of habitats around
door is shut when water fills the bladder, the world. Corals, grasslands, wetland,
The enzymes produced by the inner wall of desert, mangrove, and tropical rain forests
the bladder digest the insect. are examples of ecosystems.
 Change in climatic conditions is
 Utricularia is useful against cough, for
accompanied by a change in vegetation
dressing of wounds, as a remedy for urinary as well. Each species adapts itself to a
disease. particular kind of environment.
TARGET PT 2020 55

 As the environment changes, species best 16. Correct Option: (a)


adapted to that environment becomes
predominant. Thus the variety or diversity Explanation:
of species in the ecosystem is influenced by
Bryophytes
the nature of the ecosystem.
 These are one of the important floral groups
15. Correct Option: (a) (non-flowering) found in India.
Explanation:  They are the second-largest group of green
plants in India distributed largely in
Biogeographic Provinces of India
Eastern Himalaya, North-eastern India,
S.No. Biogeographic Biogeographic Western Himalaya, and the Western
Zones Provinces Ghats.

1 Trans Himalaya Ladakh mountain  Their plant body is differentiated into a


small stem and simple leaves, but true
Tibetan Plateau roots are absent.
Sikkim
 They usually grow in moist places.
2 The Himalaya North West Prominent examples include Liverworts,
Himalaya mosses.

West Himalaya  Mosses constitute the major component of


Indian bryoflora followed by liverworts and
Central-Himalaya
hornworts.
East-Himalaya
17. Correct Option: (d)
3 Indian Desert Thar
Katchchh
Explanation:

Floral Endemism in India


4 Semi Arid Punjab Plains
Gujarat Rajputana  Floral Endemism is the phenomenon of
flower species being unique to a defined
5 Western Ghats Malabar Plains geographical area. Its measure defines the
diversity of species at any location.
Western Ghat
mountains  In India, the sequence of floral endemism
is (in decreasing order):
6 Deccan Central Highlands
Peninsula ! Peninsular India including
Chotta Nagpur
western and Eastern Ghats (about
Eastern Highlands 2,600 species).
Central Plateau ! Eastern Himalaya and north-eastern
Deccan South region (about 2,500 species).
! North-western Himalaya (about 800
7 Gangetic Plain Upper Gangetic
species).
Plains
Lower Gangetic ! Andaman & Nicobar Islands (about
250 species).
Plains

8 Coast West Coast 18. Correct Answer: (c)


East Coast Explanation:
Lakshadweep
Insectivorous Plants
9 Northeast India Brahmaputra  These are plants that specialize in
Valley trapping insects.
North East Hills  They differ from normal plants in their
mode of nutrition.
10 Island Andaman
Nicobar  However, these plants never prey
upon humans or large animals.
56 TARGET PT 2020

 Insectivorous plants can broadly be divided ! Direct causes - such as hunting,


into two categories based on their method collection or capture, and persecution.
of trapping their prey: ! Indirect causes - such as habitat
! Active Plants: These can close their loss, modification, and fragmentation
leaf traps the moment insects land and the introduction of invasive
on them. species.

! Passive Plants: These have a ‘pitfall’ Natural Extinction


mechanism, having some kind of jar  It has been caused due to several factors:
or pitcher-like structure into which
the insect slips and falls, to eventually  Continent drifting
be digested.  Climate change
! The insectivorous plants often  Tectonic activity
have several attractions such as
 Increased volcanic activity
brilliant colors, sweet secretions
and other curios to lure their victims.  The late Ordovician global glaciations (439
Million years ago).
19. Correct Answer: (a)
 The late Cretaceous extinction assumed
Explanation: to be associated with an extra-terrestrial
impact.
Nepenthes
 The members of the Nepenthes family 21. Correct answer: (d)
are commonly known as ‘pitcher Explanation
plants’ because their leaves bear jar-like
structures. All the above statements are correct
 Its distribution is confined to the high Supplementary notes
rainfall hills and plateaus of the north-
eastern region, at altitudes ranging from  The Union Cabinet has cleared the
100 - 1500 m, particularly in Garo, Khasi Citizenship (Amendment) Bill that
and Jaintia hills of Meghalaya. seeks to grant citizenship to non-Muslim
refugees from Pakistan, Bangladesh
 Nepenthes conform to the pitfall type of
and Afghanistan if they faced religious
trap. A honey-like substance is secreted
persecution there.
from glands at the entrance of the pitcher.
Once the insect enters into the pitcher, it  The Act amends the Citizenship Act, 1955,
falls down because of the slipperiness. in order to grant Indian nationality to
Hindus, Sikhs, Buddhists, Jains, Parsis
 The inner wall, towards its lower half,
and Christians who come to India after
bears numerous glands, which secrete a
proteolytic enzyme. This enzyme digests the facing religious persecution in Bangladesh,
body of the trapped insects and nutrients Pakistan and Afghanistan.
are absorbed.  The Act doesn’t spell it out clearly, but
 Nepenthes in local medicine to treat cholera the fact that it entitles Hindus, Sikhs,
patients, the liquid inside the pitcher is Buddhists, Jains, Parsis, and Christians
useful for urinary troubles. It is also used facing religious persecution in the three
as eye drops. nations, to seek Indian citizenship,
highlights the exclusion of Muslims.
20. Correct Option: (c)  This amendment is of the Citizenship Act,
Explanation: 1955 which requires the applicant to have
resided in India for 11 of the previous
Artificial Extinction 14 years. The amendment relaxes this
requirement from 11 years to six years, for
 Even though species extinction is a natural Hindus, Sikhs, Buddhists, Jains, Parsis,
process that can happen without the
and Christians from the three nations.
intervention of humans, extinctions caused
by humans is now happening over and  The Act also amends the provisions on
above the reasonable estimate of natural registration of Overseas Citizens of India
extinction rates. (OCI).
 Species are threatened with extinction by  OCI cardholders are foreigners who are
the intervention of humans due to: persons of Indian origin.
TARGET PT 2020 57

22. Correct Answer: (b) do so, they are bound by their party whip.
The Anglo-Indian members enjoy the same
Explanation: powers as others, but they cannot vote in
 1st statement is incorrect: Article 331 of the Presidential election because they are
the Constitution Provides for nomination nominated by the President.
of two Anglo-Indians to Lok Sabha. It
says: “Notwithstanding anything in Article Who are Anglo-Indians?
81, the President may, if he is of opinion  The Anglo-Indian community in India
that the Anglo-Indian community is not traces its origins to an official policy of the
adequately represented in the House of British East India Company to encourage
the people, nominate not more than two marriages of its officers with local women.
members of that community to the House
of the People.” The 126th Amendment does  The term Anglo-Indian first appeared in
away with this. the Government of India Act, 1935.
 In the present context, Article 366(2) of the
Supplementary Notes Constitution Of India states: “An Anglo-
 Recently, Parliament passed the Indian means a person whose father or any
Constitution (126th Amendment) Bill, of whose other male progenitors in the male
extending reservation for SC/STs but doing line is or was of European descent but who
away with the provision for nomination of is domiciled within the territory of India
Anglo Indians to Lok Sabha and some state and is or was born within such territory of
Assemblies. parents habitually resident therein and not
established there for temporary purposes
 Article 331 of the Constitution Provides only…”
for nomination of two Anglo-Indians to Lok
Sabha. It says: “Notwithstanding anything  According to 2011 Census there are only
in Article 81, the President may, if he is of 296 people who identified themselves
opinion that the Anglo-Indian community as belonging to the sect Anglo Indian.
is not adequately represented in the House However, All India Anglo Indian
of the people, nominate not more than two Association contested the data and asserts
members of that community to the House that there are many more Anglo-Indians in
of the People.” The 126th Amendment the country.
does away with this.
23. Correct Answer: (a)
 The idea of such nominations is traced to
Frank Anthony, who headed the All India Explanation:
Anglo-Indian Association. Article 331 was
 4th statement is incorrect. Real estates
added in the Constitution following his
industry is organized in India.
suggestion to Jawaharlal Nehru.
 Article 333 deals with representation of Supplementary Notes
the Anglo-Indian community in Legislative
Assemblies. It says: “Notwithstanding Issues faced by Real Estate Industry
anything in Article 170, the Governor of a  Approvals and Procedural difficulties: There
State may, if he is of opinion that the Anglo- are almost 50 approvals or more need to be
Indian community needs representation taken for starting a real estate project and
in the Legislative Assembly of the State further these approvals are required form
and is not adequately represented therein, different govt. departments or authorities.
[nominate one member of that community This is one of the major causes of delays and
to the Assembly].” high amount of corruption in real estate
 Currently 14 Assemblies have one Anglo- sector. Consequently corruption and delay
Indian member each: Andhra Pradesh, cause inconvenience to customers only.
Bihar, Chhattisgarh, Gujarat, Jharkhand,  The Real Estate (Regulation and
Karnataka, Kerala, Madhya Pradesh, Development) Act, 2016
Maharashtra, Tamil Nadu, Telangana,
 It seeks to protect home-buyers as well as
Uttar Pradesh, Uttarakhand and West
help boost investments in the real estate
Bengal. The 126th Amendment does away
industry.
with this as well.
 The Act establishes Real Estate Regulatory
 According to the 10th Schedule of the
Authority (RERA) in each state for
Constitution, Anglo-Indian members of
regulation of the real estate sector and also
Lok Sabha and state Assemblies can take
acts as an adjudicating body for speedy
the membership of any party within six
dispute redressal.
months of their nomination. But, once they
58 TARGET PT 2020

 Lack of clear land titles: The land titles cities like Mumbai or Delhi despite being
are not clear because of poor record so much demand in real estate sector.
keeping and division of land in many
 Thus to handle the above issues government
parts till independence. The slow pace of
of India has launched Real Estate
modernization of land records is further
(Regulation and Development) Bill. It seeks
aggravating the problem. to regulate contracts between buyers and
 Speculation in Land and Real Estate Prices: sellers in the real estate sector to ensure
The prices of land and real estate in India consumer protection, and standardisation
have increased exponentially in last decade of business practices. It establishes
and causes overpricing of commercial or regulatory authorities at the state level to
residential property. In recent times, the register residential real estate projects.
real estate is the most favorable destination
for investment in India and far ahead than 24. Correct Option (b)
equity or gold. Further real estate agents
Explanation:
or brokers buy or sell property frequently
with their own investments and cause of  Statements 1 and 2 are correct
surging prices in property.  Statement 3 is incorrect: Deoxygenation
favours low-oxygen tolerant marine
The Real Estate (Regulation and
species.
Development) Act, 2016
 It seeks to protect home-buyers as well Supplementary Notes
as help boost investments in the real  The primary causes of deoxygenation are:
estate industry.
 Eutrophication (increased nutrient run-off
 The Act establishes Real Estate from land and sewage pollution).
Regulatory Authority (RERA) in each
state for regulation of the real estate  Nitrogen deposition from burning of fossil
sector and also acts as an adjudicating fuels.
body for speedy dispute redressal.  Widespread impacts from ocean warming.
 Ocean oxygen loss is closely related to
 Sources of Finance: Finance is the key for
ocean warming and acidification caused
development of any industry. Due to poor
by anthropogenic carbon dioxide emissions
image of Real Estate sector, banks are
and biogeochemical consequences related
becoming reluctant to provide loans and to anthropogenic fertilization of the ocean.
making regulation tougher to avoid the bad
loans. Alternate sources of finance are very  As the ocean warms, its water hold less
costly and ultimately impact total cost of oxygen and become more buoyant, resulting
the project. in reduced mixing of oxygen-rich water
near the surface with the ocean depths,
 High Input Cost: The real estate is a capital which naturally contain less oxygen.
and labour intensive industry; thus rise in
cost of labour and construction material  Nutrient pollution causes oxygen loss in
due to inflation poses many problems to coastal waters. Fertiliser, sewage, animal
real estate industry. Further real estate and aquaculture waste cause excessive
builders many times raise a question about growth of algae, which in turn deplete
unfair practices in cement industry for rise oxygen as they decompose.
of price more than 50% in quick time.  The loss of oxygen from world’s ocean is
 Further government intervention of increasingly threatening fish species and
building minimum 20% affordable housing disrupting ecosystems.
putting extra burden on developers and  Deoxygenation is starting to alter the
ultimately on the rest 80%. balance of marine life , favouring low-
 Real estate is the most famous sector for oxygen tolerant species (e.g. microbes,
soaking the black money without any jellyfish and some squid) at the expense of
ambit. low-oxygen sensitive ones (many marine
species, including most fish).
 High taxation like stamp duty, VAT etc. and
 Some of the ocean’s most productive biomes
land acquisition are other major challenges
– which support one fifth of the world’s
faced by real estate sector.
wild marine fish harvest – are formed by
 All the above issues can be verified by the ocean currents carrying nutrient-rich but
facts that more than 30% of houses and oxygen-poor water to coasts that line the
commercial spaces are lying vacant in major eastern edges of the world’s ocean basins.
TARGET PT 2020 59

 As naturally oxygen-poor systems, these Supplementary Notes


areas are particularly vulnerable to even
 Pawan – Arabian Sea
small changes in ocean oxygen.
 Idai – South-West Indian Ocean
 Impacts here will ultimately ripple out and
affect hundreds of millions of people.  Vayu – North Indian Ocean; Arabian Sea
 Species groups such as tuna, marlin and  Fani – Bay of Bengal
sharks are particularly sensitive to low  Phailin – Western Pacific Ocean
oxygen because of their large size and
energy demands.  Kyarr – North India Ocean; Arabian Sea

 These species are starting to be driven  Pabuk – Northwestern Pacific Ocean and
into increasingly shallow surface layers Indian Ocean basin
of oxygen-rich water, making them more  Hikka – Arabian Sea
vulnerable to overfishing.
 Mekunu – North Indian Ocean; affecting
 Very low ocean oxygen can also affect Oman, Socotra, Yemen
basic processes like the cycling of elements
crucial for life on Earth, such as nitrogen  Sagar – Gulf of Aden; Arabian Sea
and phosphorous.  Daye – Bay of Bengal

25. Correct Option (a)  Luban – North India Ocean; affecting


Arabian Peninsula
Explanation: Statement 1 is correct
 Titli – Arabian Sea
 Statements 2 and 3 are incorrect: Idai –
 Gaja – Gulf of Thailand
South-West Indian Ocean; Cyclone Pawan
– Arabian Sea, North Indian Oean.  Phethai – Bay of Bengal

**********
60 TARGET PT 2020

TEST
DAY - 35

Time Allowed: 30 mins Maximum Marks: 50

1. Arrange the following states in 3. It is popular for its mating dance.


increasing order of tiger population:
Select the correct option using the codes
A. Madhya Pradesh given below:
B. Karnataka (a) 1 only

C. Uttarakhand (b) 3 only


(c) 1 and 3 only
D. Maharashtra
(d) 1, 2, and 3
Select the correct option using the codes
given below: 4. Consider the following statements
(a) D-C-B-A regarding a species:

(b) B-C-A-D 1. It is a species of riverine turtle native


to Southeast Asia.
(c) D-C-A-B
2. It is classified Critically Endangered
(d) D-B-C-A by the IUCN.

2. In which of the following tiger reserves, 3. In India, it is found in West Bengal &
the number of tigers is zero? Odisha.
1. Dampa 4. It nests on the ground.
2. Buxa Which of the following species has been
3. Palamau described above?
4. Valmiki (a) Olive Ridley
Select the correct option using the codes (b) Hawksbill
given below:
(c) Terrapin
(a) 1 and 4 only
(d) Leatherback
(b) 3 and 4 only
5. Consider the following statements
(c) 1, 2 and 3 only
regarding the Greater one-horned
(d) 1, 2, 3 and 4 rhino (Rhinoceros unicornis):

3. Which of the following statements 1. It has been categorized as ‘Endangered’


regarding ‘Bengal Florican’ is/are according to the IUCN- Red Data
correct? Book.
1. It is one of the only four bustards found 2. It is the least populated rhino in the
in India. world.
2. About 50% of this species are in West 3. Kaziranga National Park in Assam has
Bengal. the highest density of this species.
TARGET PT 2020 61

Which of the above statements is/are 9. Which of the following statements


incorrect? regarding Pygmy hog is/are correct?
(a) 1 only 1. It is the world’s smallest wild pig.
(b) 2 only 2. It is endemic to Manas National Park.
3. It has been categorized as ‘Endangered’
(c) 3 only
according to the IUCN- Red Data
(d) 1, 2, and 3 Book.

6. Which of the following categories does Select the correct option using the codes
not fall in the ‘threatened categories’ given below:
as per the International Union for
(a) 1 only
Conservation of Nature?
(b) 1 and 3 only
(a) Least Concern
(c) 2 and 3 only
(b) Data deficient
(d) 1, 2, and 3
(c) Extinct in wild
(d) All of the above 10. Consider the following statements
regarding a species:
7. Consider the following statements 1. It is endemic to the largest biosphere
regarding a species: reserve in India.
1. It is a medium-sized, nocturnal and 2. It is listed as endangered in IUCN red
burrowing rodent endemic to India. list.
2. It is found only in the Eastern Ghats of 3. Its other names are Wanderloo and
Tamil Nadu. Bruh.
3. It has been categorized as ‘Critically
Endangered’ according to the IUCN- Which of the following species has been
Red Data Book. described?
(a) Nilgiri Tahr
Which of the following species has been
described? (b) Lion-tailed Macaque
(a) Elvira Rat (c) Malabar giant squirrel
(b) Malabar Civet (d) Gray langur
(c) Namdapha Flying Squirrel 11. Consider the following statements
(d) Andaman shrew regarding a species:
1. It is a nocturnal burrowing rodent that
8. Arrange the following states in is found only in India.
increasing order of elephant
2. It is categorized as “Critically
population:
Endangered” as per IUCN.
A. Kerala 3. It is endemic to a small plateau near
Pune, Maharashtra.
B. Karnataka
Which of the following species has been
C. Assam
described?
D. Tamil Nadu (a) Elvira Rat
Select the correct option using codes given (b) Kondana Rat
below: (c) Namdapha Flying Squirrel
(a) A-B-C-D (d) Malabar Civet
(b) D-C-A-B
12. Consider the following pairs regarding
(c) D-A-C-B hotspots and their corresponding
(d) B-D-A-C regions:
62 TARGET PT 2020

1. Sundaland: Andaman group of islands B. Local Extinction 2. Continuous


2. Western Ghats: Sri Lanka process of
natural
3. Indo-Burma: Nicobar group of Islands and low-level
4. Himalayas: Assam extinction

Which of the above pairs are correctly C. Ecological 3. Species no longer


matched? Extinction found anywhere
(a) 2 and 3 only on earth

(b) 1 and 4 only D. Background 4. More than 65% of


Extinction all species
(c) 1, 2, and 3 only
become extinct
(d) 1,2, 3, and 4
E. Mass Extinction 5. Species no longer
13. Which among the following statements found in area it
is not a Mitigative strategy for Man- once inhabited
Animal Conflict?
Select the correct answer using the code
(a) Increasing alternate crops preys or given below:
water points
(b) Wildlife translocation A B C D E

(c) Wildlife conservation education for (a) 4 5 2 1 3


local populations
(b) 3 5 1 2 4
(d) Creation of Artificial and Natural
barriers (physical and biological) (c) 4 2 1 5 3

14. What do you understand by high-value (d) 2 1 5 4 3


fish?
17. Identify the incorrect with respect to
(a) A fish that is captured prior to Sarus Crane:
completing its life cycle.
(a) It is the tallest flying bird in the
(b) A fish that is captured after the world.
completion of its life cycle.
(b) IUCN has categorized it as Endangered
(c) A fish whose cost is high in the market species.
of commercial fishing.
(c) It is the state bird of Uttar Pradesh.
(d) A fish whose demand is high among
consumers due to its good quality. (d) It is India’s only resident breeding
crane.
15. Many species like Stellar’s sea cow,
Passenger pigeon, etc. extinct due to: 18. Find the correct statement related to
Sumatran Rhinoceros species:
(a) Habitat loss and fragmentation
(a) It is the largest of all five Rhino species
(b) Over-exploitation
found in the world.
(c) Alien species invasions
(b) It is categorized as Endangered species
(d) Co-extinction as per the IUCN Red List.

16. Match the following: (c) It is regionally extinct in India.


(d) None of the above
List I List II
19. Which of the following criteria is
A. Biological 1. Species, so few
required for a species to be considered
Extinction in number that
as ‘Endangered’?
it no longer plays
its normal role in (a) Reduction in population size by 70%
the community over the last 10 years.
TARGET PT 2020 63

(b) Population size estimated to number (a) 1 and 2 only


fewer than 10,000 mature individuals.
(b) 2 only
(c) When a species is known only to
survive in cultivation, in captivity or as (c) 3 only
a naturalized population well outside (d) 1, 2 and 3
the past range.
(d) None of the above 23. Aditya mission of ISRO is about
(a) To study the outermost region of the
20. Which of the following national park
sun, called corona.
is a part of the Vulture Safety zone in
India? (b) To study moon
(a) Sundarban National Park (c) To study the Indian ocean
(b) Jim Corbett (d) To study the monsoon patterns
(c) Gir Forest National Park
24. Avangard Hypersonic Missile System
(d) All of the above was in news recently. It is developed
by which country?
21. With reference to PM – KISAN scheme,
consider the following statements (a) Russia
1. It is a Centrally Sponsored scheme. (b) China
2. The fund is directly transferred to the
(c) USA
bank accounts of the beneficiaries.
3. The entire responsibility of identification (d) Britain
of beneficiary farmer families rests
25. Consider the following statements
with the State / UT Governments.
about Budapest Convention-
Which of the following statement is/ 1. It is also known as the Convention on
are correct? Cybercrime.
(a) 1 and 2 only 2. It is the first international treaty
(b) 2 and 3 only seeking to address Internet and
computer crime by harmonizing
(c) 1 and 3 only
national laws.
(d) 1, 2 and 3
3. It is drawn by Council of Europe and is
22. Consider the following statements not open for ratification to states which
regarding Ayushman Bharat Scheme are not its members.
1. It is a Central Sector Scheme. Which of the above statements is/are
2. Benefit cover of Rs. 5 lakh per family correct?
per year.
(a) 1 only
3. The benefit cover will include pre-
hospitalisation expenses only. (b) 2 only
(c) 1 and 2 only
Which of the following statement is/
are correct? (d) 1,2 and 3

**********
64 TARGET PT 2020

ANSWER HINTS
DAY - 35

1. Correct Option: (a) Andhra Pradesh 68 48


Explanation: Telangana – 26
Tiger Census 2018 Chhattisgarh 46 19
 The Total population of Royal Bengal Tiger
Jharkhand 3 5
in India is 2967 as per the Tiger Census
2018, which is more than double of 2006. Madhya Pradesh 308 526

Maharashtra 190 312

Odisha 28 28

Rajasthan 45 69

Goa 5 3

Karnataka 406 524

Kerala 136 190

Tamil Nadu 229 264

Arunachal Pradesh 28 29
 Madhya Pradesh became the Tiger
state of India with 526 Tigers whereas
Assam 167 190
Karnataka, who was holding the Tiger Mizoram 3 0
State salutation since 2010 slipped
to the second position with 524 Nagaland – –
Royal Bengal Tiger population and
Uttrakhand is third with 442 Tigers Northern WB 3 0
Population. Maharashtra, Kerala, and
Sunderbans 76 88
Uttar Pradesh follow these states.

 Statewise Tiger Population in India 2. Correct Option: (c)


(Comparision chart) Explanation:
State 2014 2018 Tiger Census 2018
Census Census  According to the latest census(2018), there
is no tiger in wild is left in 3 tiger reserves
Bihar 28 31 viz. Dampa, Palamau, and Buxa.
Uttrakhand 340 442  Thus, there is zero tiger is in Mizoram. In
Jharkhand and Wst Bengal, few tigers are
Uttar Pradesh 117 173 there in other tiger reserves of the states.
TARGET PT 2020 65

 This rare bustard species is very well


known for its mating dance. Among the
tall grasslands, secretive males advertise
their territories by springing from the
ground and flitting to and fro in the air.

4. Correct Option: (b)


Explanation:
Northern River Terrapin (Batagur baska)
3. Correct Option: (c)
 India harbors 24 species of freshwater
Explanation: turtles and 5 species of saltwater turtles,
with areas in Northeast and North
Bengal Florican (Houbaropsis India considered in the top three Turtle
bengalensis) Biodiversity Hotspots of the world.
 The Bustards are an extremely endangered  The northern river terrapin is one
group of birds dependent on grassland of Asia’s largest freshwater and
ecosystems. Once upon a time, they used backwater turtles.
to occur in the arid, semi-arid and moist
grasslands across the country. There are  It is a species of riverine turtle native
four species of Bustards in India Great to Southeast Asia. It is classified
Indian Bustard, Lesser Florican, Bengal Critically Endangered by the IUCN.
Florican, and Houbara Bustard. They In India, it is found in West Bengal &
are among the most threatened of the 22 Odisha.
Bustards found in the world.  It is strongly aquatic but uses terrestrial
 The Bengal Florican (Houbaropsis nesting sites, frequenting the tidal areas
bengalensis) is Critically Endangered, of estuaries, large rivers, and mangrove
due to rapid habitat loss and hunting. It forests.
occurs patchily from India to Vietnam, with
the majority of the world’s population
breeding around the Tonle Sap Great
Lake in Cambodia.
 Within India, the Bengal florican survives
in highly fragmented numbers in Manas
National Park, Kaziranga National Park,
Orang National Park, Sonai-Rupai Wildlife
Sanctuary, Dibru-Saikhoa National Park
and Burachapori Wildlife Sanctuary
in Assam, D’Ering Wildlife Sanctuary
in Arunachal Pradesh, Jaldapara and
Northern River Terrapin (Batagur baska)
Gorumara Wildlife Sanctuary in West
Bengal and Dudwa National Park in Uttar  Olive Ridley (Lepidochelys olivacea),
Pradesh. Green (Chelonia mydas), Hawksbill
66 TARGET PT 2020

(Eretmochelys imbricata), Loggerhead of the global conservation status of plant


(Caretta caretta) and the Leatherback and animal species.
(Dermochelys coriacea) turtles are the sea
 Out of nine categories, the “threatened” is
turtles of India.
a grouping of only three categories viz.
Critically Endangered, Endangered,
5. Correct Option: (d)
and Vulnerable.
Explanation: 7. Correct Option: (a)
Greater one-horned rhinos (Rhinoceros Explanation:
unicornis)
Elvira Rat (Cremnomys elvira)
 The Greater one-horned rhino (or “Indian
rhino”) is the largest of the rhino species.  It is a medium-sized, nocturnal and
burrowing rodent endemic to India.
 It is commonly found in Nepal, Bhutan,
Pakistan and in Assam, India. It is confined  Its habitat is the tropical dry deciduous
to the tall grasslands and forests in the shrubland forest, seen in rocky areas.
foothills of the Himalayas.
 It is known only from Eastern Ghats of
Tamil Nadu. Recorded from an elevation of
Rhinos Population about 600 m above mean sea level.
Javan rhino 72  It has been categorized as ‘Critically
Endangered’ according to the IUCN- Red
Sumatran rhino <80 Data Book.
Between 5,366 and
Black rhino
5,627

Greater one-horned rhino 3,588

Between 17,212
White rhino
and 18,915

8. Correct Option: (c)


Explanation:
Elephant Census
 Elephant census is conducted once in 5
years under the aegis of Project elephant.
 Out of an estimated 2,645 in Assam,  The estimated population of Asian
Kaziranga has 2,413 rhinos, according to Elephants in India is 29,964 as per the
the Rhino census 2018. census conducted in 2017.
 Pobitora Wildlife Sanctuary (102) shelters  Among the states, the highest
the highest density of Indian rhinos in the population has been recorded in
world. Karnataka followed by Assam, Kerala,
Tamil Nadu, Odisha, etc.
 It is listed as Vulnerable on the IUCN
Red List. Mammoth Count
 Javan rhinos and Sumatran rhinos
are the least populated rhinos. Table List State-wise population estimates of
elephants as per census conducted during 2017
6. Correct Option: (d)
State Count State Count
Explanation:
Karnatka 6,049 Uttar 232
IUCN red data book Pradesh
 The IUCN Red List of Threatened Species™
Assam 5,719 Tripura 102
is the world’s most comprehensive inventory
TARGET PT 2020 67

home range stretching from Gibraltar to


Kerala 5,706 Andhra 65
Japan. However, this particular primate
Pradesh
is confined to tiny, isolated pockets of
Tamil Nadu 2,761 Andaman 25 evergreen tropical forest in the Western
& Nicobar Ghat Mountains in India.
Islands  Also known as Wanderloo and Bruh,
Odisha 1,976 Bihar 25 they are endemic to the Nilgiri Biosphere
Reserve, the largest biosphere in India.
Uttrakhand 1,839 Manipur 9
 The reasons for being endangered
Arunachal 1,614 Mizoram 7 are habitat destruction and their slow
Pradesh reproduction cycle.
W.B. (North + 682 Madhya 7
South) Pradesh

Jharkhand 679 Haryana 7

Nagaland 446 Himachal 7


Pradesh

Chhattisgarh 247 Maharashtra 6

9. Correct Option: (b)


Explanation:
Pygmy hog (Porcula salvania)
 Today, the Pygmy Hog is found in just Wanderloo
three places in Assam—Manas, Sonai
Rupai and Orang. 11. Correct Option: (b)
 It is the world’s smallest wild pig. Explanation:
 It is one of the most useful indicators of the Kondana Rat
management status of grassland habitats.
 It is a nocturnal burrowing rodent
 Pygmy hog-sucking Louse, a parasite that is found only in India, at the small
that feeds only on Pygmy also falls in the Sinhagarh Plateau near Pune in
category of endangered as its survival is Maharashtra.
linked to that of the host species.
 It is categorized as “Critically
 According to the latest assessment Endangered” as per IUCN.
by IUCN, it has been put under the  It is sometimes known to build nests.
‘endangered’ category.

Porcula salvania Kondana Rat

10. Correct Option: (b) 12. Correct Option: (a)


Explanation: Explanation:

Lion-tailed Macaque Biodiversity hotspots in India


 The Lion-tailed Macaque is one of 16  The British biologist Norman Myers coined
Macaque species, which have an extensive the term “biodiversity hotspot” in 1988 as a
68 TARGET PT 2020

biogeographic region characterized both by  Wildlife conservation education for


exceptional levels of plant endemism and local populations.
by serious levels of habitat loss.
 Better sharing of information among the
 According to the Conservation International, forest conservator stakeholders.
to qualify as a hotspot a region must meet
two strict criteria: it must contain at least 14. Correct Option: (a)
1,500 species of vascular plants (> 0.5% of
the world’s total) as endemics, and it has Explanation:
to have lost at least 70% of its original
habitat. High-Value Fish

 There are currently 36 recognized  It is captured prior to completing its


biodiversity hotspots, out of which 4 are in life cycle. Fish species of high value
India. can be raised in fishponds at more
profitable rates.
 These are as follows:
 For example, seabass which sells at Pounds
! Himalaya: It includes the entire
Indian Himalayan region (and that 250 and above per kilo, which costs more
falling in Pakistan, Tibet, Nepal, than milkfish and tilapia.
Bhutan, China and Myanmar). In the
North-East, it includes regions that 15. Correct Option: (b)
are north of the Brahmaputra river Explanation:
including Assam.
! Indo-Burma: It includes entire Over-exploitation
North-eastern India that is south of  Due to over-hunting and over-exploitation
Brahmaputra, and Andaman group of various plants and animals by humans,
of Islands (and Myanmar, Thailand,
many species have become endangered or
Vietnam, Laos, Cambodia, and
extinct.
southern China).
 Many species like Stellar’s sea cow,
! Sundalands: It includes the Nicobar
Passenger pigeon, etc. extinct due to over-
group of Islands (and Indonesia,
Malaysia, Singapore, Brunei, exploitation.
Philippines).
16. Correct Option: (b)
! Western Ghats: it includes entire
western ghats and Sri Lanka. Explanation:

13. Correct Option: (d) Extinction of Species

Explanation: List I List II

Preventive strategies for Man-Animal A. Biological 3. Species no longer


Conflict Extinction found anywhere
 Creation of Artificial and natural on earth
barriers (physical and biological).
B. Local Extinction 5. Species no longer
 Alternative high-cost livestock husbandry found in area it
practices. once inhabited
 Voluntary relocation by the human
C. Ecological 1. Species is so few
population and resettlement.
Extinction in number that it
Mitigative strategies for Man-Animal no longer plays in
Conflict normal role
 Insurance programs for compensation of D. Background 2. Continuous
loss due to destruction caused by animals. Extinction process of
 Community-based natural resource natural and low-
management schemes (CBNRMS). level extinction
 Increasing alternate crops preys or E. Mass Extinction 4. More than 65% of
water points. all species become
 Wildlife translocation. extinct
TARGET PT 2020 69

Extinction of Species 17. Correct Option: (b)


 A known species has gone extinct Explanation:
if no member of the species is found
anywhere on Earth. This is known Sarus Crane
as biological extinction and is  It is the tallest flying bird in the world.
irreversible.
 IUCN has categorized it as Vulnerable
 Before a species goes biologically extinct, it species.
goes through two other stages:
 It is the state bird of Uttar Pradesh.
! Local Extinction: Species is no
 It is India’s only resident breeding crane.
longer found in the area it once
inhabited, though it is present  The population of Sarus Crane has been
elsewhere in the world. distributed in the Indian sub-continent,
southeast
! Ecological Extinction: So few
members of the species are left  Asia and northern Australia.
that it can no longer play its  The population of Sarus crane soared as
normal ecological role in the per the recent 2018 census.
community.
 Background Extinction is a process of 18. Correct Option: (c)
natural and low-level extinction that Explanation:
goes on continuously due to changes
in the environmental conditions; such The Sumatran Rhinoceros (Dicerorhinus
changes may be small or big, gradual or sumatrensis)
sudden. When such changes occur, the  There are five species of rhino: White rhino,
local species must adapt itself, move to a Black rhino, Sumatran rhino, Greater
more favorable area or become extinct. one-horned rhino (or Indian rhino)
 The background extinction has always been and Javan rhino.
happening and biologists say that 99.9% of  Sumatran Rhinoceros is the smallest and
all species that ever lived are extinct. most endangered of the five rhinoceros
 The rate of background extinction has been species.
generally uniform over long geological  It is now thought to be regionally extinct in
periods. At some points in time, however, India, though it once occurred in the foothills
mass extinctions have occurred on earth. of the Himalayas and north-east India.
 A mass extinction is a global,  The Javan Rhinoceros (Rhinoceros
catastrophic event with more than sondaicus) is also believed to be extinct in
65% of all species becoming extinct. India and only a small number survive in
Java and Vietnam.
 There have been 5 mass extinctions over
the past 500 million years and in each case  Sumatran Rhinoceros is categorized as
there was a huge loss of biodiversity. Critically Endangered as per IUCN red
list.
 Both environmental and biological factors
have led to mass extinctions. The suggested 19. Correct Option: (a)
causes include global cooling, falling sea
levels, predation and competition. Explanation:

 Our planet is now in the midst of its sixth Endangered Category (EN)
mass extinction of plants and animals —  A species is categorised in the Endangered
the sixth wave of extinctions in the past category if it meets any of the following
half-billion years. criteria:
 We’re currently experiencing the worst ! Reduction in population size by
spate of species die-offs since the loss of the 70% over the last 10 years.
dinosaurs 65 million years ago. Although
extinction is a natural phenomenon, it ! Population size is estimated to
number fewer than 250 mature
occurs at a natural “background” rate of
individuals.
about one to five species per year. Scientists
estimate we’re now losing species at up ! Quantitative analysis showing the
to 1,000 times the background rate, with probability of extinction in the wild in
literally dozens going extinct every day. at least 20% within 20 years.
70 TARGET PT 2020

Extinct in the Wild (EW)  For enrollment, the farmer is required to


approach the local patwari / revenue officer
 A species is considered Extinct in the
/ Nodal Officer (PM-Kisan) nominated by
Wild when it is known only to survive in
cultivation, in captivity or as a naturalized the State Government.
population well outside the past range and  The Common Service Centres (CSCs) have
exhaustive surveys have failed to record an also been authorized to do registration of
individual at their expected habitat. the farmers for the Scheme upon payment
of fees.
20. Correct Option: (b)
 Farmers can also do their self-registration
Explanation: through the Farmers Corner in the portal.
Vulture Safety Zone  Farmers can also edit their names in
PM-Kisan database as per their Aadhaar
 The concept of a Vulture Safety Zone is
database / card through the Farmers
unique for the Asian continent but similar
Corner in the portal.
VSZ is in operation in both Europe and
Africa.  Farmers can also know the status of their
payment through the Farmers Corner in
 Aim of developing Vulture Safety Zone is
to establish targeted awareness activities the portal.
surrounding a 150 km radius of vultures’
colonies so that no diclofenac or the Major Objective of the scheme
veterinary toxic drugs are found in cattle  With a view to provide income support to
carcasses, the main food of vultures(to all land holding eligible farmer families,
provide safe food). the Government has launched PM-KISAN.
 The Vulture Safety Zone has spread around  The scheme aims to supplement the
in several hundred kilometers covering
financial needs of the farmers in procuring
the Jim Corbett in Uttarakhand,
various inputs to ensure proper crop health
Dudhwa and Kartarniaghat forest
and appropriate yields, commensurate with
reserves in Uttar Pradesh which is
the anticipated farm income.
adjoining the Indo-Nepal border. Nepal has
already set up VSZ on the Indian borders.
Definition of farmer’s family
21. Correct option: (b)  A landholder farmer’s family is defined as
“a family comprising of husband, wife and
Explanation minor children who own cultivable land as
 Statement 1 is incorrect: Pradhan per land records of the concerned State/
Mantri Kisan Samman Nidhi (PM- UT”.
KISAN) is a Central Sector scheme
 The existing land-ownership system will be
with 100% funding from Government
used for identification of beneficiaries for
of India.
calculation of benefit.
Supplementary notes
22. Correct option: (b)
PM – KISAN scheme
Explanation
 Pradhan Mantri Kisan Samman Nidhi
(PM-KISAN) is a Central Sector scheme  Statement 1 is incorrect: Ayushman
with 100% funding from Government of Bharat - Pradhan Mantri Jan Arogya
India. Yojana (AB-PMJAY) is a Centrally
 Under the Scheme an income support of Sponsored Scheme
Rs.6000/- per year is provided to all farmer  Statement 3 is incorrect: The benefit
families across the country in three equal cover will also include pre and post-
installments of Rs.2000/- each every four hospitalisation expenses.
months.
Supplementary notes
 The entire responsibility of identification of
beneficiary farmer families rests with the Ayushman Bharat Scheme
State / UT Governments.
 Ayushman Bharat - Pradhan Mantri Jan
 The fund is directly transferred to the bank Arogya Yojana (AB-PMJAY) is a Centrally
accounts of the beneficiaries. Sponsored Scheme having central sector
 Farmers covered under the Exclusion component under Ayushman Bharat
Criteria of the Operational Guidelines are Mission anchored in the Ministry of Health
not eligible for the benefit of the Scheme. and Family Welfare (MoHFW).
TARGET PT 2020 71

 It is an umbrella of two major health Earlier, this mission was planned to


initiatives, namely Health and wellness launch in an 800 km low earth orbit. But, it
Centres and National Health Protection was found that a satellite placed in a halo
Scheme. orbit around the Lagrangian Point 1 (L1)
can give a big advantage of viewing Sun.
 AB-PMJAY provides a defined benefit cover
Thus, this mission was renamed Aditya
of Rs. 5 lakh per family per year. This cover
L-1 Mission.
will take care of almost all secondary care
and most of tertiary care procedures.  The temperature of the solar corona goes
beyond million degrees. From the ground,
 To ensure that nobody is left out (especially
the corona could be seen only during total
women, children and elderly) there will
solar eclipses mainly due to the bright solar
be no cap on family size and age in the
disc and the scattering of the sunlight by
scheme.
the earth’s atmosphere. To mask the bright
 The benefit cover will also include pre and solar disc and study the corona, one has to
post-hospitalisation expenses. go beyond the atmosphere.
 Benefits of the scheme are portable across Objectives
the country and a beneficiary covered
under the scheme will be allowed to take  The major scientific objectives of Aditya-1
cashless benefits from any public/private mission are to achieve a fundamental
empanelled hospitals across the country. understanding of the physical processes
that heat the solar corona; accelerate the
 The beneficiaries can avail benefits in both solar wind; and produce coronal mass
public and empanelled private facilities. All ejections (CMEs).
public hospitals in the States implementing
AB-PMJAY, will be deemed empanelled for Features of Aditya L1
the Scheme.
 There will be six scientific payloads on this
 For beneficiaries, it will be a cashless, 400 kg satellite.
paper less transaction. Keeping in view the
 It will be placed in the halo orbit near the
State specific requirements, States/ UTs
L1 point of the Sun-Earth system.
will have the flexibility to modify these
rates within a limited bandwidth.  The Aditya-L1 can now provide observations
of the Sun’s photosphere (soft and solid
 AB-PMJAY is an entitlement based scheme
X-rays), chromosphere (UV) and the corona
with entitlement decided on the basis of
(visual and NIR) along with additional
deprivation criteria in the SECC database.
experiments.
 At the national level to manage, a National
 The 20 cm coronagraph, having a field
Health Agency has been set up. States/
of view of corona from 1.05 R to 3.0 R,
UTs are advised to implement the scheme will use an off axis parabolic mirror. The
by a dedicated entity called State Health payload will have three CCD detectors
Agency (SHA). system with a capability of simultaneous
imaging in 6374 Å, 5303 Å and in 5800 Å
23. Correct Answer: (a)
for continuum/broadband.
Explanation: Option (a) is correct
6 Payloads of Aditya L-1 Mission
Supplementary Notes
 Visible Emission Line Coronagraph
 Prime Minister Narendra Modi recently (VELC): It will help to study the origin
highlighted in Mann Ki Baat program that of Coronal Mass Ejection, diagnostic
ISRO is planning to launch its first Sun parameters of solar corona and its
Mission Aditya L1. It is an ambitious plan dynamics.
of Indian Space Research Organisation
 Solar Ultraviolet Imaging Telescope
(ISRO).
(SUIT): This payload will help to find
 Aditya - L1 is the first Indian mission to out the image of spatially resolved Solar
study the outermost region of the sun, Photosphere as well as to measure solar
called corona. irradiance variations.
 Aditya-1 to Aditya L-1 Mission: The  Plasma Analyser Package for Aditya
concept of the Aditya-1 mission was to (PAPA): It has been designed to understand
carry 400 kg of the payload called Visible the energy distribution and composition of
Emission Line Coronagraph (VELC). solar winds.
72 TARGET PT 2020

 Solar Low Energy X-ray Spectrometer Strategic Implications


(SoLEXS): It will monitor X-ray flares of
 Avangard is a strategically valuable weapon
Solar system to study the heating system
for two main reasons, its maneuverability
of solar corona.
and its versatility.
 High Energy L1 Orbiting X-ray  The weapon is capable of performing sharp
Spectrometer (HEL1OS): It will help maneuvers on its way to targets making
to observe the various dynamic actions it absolutely invulnerable for any missile
happen in solar corona to provide an defense system.
estimate of solar energy.
 It also boasts the ability to deploy
 Magnetometer: This payload will monitor countermeasures during flight allowing
and measure the magnitude of the nature it to penetrate air and missile defenses
of Interplanetary Magnetic Field in the virtually undetected.
solar system.
 As for its versatility, Avangard can be
24. Correct Answer: (a) fitted with both nuclear and conventional
payloads depending on a situation.
Explanation: Option (a) is correct. Moreover, even without an explosive
payload, the precision and speed of the
Supplementary Notes weapon is believed to have enough force to
 Recently, Russia’s first regiment of obliterate smaller targets, such as vehicles
Avangard hypersonic missiles has been put or bases, making it an invaluable weapon
into service sending shock waves around in the Russian arsenal.
the world.
25. Correct Answer- (c)
 Avangard is a hypersonic glide vehicle
developed by Russia. It’s designed to Explanation-
be carried as a multiple independently
targetable reentry vehicle (MIRV) payload Statement (1) and (2) are correct-
by the UR-100UTTKh, RS-26 Rubezh and The Convention on Cybercrime, also
RS-28 Sarmat super-heavy ICBM. known as the Budapest Convention on
Cybercrime or the Budapest Convention,
 Avangard can presumably reach
speeds up to Mach 20 and can be used to is the first international treaty seeking
deliver nuclear and conventional payloads. to address Internet and computer
crime (cybercrime)by harmonizing national
 It’s designed to sit atop an intercontinental laws, improving investigative techniques,
ballistic missile (ICBM) and, once launched,
and increasing cooperation among nations.
it uses aerodynamic forces to sail on top of
the atmosphere. It was drawn up by the Council of
 Russia has successfully tested the missile Europe in Strasbourg, France, with the
at least twice and it is expected to enter active participation of the Council of
service in late 2018 or early 2019. Europe’s observer states Canada, Japan,
 During the annual state-of-the-nation Philippines, South Africa and the United
in March 2018, the President of Russia, States.
Vladimir Putin unveiled the Avangard and
described the Hypersonic Missile as one of Statement (3) is incorrect- It is open
the six next-generation weapons that are for ratification even to states that are not
under development. members of the Council of Europe.
 The initial research on hypersonic warheads Supplementary notes
started back in the mid-1980s at the time
of the USSR or the Soviet Union which was
Budapest Convention
ceased at the time of the dissolution of the  Since it entered into force, important
Soviet Union in 1991. Later around the countries like Brazil and India have
mid-1990s, Russia started working back on declined to adopt the Convention on the
the project under the name ‘Project 4202’. grounds that they did not participate in its
Since then, Russia has made around 14 drafting.
reported tests of the Avangard Hypersonic
 Russia opposes the Convention, stating
Missile. A Hypersonic Missile means
that adoption would violate Russian
that can travel at speed of above Mach sovereignty, and has usually refused to
5 (Mach 5 speed is 5 times more than cooperate in law enforcement investigations
the speed of sound). relating to cybercrime.
TARGET PT 2020 73

 It is the first multilateral legally binding pornography, hate crimes, and violations
instrument to regulate cybercrime. of network security.
 Since 2018, India has been reconsidering  It also contains a series of powers and
its stand on the Convention after a surge in procedures such as the search of computer
cybercrime, though concerns about sharing networks and lawful interception.
data with foreign agencies remain.
 Its main objective, set out in the
 The Convention is the first international preamble, is to pursue a common
treaty on crimes committed via the criminal policy aimed at the protection
Internet and other computer networks, of society against cybercrime, especially
dealing particularly with infringements of by adopting appropriate legislation and
copyright, computer-related fraud, child fostering international cooperation.

**********
74 TARGET PT 2020

TEST
DAY - 36

Time Allowed: 30 mins Maximum Marks: 50

1. Consider the following statements 2. Its decisions are merely advisory in


regarding Earth Overshoot Day: nature.
1. It is not a fixed date as it keeps changing 3. CoP 13 was held in China.
each year.
Which of the above statements is/are
2. In 2019, it was observed as the earliest correct?
ever.
(a) 2 only
Which of the above statements is/are
(b) 1 and 3 only
correct?
(c) 2 and 3 only
(a) 1 only
(d) 1, 2 and 3
(b) 2 only
(c) Both 1 and 2 4. Where is India’s first garbage café
situated?
(d) Neither 1 nor 2
(a) (a) Uttar Pradesh
2. Consider the following statements (b) (b) Chhattisgarh
regarding Secure Himalaya project:
(c) (c) Jharkhand
1. It has been launched by MOEFCC and
IUCN. (d) (d) West Bengal
2. Its main objective is to protect Kashmiri 5. Which of the following statements is/
Stag/Hangul. are correct?
3. North-Eastern states are not covered 1. Deforestation means a decrease in
in this project. the area covered by forests whereas,
forest degradation does not involve a
Which of the above statements is/are
reduction in the forest area.
incorrect?
2. Land Degradation Neutrality is one of
(a) 1 only the targets of the SDGs.
(b) 1 and 2 only
Select the correct option using the codes
(c) 3 only given below:
(d) 2 and 3 only (a) 1 only

3. The CoP 14 to the United Nations (b) 2 only


Convention to Combat Desertification (c) Both 1 and 2
was held in India. With reference to this,
(d) Neither 1 nor 2
consider the following statements:
1. UNCCD is one of the results of the 6. Which of the following conservation
Earth Summit at the Rio Conference, strategies does not involve community
1992. participation directly?
TARGET PT 2020 75

(a) Joint Forest Management Select the correct option using the codes
given below:
(b) BeejBachaoAndolan
(c) Chipko Movement (a) 2 only

(d) Demarcation of Wildlife sanctuaries (b) 3 only


(c) 1 and 3 only
7. Which of the following is called climate
change’s equally evil twin? (d) 1, 2 and 3

(a) Ozone depletion 11. Consider the following statements:


(b) Ocean acidification 1. Yokkaichi asthma can be occurred due
(c) Ocean deoxygenation to the burning of petroleum.
2. Yusho Poisoning is due to the
(d) Forest degradation
contamination of polychlorinated
8. How does the ocean acidification biphenyls.
impact Coral reefs?
Select the correct option using the codes
1. It limits the formation of Calcium given below:
bicarbonate skeleton.
(a) 1 only
2. They do not affect the pre existing
skeleton but hinders the process of new (b) 2 only
coral formation. (c) Both 1 and 2
3. It may also impact corals before they (d) Neither 1 nor 2
even begin constructing their homes.
12. Which of the following pairs is/are
Select the correct option using the codes
incorrectly matched?
given below:
Diseases: Causes
(a) 3 only
1. Itai-Itai: Cadmium
(b) 2 only
2. Blue baby syndrome: Fluorides
(c) 1 and 2 only
3. Black Lung disease: Coal dust
(d) 1, 2, and 3
Select the correct option using the codes
9. Which of the following is not a given below:
consequence of the deforestation:
(a) 2 only
1. Landslides in hilly areas
(b) 3 only
2. Silting of rivers and lakes
(c) 1 and 3 only
3. Loss of CO2 sink
4. Desertification (d) 2 and 3 only

Select the correct option using the codes 13. Which of the following pair is incorrectly
given below: matched pertaining to the e-waste
metals and their respective sources?
(a) 1 only
(a) Beryllium – Glass panels
(b) 3 only
(b) Mercury – Thermostats
(c) 2 and 4 only
(c) Hexavalent Chromium – PVC
(d) None of the above
(d) Barium – Cathode Ray tube
10. Identify the correct human activity
which degrades the environment: 14. Solid wastes are the discarded
1. Rapid industrializations materials which needs to be treated
and disposed. In light of this statement,
2. Overuse of consumer durable goods which of the following are the best
3. Excessive irrigation ways to dispose or treat solid waste?
76 TARGET PT 2020

(a) Incineration plants, Pyrolysis, Sanitary (c) 2 and 3 only


landfills and Open dumps
(d) 1, 2 and 3
(b) By in-situ bioremediation techniques
18. World Day to Combat Desertification
(c) Mycromediation and Mycofiltration and Drought (WDCD) is observed on
(d) All of the above June 17th every year. Consider the
following statements with reference to
15. Which of the following statements it:
correctly defines the Putrescibility? 1. It aims to promote public awareness to
(a) It is the process of decomposition of combat land degradation and expansion
organic matter in water. of existing deserts.

(b) It is a process in which hemoglobin reacts 2. Its 2019 theme is ‘Land has true value.
Invest in it.’
with non-functional methaemoglobin
and impairs oxygen transport. 3. Agenda for Sustainable Development
Goal 15 emphasizes the need to halt
(c) It is a process in which water is purified and reverse land degradation.
by extracting toxic materials and heavy
metals. Which of the above statements is/are
(d) It is a process of treatment of sewage correct?
water. (a) 1 only

16. Which among the following statement (b) 1 and 3 only


is not correct pertaining to the Waste (c) 3 only
Minimization Circles (WMC)?
(d) 2 and 3 only
(a) It is implemented under the United
Nations Environment Programme 19. Consider the following statements
(UNEP) with the assistance of the about Rotterdam Convention.
Central Pollution Control Board. 1. It is a multilateral treaty on the
(b) It aims to realize the objectives of the Prior Informed Consent Procedure
Policy Statement for Abatement of for Certain Hazardous Chemicals and
Pesticides in International Trade
Pollution (1992).
2. It is jointly administered by the World
(c) It emphasizes the participation Health Organisation and the United
of citizens and the government in Nations Environment Programme
environmental monitoring. (UNEP).
(d) None of the above
Which of the above statements is/are
17. With reference to a Special Report on correct?
Climate Change and Land as seen in (a) 1 only
news recently, consider the following
(b) 2 only
statements:
1. It is published by Intergovernmental (c) Both 1 and 2
Panel on Climate Change (IPCC). (d) Neither 1 nor 2
2. The report claims that global food
wastage is also a contributor to climate 20. Which of the following statements is/
change. are correct with respect to National
Afforestation Program (NAP)?
3. As per the report, land degradation
1. It is being operated as a 100% Central
is both the cause and consequence of
Sector Scheme.
climate change.
2. The objective of the scheme is to
Which of the above statements is/are develop the forest resources with
correct? people’s participation, with focus on
improvement in livelihoods of the
(a) 1 only
forest-fringe communities, especially
(b) 1 and 2 only the poor.
TARGET PT 2020 77

Select the correct answer using the code 23. Which of the following activities can be
given below: said to be a cause of climate change?
(a) 1 only 1. Land degradation

(b) 2 only 2. Industrial heat


3. Meat consumption
(c) Both 1 and 2
(d) Neither 1 nor 2 Choose the correct answer:
(a) Only 1
21. Consider the following statements
regarding Prime Minister Van (b) 1 and 2 only
DhanYojana (c) 2 and 3 only
1. The Van Dhan Scheme is an initiative (d) 1, 2 and 3
of the Ministry of Environment Forest
and Climate Change 24. ISRO recently launched spy satellite,
2. It is a Market Linked Tribal RISAT-2BR1, in the orbit. Which of the
following statements are correct about
Entrepreneurship Development
this launch?
Program.
1. RISAT-2BR1 was launched on PSLV-
Which of the following statement is/ C48
are correct? 2. This launch carried customer satellites
(a) 1 only of United States, France, Japan, and
Israel.
(b) 2 only
3. RISAT-2BR1 will be used in fields of
(c) Both 1 and 2 agriculture and disaster management
(d) Neither 1 nor 2 support.

Choose the correct answer:


22. What are the major problems that
plagued the effective implementation (a) Only 1
of mid-day meal scheme? (b) 1 and 2 only
1. There are too many layers of government
(c) 1 and 3 only
involved in the scheme, resulting in
poor information, coordination and (d) 1, 2 and 3
monitoring.
25. Recently, a newly discovered species
2. Rampant corrupt practices have was named Marengo sachintendulkar.
been found from food procurement to Which of the following statements
distribution of hot cooked meals among is correct regarding this newly
children. discovered specie?
3. Caste bias and discrimination continues 1. Marengo sachintendulkar is a squirrel.
in the implementation of the scheme 2. Marengo sachintendulkar is a round-
4. It has decreased enrollment in schools, bodied squirrel.
decreased attendance in schools, 3. Another, jointly discovered specie,
reduced performance of students in IndomarengoChavarapatera, was
class in terms of better attention span named after a social reformer from
and academic progress. Kerala.
Choose the correct option Choose the correct answer:
(a) 1 and 2 (a) 1 and 2 only
(b) 1, 2 and 3 (b) Only 2
(c) 1, 2, 3 and 4 (c) Only 3
(d) 2, 3 and 4 (d) 1, 2 and 3

**********
78 TARGET PT 2020

ANSWER HINTS
DAY - 36

1. Correct Option: (c) 3. Correct Option: (b)


Explanation: Explanation:

Earth Overshoot Day United Nations Convention to Combat


Desertification
 Earth Overshoot Day (EOD), previously
 The United Nations Convention to Combat
known as Ecological Debt Day (EDD), is Desertification (UNCCD) is the sole
the calculated illustrative calendar date on legally binding international agreement
which humanity’s resource consumption linking environment and development to
for the year exceeds Earth’s capacity to sustainable land management.
regenerate those resources that year.  India is a signatory to the United Nations
Convention for Combating Desertification
 Earth Overshoot Day is calculated by the (UNCCD).
Global Footprint Network.
 It is one of the three conventions of the
 It keeps changing each year and is Earth Summit at the Rio Conference,
observed earlier than the previous 1992. These are the United Nations
year. Framework Convention on Climate Change
(UNFCC), United Nations Convention
 In 2019, it was observed on July 29, the to Combat Desertification(UNCCD), and
earliest ever. Convention on Biological Diversity (CBD).
 One of the main functions of the COP
2. Correct Option: (b) is to review reports submitted by the
Parties detailing how they are carrying
Explanation: out their commitments; the COP makes
recommendations on the basis of these
Secure Himalaya project reports. It also has the power to make
amendments to the Convention or to adopt
 SECURE Himalaya project was launched
new annexes, such as additional regional
in the Global Wildlife Programme (GWP) implementation annexes. In this way, the
conference as a collaboration between the COP can guide the Convention as global
Environment, Forests and Climate circumstances and national needs change.
Change Ministry (MoEFCC) and the To assist the COP, the Convention provides
UNDP (United Nations Development for subsidiary bodies and allows the COP
to establish additional ones if necessary.
Programme)
 CoP 13 was held at Ordos, China.
 It covers the high Himalayan Ecosystem
spread over four states viz. Himachal  The permanent secretariat of the
Convention was established in Article 23 of
Pradesh, Jammu, and Kashmir,
the UNCCD. It has been located in Bonn,
Uttarakhand, and Sikkim. Germany since January 1999.
 The key focus of the project is on improving
4. Correct Option: (b)
the enforcement to ensure the protection
of snow leopard and other endangered Explanation:
species and ensuring a secure livelihood to  India’s first garbage cafe is opened in
the people in the region. Ambikapur city of Chhattisgarh.
TARGET PT 2020 79

5. Correct Option: (c) forest land managed by the forest


department.
Explanation:
 In return, the members of these
Land/forest management communities are entitled to intermediary
benefits like non-timber forest produces
 Deforestation: this involves a decrease in the
and share in the timber harvested by
area covered by forest. However, it cannot
‘successful protection’.
be so defined without adding a reference to
its use (or allocation). In point of fact, there
exist certain forms of forest utilization - and
7. Correct Option: (b)
priority objectives of forest management Explanation:
- that clear temporarily the forest cover
while guaranteeing its maintenance. This Ocean acidification
is the case of clearcutting of areas where
 Ocean acidification is sometimes
the forest will regenerate itself or be
called “climate change’s equally evil
regenerated, or of the final cut in an even-
twin,” and for good reason: it’s a significant
aged forest silvicultural treatment once
and harmful consequence of excess carbon
natural regeneration has been assured.
dioxide in the atmosphere that we don’t see
In other words, there is no deforestation
or feel because its effects are happening
if there is a guarantee of continuity in
underwater.
maintaining the forest cover;
 At least one-quarter of the carbon dioxide
 Degradation: this does not involve a
(CO2) released by burning coal, oil and gas
reduction of the forest area, but rather a
doesn’t stay in the air but instead dissolves
quality decrease in its condition, this being
into the ocean. Since the beginning of the
related to one or a number of different
industrial era, the ocean has absorbed
forest ecosystem components (vegetation
some 525 billion tons of CO2 from the
layer, fauna, soil, etc.), to the interactions
atmosphere, presently around 22 million
between these components, and more
tons per day.
generally to its functioning.
 Land Degradation Neutrality is one of the 8. Correct Option: (a)
targets of the SDGs in target number 15.3.
Explanation:
6. Correct Option: (d) Impact of ocean acidification on coral
Explanation: reefs
 Reef-building corals craft their own
Forest Conservation
homes from calcium carbonate, forming
 Chipko movement in the Himalayas has complex reefs that house the coral animals
not only successfully resisted deforestation themselves and provide habitat for many
in several areas but has also shown other organisms.
that community afforestation with
 Acidification may limit coral growth
indigenous species can be enormously
by corroding pre-existing coral
successful.
skeletonswhile simultaneously
 Farmers and citizen’s groups like the slowing the growth of new ones, and
BeejBachaoAndolan in Tehri and the weaker reefs that result will be more
Navdanya have shown that adequate vulnerable to erosion. This erosion will
levels of diversified crop production without come not only from storm waves but also
the use of synthetic chemicals are possible from animals that drill into or eat coral.
and economically viable.
 A recent study predicts that by roughly
 In India, the Joint Forest Management 2080 ocean conditions will be so acidic
(JFM) program furnishes a good example that even otherwise healthy coral reefs
for involving local communities in the will be eroding more quickly than they can
management and restoration of degraded rebuild.
forests. The program has been in formal
 Acidification may also impact corals
existence since 1988 when the state of
before they even begin constructing
Odisha passed the first resolution for joint
their homes. The eggs and larvae of
forest management.
only a few coral species have been
 JFM depends on the formation of local studied, and more acidic water didn’t
(village) institutions that undertake hurt their development while they
protection activities mostly on degraded were still in the plankton. However,
80 TARGET PT 2020

larvae in acidic water had more and paper mills pollute water in which
trouble finding a good place to settle, they dump the waste.
preventing them from reaching
 Loss of medicinal and other useful
adulthood.
plants: Unique medicinal plants grow
in certain forests. They are lost due to
deforestation. Aromatic herbs, rubber trees,
and other useful plants are also lost due to
deforestation Thus forest destruction leads
to large scale environmental degradation.

10. Correct Option: (d)


Explanation:
Impact of Industrialization on the
Environment

Industrialization led to environmental


degradation like the following reasons:
 Natural resources used as raw materials
9. Correct Option: (d) by industry are depleting rapidly.

Explanation:  Industries generate a lot of toxic gases, and


liquid effluents leading to environmental
Consequences of deforestation degradation.
 Soil erosion: Trees intercept rainfall and  Industries generate a large amount of
cutting of trees and removal of plants leads waste, which piles up in the environment.
to soil erosion. Plants’ roots hold the soil Disposal of waste not only needs land but
in place. With the loss of protecting cover also pollutes the environment and poses
of plants, topsoil, which is rich in organic hazards to human health.
matter, is washed away and the soil loses
its fertility.  Industries use up a lot of fossil fuels as a
source of energy. Accelerated consumption
 Landslides: Removal of trees from of fossils fuels is depleting their stock as
forests leads to soil erosion. Ultimately they are limited and non-renewable. But
cause landslides in hilly areas. This is the burning of fossil fuels releases CO2 in
because the roots of trees hold the soil the atmosphere leading to global warming.
in position;
 Silting: The loss of trees from forests Impact of Modernized Agriculture on
also causes silting of rivers and lakes as Environment
loose soil gets washed with rainwater
and reaches water bodies; An increase in food production to achieve
self-sufficiency is an important objective.
 Loss of wild habitat: Wild animals live
Intensive agriculture, unfortunately, may
in forests. Cutting forests means loss of
their habitat which in turn renders them lead to serious damage to the environment.
endangered or even extinct. Some of these are listed below:
 Climate Change: Deforestation results in  Forests have been cleared transformed into
a change in climate since trees make the farmlands for growing food crops.
surroundings humid. The loss of trees leads  Excessive irrigation and poor drainage
to loss of humidity. Also, transpiration from causes waterlogging and kill plants.
plants makes rain clouds and so rainfall is
reduced due to deforestation. This causes  Pollution by agrochemicals like increased
desertification. use of synthetic fertilizers and pesticides
 Loss of CO2 sink: Pollutants released  Agro-industrial wastes are generated. e.g.
by industries take CO2 are taken up by crop residues such as paddy, jawar, gram
trees. When forests are denuded, This straws, cotton straws, sugarcane trash,
CO2 sink is lost and CO2 collects in the and coconut shells, etc. pile up causing
environment. environmental degradation.
 Pollution: When trees are cut to use for  High yielding varieties (HYV) of food crops
making furniture or paper, the sawmills replaced various traditional crop varieties.
TARGET PT 2020 81

Traditional agriculture was based on a biphenyls) had accidentally leaked


multi-cropping system, i.e. growing of into the oil during the manufacturing
food crops, fodder, and firewood crops process. Health experts now refer to
together. This practice had been replaced this illness as ”Yusho,” which means ”oil
by monoculture i.e., growing of only one disease.”
kind of crop (such as wheat, etc) in a field
 Yokkaichi asthma occurred in the city
of a specific set of nutrients making soil
of Yokkaichi in Mie Prefecture, Japan
unfit for growing other crops but is being
between 1960 and 1972. The burning of
considered again.
petroleum and crude oil released large
Urbanization and Environment quantities of sulfur oxide that caused severe
smog, resulting in severe cases of chronic
Growth of cities leads to increasing demand obstructive pulmonary disease, chronic
for environmental resources leading to bronchitis, pulmonary emphysema, and
following changes-  bronchial asthma among the local
inhabitants.
 Cultivated land was lost forever for
building houses, industries, roads, and
other facilities.
12. Correct Option: (a)

 The availability of water becomes more Explanation:


and more scarce.
Environmental related diseases
 In cities, black smoke emitted from
 Itai-Itai disease was the documented case
industries, buses, truck, etc. cause air
of mass cadmium poisoning in Toyama
pollution. A large amount of garbage is
Prefecture, Japan, starting around 1912.
generated and not disposed of properly.
The cadmium poisoning caused softening
As a result, garbage remains scattered
of the bones and kidney failure. The
and unattended. Domestic and industrial
cadmium was released into rivers by
effluents are drained into rivers and lakes.
mining companies in the mountains.
High noise levels are a common feature of
the urban environment.  Blue baby syndrome is believed to be
caused by high nitrate contamination
 Inadequate facilities and lack of basic
in groundwater resulting in decreased
amenities in slums lead to unhygienic oxygen-carrying capacity of hemoglobin in
conditions and social distortion and crime. babies leading to death. The groundwater
Need for various commodities is thought to be contaminated by leaching
of nitrate generated from fertilizer used in
 Articles of everyday use such as agricultural lands and waste dumps. It may
plastic vessels, mugs, buckets, etc., also be related to some pesticides (DDT,
agricultural implements, machinery, PCBs, etc), which cause ecotoxicological
chemicals, cosmetics, etc are problems in the food chains of living
manufactured in factories. organisms, increasing BOD, which kills
 The raw materials and fossil fuels and water aquatic animals.
needed to run industries for manufacturing  Pneumoconiosis is caused due to the
these products lead to their depletion. deposit of coal dust in the lungs of coal
 Mining activities have depleted stock of miners, leads to a serious lung disease
called Black Lung disease.
mineral resources particularly fossil fuels.

11. Correct Option: (c) 13. Correct Option: (a)

Explanation: Explanation:
E-Waste Pollutants
Environmental related diseases
 E-Waste or electronic waste includes
 In 1968, more than one thousand people
discarded and end-of-life electronic products
in western Japan became seriously ill.
ranging from computers, equipment
They suffered from fatigue, headache,
used in Information and Communication
cough, numbness in the arms and legs, and
Technology (ICT), home appliances,
unusual skin sores. Pregnant women later
audio, and video products and all of their
delivered babies with birth defects. These
peripherals.
people had eaten food that was cooked in
contaminated rice oil. Toxic chemicals  E-Waste can be hazardous if it is recycled
called PCBs (polychlorinated in primitive ways.
82 TARGET PT 2020

 The related e-waste metals and their is an alternative to incineration. The gas
sources are: and liquid thus obtained can be used as
fuels.
! Beryllium – It is commonly found
on motherboards and finger clip and  Composting: Composting is a biological
is used as a copper-beryllium alloy to process in which micro-organisms, mainly
strengthen connectors. fungi and bacteria, decompose degradable
! Mercury – It is estimated that 22% organic waste into humus like substance in
of the yearly world consumption the presence of oxygen.
of mercury is used in electrical  Vermiculture: It is also known as
and electronic equipment is used earthworm farming. In this method, Earth
in thermostats, sensors, relays, worms are added to the compost. These
switches, medical equipment, lamps,
worms break the waste and the added
mobile phones, and batteries.
excreta of the worms makes the compost
! Hexavalent Chromium (Chromium very rich in nutrients.
VI) - It is used as a corrosion protector
of untreated and galvanized steel 15. Correct Option: (a)
plates and as a decorative or hardener
for steel housings Plastics (including Explanation:
PVC).
Putrescibility
! Barium – It is a soft silvery-white
metal that is used in computers in the  It is the process of decomposition of organic
front panel of a Cathode Ray Tube, matter present in water by microorganisms
to protect users from radiation. using oxygen.
! Lead – It is used in glass panels and
16. Correct Option: (a)
gaskets in computer monitors.
Explanation:
14. Correct Option: (a)
Waste Minimization Circles (WMC)
Explanation:
 WMC helps Small and Medium Industrial
Treatment and disposal of solid waste Clusters in waste minimization in their
 Open dumps: Open dumps refer to industrial plants.
uncovered areas that are used to dump solid  This is assisted by the World Bank with
waste of all kinds. The waste is untreated, the Ministry of Environment and Forests
uncovered, and not segregated. and Climate Change acting as the nodal
 Landfills: Landfills are generally located ministry. The project is being implemented
in urban areas. It is a pit that is dug in with the assistance of the National
the ground. The garbage is dumped and Productivity Council (NPC), New Delhi.
the pit is covered with soil everyday thus
 The initiative also aims to realize the
preventing the breeding of flies and rats.
Thus, every day, garbage is dumped and objectives of the Policy Statement for
sealed. After the landfill is full, the area Abatement of Pollution (1992), which
is covered with a thick layer of mud and states that the government should educate
the site can thereafter be developed as a citizens about environmental risks, the
parking lot or a park. economic and health dangers of resource
degradation and the real economic cost of
 Sanitary landfills: Sanitary landfill is natural resources.
more hygienic and built in a methodical
manner to solve the problem of leaching.  The policy also recognizes that citizens
These are lined with materials that are and non-governmental organizations
impermeable such as plastics and clay, and play a role in environmental
are also built over impermeable soil. monitoring, therefore, enabling them
 Incineration plants: The process of to supplement the regulatory system and
burning waste in large furnaces at high recognizing their expertise where such
temperature is known as incineration. exists and where their commitments and
In these plants the recyclable material is vigilance would be cost-effective.
segregated and the rest of the material is
burnt and ash is produced. 17. Correct Option: (d)
 Pyrolysis: It is a process of combustion Explanation:
in absence of oxygen or the material burnt
under controlled atmosphere of oxygen. It Climate Change and Land
TARGET PT 2020 83

 The IPCC approved and accepted as changes in the distribution and intensity
Climate Change and Land: an IPCC of extreme events.
special report on climate change,
 Degraded land is less productive which
desertification, land degradation,
reduces its ability to absorb carbon thus
sustainable land management, food
exacerbating climate change.
security, and greenhouse gas fluxes
in terrestrial ecosystems at its 50th
18. Correct Option: (c)
Session held on 2 – 7 August 2019.
Explanation:
 This is the first time that IPCC has
solely focused on land sector.  Statement 1 is incorrect: Desertification
is the degradation of land in arid, semi-arid
 The current report talks about the
and dry sub-humid areas. it doesn’t refer to
contribution of land-related activities to the expansion of existing deserts.
global warming — how the different uses
of land, like agriculture, industry, forestry,  Statement 2 is incorrect: WDCD 2019
cattle-rearing, and urbanization, was marks the 25th anniversary of the UN
affecting emissions of greenhouse gases. Convention to Combat Desertification
(UNCCD), So this year campaign will run
 An important part of the report talks about under the slogan 25 years - Let’s grow the
the manner in which even existential future together.’
activities like food production contributes
to global warming and is also affected by World Day to Combat Desertification and
it. Drought (WDCD)
 The report says that if pre-production  World Day to Combat Desertification
activities like cattle rearing and post- and Drought Observed every year on 17
production activities like transport, energy June, to promote public awareness
and food processing, is taken into account, of international efforts to combat
then food production could contribute as desertification and land degradation
much as 37 per cent of all greenhouse gas neutrality is achievable through
emissions every year. problem-solving, strong community
involvement and co-operation at all
 It points out that nearly 25 per cent levels.
of all food produced is either lost or
wasted. And even the decomposition  Desertification is the degradation of land in
of the waste releases emissions. arid, semi-arid and dry sub-humid areas. it
doesn’t refer to the expansion of existing
 The report shows that sustainable land deserts.
management including sustainable
forest management can help reduce  The Day was declared by the United
land degradation and also tackle climate Nations General Assembly in 1995.
change. Many countries, groups and individuals
celebrate and observe WDCD by organizing
 Coordinated efforts to tackle climate a variety of outreach activities and
change will also help improve land, food awareness raising events.
security and nutrition etc.
 WDCD 2019 marks the 25th anniversary
 Reducing over-consumption and waste of of the UN Convention to Combat
food, eliminating the clearing and burning Desertification (UNCCD), So this year
of forests, preventing over-harvesting of campaign will run under the slogan 25
fuel wood, and reducing greenhouse gas years - Let’s grow the future together.’
emissions will help to address land related
 The 2030 Agenda for Sustainable
climate change issues. Development especially the Goal 15
emphasizes the need to halt and
Land-Climate Link:
reverse land degradation. Sustainable
 Land use and changes in land use have Development Goals SDG Target 15.3
always been an integral part of the aims to achieve a Land Degradation
conversation on climate change. That is Neutral World by the year 2030, by
because land acts as both the source as maintaining and increasing the amount of
well as a sink of carbon. healthy and productive land resources.
 Land Degradation is both the cause
19. Correct Option: (a)
and consequence of climate change.
Explanation:
 Climate change causes the land to
degrade through both gradual changes in  Statement 2 is incorrect: It is jointly
temperature and rainfall patterns, as well administered by the United Nations Food
84 TARGET PT 2020

and Agriculture Organization (FAO) improvement in livelihoods of the forest-


and the United Nations Environment fringe communities, especially the poor.
Programme (UNEP).
 NAP aims to support and accelerate
Rotterdam Convention the on-going process of devolving forest
conservation, protection, management
 Rotterdam Convention is also called as and development functions tothe Joint
the Prior Informed Consent Procedure Forest Management Committees (JFMCs)
for Certain Hazardous Chemicals and at the village level, which are registered
Pesticides in International Trade societies.
What it does?  The scheme is implemented by three tier
 The convention promotes open exchange of institutional setup through the State Forest
information between importers-exporters Development Agency (SFDA) at the state
of hazardous chemicals. level, Forest Development Agency (FDA)
at the forest division level and JFMCs at
 Calls on exporters of hazardous chemicals village level.
to use proper labeling, include directions
on safe handling, and inform purchasers of  The major components of the scheme
any known restrictions or bans. includes afforestation under Seven
plantation models, maintenance of previous
 Signatory nations can decide whether to years plantations and Ancillary Activities
allow or ban the importation of chemicals like soil and moisture conservation
listed in the treaty
activities (SMC), fencing, overheads,
 Exporting countries are obliged to make monitoring and evaluation (M&E), micro-
sure that producers within their jurisdiction planning, awareness raising, Entry Point
comply. Activities (EPA) etc.
 It is jointly administered by the  The Scheme is demand driven and
United Nations Food and Agriculture afforestation area is sanctioned on the basis
Organization (FAO) and the United of past performance, potential degraded
Nations Environment Programme forest land available for eco-restoration
(UNEP). and availability of budget.

20. Correct Option: (b)  The Annual Plan of Operation (APO) of


SFDAs is approved as per Guidelines of
Explanation: NAP. NAP is a centrally sponsored scheme
which is implemented with the fund sharing
National Afforestation Program pattern of 60: 40 percent between Centre
 The Ministry of Environment, Forest and and States wherein the sharing pattern for
Climate Change (MoEFCC) is implementing Northeastern and hilly States is 90:10.
plantation/afforestation schemes in the
 The central share of funds are released
forest areas with participatory approach.
through State Government and state
 The plantation species under the schemes government transfers the funds to SFDA
is selected by the implementing agencies/ along with its state share which sometime
the members of Joint Forest Management causes delay in fund availability to SFDAs
Committees (JFMC) on the basis of their for implementation of NAPcausing delay
needs, ecological conditions and other local in submission of mandatory documents for
factors in consultation with the Forest subsequent release of funds.
Department.
 The native forest species are encouraged 21. Correct option: (b)
for plantation in the forest areas giving
Explanation
importance to trees with multiple uses.
 Statement 1 is incorrect: The Van Dhan
 MoEFCC has not issued any specific
Scheme is an initiative of the Ministry of
direction for plantation of fruit bearing
Tribal Affairs and TRIFED.
trees as it is decided by the JFM Committees
considering local conditions and the micro Supplementary notes
plan of the area.
 Prime Minister Van DhanYojana
 The overall objective of the National
Afforestation Programme (NAP) scheme  TRIFED under Ministry of Tribal Affairs is
is ecological restoration of degraded going to forge a partnership with RituBeri
forests and to develop the forest resources Designs to promote Indian tribal crafts and
with peoples’ participation, with focus on culture across the country and the world.
TARGET PT 2020 85

 Ms. Beri will be the Chief Design Consultant  No special structure is there to look after the
for Tribes India. quality of food served, hygienic conditions
and other aspects of the scheme.
 PradhanMantri Van DhanYojana (PMVDY)
is a Market Linked Tribal Entrepreneurship Corrupt practices
Development Program for forming clusters
of tribal SHGs and strengthening them  Rampant corrupt practices have been found
into Tribal Producer Companies has been from food procurement to distribution of
hot cooked meals among children.
launched with participation from all the 27
States from the Country. ! A video showed plain chapatis being
served with salt in a school.
 The Van Dhan Scheme is an initiative of the
Ministry of Tribal Affairs and TRIFED. ! Another video revealed how a litre of
milk was mixed with water so that
 It was launched in 2018 and seeks to
it would suffice for the more than 80
improve tribal incomes through value
children present that day in school.
addition of tribal products.
! As per the government norms, every
 The scheme will be implemented through child is entitled to receive 150 ml of
Ministry of Tribal Affairs as Nodal milk as part of the mid-day meal.
Department at the Central Level and
TRIFED as Nodal Agency at the National Social Discrimination
Level.
 Caste bias and discrimination continues in
 At State level, the State Nodal Agency the implementation of the scheme
for MFPs and the District collectors are
 It has been found in many schools that
envisaged to play a pivot role in scheme
foods cooked by lower caste cooks were not
implementation at grassroot level. eaten by children.
 Locally the Kendras are proposed to be  Food is central to the caste system, so in
managed by a Managing Committee (an many schools, children are made to sit
SHG) consisting of representatives of Van separately according to their caste status.
Dhan SHGs in the cluster.
Unhygienic issue
22. Correct Answer: (b)
 There have been multiple incidents of
 Explanation: 4th statement is incorrect: Mid unhygienic mid-day meals.
day meal scheme has increased enrollment
 Dead rats, worms, lizards and insects have
in schools, increased attendance in schools,
been spotted in mid-day meals served at
improved performance of students in class
schools across India.
in terms of better attention span and
academic progress.  Recently, the CAG found that schools in
Punjab were noted to have no water testing,
Supplementary Notes no use of gloves, and violation of prescribed
menus.
 Issues with Mid-Day Meal Scheme
 Children were made to wash utensils,
 Aim: The program aims to “enhance
despite the school having hired staff to do
enrollment, retention and attendance
so
and simultaneously improve nutritional
levels among children.” The program has Fortification Issue
the potential to end chronic malnutrition
and starvation. However, the program has  Sub-standard food is served in many schools
many issues which the government must because there is not enough monitoring and
accountability at the lower levels
immediately address.
 85% of Indian children between the ages of
Organizational problems: 7 to 12 have high rates of micronutrients
 There are too many layers of government deficiencies (in iron, folic acid, Vitamin A).
involved in the scheme, resulting in poor These can impair cognitive developments;
information, coordination and monitoring. impair concentration, cause school
absenteeism and even illness. As successful
 A few examples of poor monitoring and pilot projects in Odisha have shown, staple
coordination show the issues this scheme essentials of Mid-Day meals like rice and
encounters. In July, 2013, 23 children from wheat like can fortified with micronutrients.
the Indian state of Bihar died as a result of Micronutrient premixes can be added to
unsafe food preparation. cooked meals. These have shown success in
86 TARGET PT 2020

reducing anaemia.
 For Primary students:
 However, fortification has only made ! Calories 450
its debut in some states like Karnataka ! Protein 12 gms
instead of a pan-India rollout.  For Upper Primary students:
! Calories 700
Irregular Food Supplies
! Protein 20 gms
 Constant food and fuel supplies are not
 Adequate quantities of micro-nutrients
provided to the schools like Iron, Folic Acid and Vitamin-A.

 It is also a difficult challenge to deliver food  Teachers face a number of problems such
to rural areas. Roads are not paved and as problem of management of the mid-
day meal, wastage of food by the students,
the infrastructure is lacking. Even if food
insufficient and delayed receipt of funds,
makes it to these remote areas, kitchens increased workload of teachers, procuring
to cook the food in are not available within dry ration from the retail shops, lack of
infrastructure for storage, cooking and
the schools. serving food, lack of safety provisions,
unhygienic surroundings, etc.
Problems faced by Teachers
Lack of Documentation
 Teachers play a key role in the successful
 A record of what students eat is not
implementation of mid-day meal scheme at maintained.
school level.  In some regions, one can see the daily menu
painted on the school wall.
Mid-Day Meal Scheme - Facts  Writing letters to authorities and
documenting the gap between the painted
 Scheme covers all children studying in
menu and what is actually served might be
class I to VIII.
a great activity.
 The programme supplies free lunches on
working days for children in primary and Other issues
upper primary classes in government,
 Other issues range from delayed payments,
government aided, local body, Education
poor food quality, cooks not receiving
Guarantee Scheme, and alternate
innovative education centres, Madarsa pay and food being wasted. There is even
and Maqtabs supported under SSA and embezzlement of the money by way of fake
National Child Labour Project schools enrollments.
run by the ministry of labour.  The scheme is perceived as charity, not a
 MDM is covered by National Food civic responsibility. With the growing shift
Security Act, 2013. of the better-off parents to private schools,
government schools are viewed as places
 Ministry/Department : Department of
for the poor. Therefore, the mid-day meal
School Education & Literacy, Ministry of
is associated with poverty both in public
Human Resource Development
perception and state policies.
 Objective: To enhance, retention and
attendance and simultaneously improving Achievements of Mid-day Meal Scheme
nutritional levels among children.
 Increased Enrollment and Attendance:
Factual Information: The meal scheme is generally a successful
one as it has led to “increased enrollment
 Started in 1995 as National Programme in schools, increased attendance in schools,
of Nutritional Support to Primary
improved performance of students in class
Education
in terms of better attention span and
 To achieve the above objectives, a academic progress.
cooked mid-day meal with the following
nutritional content is provided to all  Improved nutritional status of stu-
eligible children. dents: Studies have found that the
schemereduced the daily protein deficiency
TARGET PT 2020 87

of a primary-school student by 100 percent,  Disregard for ocean health: Overfishing,


the calorie deficiency by almost 30 percent plastic pollution, micro-plastics, flow of
and the daily iron deficiency by nearly 10 fertilisers and chemicals etc. is suffocating
percent. fish and damaging ocean health. More than
1 billion people depend on the oceans. Ocean
 There is a dramatic fall in underweight health is vital to biodiversity, healthy
and stunted children (from 48 % to 39 % fisheries and to regulate the climate.
(2005-6 and 2013-14).
24. Correct Option (c)
23. Correct Option (d)
Explanation: Statement 1 and 3 are
Explanation: All statements are correct correct
Supplementary Notes
 Statement 2 is incorrect: This launch carried
 Co2 emission on rise: According to the customer satellites of United States, Japan,
Global Carbon Project, CO2 emissions are Italy, and Israel. ‘Italy’ not ‘France’.
on rise by 0.6% in 2019 (2.1% in 2018). The
reductions are not enough to stop global Supplementary Notes
warming. Despite a significant decline in  IRISAT-2BR1 is a synthetic-aperture
coal consumption in US and Europe, the radar (SAR) imaging satellite for
higher global emissions are attributed to reconnaissance built by Indian Space
growth in natural gas and oil usage. Research Organisation(ISRO).
 Industrial heat: Industrial products are  It is part of India’s RISAT series of SAR
essential to construction, infrastructure imaging space crafts and 4th satellite in
and manufacturing, but making them re- the series.
quires a lot of heat—heat that emits more  RISAT-2BR1 was launched in December
carbon dioxide than all the world’s cars and 2019 aboard PSLV-C48 from First Launch
planes. Many industrial processes start Pad of SatishDhawan Space Centre.
with melting rocks by burning fossil fuels,
 It was the 50th launch of Polar Satellite
and development of alternative technolo-
Launch Vehicle (PSLV) and 75th launch
gies is far behind and expensive.
from SatishDhawan Space Centre.
 Land degradation: Land degradation,  RISAT-2BR1 usage: Apart from being used
mainly due to human activities like defor- for military purposes, RISAT-2BR1 has
estation, mining/quarrying, construction, applications in fields such as agriculture
roads, other infrastructure for economic and disaster management support.
development, human settlements for in-
creasing population, etc., is a contributing  RISAT-2BR1 was launched with nine other
factor to climate change. Even agriculture ride-sharing commercial satellites.
and related activities are degrading land,  Nine commercial ridesharing satellites
including groundwater resources. weighed 157.6 kg cumulatively. These
customer satellites included six from
 Meat consumption: IPCC report ‘Climate
United States, and one each from Japan,
Change and Land’ emphasises the ever- Italy and Israel.
increasing global meat consumption and
the resulting distorted land-use pattern to  RISAT 2B, RISAT 2BR1 and RISAT 2BR2
meet this requirement, as a cause contrib- (Radar Imaging Satellite) are satellite
uting to climate change. The EAT-Lancet imaging missions of ISRO using an
Commission report also supports this; it active SAR (Synthetic Aperture Radar)
imager to provide continuity of service for
adds biodiversity loss, natural water deple-
RISAT-2.
tion and carbon emission to the associated
risks.  Objective of the RISAT mission is to
use the all-weather and day-and-night
! Creating pastures to feed cattle caus- SAR observation capability in applications
es huge deforestation. Processing, such as agriculture, forestry, soil moisture,
preservation and packaging of cattle geology, sea ice, coastal monitoring, object
slaughtered is also highly GHG-gen- identification, and flood monitoring, and
eration intensive process. also for military surveillance.
! Cattle itself is responsible for produc-  Other significant PSLV launches:
ing high quantities of methane, which Chandrayaan-1, the Mars Orbiter Mission
has a far greater carbon footprint com- (MOM) and a record launch of 104 satellites
pared to carbon dioxide. in one go.
88 TARGET PT 2020

25. Correct Option (c) who was a campaigned towards creating


awareness about education in Kerala.
Explanation: Statement 3 is correct
 His discovery of two flat-bodied spiders
 Statements 1 and 2 are incorrect: Marengo has been included in World Spider
sachintendulkar is a flat-bodied spider. Catalogue also.
Supplementary Notes  The two new species belong to Indomarengo
 DhruvPrajapati, a junior researcher genus and Marengo genus of Asian jumping
pursuing a PhD in spider taxonomy with spiders.
Gujarat Ecological Education &  Marengo sachintendulkar: It was found in
Research (GEER) Foundation, recently Kerala, Tamil Nadu, and Gujarat.
discovered two new species of spiders.
 IndomarengoChavarapatera: It was found
 He has named one of the species as in Kerala.
Marengo sachintendulkar after Indian
crickterSachin Tendulkar and other as  There are nearly 48,000 types of spiders
IndomarengoChavarapatera which is found worldwide, spread across 120
inspired by Saint Kuriakose Elias Chavara families.

**********
TARGET PT 2020 89

TEST
DAY - 37

Time Allowed: 30 mins Maximum Marks: 50

1. Which of the following are not the 4. Which of the correct statements
secondary pollutant? regarding Zero Liquid Discharge is/
1. SO2 are correct?
2. Ozone 1. It completely eliminates liquid
discharge from the water system.
3. NOx
4. PANs 2. Removal of organic and suspended
dissolved solids is not a prerequisite
Select the correct option using the codes for this treatment of water.
given below: 3. Only a small amount of energy is
(a) 1 and 2 only needed for this process.
(b) 1 and 3 only Select the correct option using the codes
(c) 2 and 4 only given below:
(d) 1, 2, and 3 only (a) 1 only

2. Which of the following chemicals are (b) 2 and 3 only


primarily present in the fly ash? (c) 1 and 3 only
(a) SiO2 (d) 1, 2, and 3
(b) CaO
5. Regarding the controlling measures of
(c) Al2O3 water pollution, consider the following
(d) All of the above statements:
1. Arsenic contamination can be removed
3. Which of the following statements is/ by Coagulation.
are incorrect?
2. To make the quality of water better,
1. The Comprehensive Environmental
fluorides are either added or removed.
Pollution Index has been developed
by the CPCB for the environmental 3. Colors of the water can be removed
assessment of Industrial Clusters only by reduction.
only.
Which of the above statements is/are
2. The areas where the pollution level
correct?
is more than 70%, are categorized as
severely polluted areas. (a) 3 only

Select the correct option using the codes (b) 1 and 2 only
given below: (c) 2 and 3 only
(a) 1 only (d) 1, 2, and 3
(b) 2 only
6. Which of the following statements
(c) Both 1 and 2
regarding water pollution is/are
(d) Neither 1 nor 2 correct?
90 TARGET PT 2020

1. Domestic waste and sewage is the 9. Which of the following is/are included
biggest polluter of water sources in under the Bio-medical wastes?
India. 1. Soiled wastes
2. Thermal pollution is one of the causes 2. Anatomical wastes
of water pollution.
3. Waste sharps
Select the correct option using the codes 4. Discarded medicines
given below:
Select the correct option using the codes
(a) 1 only given below:
(b) 2 only (a) 1, 2, 3, and 4
(c) Both 1 and 2 (b) 2, 3 and 4 only
(d) Neither 1 nor 2 (c) 2 and 4 only

7. Which of the following are the effects (d) 1 and 3 only


of tropospheric ozone?
10. With reference to the new technologies
1. It interferes with photosynthesis. for the treatment of bio-medical waste,
2. It assists in the formation of consider the following statements:
Peroxyacetylnitrate (PAN). 1. In Plasma Pyrolysis, bio-medical waste
3. It causes bronchitis. is treated at high temperatures.
4. It is toxic for soft tissues and bones. 2. The disposal of the treated wastes is
not needed in the Plasma Pyrolysis
Select the correct option using the codes technology.
given below:
3. Waste Sharp Dry Heat Sterilization
(a) 2 only & Encapsulation’ technology is also
called autoclaving.
(b) 2 and 4 only
4. Shredding cum Chemical Disinfection
(c) 1, 2, and 3 only
is a non-burn technology.
(d) 1, 2, 3, and 4
Which of the above statements is/are
8. “Noise pollution is an unpleasant noise incorrect?
created by people or machines that can (a) 1 and 3 only
be annoying, distracting, intrusive,
and/or physically painful”. Which of (b) 2 and 4 only
the following is/are not the effects of (c) 1 and 4 only
noise pollution?
(d) 2, 3 and 4 only
1. Excessive adrenalin in the
bloodstream 11. Identify the incorrect statement with
respect to Biological Oxygen Demand
2. Birth defects
(BOD):
3. Digestive irregularities
(a) It is the measure of oxygen equivalent
4. Decrease in the rate of color of the requirement of oxidation of total
perception organic matter present in water.
Select the correct option using the codes (b) BOD is limited to biodegradable
given below: materials only.
(a) 1 only (c) Chemical oxygen demand (COD) is a
better mode to measure pollution load
(b) 3 only
in water than BOD.
(c) 2 and 4 only
(d) All of the above statements are
(d) None of the above correct.
TARGET PT 2020 91

12. What do you understand by Radioactive Which of the above statements is/are correct?
Pollution?
(a) 1 only
(a) It is a phenomenon of spontaneous
emission of short wave electromagnetic (b) 2 only
waves only. (c) Both 1 and 2
(b) It is a phenomenon of spontaneous (d) Neither 1 nor 2
emission of proton, electrons and
gamma rays only. 16. Which of the following statements is /
(c) It is a phenomenon of spontaneous are correct about the point source of
emission of alpha and beta particles water pollution?
only.
1. Pollutant travels directly from source
(d) It is a phenomenon of spontaneous to water.
emission of alpha and beta particles
2. These sources are difficult to regulate.
and cosmic rays.
Select the correct answer using code given
13. Identify an incorrect pair regarding
the pollutants and their respective below:
components: (a) 1 only
(a) Gaseous pollutants: Oxides of carbon (b) 2 only
(b) Particulate matter: Radioactive
(c) Both 1 and 2
substances
(d) Neither 1 nor 2
(c) Burning of plastics: Poisonous gases
like phosgene (COCl2)
17. Which of the following statements
(d) All of the above correctly defines the Putrescibility?

14. Consider the following statements (a) It is the process of decomposition of


regarding Environmental Pollutants: organic matter in water.
1. Peroxyacetyl nitrate (PAN), (b) It is a process in which hemoglobin reacts
Dichlorodiphenyltrichloroethane with non-functional methaemoglobin
(DDT) and Plastic are examples of and impairs oxygen transport.
secondary pollutants.
(c) It is a process in which water is purified
2. Qualitative Pollutants are man-made.
by extracting toxic materials and heavy
3. Glass, salts of heavy metals and metals.
radioactive substances are examples of
Biodegradable Pollutants. (d) It is a process of treatment of sewage
water.
Which of the above statements is/are
correct? 18. With reference to the effects of water
(a) 1 and 2 only pollution, which of the following
statements is incorrect?
(b) 2 only
(c) 1 and 3 only (a) Water contaminated with cadmium
can cause itai-itai.
(d) 1, 2 and 3
(b) Excess fluoride in drinking water
15. Consider the following statements causes skeletal fluorosis.
about the Photochemical Smog:
(c) Hot waters discharged from industries
1. It is a term used to describe air pollution
when added to water bodies, increases
that is a result of the interaction of
its Dissolved Oxygen (DO) content.
sunlight with certain chemicals in the
atmosphere. (d) The presence of excess nitrate in
2. During the winter it leads to a decrease drinking water causes blue-baby
in pollution levels. syndrome.
92 TARGET PT 2020

19. Which among the following pair is 1. It recommended to Increase budgetary


incorrectly matched? allocations for family planning
(a) Phytoextraction: Use of microorganisms 2. Treating population stabilisation and
to degrade the environmental family planning as a national priority
contaminants into less toxic forms. is one of its outcome
(b) Phytodegradation: Uptake of organic 3. India currently has the second largest
contaminants from soil and their population in the world.
transformation to less toxic forms.
Which of the following statements is/are
(c) Phytostabilization: It is a technique in
correct?
which plants reduce the mobility and
migration of contaminated soil. (a) 1 and 2 only
(d) Rhizodegradation: It is the breakdown (b) 2 only
of contaminants through the activity
(c) 3 only
existing in the rhizosphere.
(d) 1, 2 and 3
20. Which of the following are man-made
sources of radioactive pollution?
23. Recently, many developments have
1. Terrestrial radiations from radio-
been made in the field of bio-printing.
nuclides present in earth’s crust
Which of the following is not a
2. Uranium mining limitation of 3D bio-printing?
3. Radiation therapy 1. The abundant and branched nature
4. Transportation of nuclear material of the vasculature is difficult to
reproduce
Select the correct answer using the code
given below: 2. Risk of immune rejection
3. Accurately printing tissues on the
(a) 1 and 2 only
micro-scale
(b) 1, 2 and 3 only
Choose the correct answer:
(c) 2 and 4 only
(a) 1 only
(d) 2, 3 and 4 only
(b) 1 and 2 only
21. Recently Breast Milk Banks are been
established in the country, consider (c) 3 only
the following statements regarding it (d) 1, 2 and 3
1. They are been established under
Ministry of women and child 24. Match the following pairs correctly:
development.
1. Hydrogen A. They use energy
2. Breast Milk Contains Important based cars stored in batteries
Antibodies. to power one or
3. Breastfeeding May Prevent more electric
Menstruation. motors.

Which of the following statements is/are 2. Electric cars B. They burn fuel
correct? in an internal
(a) 1 and 2 only combustion
engine.
(b) 2 and 3 only
3. Conventional C. They create the
(c) 3 only
Cars electricity in an
(d) 1, 2 and 3 onboard fuel cell,
usually using
22. NITI Aayog recently drafted a roadmap oxygen from the
for achieving population stabilisation, air and stored
with reference to that consider the
hydrogen.
following statements
TARGET PT 2020 93

(a) 1–A, 2-B, 3-C core components and materials to


(b) 1-C, 2-A, 3-B increase the domestic value addition
(c) 1-B, 2-C, 3-A and reduce dependence on import of
(d) 1-C, 2-B, 3-A electronic goods by focusing on scale,
skill and technology.
25. Which of the following is/are the
objectives of National Electronics Which of the following statement(s) given
Policy (NEP), 2019? above is/are correct?
1. To position India as a global hub
for Electronics System Design and (a) 1 only
Manufacturing (ESDM) by creating an
enabling environment for the industry (b) 2 only
to compete globally.
(c) Both 1 and 2
2. Promote domestic manufacturing in the
entire value-chain of ESDM, including (d) Neither one nor two

**********
94 TARGET PT 2020

ANSWER HINTS
DAY - 37

1. Correct Option: (b)  Magnesium, potassium, sodium, titanium,


and sulfur are also present to a lesser
Explanation: degree.
Primary pollutant
3. Correct Option: (b)
 A primary pollutant is a pollutant
emitted directly from a source. It persists Explanation:
in the form in which they are added to the
Comprehensive Environmental Pollution
environment. Example-CO, CO2, SO2,
Index (CEPI)
Oxide of nitrogen (NOx), Particulate
matter, etc.  The Central Pollution Control Board
(CPCB) has developed a Comprehensive
Environmental Pollution Index
(CEPI). CPCB has done a nationwide
environmental assessment of
Industrial Clusters based on CEPI.
 The index captures the various health
dimensions of the environment including
air, water and land.
 Out of identified 88 prominent industrial
clusters, 43 industrial clusters in 16 States
having a CEPI score of 70 and above
were identified as Critically Polluted
areas, while industrial clusters with CEPI
 Secondary Pollutants are formed by
interaction among the primary pollutants. scores between 60 & 70 are categorized
They are more toxic. Example- Smog, as severely polluted areas.
Tropospheric Ozone, Peroxy Acetyl
Nitrates, etc. 4. Correct Option: (a)
Explanation:
2. Correct Option: (d)
Zero Liquid Discharge (ZLD)
Explanation:
 Zero liquid discharge (ZLD) is a
Fly Ash wastewater treatment that completely
 Fly ash is the finely divided residue that eliminates liquid discharge from a
results from the combustion of pulverized system. ZLD represents the ultimate
coal and is transported from the combustion cutting-edge treatment system for the total
chamber by exhaust gases. elimination of wastewater effluent into
neighboring waterways. The goal of any
 It is produced from the combustion of coal well-designed ZLD system is to minimize
in electric utility or industrial boilers, the volume of wastewater that requires
produced by coal-fired electric and steam
further treatment, process wastewater in
generating plants.
an economically feasible manner, while
 It consists primarily of oxides of also producing a clean stream suitable for
silicon, aluminum, iron, and calcium. re-use elsewhere in the facility.
TARGET PT 2020 95

 The first step to achieve ZLD is to look for Miscellaneous treatments


ways to limit the amount of wastewater
that needs to be treated. The equipments Removal of colors, odors, and taste from
needed to achieve ZLD varies depending water
on the characteristics of the wastewater  The special treatments for the removal of
as well as the wastewater volume. colors, odors, and tastes are
Typical waste streams in an industrial
setting include wastewater rejects ! Aeration – application of oxygen (O2)
typically from reverse osmosis (RO) or ion ! Activated carbon treatment (reduction)
exchange, cooling tower blowdown, spent – Specially treated carbon which
coolants, DI regenerant, metal finishing possesses the property of absorbing
wastewaters, tank or equipment washing and attracting impurities, such as
wastewaters, and other miscellaneous gases, liquids, and finely dissolved
industrial wastewaters such as compressor solids.
condensate and floor scrubber wash
waters. A traditional approach to ZLD is ! Treatment with copper sulfate
to use some sort of filtration technology, (CuSO4·7H2O) – It helps in removing
funnel the reject waters to an evaporator, colors, tastes, and odors from water.
and send the evaporator concentrate to a ! Treatment with oxidizing agents –
crystallizer or spray dryer. Oxidizing agents used are KMnO4,
Cl2, O3, etc.
Advantages:
Removal of salt and dissolved solids from
 Better management of wastewater.
water i.e. Desalination
 Less environmental pollution.
 The process of removing the salt content
 Treatment and recovery of valuable is known as desalination. The various
products from waste streams. methods used are
 It avoids wastage and spurs recycling ! By evaporation and distillation
by conventional and far less expensive
! Electrodialysis method
solutions.
! Reverse osmosis method
Disadvantages:
! Freezing process
 Removal of organic and suspended
dissolved solids is a prerequisite. ! Solar distillation method

 High amount of energy utilization for ! Other methods


the evaporation process.
Removal of iron (Fe) and manganese (Mn)
 Management of solid waste needs from water
consideration due to its potentially
 The Fe and Mn may be present in water
hazardous nature. either in combination with organic matter
 Capital cost is high. or without such combination. When present
without combination with organic matter,
5. Correct Option: (b) they can be easily removed by aeration,
followed by coagulation, sedimentation,
Explanation: and filtration. On the other hand, when Fe
and Mn are present in combination with
Controlling measures of water pollution
organic matter, removal becomes difficult.
 The various methods or the techniques In such cases, the addition of lime, Cl2, or
which may be adopted for purifying the KMnO4 is useful.
public water supplies are:
Addition to and removal of fluorides from
! Screening water
! Plain sedimentation  Fluoride (F¯) content in water should
! Sedimentation aided with coagulation be about 1 mg/L. To ensure this,
fluorides are either added in water
! Filtration (i.e. Fluoridation) or removed from
! Disinfection this (i.e. Defluoridation).

! Softening Arsenic contamination and its removal


! Miscellaneous treatments.  The maximum permissible limit of
96 TARGET PT 2020

Arsenic (As) specified by the World Health will double the rate of many chemical
Organization (WHO) as well as the Bureau reactions and so the decay of the organic
of Indian Standards (BIS) is 0.01 mg/L (10 matter, rusting of iron, and the solution
ppb). If there is excess concentration of As rate of salts are also accelerated by
in water, then it needs to be removed using calefaction. All organisms have a range
any one of the following methods: of temperature tolerance beyond which
! Coagulation – precipitation they either die or move to more congenial
technique by using aluminum and conditions downstream.
ferric salts
! Adsorption technique by using
activated alumina or ion exchange
resins
! Membrane technology like reverse
osmosis and electrodialysis

6. Correct Option: (c)


Explanation:
Water pollution
 Domestic waste and sewage is the biggest
polluter of surface and groundwater
sources in India. This is due to the big 7. Correct Option: (c)
lacuna between the amount of sewage
Explanation:
generation and the facilities to dispose
it off. The problem is not only the lack of
Effects of air pollutants
facilities; rather, the non-functioning of
existing facilities/treatment plants is more  The effects of various pollutants can be
critical. understood in terms of health aspects and
environmental aspects. The effects are
summarized in the following table:
Pollutants Major effects
Health effects Environmental
effects
Sulfur oxides Respiratory Acid rain
(SOx) problems,
Heart and
lung disorders,
Visual
impairment
Nitrogen Pulmonary Precursor of ozone
oxides (NOx) disorders formation in the
Thermal pollution increased troposphere, Aerosol
 Temperature above the normal range is susceptibility formation
to respiratory
called thermal pollution or Calefaction.
infections
Thermal pollution occurs as a result of the
entry of heated water from industries and Particulate Respiratory Visibility reduction
matter (PM) problems, liver
power generation plants. Various processes fibrosis, lung/
involved in generating thermal pollution liver cancer,
are Heart stroke,
Bone problems
! Water for cooling condensers
Carbon Anoxemia -
! Feeding boilers for steam generation monoxide (CO) leading to
various
! Auxillary plant cooling cardiovascular
problems.
! Ash handling Infants,
! Gas washing, etc. pregnant
women, and
 The immediate effect of an increase in elderly people
temperature is a decrease in the oxygen are at higher
risk
concentration. A temperature rise of 10°C
TARGET PT 2020 97

Ozone (O3) Respiratory O3 in upper which is responsible for high blood


problems, troposphere causes pressure,
Asthma, greenhouse effects,
bronchitis, etc. Harmful effects on ! Blaring sounds are known to cause
plants as it interferes mental distress,
in photosynthesis and
results in the death of
! Heart attacks, neurological problems,
plant tissues since it birth defects, and abortion.
assists in the formation
of Peroxyacety-
 Muscle contraction leading to a nervous
lnitrate (PAN) breakdown, tension, etc.
Lead (Pb) Serious effects -  The adverse reactions are coupled with a
on the central change in hormone content of blood, which
nervous in-turn increases heartbeat, constriction
system since
it is absorbed
of blood vessels, digestive spasms, and
rapidly in dilation of the pupil of the eye.
bloodstream,
Anemia,
 Adversely affects health, work efficiency,
toxic for soft and behavior. Noise pollution may cause
tissues and damage to the heart, brain, kidneys, liver,
bones and may produce emotional disturbance.
Ammonia Immediate -  The most immediate and acute effect of
(NH3) effects lead to
noise is the impairment of hearing that
the burning
of eyes, nose, diminishes some part of the auditory
throat, and system. The eardrum is damaged when
respiratory exposed to very loud and sudden noises. The
tract. hair cells in the inner ear are chronically
Prolonged damaged. Prolonged exposure to noise of
effects result
in blindness, certain frequency patterns leads to chronic
lung damage, damage to the inner ear and leads to
or death. hearing loss.
 Impulsive noise may cause psychological
8. Correct Option: (d) and pathological disorders.
Explanation:  Ultrasonic sound can affect the
digestive, respiratory, cardiovascular
Effects of noise pollution system, and semi-circular canals of
 Problems caused by noise pollution include the internal ear.
stress-related illnesses, speech interference,  The brain is adversely affected by loud and
hearing loss, sleep disruption, and lost sudden noise by jets and airplanes. People
productivity. The hazards are summarized are subjected to psychiatric illness.
below:
 Recent reports suggest that blood is
thickened by excessive noise. Eosinophilia,
Effects of hyperglycemia, hypokalaemia, and
Noise Pollution
hypoglycemia are caused by alteration in
the blood due to noise.
 The optical system of human beings is also
Auditory Effetcs Non Auditory affected by noise pollution. Severe noise
Effetcs
pollution causes:
Annoyance Physical ! Pupillary dilation
disorder like
Loss of increase in ! Impairment of night vision and
Auditory
Deafness working heart beats,
fatigue Interference
efficiency B.P. etc. ! Decrease in the rate of color
in speech
communication perception.

 Noise pollution affects both human and 9. Correct Option: (a)


animal health. It leads to: Explanation:
! contraction of blood vessels,
Bio-medical waste
! making skin pale,
 According to Biomedical Waste
! Excessive adrenalin in the bloodstream (Management and Handling) Rules, 1998 of
98 TARGET PT 2020

India “Any waste which is generated during at high temperatures under controlled
the diagnosis, treatment or immunization conditions to form gases like methane,
of human beings or animals or in research hydrogen and carbon monoxide which are
activities pertaining thereto or in the subjected to combustion (oxidation) in the
production or testing of biological”. secondary chamber.
 The bio-medical waste consists of  In this process, waste is converted into
small clinker which can be disposed of
! Human anatomical waste like
in secured landfills.
tissues, organs and body parts
! Animal wastes generated during
research from veterinary hospitals
! Microbiology and biotechnology
wastes from laboratory, culture stocks
or specimens of micro-organisms, live
or attenuated vaccines, human and
animal cell culture used in research
! Waste sharps like hypodermic
needles, syringes, scalpels, and
broken glass
! Discarded medicines and
cytotoxic drugs comprising of Waste Sharp Dry Heat Sterilization &
outdated, contaminated and discarded Encapsulation’ technology
medicines
 It is based on ‘dry heat sterilization’,
! Soiled waste such as cotton balls contrary to the autoclaving where the
used while blood collection or used high-pressure steam is used, especially
as absorbent material for accidental for the treatment of waste category no. 04
blood and body fluid spillage (i.e. waste sharps) as listed under Schedule-I
! Solid waste such as dressing, bandages, of the BMW Rules, 1998. Approval to this
plaster casts, material contaminated technology is accorded by CPCB under
with blood, tubes, and catheters the Bio-medical Waste (Management &
Handling) Rules, 1998 as amended subject
! Liquid waste from any of the infected to the conditions. Dry Heat Sterilization &
areas Encapsulation of Waste Sharps treatment
! Incineration ash from incineration of equipment and the canister before used
any bio-medical waste for the collection of waste sharps and the
canister after treatment.
! Chemical wastes generated while
testing and analysis. Shredding cum Chemical Disinfection of
Bio-medical Waste
10. Correct Option: (a)
 This technology is based on ‘shredding
Explanation: followed by chemical disinfection’ of
biomedical waste. Provisional approval
New technologies for the treatment of bio- of this technology is accorded by CPCB
medical waste for the treatment of bio-medical waste on
 CPCB has granted conditional or a trial basis for the evaluation / efficacy
provisional approval to new Technologies of the technology. This is a non-burn
(other than notified under BMW Rules) for technology, in which bio-medical
treatment of bio-medical waste, under the waste is shredded and sterilized so as
BMW Rules as under: to make it suitable for disposal along with
municipal solid waste.
! Plasma Pyrolysis;
! Waste Sharps Dry heat sterilization 11. Correct Option: (a)
and encapsulation
Explanation:
! Shredding cum Chemical disinfection
(Static/Mobile) Biological Oxygen Demand (BOD)
 Water pollution by organic wastes
Plasma Pyrolysis Technology
is measured in terms of Biochemical
 In this, bio-medical waste is treated Oxygen Demand (BOD).
TARGET PT 2020 99

 Biological Oxygen Demand is the amount water with nutrients particularly


of dissolved oxygen needed by bacteria in nitrates and phosphates triggers the
decomposing the organic wastes present in explosive growth of green algae) that
water. may take place in water bodies and
kill aquatic life.
 It is expressed in milligrams of oxygen
per litre of water.  The use of pesticides not only kills
pests that destroy crops but may also
 The higher value of BOD indicates low
kill many non-pest organisms which
Dissolved Oxygen content of water.
may include even useful species of
 BOD is limited to biodegradable materials insects such as pollinators, birds, and
only, hence it is not considered a much helpers in the dispersal of plant seeds.
reliable method of measuring pollution Pesticides tend to accumulate and
load in the water. their concentration increases through
 Chemical oxygen demand (COD) is a the food chain and reaches toxic levels
slightly better mode used to measure in eggs, milk and other food items
pollution load in the water. COD is the (biomagnification).
measure of oxygen equivalent to the  All these industries discharge several
requirement of oxidation of total waste gases and particulate pollutants
organic matter (i.e. biodegradable and into the atmosphere. Some of them are
non-biodegradable) present in water. as follows:-

12. Correct Option: (b) ! Gaseous pollutants: Oxides of carbon,


nitrogen, and sulphur.
Explanation:
! Particulate matter: Fine metal dust, fly
Radioactive Pollution ash, soot, cotton dust, and radioactive
substances.
 Radioactivity is a phenomenon of
spontaneous emission of proton ! Burning of plastics: Emit
(a-particles), electrons (ß-particles) and polychlorinated biphenyls (PCBs)
gamma rays (short wave electromagnetic which are harmful to lungs and
waves) due to the disintegration of vision.
atomic nuclei of some elements. These ! Accidental release of some poisonous
cause radioactive pollution. gases like phosgene (COCl2) and
 Alpha particles can be blocked by a methyl isocyanate (as it happened in
piece of paper and human skin. Bhopal) is fatal.

 Beta particles can penetrate through ! Secondary air pollutants formed


the skin, while can be blocked by some from complex reactions between
pieces of glass and metal. primary pollutants, such as smog
and acid rain, which are harmful to
 Gamma rays can penetrate easily to all living organisms, buildings, and
human skin and damage cells on its monuments.
way through, reaching far, and can
only be blocked by a very thick, strong,
14. Correct Option: (b)
massive piece of concrete.
Explanation:
13. Correct Option: (c)
Classifications of Pollutants
Explanation:
 According to the form in which they persist
Pollution by agrochemicals after release into the environment:
 Increased use of synthetic fertilizers ! Primary Pollutants: These persist
causes serious environmental in the form in which they are
problems. For example, unused added to the environment e.g.
fertilizers from agricultural fields are DDT, plastic.
carried away by runoff waters into
! Secondary Pollutants: These
lakes and rivers causing pollution.
are formed by interaction
 These agrochemicals may even among the primary pollutants.
seep through the soil and pollute For example, peroxyacetyl
groundwater. Excessive nutrients nitrate (PAN) is formed by the
enrichment of water bodies leads to interaction of nitrogen oxides and
‘eutrophication’ (i.e. enrichment of hydrocarbons.
100 TARGET PT 2020

 According to their existence in nature: Point Sources


! Quantitative Pollutants: These occur  It is directly attributable to one
in nature and become pollutants when influence.
their concentration reaches beyond a  Here pollutant travels directly from source
threshold level. E.g. carbon dioxide, to water.
nitrogen oxide.
 Point sources are easy to regulate.
! Qualitative Pollutants: These do
not occur in nature and are man- 17. Correct Option: (a)
made. E.g. fungicides, herbicides,
DDT, etc. Explanation:
 According to their nature of disposal: Putrescibility
! Biodegradable Pollutants: Waste  It is the process of decomposition
products, which are degraded by of organic matter present in water by
microbial action. E.g. sewage. microorganisms using oxygen.
! Non-biodegradable Pollutants:
18. Correct Option: (c)
Pollutants, which are not
decomposed by microbial action. Explanation:
E.g. plastics, glass, DDT, salts
of heavy metals, radioactive Effects of Water Pollution
substances, etc.  Effects on aquatic ecosystem
 According to their origin: ! Polluted water reduces Dissolved
! Natural: E.g. carbon dioxide, nitrogen Oxygen (DO) content, thereby,
oxide eliminates sensitive organisms
like plankton, molluscs, and fish,
! Anthropogenic: E.g. fungicides, etc.
herbicides, DDT, etc.
! Biocides, polychlorinated
biphenyls (PCBs) and heavy
15. Correct Option: (a)
metals directly eliminate sensitive
Explanation: aquatic organisms.
! Hot waters discharged from industries,
Photochemical Smog
when added to water bodies, lowers its
! Photochemical smog (smog) is a DO content.
term used to describe air pollution
 Effects on human health:
that is a result of the interaction of
sunlight with certain chemicals in the ! The polluted water usually
atmosphere. contains pathogens like viruses,
bacteria, parasitic protozoa,
! Its occurrences are often linked to and worms, therefore, it is a
heavy traffic, high temperatures, source of water-borne diseases
and calm winds. like jaundice, cholera, typhoid,
! During the winter, wind speeds are amoebiasis, etc.
low and cause the smoke and fog ! Mercury compounds in
to stagnate near the ground; hence wastewater are converted by
pollution levels can increase near bacterial action into extremely
ground level. toxic methyl mercury, which can
cause numbness of limbs, lips
! Smoke particles trapped in the
and tongue, deafness, blurring of
fog gives it a yellow/ black color
vision and mental derangement.
and this smog often settled over
cities for many days. ! Water contaminated with cadmium
can cause itai itai disease also called
16. Correct Option: (a) ouch-ouch disease (a painful disease
of bones and joints) and cancer of
Explanation: lungs and liver.
Water Pollution - Types of sources ! The compounds of lead cause
anemia, headache, loss of muscle
 Point Sources
power and bluish line around the
 Diffuse or non-point source gum.
TARGET PT 2020 101

 Hazards of groundwater pollution: rhizosphere. This activity is due to the


presence of proteins and enzymes produced
! Excess nitrate in drinking water
by the plants or by soil organisms such as
reacts with hemoglobin to form
bacteria, yeast, and fungi.
non-functional methaemoglobin
and impairs oxygen transport.  Rhizofiltration is a water remediation
This condition is called technique that involves the uptake of
methaemoglobinemia or blue baby contaminants by plant roots. Rhizofiltration
syndrome. is used to reduce contamination in natural
wetlands and estuary areas.
! Excess fluoride in drinking water
causes neuro-muscular disorders, 20. Correct Option: (d)
gastro-intestinal problems, teeth
deformity, hardening of bones and stiff Explanation:
and painful joints (skeletal fluorosis).
Radioactive Pollution
! Overexploitation of groundwater
 Radioactivity is a phenomenon of
may lead to leaching of arsenic
spontaneous emission of proton
from soil and rock sources and
contaminate groundwater. (a-particles), electrons (ß-particles)
Chronic exposure to arsenic and gamma rays (short wave
causes black foot disease. It also electromagnetic waves) due to the
causes diarrhoea, peripheral disintegration of atomic nuclei of some
neuritis, hyperkerotosis and also elements. These cause radioactive
lung and skin cancer. pollution.

 Biological Magnification  Sources

 Eutrophication. ! Natural
 They include cosmic rays from
19. Correct Option: (a) space and terrestrial radiations
Explanation: from radio-nuclides present in
earth’s crust such as radium-
Genetic engineering approaches 224, uranium-238, thorium-232,
potassium- 40, carbon-14, etc
Phytoremediation
! Man-made
 Phytoremediation is the use of plants to
remove contaminants from soil and water.  Nuclear power plants
 Nuclear weapon
Types
 Transportation of nuclear
 Phytoextraction / phytoaccumulation
material
is the process by which plants
accumulate contaminants into the  Disposal of nuclear waste
roots and above-ground shoots or
 Uranium mining
leaves.
 Phytotransformation or
 Radiation therapy
phytodegradation refers to the
uptake of organic contaminants from 21. Correct option: (b)
soil, sediments, or water and their Explanation
transformation to a more stable, less
toxic, less mobile form.  Statement 1 is incorrect: Breast Milk
Banks are established under “National
 Phytostabilization is a technique in Guidelines on Establishment of
which plants reduce the mobility Lactation Management Centres
and migration of contaminated soil. in Public Health Facilities” by the
Leachable constituents are adsorbed and Ministry of Health and Family
bound into the plant structure so that they
Welfare.
form an unstable mass of plant from which
the contaminants will not re-enter the Breast Milk Banks
environment.
 Breast Milk Banks are established
 Phytodegradation or rhizodegradation under “National Guidelines
is the breakdown of contaminants on Establishment of Lactation
through the activity existing in the Management Centres in Public Health
102 TARGET PT 2020

Facilities” by the Ministry of Health  The NITI Aayog is going to draft a


and Family Welfare. roadmap for achieving population
stabilisation in collaboration with the
 It is established for the purpose of
Population Foundation of India (PFI).
collecting, screening, processing,
storing and distributing donor  It is organising a National Consultation
human milk at Government Medical titled “Realizing the vision of
Colleges or District Hospitals with population stabilization: leaving no
high delivery load and availability of one behind”.
newborn treatment units.
About the Draft-
 It has a collection and storage facility
of mother’s own milk in District  The working paper is expected to
Hospital/Sub-district Hospital with at address key gaps in India’s family
least 12 beds. planning programmes. India, with a
 These are established at all delivery current population size of 1.37 billion,
points to provide breastfeeding has the second largest population in
support, lactation counselling and the world.
Kangaroo Mother Care (KMC) support  It will offer constructive
to mothers. recommendations to address regional
 The first Breast Milk Bank of India disparities in outcomes by focusing
was established in 1989 at Mumbai. on adolescents and youths, inter-
departmental convergence, demand
 MAA - “Mothers Absolute Affection”
generation, access to contraceptive
is a nationwide programme of the
services and quality of care.
Ministry of Health and Family Welfare
to promote breastfeeding.  The Aayog said that India is at a stage
where birth rates are falling but the
Importance of Breast Milk population continues to grow due to
 Breast milk is the optimum source of the fact that more than 30 per cent
nutrition for the first six months of life of the population is young and in the
of an infant. reproductive age group.
 It prevents infections like diarrhoea
and acute respiratory infections in
23. Correct Option (c)
early infancy and thus reduces infant Explanation: Statements 1 and 2 are
mortality. correct
 It decreases the risk of mothers
 Statement 3 is incorrect: While bio-
developing breast cancer, ovarian
printed organs are still far from accurate
cancer, type 2 diabetes, and heart
development, tissues have successfully
disease.
been printed on micro-scale.
 Breast Milk Contains Important
Antibodies. 3D bio-printing
 Breast Milk Promotes a Healthy  Bio-printing is an additive
Weight. manufacturing process where
 Breastfeeding May Make Children biomaterials such as cells and growth
Smarter. factors are combined to create tissue-
like structures that imitate natural
 Breastfeeding May Help You Lose tissues.
Weight.
! Bio-ink is used to create these
 Breastfeeding Helps the Uterus structures in a layer-by-layer
Contract. manner.
 Mothers Who Breastfeed Have a Lower  Bio-ink: Bio-ink is a combination of
Risk of Depression. living cells and a compatible base, like
 Breastfeeding May Prevent collagen, elastin, gelatin, hyaluronan,
Menstruation. silk, alginate or nanocellulose.
 But the process can essentially be
22. Correct option: (d)
summarized into three key steps:
Explanation ! Pre bio-printing involves creating
 NITI Aayog to draft roadmap for achieving the digital model that the printer
population stabilisation will produce. The technologies
TARGET PT 2020 103

used are computed tomography 24. Correct Answer: (b)


(CT) and magnetic resonance
imaging (MRI) scans. Explanatory Notes: Option (b) is correct.

! Bio-printing is the actual printing  Electric Vehicles V. Hydrogen Vehicles


process, where bio-ink is placed in 25. Correct Option: (c)
a printer cartridge and deposition
takes place based on the digital Explanation:
model.  Statement 1 & 2 are correct:
! Post bio-printing is the mechanical Supplementary Notes
and chemical stimulation of
printed parts so as to create stable  National Electronics Policy (NEP)
structures for the biological is launched in 2019 and replaced the
material. earlier National Electronics Policy,
2012.
 Application:
 It aims at making India a global hub
! Medicine and Bio-engineering: for Electronics System Design and
Advancements in the production Manufacturing (ESDM) by creating an
of cartilage tissue for use in enabling environment for the industry
reconstruction and regeneration. to compete globally.
! The process has potential  It promotes domestic manufacturing
to eradicate in future the in the entire value-chain of ESDM,
need of organ donation and including core components and
transplantation. Treatment for materials to increase the domestic value
diseases can be tested using addition and reduce dependence on
artificially affected tissues. import of electronic goods by focusing
on scale, skill and technology.
! Bone tissue regeneration as
well as prosthetics and dental  NEP 2019 primarily targets increasing
applications. the electronic goods exports. The
rising trend has been seen in
! Pharmaceutical testing and electronics exports amid slowdown
reduced need for animal trials. in Indian economy. The total value
of production of electronic goods has
! Cosmetic surgery
increased from $31.2 billion in FY15
 Limitation: to $65.5 billion in FY19. Electronics
exports are led by mobile phones.
! The vasculature still has to be
developed to allow lab skin to  India has become the 2nd largest
integrate with the human body’s producer of mobile phones, replacing
blood vessels. Vietnam.

! Stem cell engineering hasn’t yet  India started to become an alternate


evolved to grow all cells of an production destination because of:
organ in a personalized way to 1). Pull Factors include potential
avoid rejection by recipient’s domestic demand and government
immune. policies to boost electronic exports. 2).
Push Factors include trade tensions
! Current not evolved to ensure between the US and China. This is
that a lab organ works well with expected that the positive trend in
all other human organs. India’s electronic exports to continue.

**********
104 TARGET PT 2020

TEST
DAY - 38

1. Which of the following pairs regarding (b) 2 and 3 only


environmental schemes and their
(c) 1, 2, and 4 only
launching years are correctly
matched? (d) 1, 3, and 4 only
1. National Action Plan on Climate
4. Which of the following statements is/
Change: 2008
are correct?
2. Climate Change Action Programme:
1. Operation Save Kurma is specific to
2018
the turtles only.
3. National Adaptation Fund on Climate
2. Operation Thunderbird is specific to
Change: 2015
the Great Indian Bustard only.
4. National Clean Air Programme: 2019
Select the correct option using the codes
Select the correct answer from the code given below:
given below the lists:
(a) 1 only
(a) 1 and 2 only
(b) 2 only
(b) 2, 3 and 4 only
(c) Both 1 and 2
(c) 1, 3 and 4 only
(d) Neither 1 nor 2
(d) 1, 2, 3 and 4
5. Arrange the following policies/plans in
2. Which of the following pairs regarding the chronological order:
environmental laws is incorrectly
matched? 1. National Environmental Policy

(a) Environment Protection Act: 1986 2. National Agroforestry Policy


3. National Forest Policy
(b) Water (Prevention and Control of
Pollution) Cess Act: 1974 4. National Biodiversity Action Plan
(c) Air (Prevention and Control of Select the correct option using the codes
Pollution) Act: 1981 given below:
(d) Public Liability Insurance Act, 1991 (a) 2-3-4-1
3. Which of the following pairs regarding (b) 3-1-2-4
projects for the conservation of species (c) 3-1-4-2
is correctly matched?
(d) 4-2-1-3
1. Project Tiger: 1973
2. Project Snow leopard: 1999 6. Dr. Shailesh Nayak Committee is
3. Project Crocodile: 1995 related to____
4. Project elephant: 1992 (a) The Himalayan ecosystem
(b) Western Ghats ecosystem regulations
Select the correct option using the codes
given below: (c) Coastal Regulation Zone
(a) 1 and 4 only (d) Reintroduction of cheetahs in India
TARGET PT 2020 105

7. Which of the following statements 3. Earth Day: 22 March


regarding the Wildlife Crime Control 4. World Wetlands Day: 2 February
Bureau is/are correct?
1. Wildlife Crime Control Bureau (WCCB) Select the correct option from the code given
is a statutory body. below the lists:
2. It was constituted by amending the (a) 1 and 4 only
Environment Protection Act, 1986. (b) 2 and 3 only
3. The Bureau has its headquarter in (c) 1 and 2 only
Chennai.
(d) 3 and 4 only
Select the correct option using the codes
given below: 11. Match the following lists:
(a) 1 only State butterflies States
(b) 2 only A. Blue Mormon Karnataka
(c) 1 and 2 only
B. Common peacock Uttarakhand
(d) 2 and 3 only
C. Sahyadri Birdwing Maharashtra
8. Consider the following statements:
D. Papilio buddha Kerala
1. State Governments cannot declare any
wild animal a vermin. Select the correct option from the code given
2. The Vermins are included in the below the lists:
Schedule-VI of the Wildlife (Protection) A B C D
Act, 1972.
(a) 1 2 4 3
Which of the above statements is/are
(b) 3 2 1 4
correct?
(c) 4 1 3 2
(a) 1 only
(d) 3 4 1 2
(b) 2 only
(c) Both 1 and 2 12. Consider the following statements:

(d) Neither 1 nor 2 1. Recently, India has decided to


reintroduce the Asian Cheetah from
9. Consider the following statements Iran.
regarding “National Citrus Gene 2. It will be introduced in the Jim Corbett
Sanctuary”: National Park.
1. It is the world’s first citrus gene Which of the above statements is/are
sanctuary. correct?
2. It is situated in Nasik, Maharashtra.
(a) 1 only
Which of the above statements is/are (b) 2 only
correct?
(c) Both 1 and 2
(a) 1 only
(d) Neither 1 nor 2
(b) 2 only
13. Which of the following activities are
(c) Both 1 and 2
permitted in a Coastal Regulated Zone
(d) Neither 1 nor 2 (CRZ)?
1. Sand and Rock mining
10. Which of the following pairs are
correctly matched? 2. Creation of Storm Water Drains
1. International day on Biodiversity: 22 3. Atomic Energy projects
May 4. Land-filling
2. World Environment Day: 8 June 5. Conveying systems and pipelines
106 TARGET PT 2020

Select the correct answer using the code 1. It is a process of evaluating the likely
given below: environmental impacts of a proposed
(a) 1, 2 and 4 only project or development.
2. It takes into account inter-related
(b) 2 and 4 only
socio-economic, cultural and human-
(c) 2, 3 and 5 only health impacts, both beneficial and
(d) 1, 2, 3, 4 and 5 adverse.
3. It is a tool used to identify the
14. Which of the following statements environmental, social and economic
regarding the Air (Prevention and impacts of a project prior to decision-
Control of Pollution) Act, 1981 is/are making.
correct?
4. It aims to predict environmental
1. The Act makes provisions for the
impacts at an early stage in project
establishment of the Central Pollution
Control Board (CPCB). planning and design, find ways and
means to reduce adverse impacts.
2. The act doesn’t cover noise pollution.
3. The act says that no person shall, Which of the above statements are correct?
without the previous consent of the (a) 1, 3 and 4 only
State Board, establish or operate any
industrial plant in an air pollution (b) 1 and 4 only
control area. (c) 2 and 3 only
Select the correct answer using the code (d) 1, 2, 3 and 4
given below:
17. With reference to PARIVESH, consider
(a) 3 only
the following statements:
(b) 1 and 2 only 1. It is an environmental single window
(c) 1 and 3 only hub for Environment, Forest, Wildlife
and CRZ clearances.
(d) 1, 2 and 3
2. It is a workflow-based application, based
15. With reference to the Forest Rights on the concept of web architecture.
Act (FRA), consider the following
statements: Which of the above statements is/are
1. The Ministry of Environment, Forest correct?
and Climate Change (MoEFCC) is the (a) 1 only
nodal agency to implement this act.
(b) 2 only
2. The act provides for diversion of
forest land for public utility facilities (c) Both 1 and 2
managed by the Government. (d) Neither 1 nor 2
3. It concerns the rights of forest-dwelling
communities to land and other 18. Which of the following is/are the key
resources. features of the National Ganga River
Basin Authority (NGRBA)?
Which of the above statements is/are
1. The Ministry of Water Resources, River
incorrect?
Development and Ganga Rejuvenation
(a) 1 only (MoWR, RD & GR) is the nodal Ministry
(b) 3 only for the NGRBA.

(c) 1 and 2 only 2. One of the Chief Ministers of the


States through which the Ganga flows
(d) 2 and 3 only becomes the Chairman of NGRBA on a
rotation basis.
16. Consider the following statements
with respect to Environmental Impact Select the correct option using the code
Assessment: given below:
TARGET PT 2020 107

(a) 1 only 2. Under JCPOA, EU refrains from re-


(b) 2 only introducing sanctions terminated
under JCPOA.
(c) Both 1 and 2
3. JCPOA did not limit Iran’s ballistic-
(d) Neither 1 nor 2 missile development.

19. With reference to the Environment Choose the correct answer:


Pollution Control Authority (EPCA),
(a) Only 1
consider the following statements:
1. EPCA is a supreme court mandated (b) 1 and 2 only
non-statutory body. (c) 2 and 3 only
2. The Minister of Environment & Forests
(d) 1, 2 and 3
(MoEF) holds the chairmanship of
EPCA. 22. Which of the following statements
3. It has the power to take suo-moto regarding National intelligence Grid
cognizance of matters related to (NATGRID) is/are correct?
environmental pollution. 1. It is an attached office to Ministry of
Which of the above statements are Home Affairs.
incorrect? 2. It is an integrated intelligence grid
(a) 1 and 2 only which will connect databases of core
security agencies.
(b) 2 and 3 only
3. It was conceptualized in wake of
(c) 1 and 3 only Mumbai terror attacks in 2009.
(d) 1, 2 and 3
Select the correct answer using the code
20. Consider the following statements: given below:
1. The World Wide Fund for Nature (WWF) (a) 1 and 3 only
is an international governmental
(b) 2 only
organization.
(c) 2 and 3 only
2. The living planet report is published
every three years by the World Wide (d) 1, 2 and 3
Fund for Nature (WWF).
23. Which of the following statements
3. World Wide Fund for Nature is working
regarding National intelligence Grid
for the reduction of humanity’s footprint
on the environment. (NATGRID) is/are correct?
1. It is an attached office to Ministry of
Which of the above statements is/are Home Affairs.
correct?
2. It is an integrated intelligence grid
(a) 3 only which will connect databases of core
(b) 1 and 3 only security agencies.
(c) 2 and 3 only 3. It was conceptualized in wake of
Mumbai terror attacks in 2009.
(d) 1, 2 and 3
Select the correct answer using the code
21. Recently, USA exited from Joint given below:
Comprehensive Plan of Action
(JCPOA). Which of the following (a) 1 and 3 only
statements are correct about JCPOA? (b) 2 only
1. JCPOA was an agreement reached
(c) 2 and 3 only
between Iran and the UN Security
Council in 2015. (d) 1, 2 and 3
108 TARGET PT 2020

24. Consider the following statements 25. Consider the following statements
about National Pharmaceutical regarding The Draft National
Pricing Authority (NPPA) that was in Statistical Commission Bill
news recently:
1. The draft bill is aimed at empowering
1. National Pharmaceutical Pricing
the National Statistical Commission
Authority deals with the issue of
affordability and availability of (NSC) to become the nodal body for all
medicines. core statistics in the country.
2. Recently, National Pharmaceutical 2. The Chairman and the members of the
Pricing Authority (NPPA) has raised Commission shall be appointed by the
the ceiling prices of 21 essential President.
medicines by 80%.
Which of the following statement is/are
Which of the above statement(s) is/are
correct?
correct?
(a) 1 only (a) 1 only

(b) 2 only (b) 2 only

(c) Both 1 and 2 (c) Both 1 and 2


(d) Neither 1 nor 2 (d) Neither 1 nor 2

**********
TARGET PT 2020 109

ANSWER HINTS
DAY - 38

1. Correct Option: (c) ! Water (Prevention and Control of


Pollution) Cess Act, 1977.
Explanation:
! Air (Prevention and Control of
Environmental schemes Pollution) Act, 1981
 National Action Plan on Climate ! The Water (Prevention and Control of
Change (NAPCC), launched in 2008, Pollution) Cess Act, 1977
formulated in the backdrop of India’s
voluntary commitment to reduce emission ! Environment (Protection) Act, 1986
intensity of its GDP by 20 to 25 percent by and Rules thereunder
2020 over 2005 levels. It was also meant ! Public Liability Insurance Act, 1991
to focus on key adaptation requirements
and the creation of scientific knowledge ! National Green Tribunal Act, 2010
and preparedness for dealing with climate
change. 3. Correct Option: (a)
 Climate Change Action Programme Explanation:
(CCAP), a central sector scheme, has
been launched in 2014 with the objective Conservation of species in India
to build and support capacity at central  Project Tiger centrally sponsored scheme
and state levels, strengthening scientific was launched in 1973, supervised by
and analytical capacity for climate change National Tiger Conservation Authority.
assessment, establishing an appropriate
institutional framework and implementing  Project Crocodile was launched in 1975
climate-related actions in the context of with support from the United Nations
sustainable development. Development Programme and the Food
and Agriculture Organisation.
 National Adaptation Fund on Climate
Change was established in 2015 to meet  Project Elephant was launched in
the cost of adaptation to climate change February 1992 as a centrally sponsored
for the State and Union Territories that scheme to assist states having free-ranging
are particularly vulnerable to the adverse populations of wild elephants and to ensure
effects of climate change. long term survival of identified viable
 MoEF&CC has launched NCAP in populations of elephants in their natural
2019 as a pan India time-bound national- habitats.
level strategy for prevention, control  Project Snow Leopard was launched in
and abatement of air pollution besides 2009 to promote a knowledge-based and
augmenting the air quality monitoring adaptive conservation framework that fully
network across the country. involves the local communities

2. Correct Option: (b) 4. Correct Option: (a)


Explanation: Explanation:
Environmental laws in India Initiatives
 The important Environmental Laws in the
 Operation Thunderbird is the code-
country are given below:
name of INTERPOL’s (International
! Water (Prevention and Control of Criminal Police Organization)
Pollution) Act, 1974. multi-national and multi-species
110 TARGET PT 2020

enforcement operation for wildlife The notification was amended from time to
protection. The operation brought time based on representations received.
about a unanimous approach by the state
 A need was felt overtime to undertake a
enforcement agencies in the fight against
comprehensive revision of the notification
wildlife crime in the country.
on the basis of number of representations
 Operation Save Kurma was a species- from the various Coastal States/UTs, besides
specific operation on turtles. Under its other stakeholders particularly related to
total of 15,739 live turtles were recovered the management and conservation of marine
from 45 suspects, having inter-state and coastal ecosystems, development
linkages. in coastal areas, eco-tourism, livelihood
options and sustainable development of
5. Correct Option: (c) coastal communities, etc.

Explanation:  Therefore, the Ministry of Environment,


Forest & Climate Change constituted
Environmental Polices a Committee in June 2014 under the
Chairmanship of Dr. Shailesh Nayak
 The principal aim of National Forest (Secretary, Ministry of Earth Sciences) to
Policy, 1988 is to ensure environmental examine the various issues and concerns of
stability and maintenance of ecological Coastal States/UTs and other stakeholders
balance including atmospheric equilibrium for recommending appropriate changes in
which is vital for sustenance of all life the CRZ Notification, 2011.
forms, human, animal, and plant. The Draft
National Forest Policy 2018 is now open for  The Committee submitted its
public comments and will replace the older recommendations in 2015.
1988 policy once it comes into force.
7. Correct Option: (b)
 India’s National Environmental Policy
(NEP) was adopted in 2006 and was built Explanation:
upon on the existing policies ( e.g. National
Wildlife Crime Control Bureau (WCCB)
Forest Policy, 1988; National Conservation
Strategy and Policy Statement on  Wildlife Crime Control Bureau is
Environment and Development, 1992; a statutory multi-disciplinary body
and the Policy Statement on Abatement established in 2007, by the Government of
of Pollution,1992; National Agriculture India under the Ministry of Environment
Policy, 2000; National Population Policy, and Forests, to combat organized wildlife
2000; National Water Policy, 2002, etc). crime in the country, by amending the
Wildlife (Protection) Act, 1972, a special
 is a response to India’s national commitment Act to protect the wildlife in the country.
to a clean environment.
 The Bureau has its headquarter in
 Following India’s adoption of the National New Delhi and five regional offices at
Environment Policy (NEP) in 2006, a Delhi, Kolkata, Mumbai, Chennai and
National Biodiversity Action Plan Jabalpur; three sub-regional offices at
(NBAP) was prepared in 2008. Guwahati, Amritsar, and Cochin; and
 The National Agroforestry Policy, five border units at Ramanathapuram,
2014 promotes tree plantation in Gorakhpur, Motihari, Nathula, and
complementarity with crops and livestock Moreh.
to improve productivity, employment,  Under Section 38 (Z) of the Wild Life
achieve efficient nutrient cycling and (Protection) Act, 1972, it is mandated to
organic matter addition for sustainable collect and collate intelligence related to
agriculture, etc. organized wildlife crime activities and to
disseminate the same to State and other
6. Correct Option: (c) enforcement agencies for immediate
action so as to apprehend the criminals;
Explanation: to establish a centralized wildlife crime
data bank; co-ordinate actions by various
Shailesh Nayak Committee
agencies in connection with the enforcement
 With the objective of conservation and of the provisions of the Act; assist foreign
protection of the coastal environment, authorities and international organization
the Ministry of Environment and Forest concerned to facilitate co-ordination and
and Climate Change notified the Coastal universal action for wildlife crime control;
Regulation Zone Notification in 1991, capacity building of the wildlife crime
which was subsequently revised in 2011. enforcement agencies for scientific and
TARGET PT 2020 111

professional investigation into wildlife  International day on Biodiversity is


crimes and assist State Governments to celebrated on 22 May to commemorate
ensure success in prosecutions related to the adoption of the text of the Convention
wildlife crimes; and advise the Government of Biodiversity on 22 May 1992 by the
of India on issues relating to wildlife Nairobi Final Act.
crimes having national and international
 World Environment Day is celebrated
ramifications, relevant policy and laws.
on 5 June to commemorate the United
 It also assists and advises the Customs Nations Conference on the Human
authorities in inspection of the consignments Environment, which took place from 5
of flora & fauna as per the provisions of June to 16 June 1972.
Wild Life Protection Act, CITES and EXIM  Earth Day is celebrated on 22 April
Policy governing such an item. since 1970.
8. Correct Option: (a)  World Wetlands Day is celebrated on
2 February to commemorate the date of
Explanation: the adoption of the Ramsar Convention on
wetland on February 02, 1971, in the city
Wildlife (Protection) Act, 1972 of Ramsar, Iran.
 With the amendment of the Act in 1991,
powers of the State Governments have been 11. Correct Option: (b)
withdrawn almost totally. Now the State
Explanation:
Governments are not empowered to
declare any wild animal a vermin. State butterflies of India
 As per Section 62 of the Wildlife Protection  Maharashtra was the 1st state to
Act, 1972, States can send a list of officially declare Blue Mormon (Papilio
wild animals to the Centre requesting polymnestor) as its state butterfly, way
it to declare them vermin for selective back in 2015 followed by Uttarakhand
slaughter. (Common peacock), Karnataka
 The Central Government may by (Sahyadri/Southern Birdwing) and
notification, declare any wild animal Kerala (Malabar banded peacock or,
other than those specified in Schedule I Papilio buddha).
and part 11 of Schedule H of the law to be
vermin for any area for a given period of
time.
 Vermins are included in the Schedule-V
of the Act.

9. Correct Option: (c)


Explanation:
Citrus gene sanctuary
 It is in Meghalaya.
 A section of the Nokrek biosphere
reserve was declared the National Citrus Blue Mormon
Gene Sanctuary in 1984.
 This was done due to the finding of Indian
wild oranges (Citrus indica Tanaka) in the
biosphere reserve.
 It is said to be the only place in the world
to house the mother plant of citrus-indica
(orange). And for this, the world’s first
citrus gene sanctuary was established
in this reserve.

10. Correct Option: (a)


Explanation:
Important days Sahyadri/Southern Birdwing
112 TARGET PT 2020

is considered to be an evolutionary link


between small and big cats.
 Four big cat species are found in India in
wild viz. Gir Lion, Bengal tiger, Indian
leopard, Snow leopard. Further, clouded
leopard is also found in India.

Common peacock

Clouded leopard
 The Big Cats that are not found in their
natural habitats in India are Jaguar and
Cheetah. Cheetah got extinct as back as
the 1940s.
 The National Tiger Conservation Authority
(NTCA) had previously told the Supreme
Court that African cheetahs would be
translocated in India from Namibia and
Papilio Buddha would be kept at Nauradehi wildlife
sanctuary in Madhya Pradesh.

Cheetah
Tamil Yeoman
 Nauradehi was found to be the most
 Recently, Tamil Nadu became the fifth state suitable area for the cheetahs as its
by declaring Tamil Yeoman (Cirrochroa forests are not very dense to restrict
thais) as its state butterfly. the fast movement of the spotted cat.
12. Correct Option: (d)  The sanctuary is located in the center
of Madhya Pradesh covering parts of
Explanation: Sagar, Damoh, Narsinghpur, and Raisen
Districts.
Cheetah reintroduction in India
 Taxonomically, Big Cats are members of 13. Correct Option: (c)
the Felidae family represented by three
genera viz. Panthera (included Lion, Tiger, Explanation:
Jaguar, Leopard and Snow Leopard),
Coastal Regulated Zone (CRZ)
Acinonyx (includes Cheetah) and Puma
(includes Cougar). Clouded Leopard  The salient features of the draft CRZ
(Neofelis nebulosa ) is not a big cat but Notification, 2018 and changes with
TARGET PT 2020 113

respect to CRZ Notification, 2011, are as  As per CRZ, 2011 Notification, for CRZ-
under:- II areas, Floor Space Index (FSI) or
the Floor Area Ratio (FAR) had been
! The High Tide Line (HTL) has
frozen at 1991 Development Control
been demarcated by the National
Regulation (DCR) levels. In the Draft
Centre for Sustainable Coastal
CRZ, 2018 Notification, it has been proposed
Management (NCSCM) and shall be
to de-freeze the same and permits FSI
reckoned as a universal standard for
for construction projects, as prevailing
the HTL for all regulatory purposes
on the date of the new Notification.
under the CRZ Notification, 2018.
 Temporary tourism facilities such
! Hazard line mapping has also as shacks, toilet blocks, change rooms,
been carried out by Survey of India.
drinking water facilities, etc. have been
The Hazard Line has, however, been
proposed in beaches. Such temporary
delinked from the CRZ regulatory
tourism facilities are also proposed to be
regime and shall be used only as permissible in the No Development
a tool for Disaster Management Zone (NDZ) of the CRZ-III areas.
and planning of adaptive and
mitigation measures.  Wherever there is a National or State
Level Highway passing through the
! CRZ limits on land along the NDZ in CRZ-III areas, temporary
tidally influenced water bodies tourism facilities have been proposed to
have been proposed to be reduced from be taken up on the seaward side of the
100 meters or the width of the creek, roads. On the landward side of such
whichever is less, to 50 meters or roads in the NDZ, Resorts/Hotels and
the width of the creek, whichever other tourism facilities have also been
is less. proposed to be permitted subject to the
! A No Development Zone (NDZ) of extant regulations of the concerned State.
20 meters has been proposed to be  Regulated limestone mining is
stipulated for all Islands close to the proposed to be permitted, subject to strict
mainland coast and for all Backwater Environmental safeguards, in areas
Islands in the mainland. adequately above the height of HTL,
! For CRZ-III areas, two separate based on recommendations of reputed
categories have been proposed viz.: National Institutes in the Mining field.

 CRZ-III A – Densely populated  The prohibitive activities along


rural areas with a population Coastal Regulation Zones are setting
density of 2161 per square up new industries and expanding existing
kilometer as per the 2011 Census. ones, except projects of the Department of
Such areas shall have an NDZ Atomic Energy, setting up and expanding
of 50 meters from the HTL as units for the disposal of waste and
against 200 meters from the effluents. However, exceptions include
HTL stipulated in the CRZ stormwater drains and facilities required
Notification, 2011. for discharging treated effluents. Though
prohibitive, these activities are permitted
 CRZ-III B – Rural areas with a under certain safeguards.
population density of below 2161
per square kilometer as per the  Also dumping of the city or town
2011 Census. Such areas shall waste for the purposes of land-
continue to have an NDZ of 200 filling, discarding ash or any other
meters from the HTL. waste from thermal power stations
and mining of sand, rocks, and other
 The procedure for CRZ clearances has substrate materials constitute the
been simplified and delegations have been non-permissive activities.
made at various levels for recommending/
according to CRZ clearances to the 14. Correct Option: (a)
projects/activities. Only such projects/
activities, which are located in the Explanation:
CRZ-I & IV areas, shall be dealt with
Air (Prevention and Control of Pollution)
for CRZ clearance by the MoEFCC.
Act 1981
For all other project activities located
in CRZ-II/III areas, CRZ clearance  The Air (Prevention and Control of Pollution)
shall be considered at the level of the Act, 1981 an Act of the Parliament of India
CZMA. for prevention, control, and abatement
114 TARGET PT 2020

of air pollution in India. It extends to the dispensaries, fair price shops, electricity
whole of India. and telecommunication lines, water tanks,
etc. with the recommendation of Gram
 The Water Prevention and Control of
Sabhas.
Pollution Act, 1974 makes provisions for
the establishment of the Central Pollution  Under Section 3(1)(h) of the Scheduled
Control Board (CPCB). Tribes and Other Traditional Forest
Dwellers (Recognition of Forest Rights)
 According to this act, the “air pollutant”
Act, 2006, the rights of settlement and
means any solid, liquid or gaseous conversion of all forest villages, old
substance (including noise) present in habitations, un-surveyed villages and
the atmosphere in such concentration as other villages in forest, whether recorded,
may be or tend to be injurious to human notified, or not, into revenue villages have
beings or other living creatures or plants or been recognized as one of the forest rights
property or environment. of forest-dwelling Scheduled Tribes and
 The further provisions of the act say that no other traditional forest dwellers on all
person shall, without the previous consent forest lands.
of the State Board, establish or operate any  As per the provisions of the Act and the
industrial plant in an air pollution control rules framed thereunder, the forest right
area. Every person to whom consent has related to the conversion of forest villages
been granted by the State Board shall into revenue villages is to be adjudicated
comply with the conditions and norms by the Gram Sabha, Sub-Divisional
prescribed by the board such as prevention Level Committee and the District Level
and control of the air pollution. Failure to Committee as per the laid down procedure,
do so brings penalty including a jail term of like any other forest right specified in the
at least 1.5 years. Act.
 The Ministry of Tribal Affairs has issued
15. Correct Option: (a)
guidelines impressing upon all the State/ UT
Explanation: Governments to convert all such erstwhile
forest villages, un-recorded settlements
Forest Rights Act, 2006 and old habitations into revenue villages
with a sense of urgency in a time-bound
 Scheduled Tribes and Other Traditional
manner. The conversion would include the
Forest Dwellers (Recognition of Forest
actual land use of the village in its entirety,
Rights) Act, 2006, has been enacted to
including land required for current or
recognize and vest the forest rights and
future community uses, like, schools,
occupation of forest land in forest-dwelling
health facilities, public spaces, etc.
Scheduled Tribes and other traditional
forest dwellers, who have been residing
16. Correct Option: (d)
in such forests for generations, but whose
rights could not be recorded. Explanation:
 The Ministry of Tribal Affairs (MoTA), is Environmental Impact Assessment
the nodal agency to implement this act.
 According to the Convention of Biological
 This Act not only recognizes the rights Diversity, Environmental Impact
to hold and live in the forest land under Assessment (EIA) is a process of evaluating
the individual or common occupation the likely environmental impacts of a
for habitation or for self-cultivation for proposed project or development, taking
livelihood but also grants several other into account inter-related socio-economic,
rights to ensure their control over forest cultural and human-health impacts, both
resources which, inter-alia, include right beneficial and adverse.
of ownership, access to collect, use and
dispose of minor forest produce, community  United Nations Environment Programme
rights such as nistar; habitat rights for (UNEP) defines Environmental Impact
primitive tribal groups and pre-agricultural Assessment (EIA) as a tool used to identify
communities; right to protect, regenerate the environmental, social and economic
or conserve or manage any community impacts of a project prior to decision-
forest resource which they have been making. It aims to predict environmental
traditionally protecting and conserving for impacts at an early stage in project
planning and design, find ways and means
sustainable use.
to reduce adverse impacts, shape projects
 The Act also provides for diversion of forest to suit the local environment and present
land for public utility facilities managed the predictions and options to decision-
by the Government, such as schools, makers.
TARGET PT 2020 115

 By using EIA both environmental and 17. Correct Option: (c)


economic benefits can be achieved, such
as reduced cost and time of project Explanation:
implementation and design, avoided  PARIVESH (Pro-Active and Responsive
treatment/clean-up costs and impacts of facilitation by Interactive, Virtuous and
laws and regulations. Environmental Single-window Hub)
 Although legislation and practice vary  It is an environmental single window
around the world, the fundamental hub for Environment, Forest, Wildlife
components of an EIA would necessarily and CRZ clearances.
involve the following stages:
 This Single-Window Integrated
! Screening to determine which projects Environmental Management System has
or developments require a full or been developed in pursuance of the spirit
partial impact assessment study; of ‘Digital India’ and capturing the essence
! Scoping to identify which potential of Minimum Government and Maximum
impacts are relevant to assess Governance.
(based on legislative requirements,  “PARIVESH” is a workflow-based
international conventions, expert application, based on the concept of
knowledge, and public involvement), web architecture.
to identify alternative solutions that
 It has been rolled out for online submission,
avoid, mitigate or compensate adverse
monitoring and management of proposals
impacts on biodiversity (including
submitted by Project Proponents to the
the option of not proceeding with
Ministry of Environment, Forest and
the development, finding alternative
Climate Change (MoEFCC), as well as to
designs or sites which avoid the
the State Level Environmental Impact
impacts, incorporating safeguards in
Assessment Authorities (SEIAA).
the design of the project, or providing
compensation for adverse impacts),  It seeks to give various types of clearances
and finally to derive terms of reference (e.g. Environment, Forest, Wildlife and
for the impact assessment; Coastal Regulation Zone Clearances)
from Central, State and district-level
! Assessment and evaluation of impacts
authorities.
and development of alternatives,
to predict and identify the likely  The system has been designed, developed
environmental impacts of a proposed and hosted by the Ministry of Environment,
project or development, including the Forest and Climate Change, with technical
detailed elaboration of alternatives; support from National Informatics Centre,
(NIC).
! Reporting the Environmental
Impact Statement (EIS) or EIA  It provides single registration and single
report, including an environmental sign-in for all types of clearances (i.e.
management plan (EMP), and a non- Environment, Forest, Wildlife and CRZ),
technical summary for the general unique-ID for all types of clearances
audience. required for a particular project and a
single Window interface for the proponent
! Review of the Environmental Impact to submit applications for getting all types
Statement (EIS), based on the terms
of clearances.
of reference (scoping) and public
(including authority) participation.
18. Correct Option: (a)
! Decision-making on whether to
Explanation:
approve the project or not and under
what conditions; and National Ganga River Basin Authority
! Monitoring, compliance, enforcement  It is a planning, financing, monitoring and
and environmental auditing. Monitor coordinating body of the centre and the
whether the predicted impacts and states.
proposed mitigation measures occur
as defined in the EMP. Verify the  The Ministry of Water Resources,
compliance of proponent with the River Development and Ganga
EMP, to ensure that unpredicted Rejuvenation(MoWR, RD & GR) is the
nodal Ministry for the NGRBA
impacts or failed mitigation measures
are identified and addressed in a  It is chaired by the Prime Minister and
timely fashion. has as its members the Union Ministers
116 TARGET PT 2020

concerned, the Chief Ministers of the the reduction of humanity’s footprint on


States through which Ganga flows, viz., the environment.
Uttarakhand, Uttar Pradesh, Bihar,
 It works in partnership with foundations,
Jharkhand, and West Bengal, among
governments, businesses, communities,
others.
individuals and more than six million
 This initiative is expected to rejuvenate members, to conserve many of the world’s
the collective efforts of the Centre and the most ecologically important regions.
States for cleaning the river. It is the world’s largest conservation
organization with over five million
19. Correct option: (a) supporters worldwide, working in more
than 100 countries, supporting around
Explanation: 1,300 conservation and environmental
projects. WWF is a foundation, with 55%
Environment Pollution Control Authority of funding from individuals and bequests,
(EPCA) 19% from government sources (such as the
 EPCA is a Supreme Court-empowered World Bank, DFID, USAID) and 8% from
body which is tasked with taking various corporations in 2014.
measures to tackle air pollution in the  The living planet report is published every
National Capital Region. It is constituted two years by WWF since 1998, it is based on
with the objective of protecting and living planet index and ecological footprint
improving the quality of the environment calculation. WWF publishes the Living
and preventing and controlling the Planet Index in collaboration with the
environmental pollution in the National Zoological Society of London. Along with
Capital Region and also assists the apex ecological footprint calculations, the Index
court in various environment-related is used to produce a bi-yearly Living Planet
matters in the region. Report giving an overview of the impact of
human activity on the world.
 It was notified in 1988 by Environment
ministryunder Environment Protection Objectives:
act, 1986.
 Protect and restore species and their
 Besides the chairman, the EPCA has 20 habitats
members. Former secretary Bhure Lal is
the current chair of EPCA.  Strengthen local communities’ ability to
conserve the natural resources they depend
 The authority has the power suo-moto, or upon
on the basis of complaints made by any
 Transform markets and policies to
individual, association, company, public
reduce the impact of the production and
undertaking or local body carrying on any
consumption of commodities
industry, operation or process.
 Ensure that the value of nature is
 Functions include:
reflected in decisions made by individuals,
! To protect and improve quality of communities, governments, and
environment and prevent and control businesses
environmental pollution in National  Mobilize hundreds of millions of people to
Capital Region. support conservation
! To enforce graded Response Action
Plan (GRAP) in NCR as per the 21. Correct Option (c)
pollution levels.
Explanation:
20. Correct Option: (a)  JCPOA was an agreement reached between
Iran and P5+1 countries in 2015.
Explanation:
22. Correct Option: (d)
World Wide Fund for Nature (WWF)
Explanation
 WWF is an international non-governmental
organization founded in 1961. It was All statements are correct
formerly named the World Wildlife Fund,
which remains its official name in Canada Supplementary notes:
and the United States.
NATGRID
 It is working in the field of wilderness
preservation, reduce the most pressing  The NATGRID project has also been in the
threats to the diversity of life on Earth and news for reportedly seeking to link social
TARGET PT 2020 117

media accounts to the huge database of  It is an integrated intelligence grid


records related to immigration entry and which will connect databases of core
exit, banking and telephone details among security agencies with an aim to collect
other data. comprehensive patterns of intelligence
that can be readily accessed by intelligence
 National Intelligence Grid (NATGRID), an agencies.
attached office of Ministry of Home Affairs,
has been created as an IT platform to  It will link 10 user agencies with certain
assist the intelligence and law enforcement databases that would be procured from
agencies in ensuring national and internal 21 organizations. The agencies concerned
security, with the ultimate aim to counter include the Intelligence Bureau, local police,
terror. and revenue and customs departments.

 NATGRID, which was conceptualized in  NATGRID, like a number of other


2009, is said to be the brainchild of the government initiatives (UIDAI), is
being established through governmental
then home minister P Chidambaram, who
notifications rather than legislation passed
mooted the idea in the wake of the 2008
in Parliament.
Mumbai attacks.
 It is an integrated intelligence grid 24. Correct Answer: (a)
which will connect databases of core  Explanation: 2nd statement is incorrect.
security agencies with an aim to collect Recently, National Pharmaceutical Pricing
comprehensive patterns of intelligence Authority (NPPA) has raised the ceiling
that can be readily accessed by intelligence prices of 21 essential medicines by 50% and
agencies. not 80%.
 It will link 10 user agencies with certain
Supplementary Notes
databases that would be procured from
21 organizations. The agencies concerned  NPAA was established by a Government
include the Intelligence Bureau, local police, of India Resolution in 1997. It is an office
and revenue and customs departments. attached to Department of Pharmaceuticals
(DoP), Ministry of Chemicals & Fertilizers.
 NATGRID, like a number of other
government initiatives (UIDAI), is  It is headquartered in New Delhi.
being established through governmental  Function: It is an independent Regulator
notifications rather than legislation passed for pricing of drugs. It ensures accessibility
in Parliament. and availability of medicines to people at
affordable prices.
23. Correct Option: (d)
 The NPPA currently fixes price of
Explanation drugs on National List of Essential
Medicines (NLEM) under schedule-I
All statements are correct of Drug Price Control Orders (DPCO).
Supplementary notes:  Recently, National Pharmaceutical Pricing
Authority (NPPA) has raised the ceiling
NATGRID prices of 21 essential medicines by 50%.
 The NATGRID project has also been in the
news for reportedly seeking to link social 25. Correct option: (a)
media accounts to the huge database of Explanation
records related to immigration entry and
exit, banking and telephone details among  Statement 2 is incorrect: The Chairman and
other data. the members of the Commission shall be
appointed by the central government on the
 National Intelligence Grid (NATGRID), an recommendation of a search committee.
attached office of Ministry of Home Affairs,
has been created as an IT platform to Supplementary notes
assist the intelligence and law enforcement
agencies in ensuring national and internal The Draft National Statistical Commission
security, with the ultimate aim to counter Bill
terror.  Draft Bill proposes autonomy for National
 NATGRID, which was conceptualized in Statistical Commission which gives
2009, is said to be the brainchild of the government power to make final decision.
then home minister P Chidambaram, who  It is put out by Ministry of Statistics &
mooted the idea in the wake of the 2008 Programme Implementation and is open
Mumbai attacks. for public suggestions until 19 January.
118 TARGET PT 2020

About the Bill- any matter relating to official statistics.


 The draft bill is aimed at empowering the  However, central government or a state
National Statistical Commission (NSC) to government may issue directions as
become the nodal body for all core statistics necessary to any government agency under
in the country. its administrative control along with a
 Core statistics include national income report on reasons for not accepting any
statistics like GDP, jobs data, industry advice to the commission.
data and budgetary transactions data.  A report on reasons for not accepting any
Features of the Bill- advice of the commission shall be laid
before Parliament or a state legislature for
 As per the draft Bill, the NSC will have a total period of thirty days.
a Chairperson, five whole time members
along with Deputy Governor of Reserve  The draft Bill states that the NSC shall have
Bank of India (RBI), Chief Statistician of power to review the statistical system of any
India (CSI) as other members and Chief government agency in the light of concepts,
Economic Advisor, Ministry of Finance, as definitions, standards, methodologies
the ex-officio member. and established policies, and recommend
 The Chairman and the members of the measures for enhanced performance; to
Commission shall be appointed by the prescribe a code of practice.
central government on the recommendation  Also, the Commission shall participate
of a search committee. in consultation with the central
 Along with retaining the advisory nature of government and coordinate with national
NSC, the draft Bill states that the decision statistical organisations on matters of
of the central government, whether a statistical standards, methodologies and
question is of policy or not, shall be final. classifications.
 As per the experts, this proposal goes
Timing of the Draft bill-
against the long pending demand to grant
more powers to the NSC.  The draft bill has been released at a time
when the government has been facing
 The draft Bill also seeks to change the
composition of the Commission by replacing criticism from several quarters over
NITI Aayog Chief Executive Officer with interference in the release of crucial data
the Finance Ministry’s Chief Economic unfavourable to it.
Advisor as the ex-officio member along with  For example, the government withheld
giving member status to Chief Statistician jobs data that showed the unemployment
of India from the current status of secretary rate at a 45-year high, releasing it only six
to the NSC.
months later, after the general elections.
 As per the draft Bill the central government
 The Bill comes at a time when several
may, from time to time, issue directions to
statistical reports such as the unemployment
the Commission as it may think “necessary
in the interest of the sovereignty and survey were withheld and consumption
integrity of India, the security of the State, expenditure survey was decided to be not
friendly relations with foreign states, released by the government.
public order, decency or morality”.
About NSC:
 The draft also specifies that the Commission
“shall in exercise of its powers or the  The National Statistical Commission (NSC)
performance of its functions, be bound by of India is an autonomous body which was
such directions or questions as the Central formed in July 2005.
Government may give in writing to it from  The objective of its commission is to reduce
time to time”. the problems faced by statistical agencies
 The draft Bill states that the government in the country in relation to collection of
shall seek advice from the Commission on data.

**********
TARGET PT 2020 119

TEST
DAY - 39

Time Allowed: 30 mins Maximum Marks: 50

1. The Global Warming Potential of greenhouse effect because both are


a species depends on which of the transparent to terrestrial radiation.
following factors?
Which of the above statements is/are
1. The absorption of UV-radiation by a
correct?
given species.
2. The spectral location of its absorbing (a) 1 only
wavelengths. (b) 2 only
3. The atmospheric lifetime of the (c) 3 only
species.
(d) 1, 2, and 3
Select the correct option using the codes
given below: 4. Which of the following statements is/
are correct?
(a) 1 only
1. Water vapor is the biggest contributor
(b) 1 and 3 only to the greenhouse effect.
(c) 2 and 3 only 2. CO2 does not have any role in the
(d) 3 only greenhouse effect of water vapor.

2. Methane is emitted by which of the Select the correct option using the codes
following processes? given below:
1. The decay of organic waste in municipal (a) 1 only
solid waste landfills (b) 2 only
2. The production and transport of coal
(c) Both 1 and 2
3. Livestock farming
(d) Neither 1 nor 2
Select the correct option using the codes
given below: 5. Consider the following pairs of United
Nations Framework Convention on
(a) 1 only Climate Change Conferences of the
(b) 1, 2 and 3 Parties and their places:
(c) 2 and 3 only 1. CoP-24: Katowice
(d) 3 only 2. CoP-25: Santiago
3. CoP-26: Glasgow
3. Consider the following statements
regarding GHGs and their sources: Which of the above pairs is/are correctly
1. Carbon dioxide is emitted during matched?
cement manufacturing (a) 1 only
2. Nitrous oxide is emitted during the (b) 1 and 3 only
treatment of wastewater.
(c) 2 and 3 only
3. Neither oxygen nor nitrogen plays
a significant role in enhancing the (d) 1, 2, and 3
120 TARGET PT 2020

6. Consider the following statements (a) 2 only


regarding tropospheric and (b) 1 and 3 only
stratospheric ozone:
(c) 2 and 3 only
1. Stratospheric ozone is naturally
formed in chemical reactions involving (d) 1, 2, and 3
ultraviolet sunlight and oxygen
molecules. 9. What are the causes of coral
bleaching?
2. Tropospheric ozone is naturally
1. Invasive alien species
formed in chemical reactions involving
hydrocarbon and nitrogen oxide 2. Mixing of river waters
without involving ultraviolet sunlight. 3. Extreme low tides
3. Unlike stratospheric ozone, the 4. Overexposure to sunlight
tropospheric ozone cannot be
destroyed. Select the correct option using the codes
given below:
Which of the above statements is/are
(a) 1, 2, 3
correct?
(b) 2, 3, 4
(a) 1 only
(c) 1, 3, 4
(b) 2 and 3 only
(d) 1, 2, 4
(c) 1 and 3 only
(d) 1, 2, and 3 10. Why has an “ozone hole” appeared over
Antarctica and not over the Arctic?
7. Consider the following statements: 1. Lower temperature in Antarctica than
1. All halones must include at least one in the Arctic.
atom of bromine. 2. Polar vortex is strong in the Arctic than
2. Bromine is less effective at destroying Antarctica.
ozone than chlorine. 3. The denitrification of the Polar
3. Dobson unit is a unit of measurement Stratospheric Clouds is more in
for the total amount of ozone depleted. Antarctica than the Arctic.
4. The dehydration is rare in the Antarctic
Which of the above statements is/are than in the Arctic.
incorrect?
Select the correct option using the codes
(a) 1 only given below:
(b) 2 only (a) 1 and 3 only
(c) 2 and 3 only (b) 2 and 4 only
(d) 1, 2, and 3 (c) 1,2, and 4 only
8. Consider the following statements (d) 1, 2, 3, and 4
regarding Agenda 21:
11. Which of the following can be the
1. It is an action plan of the United possible impacts of climate change on
Nations (UN) related to sustainable agriculture?
development.
1. Increased crop yields in temperate
2. It was an outcome of the Rio+20. regions.
3. The number 21 refers to the number 2. Decreased amount of arable land in the
of parties supporting the Agenda i.e. high-latitude region.
G-20 plus Europian Union.
3. Decreased fertilizer requirement.
Which of the above statements is/are 4. Increase in the agricultural production
incorrect? variability.
TARGET PT 2020 121

Select the correct answer using the code component gases of Green House Gas
given below: emission?
(a) 1 and 3 only 1. Carbon dioxide
(b) 2 and 3 only 2. Methane

(c) 1 and 4 only 3. Nitrogen

(d) 1 and 2 only 4. Nitrous oxide

Select the correct answer using the code


12. Which of the following are the causes
given below:
of the urban heat island?
1. The high albedo of cities (a) 1, 2 and 3 only

2. Lack of vegetation (b) 1, 2 and 4 only


3. Waste heat (c) 3 and 4 only

Select the correct answer using the code (d) 1, 2, 3 and 4


given below:
16. Increased level of carbon dioxide in the
(a) 1 and 2 only atmosphere would impact the plants
(b) 2 and 3 only in which of the following ways?

(c) 1 and 3 only 1. A decrease in the photosynthetic


productivity of plants.
(d) 1, 2 and 3
2. The proliferation of weeds.
13. In the process of Ocean 3. Increase in the number of insects and
acidification: other pests.
(a) Oceans become more alkaline Select the correct answer using the code
(b) The concentration of carbonate ions given below.
increases
(a) 1 and 2 only
(c) pH level decreases
(b) 1 and 3 only
(d) The concentration of hydrogen ions in
(c) 2 and 3 only
the ocean increases
(d) 1, 2 and 3
14. Increased level of carbon dioxide
in the atmosphere would impact 17. With reference to extra-terrestrial
the plants in which of the following causes of climate change, consider
ways? the following statements:
1. A decrease in the photosynthetic 1. An increase in the number of sunspots
productivity of plants. leads to warmer and drier conditions
2. The proliferation of weeds. on Earth.

3. Increase in the number of insects and 2. As the orbital eccentricity of earth


other pests. increases, summer days increase in
both the hemispheres.
Select the correct answer using the code
3. The axial tilt of Earth’s rotational axis
given below.
controls the duration of seasons.
(a) 1 and 2 only
Which of the above statements is/are
(b) 1 and 3 only correct?
(c) 2 and 3 only (a) 1 only
(d) 1, 2 and 3 (b) 3 only
15. Which of the following are the main (c) 2 and 3 only
122 TARGET PT 2020

(d) 1, 2 and 3 1. The Zoological Survey of India (ZSI)


was established to promote the survey,
18. Which of the following can be the exploration and research of the fauna
possible impacts of climate change in the region
on agriculture? 2. The Botanical Survey of India (BSI) is
1. Increased crop yields in temperate the apex research organization under
regions. the MOEF for carrying out taxonomic
2. Decreased amount of arable land in the and floristic studies on wild plant
high-latitude region. resources of the country.

3. Increased fertilizer requirement. Which of the above statements is/are


4. Reduction in agricultural production correct?
variability. (a) 1 only

Select the correct answer using the code (b) 2 only


given below: (c) Both 1 and 2
(a) 1 and 3 only (d) Neither 1 nor 2
(b) 2 and 3 only 21. Recently, ‘eBkray’ platform was
(c) 1 and 4 only launched for which of the following?
(d) 1 and 2 only (a) Auction of assets attached by banks
(b) Integrating e-commerce platforms
19. Match the following pairs:
(c) Integrating defense procurements
List I List II
(d) For providing audio content to visually
A. Black Carbon 1. Stored in the impaired
form of aquatic
22. Consider the following statements regarding
biomass
Parliamentary Standing Committees-
B. Brown Carbon 2. Originates from 1. Parliamentary committees draw their
the incomplete authority from Article 105 and Article
combustion of 118.
fossil fuels 2. Parliament is not bound by the
recommendations of committees.
C. Blue Carbon 3. Stored in the soil
of natural 3. The three financial committees
are the Public Accounts Committee,
ecosystems
the Estimates Committee and the
D. Green Carbon 4. Released by Committee on Public Undertakings.
the combustion
Which of the above statements is/are
combustion of correct?
organic matter
(a) 3 only
Select the correct answer using the code
(b) 1 and 2 only
given below:
(c) 1 and 3 only
A B C D
(d) 1,2 and 3
(a) 2 4 3 1
23. Consider the following statements about
(b) 2 4 1 3 Gaganyaan Mission-
(c) 4 2 1 3 1. The programme will make India the
(d) 4 2 3 1 fourth nation in the world to launch a
Human Spaceflight Mission.
20. Consider the following statements: 2. The spacecraft will be placed in a low
TARGET PT 2020 123

earth orbit of 300-400km. (NTCA)

Which of the above statements is/are 1. Project Tiger was launched in 1973 with
correct?
(a) 1 only 9 tiger reserves for conserving our

(b) 2 only national animal, the tiger.


(c) Both 1 and 2
2. The National Tiger Conservation
(d) Neither 1 nor 2
Authority (NTCA) is a statutory
24. Consider the following facts about Sambhar
lake: body of the Ministry, with an
1. The Sambhar Salt Lake is India’s largest
overarching supervisory/coordination
inland saltwater body located near
Jaipur in Rajasthan. role, performing functions as provided
2. Sambhar has not been designated as
a Ramsar site. in the Wildlife (Protection) Act, 1972.

Which of the above statement(s) is/are


correct? Choose the correct option

(a) 1 only
(a) 1 only
(b) 2 only
(c) Both 1 and 2 (b) 2 only

(d) Neither 1 nor 2


(c) Both 1 and 2
25. Consider the following statements about
National Tiger Conservation Authority (d) Neither 1 nor 2

**********
124 TARGET PT 2020

ANSWER HINTS
DAY - 39

1. Correct Option: (c)  CO2, by definition, has a GWP of 1


regardless of the time period used, because
Explanation: it is the gas being used as the reference.
Global Warming Potential CO2 remains in the climate system for
a very long time: CO2 emissions cause
 Global warming potential (GWP) is a increases in atmospheric concentrations of
measure of how much heat a greenhouse CO2 that will last thousands of years.
gas traps in the atmosphere up to a specific
time horizon, relative to carbon dioxide. It 2. Correct Option: (b)
compares the amount of heat trapped by a
certain mass of the gas in question to the Explanation:
amount of heat trapped by a similar mass
of carbon dioxide and is expressed as a Methane emissions
factor of carbon dioxide.  One of the greenhouse gases responsible
 A GWP is calculated over a specific time for this global warming is methane. It is
horizon, commonly 20, 100, or 500 years. emitted naturally and anthropogenically
User related choices such as the time from different sources and its concentration
horizon can greatly affect the numerical in the atmosphere has assumed alarming
values obtained for carbon dioxide proportions. It is observed that the
equivalents. In the Fifth Assessment agricultural sector emits the highest
Report of the Intergovernmental Panel on amount of methane, followed by the energy
Climate Change, methane has a lifetime and waste sectors, respectively.
of 12.4 years and with climate-carbon
feedbacks a global warming potential of  Methane emissions also result from
86 over 20 years and 34 over 100 years in livestock and other agricultural
response to emissions. For a change in time practices and by the decay of organic
horizon from 20 to 100 years, the GWP for waste in municipal solid waste
methane, therefore, decreases by a factor of landfills.
approximately 2.5.
 It is emitted during the production
 GWP depends on the following and transport of coal, natural gas, and
factors: oil.
! the absorption of infrared  It is released from submerged soils to the
radiation by a given species atmosphere through the roots and stems
! the spectral location of its of rice plants, which corresponds to 6 to
absorbing wavelengths 29 percent of total annual anthropogenic
methane emission.
! the atmospheric lifetime of the
species 3. Correct Option: (d)
 Thus, a high GWP correlates with a large
Explanation:
infrared absorption and a long atmospheric
lifetime. The dependence of GWP on Greenhouse Gases
the wavelength of absorption is more
complicated. Even if a gas absorbs radiation  Gases that trap heat in the atmosphere
efficiently at a certain wavelength, this may are called greenhouse gases. This section
not affect its GWP much if the atmosphere provides information on emissions and
already absorbs most radiation at that removals of the main greenhouse gases to
wavelength. and from the atmosphere.
TARGET PT 2020 125

 Although the Earth’s atmosphere


consists mainly of oxygen and
nitrogen, neither plays a significant
role in enhancing the greenhouse
effect because both are essentially
transparent to terrestrial radiation.
The greenhouse effect is primarily a
function of the concentration of water
vapor, carbon dioxide (CO2), methane
(CH4), nitrous oxide (N2O), and other trace
gases in the atmosphere that absorb the
terrestrial radiation (infra-red) leaving the
surface of the Earth.
 Carbon dioxide (CO2): Carbon dioxide  However, CO2 and other greenhouse gases
enters the atmosphere through burning increase the amount of water vapor in the air
fossil fuels (coal, natural gas, and oil), by boosting the rate of evaporation.
solid waste, trees, and other biological
materials, and also as a result of certain  Since the rate of evaporation rises with
chemical reactions (e.g., manufacture temperature, the amount of water vapor in
of cement). Carbon dioxide is removed the air at any one time (and the amount of
from the atmosphere (or “sequestered”) warming it causes) is strongly related to the
when it is absorbed by plants as part of the amount of other greenhouse gases in the
biological carbon cycle. air. The addition of the non-condensable
 Methane (CH4): Methane is emitted during gases causes the temperature to increase
the production and transport of coal, natural and this leads to an increase in water vapor
gas, and oil. Methane emissions also result that further increases the temperature.
from livestock and other agricultural This is an example of a positive feedback
practices and by the decay of organic waste effect.
in municipal solid waste landfills.
 Nitrous oxide (N2O): Nitrous oxide
is emitted during agricultural and
industrial activities, combustion of
fossil fuels and solid waste, as well as
during the treatment of wastewater.
 Fluorinated gases: Hydrofluorocarbons,
perfluorocarbons, sulfur hexafluoride, and
nitrogen trifluoride are synthetic, powerful
greenhouse gases that are emitted from a
variety of industrial processes. Fluorinated
gases are sometimes used as substitutes for
stratospheric ozone-depleting substances
(For instance, chlorofluorocarbons,
hydrochlorofluorocarbons, and halons).
These gases are typically emitted in smaller
quantities, but because they are potent 5. Correct Option: (b)
greenhouse gases, they are sometimes
referred to as High Global Warming Explanation:
Potential gases (“High GWP gases”).
UNFCCC Conference of the Parties
4. Correct Option: (a)  The COP is the supreme decision-making
body of the Convention. All States that are
Explanation: Parties to the Convention are represented
Water vapor at the COP, at which they review the
implementation of the Convention and
 Water vapor is the biggest overall contributor any other legal instruments that the COP
to the greenhouse effect and humans are adopts and take decisions necessary to
not directly responsible for emitting this promote the effective implementation of
gas in quantities sufficient to change its the Convention, including institutional
concentration in the atmosphere. and administrative arrangements.
126 TARGET PT 2020

 The first UNFCCC Conference of the Stratospheric ozone formation


Parties took place from 28 March to 7 April
 These reactions occur continually wherever
1995 in Berlin, Germany.
ultraviolet sunlight is present in the
 COP 24 was held on 3–14 December stratosphere. As a result, the greatest
2018 in Katowice, Poland. ozone production occurs in the tropical
stratosphere.
 The 25th session of the Conference of
the Parties (COP 25) to the UNFCCC  The production of stratospheric
was planned to take place from 11 to 22 ozone is balanced by its destruction
November 2019 in Brazil. Upon election in chemical reactions. Ozone reacts
as President of Brazil, Jair Bolsonaro continually with a wide variety
withdrew Brazil from hosting the event. of natural and human-produced
COP 25 was then planned to take place in chemicals in the stratosphere. In
Parque Bicentenario Cerrillos in Santiago each reaction, an ozone molecule is
de Chile, Chile from 2 to 13 December with a lost and other chemical compounds
pre-sessional period from 26 November to 1 are produced. Important reactive
December 2019 with up to 25,000 delegates gases that destroy ozone are those
scheduled to attend. However, following the containing chlorine and bromine.
2019 Chilean protests, Chilean President
 Some stratospheric ozone is transported
Sebastián Piñera announced Chile’s
down into the troposphere and can
withdrawal from hosting the summit in
influence ozone amounts at Earth’s surface,
late October 2019. Then Spain offered and
particularly in remote, unpolluted regions
was appointed, as the new host. The event
of the globe.
took place in Madrid.
 The 26th session of the Conference of
the Parties (COP 26) to the UNFCCC is
expected to take place from 9-19 November
2020, in Glasgow, UK.

6. Correct Option: (a)


Explanation:
Tropospheric and stratospheric ozone
 Stratospheric ozone is naturally formed
in chemical reactions involving ultraviolet
sunlight and oxygen molecules, which make
up 21% of the atmosphere. In the first step,
sunlight breaks apart one oxygen molecule
(O2) to produce two oxygen atoms (2 O). In Stratospheric ozone depletion
the second step, each atom combines with
an oxygen molecule to produce an ozone  Near Earth’s surface, ozone is
molecule (O3). produced in chemical reactions
involving naturally occurring gases
and gases from pollution sources.
Production reactions primarily
involve hydrocarbon and nitrogen
oxide gases and require sunlight. Fossil
fuel combustion is a primary pollution
source for tropospheric ozone production.
The surface production of ozone does not
significantly contribute to the abundance of
stratospheric ozone. The amount of surface
ozone is too small in comparison, and the
transport of surface air to the stratosphere
is not effective enough.

Tropospheric ozone formation


 As in the stratosphere, ozone in the
troposphere is destroyed in naturally
TARGET PT 2020 127

occurring chemical reactions and in above a point on the earth’s surface, one
reactions involving human-produced Dobson unit being equivalent to a
chemicals. Tropospheric ozone can layer of pure ozone 0.01 mm thick at
also be destroyed when ozone reacts standard temperature and pressure.
with a variety of surfaces, such as
those of soils and plants. 8. Correct Option: (c)
Explanation:

Agenda 21
 Agenda 21 is a non-binding action plan
of the United Nations (UN) related to
sustainable development.
 It was an outcome of the United Nations
Conference on Environment and
Development (UNCED) held in Rio de
Janeiro, Brazil, in 1992 also known as
Earth Summit.
 It is a comprehensive blueprint of action
Tropospheric ozone depletion
to be taken globally, nationally and locally
by organizations of the UN, governments,
7. Correct Option: (c)
and major groups in every area in which
Explanation: humans directly affect the environment.

Halones  The number 21 refers to an agenda for


the 21st century.
 All halons contain bromine which
is 40-100 times more effective at
9. Correct Option: ()
destroying ozone than chlorine.
Synergistic interactions between bromine Explanation:
derived mainly from the halons and methyl
bromide, and chlorine in the stratosphere Causes of coral bleaching
is responsible for 30-40% of the Antarctic
ozone hole.  Change in ocean temperature:
Increased ocean temperature caused by
 The most commonly used halons are halon- climate change is the leading cause of coral
1211 (CBrClF2), halon-1301 (CBrF3), and bleaching. When water is too warm, corals
halon-2402 (C2Br2F4). The numbering
will expel the algae (zooxanthellae) living
system for the halons is different but more
in their tissues causing the coral to turn
simple than for that of the CFCs. The first
completely white. As little as 2 degrees
digit from the left is for the total number of
carbon atoms, second is for the number of Fahrenheit can cause coral to drive out
fluorine atoms, third is for the number of algae.
chlorine atoms, and last is the number of  Runoff and pollution: Storm generated
bromine atoms. precipitation can rapidly dilute ocean water
 Halon-1211 is used primarily in portable and runo can carry pollutants — these can
fire extinguishing systems in the Western bleach near-shore corals.
Hemisphere and Western Europe. Halon-
 Overexposure to sunlight: When
1301 is used in flooding systems for fire
and explosive containment. Halon-2402 is temperatures are high, high solar irradiance
used as a fire extinguishing agent in Japan, contributes to bleaching in shallow-water
Russia, China, and the former states of the corals.
Soviet Union. At the present, there are  Extreme low tides: Exposure to the
no known drop-in replacements for the
air during extreme low tides can cause
halons, although many groups are working
bleaching in shallow corals.
on potential replacements.
 Invasive alien species: There is not any
Dobson unit role of the invasive alien species in the
 Dobson unit is a unit of measurement for bleaching of corals. At most, they can alter
the total amount of ozone in the atmosphere the biodiversity of the reefs.
128 TARGET PT 2020

the entire winter (about 5 months) and in


the Arctic for only limited periods (10–60
days) in most winters.

Isolated conditions
 Stratospheric air in the polar regions is
relatively isolated from other stratospheric
regions for long periods in the winter
months. The isolation comes about because
of strong winds that encircle the poles,
forming a polar vortex, which prevents
substantial motion of air into or out of
the polar stratosphere. This circulation
10. Correct Option: (a) strengthens in winter as stratospheric
temperatures decrease, with the result
Explanation: that the isolation of air in the vortex is
much more effective in the Antarctic than
The ozone hole over Antarctica rather
the Arctic.
than the Arctic
Polar stratospheric clouds (PSCs)
Distributing halogen gases
 Reactions on the surfaces of liquid and
 Halogen source gases emitted at Earth’s
solid PSCs can substantially increase the
surface are present in comparable
relative abundances of the most reactive
abundances throughout the stratosphere
chlorine gases. These reactions convert the
in both hemispheres even though most
reservoir forms of reactive chlorine gases,
of the emissions occur in the Northern
chlorine nitrate (ClONO2) and hydrogen
Hemisphere. The abundances are
chloride (HCl), to the most reactive form,
comparable because most source gases have
ClO. ClO increases from a small fraction
no significant natural removal processes in
of available reactive chlorine to comprise
the lower atmosphere and because winds
nearly all chlorine that is available. With
and convection redistribute and mix air
increased ClO, additional catalytic cycles
efficiently throughout the troposphere on
involving ClO and BrO become active in
the timescale of weeks to months. Halogen
the chemical destruction of ozone whenever
gases (in the form of source gases and some
sunlight is available. PSCs exist in larger
reactive products) enter the stratosphere
regions and for longer time periods in
primarily from the tropical upper
the Antarctic than the Arctic. The most
troposphere. Stratospheric air motions
then transport these gases upward and common type of PSC forms from nitric
toward the pole in both hemispheres. acid (HNO3) and water condensing on
pre-existing liquid sulfuric acid-containing
Low polar temperatures particles. Some of these particles freeze to
form reactive solid particles.
 The severe ozone destruction represented
by the ozone hole requires that low Nitric acid and water removal
temperatures be present over a range
of stratospheric altitudes, over large  Once formed, PSC particles fall to lower
geographical regions, and for extended time altitudes because of gravity. The largest
periods. Low temperatures are important particles can descend several kilometers
because they allow liquid and solid PSCs or more in the stratosphere during the
to form. Reactions on the surfaces of these low-temperature winter/spring period
PSCs initiate a remarkable increase in that lasts several months in Antarctica.
the most reactive chlorine gas, chlorine Because PSCs often contain a significant
monoxide (ClO). fraction of available HNO3, their descent
removes HNO3 from regions of the ozone
 In the Antarctic winter, minimum layer. This process is called denitrification
daily temperatures are generally of the stratosphere. With less HNO3,
much lower and less variable than the highly reactive chlorine gas ClO
in the Arctic winter. These and other remains chemically active for a longer
meteorological differences occur period, thereby increasing chemical ozone
because of the unequal distribution destruction.
among land, ocean, and mountains
between the hemispheres at middle  Significant denitrification occurs each
and high latitudes. The winter winter in the Antarctic and in some,
temperatures are low enough for PSCs to but not all, Arctic winters, because
form somewhere in the Antarctic for nearly PSC formation temperatures must be
TARGET PT 2020 129

sustained over an extensive altitude  As a result of the thawing of snow,


region and time period to affect the amount of arable land in the high-
denitrification. latitude region is likely to increase
by reduction of the number of frozen
 Ice particles form when temperatures are
lands. At the same time, arable land along
a few degrees lower than PSC formation
the coastlines is bound to be reduced as a
temperatures. If ice temperatures persist
for weeks to months over extensive altitude result of rising sea levels.
regions, ice particles will also fall several  Erosion, submergence of shorelines, the
kilometers due to gravity. As a result, a salinity of the water table due to the
significant fraction of water vapor can also increased sea levels, could mainly affect
be removed from regions of the ozone layer. agriculture through the inundation of low
This process is called dehydration of the lying lands.
stratosphere. With the low temperatures
required to form ice, dehydration is  The rising temperature would increase
common in the Antarctic and rare in fertilizer requirement for the same
the Arctic winters. The removal of water production targets and result in higher
vapor does not directly affect the catalytic GHG emissions, ammonia volatilization
reactions that destroy ozone. Dehydration and cost of crop production.
indirectly affects ozone destruction by  Increased frequencies of droughts,
suppressing PSC formation, which reduces floods, storms, and cyclones are likely
ClO production in PSC reactions. to increase agricultural production
variability.

12. Correct Option: (b)


Explanation:
Urban Heat Island
 The urban heat island phenomenon “exists
in areas with a lot of densely placed
buildings and paved surfaces that absorb
heat from the sun, causing the area to
be several degrees hotter than rural or
surrounding areas.
 Heat islands form in urban and suburban
areas due to the following reasons:
11. Correct Option: (c) ! Paved and impermeable surfaces
Explanation: ! Dark surfaces(hence, low albedo
of the cities)
Impact of Climate Change on Agriculture
! Thermal mass
 Climate Change can affect crop yield as
well as the types of crops that can be grown
! Lack of vegetation
in certain areas, by impacting agricultural ! Waste heat
inputs such as water for irrigation, amounts
of solar radiation that affect plant growth, ! Climate change
as well as the prevalence of pests.  Cities have a low albedo, the reflecting
 A rise in temperatures caused by power of a surface. The increased surface
increasing greenhouse gases is likely to area of buildings results in more solar
affect crops differently from region to radiation absorption than reflection.
region. For example, moderate warming  The urban heat island effect is so strong
(an increase of 1 to 3°C in mean in Delhi, that it saw 50% less fog than
temperature) is expected to benefit surrounding areas. In Delhi, the heat
crop yields in temperate regions, while island effect also appears to be suppressing
in lower latitudes especially seasonally
the very formation of fog.
dry tropics, even moderate temperature
increases (1 to 2°C ) are likely to have
13. Correct Option: (c)
negative impacts for major cereal crops.
Warming of more than 3°C is expected to Explanation:
have a negative effect on production in all
regions. Ocean Acidification
130 TARGET PT 2020

 Oceans are an important reservoir for CO2, ! Chlorofluorocarbons


absorbing a significant quantity of it (one-
! Nitrous oxide
third) produced by anthropogenic activities
and effectively buffering climate change. ! Ozone
 Ocean acidification is the change in  Nitrogen is not the Green House Gas.
ocean chemistry – lowering of ocean
pH (i.e. increase in the concentration of 16. Correct Option: (c)
hydrogen ions) driven by the uptake of
carbon compounds by the ocean from the Explanation:
atmosphere.
Impact of Increased Carbon Dioxide on
 As the uptake of atmospheric carbon Plants
dioxide by the ocean increases, the
concentration of hydrogen ions in the  Weeds may proliferate rapidly and that too
ocean increases, the concentration of at the expense of useful plants.
carbonate ions decreases, the pH of  Insects and other pests that feed on plants
the oceans decreases and the oceans may also increase in number. The survival
become less alkaline – this process is of other organisms gets affected. Elevated
known as ocean acidification. CO2 can increase levels of simple sugars in
leaves and lower their nitrogen content.
14. Correct Option: (c) These can increase the damage caused
Explanation: by many insects, who will consume more
leaves to meet their metabolic requirements
Global Warming of nitrogen. Thus, any attack will be more
severe
 The increased CO2 concentration in
the atmosphere may increase the  The increased CO2 concentration in
photosynthetic productivity of plants the atmosphere may increase the
due to an increased rate of metabolic photosynthetic productivity of plants
activity. This, in turn, produces more due to an increased rate of metabolic
organic matter. activity. This, in turn, produces more
organic matter.
 Weeds may proliferate rapidly and that too
at the expense of useful plants. 17. Correct Option: (c)
 Insects and other pests that feed on plants
Explanation:
may also increase in number. The survival
of other organisms gets affected. Elevated Extra-Terrestrial causes of Climate
CO2 can increase levels of simple sugars in Change
leaves and lower their nitrogen content.
These can increase the damage caused  Interstellar Nebulae:
by many insects, who will consume more ! The path of the earth around the galaxy
leaves to meet their metabolic requirements is elliptical and its passage every 270
of nitrogen. Thus, any attack will be more to 400 million years is near to center
severe of the galaxy. In every 300 million
years, Earth passes through the dust
15. Correct Option: (b) lane arm of the galaxy. The arm has
Explanation: an accumulation of interstellar matter
or the Nebulae. This nebula of galaxy
Green House Gases interposes between sun and earth,
because of which there is a reduction
 Global warming is a result of a rise in
in solar radiation reaching the earth’s
the proportion of atmospheric greenhouse
surface which might bring in climate
gases (GHG) than their normal level.
change.
 The rise of GHG levels has been attributed
 Solar Irradiance:
to the process of industrialization,
urbanization, and pollution caused by ! There is a regular fluctuation in the
vehicular, industrial, domestic and amount of energy irradiated from
agricultural emission. the outer surface of the sun or the
photosphere, this alteration brings
 The main component gases of GHG
changes in the temperatures and
emissions are –
precipitation.
! Carbon dioxide
! The changes in the net amount of solar
! Methane energy received on earth may also
TARGET PT 2020 131

change because of changes in relative due to axial precision winters in the


distance between earth and sun. northern hemisphere become much
longer and warmer while summers
 Sunspots:
in the southern hemisphere become
! These are dark and cooler patches on much longer and less warm.
the sun which increase and decrease
in a cyclical manner. 18. Correct Option: (a)
! When the number of sunspots Explanation:
increases, cooler and wetter
weather and greater storminess Impact of Climate Change on Agriculture
occur. A decrease in sunspot
 Climate Change can affect crop yield as
numbers is associated with warm
well as the types of crops that can be grown
and drier conditions.
in certain areas, by impacting agricultural
 Milankovitch oscillations: inputs such as water for irrigation, amounts
of solar radiation that affect plant growth,
! This theory infers cycles in the
as well as the prevalence of pests.
variations in the earth’s orbital
characteristics around the sun, the  A rise in temperatures caused by
wobbling of the earth and the changes increasing greenhouse gases is likely to
in the earth’s axial tilt. All these alter affect crops differently from region to
the amount of insolation received from region. For example, moderate warming
the sun, which in turn, might have a (an increase of 1 to 3°C in mean
bearing on the climate. temperature) is expected to benefit
crop yields in temperate regions, while
! The orbital eccentricity of the
in lower latitudes especially seasonally
earth: The eccentricity of the earth orbit
dry tropics, even moderate temperature
is between 0.001 & 0.054. Earth takes
increases (1 to 2°C ) are likely to have
95,000 years to achieve its maximum
eccentricity from the minimum. As negative impacts for major cereal crops.
eccentricity increases, days of Warming of more than 3°C is expected to have
summer increase both in northern a negative effect on production in all regions.
and southern hemispheres.  As a result of the thawing of snow,
! Obliquity of the axial tilt of the amount of arable land in the high-
Earth’s rotational axis: The axial latitude region is likely to increase
tilt or obliquity of the earth’s rotational by reduction of the number of frozen
axis is the angle between the axis of lands. At the same time, arable land along
revolution and the axis of rotation the coastlines is bound to be reduced as a
of the Earth. The significance of result of rising sea levels.
obliquity is that it controls the  Erosion, submergence of shorelines, the
latitudinal distribution of solar salinity of the water table due to the
radiant energy and intensity and increased sea levels, could mainly affect
duration of different seasons. The agriculture through the inundation of low
change in the obliquity is directly lying lands.
related to the temperature difference
between summers and winters.  The rising temperature would increase
Smaller is the obliquity; smaller is fertilizer requirement for the same
the change in temperature between production targets and result in higher
summers and winters. GHG emissions, ammonia volatilization
and cost of crop production.
! Axial precision or Precision of
equinoxes: It is a slow and continuous  Increased frequencies of droughts, floods,
change in the orientation of the storms, and cyclones are likely to increase
rotational axis of the earth due to the agricultural production variability.
effects of gravity of the sun, the moon
and of the nearby planets. Due to the 19. Correct Option: (b)
effect of gravitational forces on earth
Explanation:
the direction of the axis of rotation
changes and it forms a complete List I List II
conical motion. This is also referred
to as wobbling of the rotational axis of A. Black Carbon 2. Originates from
the earth. This cycle of axial precision
is completed in every 26,000 years. The the incomplete
length and warmth of the season alter combustion of
due to axial precision. It is seen that fossil fuels
132 TARGET PT 2020

B. Brown Carbon 4. Released by 20. Correct Option: (c)


the combustion
Explanation
combustion of
organic matter Zoological Survey of India
C. Blue Carbon 1. Stored in the  The Zoological Survey of India (ZSI)
form of aquatic was established to promote the survey,
biomass exploration and research of the fauna
D. Green Carbon 3. Stored in the soil in the region.
of natural  The activities of the ZSI are coordinated
ecosystems by the Conservation and Survey Division
 Black Carbon under the Ministry of Environment, Forest
! Black carbon (BC) is a component of and Climate Change, Government of India
fine particulate matter of the size 2.5
 Primary objectives are: Exploring,
μm. It consists of pure carbon, which
originates from the incomplete Surveying, Inventorying and Monitoring of
combustion of fossil fuels, coal, faunal diversity in various states, selected
biofuel, biomass, wood, rubber, etc. It ecosystems and protected areas of India;
is emitted in the form of soot.
Taxonomic studies of the faunal components
! Soot is an airborne mass of impure collected; Status survey of Threatened and
carbon particles resulting from Endemic species; Preparation of Red Data
the incomplete combustion of
hydrocarbons. It originates from Book, Fauna of India and Fauna of States.
pyrolysis.  Secondary objectives are: GIS and
 Brown Carbon Remote Sensing studies on recorded
! Brown carbon is brown smoke animal diversity as well as on threatened
released by the combustion of species; Chromosomal Mapping and DNA
organic matter. It coexists with Barcoding.
black carbon when released in the
atmosphere. Botanical survey of India
! It is one of the significant warming
factors as it disturbs the temperature  The Botanical Survey of India (BSI) is
pattern of the atmosphere and the the apex research organization under
cloud forming process. It also changes the MOEF for carrying out taxonomic
the solar absorption pattern and the and floristic studies on wild plant
nature of clouds.
resources of the country.
 Blue Carbon
 The prime objectives of the Botanical Survey
! It is the carbon captured by the
world’s oceans and coastal ecosystems. of India (BSI) is to undertake intensive
This carbon is captured by living floristic surveys and collect accurate and
organisms in oceans is stored detailed information on the occurrence,
in the form of aquatic biomass. distribution, ecology and economic utility
Seagrasses, mangroves, and marshes
are types of vegetated coastal blue of plants in the country
carbon ecosystems, these habitats
have a cover of approximately 49 21. Correct answer: (a)
million hectares worldwide. Blue
carbon ecosystem act as the major Explanation:
sink for capturing atmospheric carbon
and reducing warming effects. ‘eBkray’

 Green Carbon  Recently, Union Finance Minister launches


! It is the carbon captured into terrestrial ‘eBkray’ auction platform for assets
plant biomass in photosynthesis and attached by banks.
stored in the plants and soil of
natural ecosystems and is a vital  eBkray is an e-auction platform to enable
part of the global carbon cycle. online auction of attached assets by banks.
TARGET PT 2020 133

 Indian Banks Auctions Mortgaged


22. Correct answer- (d)
Properties Information (IBAPI) Explanation-
portal
About Parliamentary Committees:
 It is an initiative of Indian Banks
Association under the policy of the In the Indian Parliament, a Standing committee is
Department of Financial Services, a committee consisting of Members of Parliament
Ministry of Finance to provide or MPs.
a platform to provide details of  It is a permanent and regular committee
mortgaged properties to be auctioned which is constituted from time to time
online by Banks, starting with PSBs. according to the provisions of an Act of
Parliament or Rules of Procedure and
 The eBkray platform provides navigational Conduct of Business.
links to all PSB e-auction sites, property
search feature and presents single-window  There are two types of Parliamentary
access to information on properties up for committee, the Standing committee and
e-auction, comparison of similar properties, the Ad hoc committee.
and also contains videos and photographs ! The Standing committees are
of the uploaded properties. constituted every year or frequently
 Buyers can use IBAPI portal to search and and they work on continuous basis.
get properties details and participate in ! Ad hoc committees are temporary
the auction process. Presently 21 banks are and created for specific task. Once
onboard on this portal
that task is completed, the ad hoc
 Currently, there are 2,457 residential, committees cease to exist.
576 commercial, 333 industrial and 18
 Parliamentary committees draw their
agricultural properties are available on
authority from Article 105 (on privileges of
eBkray platform among others.
Parliament members) and Article 118 (on
 PSBs have attached assets worth over Rs Parliament’s authority to make rules for
2.3 lakh crore in the last three fiscal years. regulating its procedure and conduct of
business).
Objectives of eBkray
 These Committees are smaller units of MPs
 To enhance user experience through from both Houses, across political parties
seamless access to information by the and they function throughout the year.
search based on the type and location of the
 These smaller groups of MPs study
property put up for e-auction by the banks
and deliberate on a range of subject
in India.
matters, Bills, and budgets of all the
 To enable online auction of attached assets ministries. Parliament is not bound by the
transparently and cleanly for the improved recommendations of committees.
realization of value by banks.
 It will provide navigational links to all
23. Correct answer- (c)
Public Sector Banks (PSBs) e-auction sites, Explanation-
property search feature and will present
single-window access to information on About the Mission-
properties up for e-auction, comparison
 The programme will make India the fourth
of similar properties, as well as contains
nation in the world to launch a Human
videos and photographs of uploaded Spaceflight Mission. So far, only the USA,
properties. Russia and China have launched human
 The platform also helps the buyer to easily spaceflight missions.
navigate to the bank e-auction site after a  ISRO has developed some critical
notified property is selected. It also helps technologies like re-entry mission
the user to search property using State- capability, crew escape system, crew
wise, District-wise and bank-wise details. module configuration, thermal protection
system, deceleration and floatation system,
Need
sub-systems of life support system etc.
 There has been information asymmetry required for this programme.
when bank attached assets are auctioned
 GSLV Mk-III launch vehicle, which
which will come to an end with the launch
has the necessary payload capability
eBkray.
for this mission, will be used to launch
 Simplify auction process Gaganyaan.
134 TARGET PT 2020

 Two unmanned Gaganyaan missions will  Causes of Mass Mortality at Sambhar


be undertaken prior to sending humans. Lake
 The mission will aim to send a three-  The possible causes for avian botulism at
member crew to space for a period of five Sambhar Lake are:
to seven days.
 Reduced water levels: This might
 The spacecraft will be placed in a low earth have increased salinity levels leading to
orbit of 300-400km.
the death of living organisms.
 The total programme cost is expected to be
 The decaying plant or animal materials are
less than Rs. 10,000 crores.
capable of hosting the bacteria for a longer
 With the ability to hold one oxygen cylinder, period of time.
the suit will allow the astronaut to breathe
in space for 60 minutes.  A bird-to-bird cycle: Since only insectivorous
and omnivorous birds were affected and
24. Correct Answer: (a) not herbivores, the birds feeding on dead
birds could have been a possible cause of
 Explanation: 2nd statement is incorrect:
Sambhar has been designated as a Ramsar such mortality.
site.  The possibility of external factors like water
pollution and eutrophication are ruled
Supplementary Notes
out as no farming is being carried out in
 Recently, the Indian Veterinary Research the vicinity of sambhar lake.
Institute (IVRI) has attributed the deaths
of migratory birds to avian botulism 25. Correct Answer: (c)
at Sambhar Lake in Rajasthan. In this
context, Sambhar lake was in news. Explanation:
 The avian botulism is a neuro-muscular  Project Tiger and National Tiger
illness caused by Botulinum (natural Conservation Authority (NTCA)
toxin) that is produced by a bacteria —
Clostridium botulinum. ! Project Tiger was launched in
1973 with 9 tiger reserves for
conserving our national animal, the
Sambhar Lake tiger. Currently, the Project Tiger
 The Sambhar Salt Lake is India's largest coverage has increased to 50, spread
inland saltwater body located near out in 18 tiger range states.
Jaipur in Rajasthan.
! The tiger reserves are constituted on
 The lake is surrounded on all sides by a core/buffer strategy. The core areas
the Aravali hills. have the legal status of a national
 It is the source of most of Rajasthan's park or a sanctuary, whereas the
salt production. buffer or peripheral areas are a mix of
forest and non-forest land, managed
 Sambhar has been designated as as a multiple use area.
a Ramsar site (recognized wetland of
international importance) because the ! It is an ongoing Centrally Sponsored
wetland is a key wintering area for tens Scheme of the Ministry of
of thousands of flamingos and other Environment, Forests and Climate
birds that migrate from northern Asia. Change providing central assistance to
the tiger States for tiger conservation
 The bacteria are commonly found in in designated tiger reserves.
the soil, rivers, and seawater. It affects both
! The National Tiger Conservation
humans and animals.
Authority (NTCA) is a statutory
 The bacteria also need anaerobic (absence of body of the Ministry, with an
oxygen) conditions and do not grow in acidic overarching supervisory/coordination
conditions. role, performing functions as provided
 It affects the nervous system of birds, in the Wildlife (Protection) Act, 1972.
leading to paralysis in their legs and ! The NTCA was launched in 2005,
wings. following the recommendations of
 The outbreaks of avian botulism tend to the Tiger Task Force. It was given
occur when average temperatures are above statutory status by 2006 amendment
21 degrees celsius, and during droughts. of Wildlife (Protection) Act, 1972.
TARGET PT 2020 135

TEST
DAY - 40

Time Allowed: 30 mins Maximum Marks: 50

1. Which of the following national parks 3. A total of 18 biosphere reserves of


are situated in Assam? India have been included in the World
1. Orang Network of Biosphere Reserves.

2. Nameri Which of the above statements is/are


3. Dibru-Saikhowa incorrect?

4. Gorumara (a) 1 only


(b) 2 only
Select the correct option using the codes
given below: (c) 1 and 3 only
(a) 2 and 3 only (d) 1, 2 and 3

(b) 1, 2, and 3 only 4. Which of the following national park


(c) 1 and 4 only has the largest area?

(d) 1, 2, 3, and 4 (a) Desert National Park


(b) Jim Corbett National Park
2. Consider the following statements
regarding a sanctuary: (c) Hemis National Park

1. It is a marine wildlife sanctuary (d) Gangotri National Park


located between the Dhamra River and
5. Which of the following statements
Brahmaniriver.
regarding Mahatma Gandhi Marine
2. It is very famous for its nesting beach National Park are correct?
for olive ridley sea turtles. 1. It was created to protect marine life
Which of the following sanctuaries has been such as the corals and nesting sea
described above? turtles prevalent in the area.
2. It is also known as Wandoor National
(a) Bhitarkanika
Park.
(b) Gahirmatha 3. It is situated in Gujarat.
(c) Chilika lake
Select the correct option using the codes
(d) Rann of Kutch given below:

3. Consider the following statements: (a) 2 and 3 only


1. Man and the Biosphere Programme (b) 1 and 3 only
(MAB) is an intergovernmental (c) 1 and 2 only
scientific program, launched in 1971
by UN Environment. (d) 1, 2, and 3

2. The latest World Congress of Biosphere 6. Which of the following national parks
Reserves took place in Lima, Peru in is also a Natural World Heritage Site
2016. in India?
136 TARGET PT 2020

(a) Jim Corbett National Park (c) Kerala


(b) Silent Valley National Park (d) Telangana
(c) KeibulLamjao National Park
11. Which of the following national parks
(d) Keoladeo Ghana National Park are in the Union Territory of Jammu
and Kashmir?
7. Which of the following pairs is/are
correctly matched? 1. Dachigam National Park
1. Largest national park: Gangotri NP 2. Kishtwar National Park
2. Largest biosphere reserve: Gulf of 3. Hemis National Park
Mannar BR
4. Salim Ali National Park
3. Largest wildlife sanctuary: Wild Ass
WLS Select the correct option using the codes
given below:
Select the correct option using the codes
given below: (a) 1 and 2 only

(a) 2 only (b) 2 and 4 only

(b) 1 and 3 only (c) 1, 2, and 4 only

(c) 1 and 2 only (d) 1, 2, 3 and 4


(d) 1, 2, and 3 12. Which of the following statements
8. Which of the following biosphere reserves regarding Hope Spots is/are correct?
is not included under UNESCO-Man and 1. It is an initiative pioneered by the
Biosphere Program? International Union for Conservation
(a) Khangchendzonga of Nature.

(b) Nokrek 2. Only the Andaman and Nicobar Islands


are the hope spots of India.
(c) Pachmarhi
(d) Manas Select the correct option using the codes
given below:
9. Which of the following statements is/
are correct? (a) 1 only

1. Pitti Wildlife Sanctuary is the (b) 2 only


only marine protected area of (c) Both 1 and 2
Lakshadweep.
(d) Neither 1 nor 2
2. It is located on the Bangaram Atoll.

Select the correct option using the codes 13. Select the correct statement(s)
given below: regarding Indian estuarine
ecosystem:
(a) 1 only
(a) In India, major estuaries are found in
(b) 2 only
the Arabian Sea
(c) Both 1 and 2
(b) Estuaries on the eastern coast are
(d) Neither 1 nor 2 smaller than those on the west coast
10. Kappatagudda wildlife sanctuary is (c) Intensive aquaculture is the reason
situated in which state? for the declining productivity of Indian
(a) Tamil Nadu estuaries
(b) Karnataka (d) Both (a) and (b)
TARGET PT 2020 137

14. Mangroves exhibit a Viviparity (c) Hydrophytes


mode of reproduction. This (d) Hydric soils
implies:
(a) Seeds germinate in the tree itself before 18. With reference to wetland classification,
falling to the ground select the correct statement:

(b) The new plant grows from an outgrowth (a) Wetlands are only found along the
in the plant body coastal regions.
(c) The new plant grows from the (b) The creek is an example of a man-made
fragments of the parent plant after wetland.
they fall on the ground
(c) A pond represents an inland wetland
(d) Seeds germinate after falling to the ecosystem.
ground.
(d) Coral reefs do not classify as a wetland
15. Coral is a living animal having as per the definition of wetlands.
a symbiotic relationship with
‘zooxanthellae’ algae. In this respect, 19. Select the correct statement regarding
select the correct statement: Montreux Record:
(a) Zooxanthellae live up on coral to get (a) It is a list of wetlands of national
adequate light. importance.
(b) Corals supply zooxanthellae with (b) It is not a part of the Ramsar
nutrients which it cannot get on the Convention.
seafloor.
(c) It contains only those sites where
(c) Zooxanthellae protect the corals from adverse changes in ecological character
ocean predators like big fish, sharks have occurred.
etc.
(d) Chilika Lake and Keoladeo National
(d) All of the above
Park are the only Indian sites in
Montreux Record.
16. Coral reefs are classified 20. Which of the following is not the
depending on their locations characteristic of an estuarine
into fringing, barrier, patch, and ecosystem?
atoll. In this context, identify the
(a) Semi-enclosed water body
correct statement?
(b) High activity of waves
(a) Patch reefs are contiguous with the
shores. (c) High productivity
(b) Fringing reefs are offshore structures (d) Highly populated
running parallel to coastlines.
(c) Atolls are circular or semi-circular
reefs. 21. With reference to Islamic Cooperation
countries (OIC), consider the following
(d) Barrier reefs are the most common statements
types of reefs.
1. It is the second largest inter-
17. Wetlands are transitional areas governmental organization after the
intermediate in character between United Nations.
deep-water and terrestrial habitats.
2. The OIC has permanent delegations to
Which of the following is not a
the United Nations and the European
characteristic of wetland ecosystem?
Union.
(a) Areas of marine water with depth at
low tide exceeding 6 metres. 3. India is one of its founding member

(b) Waterlogged soils for at least 7 days Choose the correct option from the
during the growing season. following
138 TARGET PT 2020

(a) 1 and 2 only 24. Which of the following are the objectives
of Bharat Darshan Scheme?
(b) 2 and 3 only
1. To position tourism as a major engine
(c) 1 and 3 only of economic growth and job creation;
(d) 1, 2 and 3 2. Develop circuits having tourist
potential in a planned and prioritized
22. Consider the following statements manner
regarding Good Governance Index,
3. Promote cultural and heritage value of
1. It is used to provide quantifiable data the country to generate livelihoods in
to compare the state of governance in the identified regions
all states and UTs.
4. Build rural infrastructure
2. The index does not consider Public
Health and Infrastructure. Choose the correct option
3. All the indicators are given same (a) 1, 2 and 3
weightage under one Governance (b) 1 and 2
Sector to calculate the value.
(c) 1, 2, 3 and 4
Which of the following statements is/are
(d) 1 and 3
correct?
(a) 2 and 3 only 25. Quick Reaction Surface to Air Missile
(QRSAM) system was successfully test
(b) 1 only fired by DRDO recently. Which of the
(c) 3 only following statements are incorrect
about Quick Reaction Surface to Air
(d) 1, 2 and 3 Missile (QRSAM) system?
23. With reference to Khadi and Village 1. It uses solid fuel propellant and has a
Industries Commission (KVIC), strike range of 25-30 km with capability
consider the following statements of hitting multiple targets.
1. It is a statutory body. 2. The missile is an all-weather,
all-terrain surface-to-air
1. It is an apex organisation under the missile equipped with electronic
Ministry of tribal affairs. counter measures against jamming by
2. It has the economic objective of aircraft radars
producing saleable articles. 3. It will be commissioned to Indian
Navy.
Choose the correct option from the
following Choose the correct option
(a) 1 and 2 only (a) Only 1
(b) 2 and 3 only (b) 1 and 2 only
(c) 1 and 3 only (c) 1, 2 and 3
(d) 1, 2 and 3 (d) Only 3

**********
TARGET PT 2020 139

ANSWER HINTS
DAY - 40

1. Correct Option: (b) 4. Correct Option: (c)


Explanation: Explanation:

National parks in Assam National parks

 There are five national parks in Assam.  The top five largest national parks are Hemis
National Park(Jammu and Kashmir),
 These are Dibru-Saikhowa national park,
Desert National Park(Rajasthan), Gangotri
Kaziranga NP, Manas NP, Nameri NP,
National Park(Uttarakhand), Namdapha
and Rajiv Gandhi Orang NP.
National Park(Arunachal Pradesh), and
 Gorumara national park is in West Khangchendzonga National Park(Sikkim).
Bengal.
 Jim Corbett National Park is in 9th place.

2. Correct Option: (b) 5. Correct Option: (c)


 The above statements are about
Explanation:
Gahirmatha marine sanctuary
 It is situated in Odisha between the Mahatma Gandhi Marine National Park
Dhamra River and Brahmaniriver.  It is situated near Wandoor in South
 It is very famous for its nesting beach for Andaman.
olive ridley sea turtles  The park was created on 24 May 1983
under the Wildlife Protection Act of 1972 to
3. Correct Option: (c) protect marine life such as the corals and
Explanation: nesting sea turtles prevalent in the area.
 There are 2 major island groups in the
Man and the Biosphere Programme
park viz. the Labyrinth Islands and the
 Man and the Biosphere Programme (MAB) Twin Islands.
is an intergovernmental scientific program,
launched in 1971 by UNESCO. 6. Correct Option: (d)

 The latest World Congress of Biosphere Explanation:


Reserves took place in Lima, Peru, from 14
Keoladeo Ghana National Park
to 17 March 2016.
 Out of the total 38 World Heritage Sites,
 The program has identified so far, 142
there are seven natural world heritage
biosphere reserves across the globe as a
sites in India.
part of the World Network of Biosphere
Reserves.  These are Great Himalayan National
Park, Kaziranga National Park, Keoladeo
 India has a total of 18 biosphere reserves of
National Park, Manas Wildlife Sanctuary,
which 11 are included in the Network.
Nanda Devi and Valley of Flowers National
 The latest addition is the Agasthyamalai Parks, Sundarbans National Park, and the
Biosphere Reserve. Western Ghats.
140 TARGET PT 2020

7. Correct Option: (a) 10. Correct Option: (b)


Explanation: Explanation:

Protected areas in India Kappatagudda wildlife sanctuary

 Presently, there are 18 notified biosphere  It is in Karnataka.


reserves in India. Gulf of Mannar BR is the  On 16 May 2019, the state government
largest biosphere reserve with 10,500 km2 changed its status from the ‘Conservation
though the core area is of only 5.55 km2. Reserve’ to the ‘Wildlife Sanctuary’.

 There are 551 existing wildlife sanctuaries  The sanctuary encompasses Gadag,
Mundargi, and Shirahatti, and is spread
in India covering an area of 119775.80
across 244.15 sq km.
km2, which is 3.64 % of the geographical
area of the country. Kachchh Desert WLS 11. Correct Option: (c)
is the largest with the area of 7506.22 km2.
Wild Ass WLS is the second largest. Explanation:

 There are 104 existing national parks National parks in the Union Territories of
in India covering an area of 40,501 km2, Jammu & Kashmir and Ladakh
which is 1.23% of the geographical area of  There were 4 national parks in the Jammu
the country. Desert NP with 3162 km2 is and Kashmir viz. Dachigam National Park,
the largest national park,Gangotri NP is Kishtwar National Park, Hemis National
the second largest. Park, and Salim Ali National Park.
 After the delimitation, 3 are in J&K and 1
8. Correct Option: (d) in Ladakh.

Explanation:  Hemis (or Hemis High Altitude National


Park) is a high altitude national park in
Man and Biosphere Program network in the eastern Ladakh. Globally famous for
India its snow leopards, it is believed to have the
highest density of them in any protected
 Launched in 1971, UNESCO’s Man and area in the world. It is the only national
the Biosphere Programme (MAB) is an park in India that is north of the Himalayas
Intergovernmental Scientific Programme inside the Palearctic ecozone.
that aims to establish a scientific basis for  Dachigam National park is located in the
the improvement of relationships between Zabarwan Range of the western Himalayas
people and their environments. in J and K.

 Out of 18 notified biosphere reserves  Kishtwar National Park is located in the


in India, 11 are included in the MAB Kishtwar district of Jammu and Kashmir.
program.  Salim Ali National Park or City Forest
National Park was a national park located
 These are Nilgiri, Gulf of Mannar, in Srinagar.
Sunderban, Nanda Devi, Nokrek,
Pachmarhi, Similipal, Achanakmar- 12. Correct Option: (d)
Amarkantak, Great Nicobar, Agasthyamala,
Explanation:
and Khangchendzonga.
Hope spots
9. Correct Option: (a)
 Hope Spots are special places that are
Explanation: critical to the health of the ocean — Earth’s
blue heart. They are about recognizing,
Pitti Wildlife Sanctuary empowering and supporting individuals
and communities around the world in their
 Pitti Bird Island, which is important for efforts to protect the ocean.
breeding and migratory populations of
 Dr. Sylvia Earle introduced the concept in
many marine terns, was declared as a
her 2009 TED talk and since then the idea
wildlife sanctuary in 2000. has inspired millions across the planet. Her
 It is the only marine protected area of initiative, Mission Blue has partnered with
Lakshadweep. the International Union for Conservation of
Nature (IUCN) and receives support from
 It is on the Agatti Atoll (coral reef). National Geographic, Rolex, and Google.
TARGET PT 2020 141

 Hope Spots are often areas in the ocean  Submergence of catchment areas due to
that need new protection, but they can rising in water level
also be existing MPAs where more action
 Change in biodiversity profile, affecting the
is needed. They can be large, they can be
small, but they all provide hope due to: production and productivity

! A special abundance or diversity of 14. Correct Option: (a)


species, unusual or representative
species, habitats or ecosystems Explanation:
! Particular populations of rare Characteristics of Mangroves
threatened or endemic species
 Mangroves exhibit Viviparity mode of
! A site with potential to reverse damage reproduction i.e. seeds germinate in the
from negative human impacts tree itself (before falling to the ground).
! The presence of natural processes  This is an adaptive mechanism to overcome
such as major migration corridors or the problem of germination in saline
spawning grounds water.
! Significant historical, cultural or
spiritual values 15. Correct Option: (a)
! Particular economic importance to the Explanation:
community.
Coral Reefs
! The two island groups (Lakshadweep
and Andaman & Nicobar Islands) are  Coral is actually a living animal.
the first hope spots in India.
 Coral has a symbiotic relationship
(each gives something to the other and
13. Correct Option: (c) gets something back in return) with
Explanation: ‘zooxanthellae’ microscopic algae which
live on coral [i.e. instead of living on the
Indian Estuarine Ecosystem seafloor, the algae lives upon the coral
 India has 14 major, 44 medium and 162 which is closer to the ocean surface and so
minor rivers drains into the sea through that the algae gets adequate light].
various estuaries.  Zooxanthellae assist the coral in nutrient
 Estuaries are an important and distinct production through its photosynthetic
component of the coastal landscape with activities. These activities provide the
highly complex ecosystems, varying coral with fixed carbon compounds for
physical-chemical properties and having energy, enhance calcification, and mediate
highly diverse flora and fauna. elemental nutrient flux.
 Major estuaries occur in the Bay of  The tissues of corals themselves are
Bengal. actually not the beautiful colors of the
coral reef but are instead clear (white).
 Many estuaries are locations of some of the
The corals receive their coloration from the
major seaports.
zooxanthellae living within their tissues.
 Most of India’s major estuaries occur on
 The host coral polyp in return provides its
the east coast. In contrast, the estuaries on
the west coast are smaller. zooxanthellae with a protected environment
to live within, and a steady supply of carbon
Issues with Indian Estuarine Ecosystem dioxide for its photosynthetic processes.
 Changes in water flow- either in excess or
16. Correct Option: (c)
less than required in various estuaries like
Narmada, Krishna, Godavari, Pulicat Explanation:
 Pollution through industries and combined Classification of Coral Reefs
city sewage
 The coral reefs are classified depending on
 Dredging activities like in Hooghly
their locations into fringing, patch, barrier,
 Changing land-use and expanding rural- and atoll.
urban settlements
 The fringing reefs are contiguous with the
 Reclamation of the fringed areas for shore and they are the most common - by
intensive aquaculture occurring reef form, found in Andaman.
142 TARGET PT 2020

 Patch reefs are isolated and discontinuous  It highlights those sites where adverse
patches, lying shoreward of offshore reef changes in ecological character have
structures as seen in the Palk Bay, Gulf of occurred, are occurring, or are likely to occur
Mannar and Gulf of Kutch as a result of technological developments,
pollution or other human interference and
 Barrier reefs are linear offshore reef which are therefore in need of priority
structures that run parallel to coastlines conservation attention.
and arise from submerged shelf platforms.
The water body between the reef and the  It is maintained as part of the Ramsar List.
shore is termed as a lagoon. Barrier reefs  Montreux Record is employed to identify
are seen in Nicobar and Lakshadweep. priority sites for positive national and
 Atolls are circular or semicircular reefs that international conservation attention.
arise from subsiding seafloor platforms Sites may be added to and removed from
as coral reef-building keeps ahead of the Record only with the approval of the
Contracting Parties in which they lie.
subsidence. The examples are the atolls of
Lakshadweep and Nicobar.  Chilika Lake, Orissa was placed on the
Montreux Record in 1993 due to siltation,
 Among the four major reef areas of India,
which was choking the mouth of the lake.
Andaman and Nicobar Islands are found
Following the rehabilitation efforts of the
to be very rich in species diversity followed government, it was removed from the
by the Lakshadweep Islands, the Gulf of Record in 2002.
Mannar and finally the Gulf of Kutch
 Keoladeo National Park, Rajasthan was
17. Correct Option: (a) placed on the Montreux Record in 1990 due
to water shortage and unbalanced grazing
Explanation: regime around it.

Characteristics of Wetlands  Loktak Lake, Manipur was included on


the Montreux Record in 1993 (signifying
 Wetlands are areas of marsh, fen, peatland/ habitat degradation), as a result of
water, whether natural (or) artificial, ecological problems such as deforestation
permanent (or) temporary with water that in the catchment area, the infestation of
is static (or) flowing, fresh, brackish (or) water hyacinth and pollution.
salt, including areas of marine water the
depth of which at low tide does not exceed 20. Correct Option: (b)
6 metres.
Explanation:
 Covered by water (or) has waterlogged soil
for at least seven days during the growing Characteristics of Estuaries
season.  An Estuary is a semi-enclosed coastal body
 Adopted plant life (hydrophytes) of water with one or more rivers or streams
flowing into it.
 Hydric soils (not enough O2 available for
some plants)  It has a free connection with the open sea.
 The complete salinity range from 0-35 ppt
18. Correct Option: (c) is seen from the head (river end) to the
mouth (sea end) of an estuary.
Explanation:
 An estuary has very little wave action, so it
Wetland Classification provides a calm refuge from the open sea. It
 Wetlands can be classified as Inland provides shelter for some of the animals.
and Coastal wetlands which can further  It is the most productive region as it
be divided into natural and man-made receives a high amount of nutrients from
wetlands. fresh and marine water.
 Estuaries are the most heavily populated
19. Correct Option: (a)
areas throughout the world, with about
Explanation: 60% of the world’s population living along
estuaries and the coast. The banks of
Montreux Record estuarine channels form a favored location
for human settlements, which use the
 It is a register of wetland sites on the List
estuaries for fishing and commerce.
of Wetlands of International Importance
and a principal tool under the Ramsar  They also act as a filter for some dissolved
convention. constituents in river water; these
TARGET PT 2020 143

precipitate in the zone where river water  OIC aims to preserve Islamic values,
meets seawater. safeguard and defend the national
sovereignty and independence of member
 More important is the trapping of suspended
states and to contribute to international
mud and sand carried by rivers which leads
peace and security.
to delta formations around estuaries.
 While the organisation has been known for
21. Correct option: (a) its cultural and social projects, its political
influence has been relatively limited.
Explanation
 OIC doesn’t have a unified voice because
Statement 3 is incorrect: India was most of its member countries are not
invited for the first time ever in the recent democracies. So, while their populations
summit. It is not a founding member of the may be in agreement they do not always
represent the views of their populations.
organization.
Supplementary notes 22. Correct option: (b)

Islamic Cooperation countries (OIC) Explanation


 The Organisation of Islamic Cooperation Statement 2 and 3 are incorrect: The
(OIC) is the second largest inter- GGI takes into consideration ten sectors:
governmental organization after the
United Nations with a membership of 57  Agriculture and Allied Sectors,
states spread over four continents.  Commerce & Industries,
 The Organization was established upon a  Human Resource Development,
decision of the historical summit which took
place in Rabat, Kingdom of Morocco on 12th  Public Health,
Rajab 1389 Hijra (September 25, 1969).  Public Infrastructure & Utilities,
 In 1970, the first-ever meeting of the  Economic Governance,
Islamic Conference of Foreign Minister
(ICFM) was held in Jeddah, which decided  Social Welfare & Development,
to establish a permanent secretariat.  Judicial & Public Security,
 The secretariat comprises a secretary-  Environment and
general who is the Chief Administrative
Officer of the organisation.  Citizen-Centric Governance.

 The majority of its member states are Supplementary notes


Muslim-majority countries, while others
have significant Muslim populations, Good Governance Index
including several African and South  These ten Governance Sectors are
American countries. measured on total 50 indicators. Difference
 Over the last 40 years, the membership indicators are given different weightage
has grown from its founding members of 30 under one Governance Sector to calculate
to 57 states. the value. E.g. Under Agriculture & Allied
Sector, there are 6 indicators with different
 The OIC has permanent delegations to the weightage, namely: Growth rate of
United Nations and the European Union. agriculture and allied sector (0.4), growth
The official languages of the OIC are rate of food grains production (0.1), growth
Arabic, English, and French. rate of horticulture produce (0.1), growth
rate of milk production (0.1), growth rate of
 While the 22 members of the Arab League
meat production (0.1) and crop insurance
are also part of the OIC, the organisation has
(0.2).
several significant non-Arab member states,
including Turkey, Iran and Pakistan.  The states and UTs are divided into three
groups: a). Big States, b). North-East &
 It also has five observer members, including
Hill States and c). UTs. The states and UTs
Russia and Thailand.
are ranked on all indicators separately, at
 The OIC holds an Islamic Summit once the same time composite ranking is also
every three years. calculated for these states and UTs under
their respective groups based upon these
 Member states each get a vote. Each
indicators.
member state can table a resolution and
then others can vote on it or suggest  The Good Governance Index is a uniform
tweaks. tool across States to assess the Status
144 TARGET PT 2020

of Governance and impact of various up of a strong rural community spirit.


interventions taken up by the State
Government and UTs. 24. Correct Answer: (a)
 The objectives of GGI are to provide
Explanatory Notes: 4th statement is
quantifiable data to compare the state of
governance in all states and UTs, enable incorrect. Bharart Darshan Scheme does
states and UTs to formulate and implement not aim at building rural infrastructure.
suitable strategies for improving governance
Supplementary Notes
and shift to result oriented approaches and
administration. Salient Features of Bharat Darshan
Scheme
23. Correct option: (c)
 100% centrally funded for the project
Explanation components undertaken for public
funding.
Statement 2 is incorrect: It is an apex
organisation under the Ministry of Micro,  To leverage the voluntary funding available
Small and Medium Enterprises, with for Corporate Social Responsibility
regard to khadi and village industries (CSR) initiatives of Central Public Sector
within India. Undertakings and corporate sector.
 Funding of individual project will vary from
Supplementary notes
state to state and will be finalised on the
Khadi and Village Industries Commission basis of detailed project reports prepared
(KVIC) by PMC (Programme Management
Consultant). PMC will be a national level
 The Khadi and Village Industries consultant to be appointed by the Mission
Commission (KVIC) is a statutory body Directorate.
formed by the Government of India, under
the Act of Parliament, ‘Khadi and Village  A National Steering Committee (NSC) will
Industries Commission Act of 1956’. be constituted with Minister in charge
of M/O Tourism as Chairman, to steer
 It is an apex organisation under the
the mission objectives and vision of the
Ministry of Micro, Small and Medium
scheme.
Enterprises, with regard to khadi and
village industries within India.  A Mission Directorate headed by the
Member Secretary, NSC as a nodal officer
 It seeks to - “plan, promote, facilitate,
organise and assist in the establishment will help in identification of projects
and development of khadi and village in consultation with the States/ UTs
industries in the rural areas in coordination governments and other stakeholders.
with other agencies engaged in rural
Scheme Objectives
development wherever necessary.
 To position tourism as a major engine of
 The KVIC may also undertake directly or
economic growth and job creation;
through other agencies studies concerning
the problems of Khadi and/or village  Develop circuits having tourist potential in
industries besides research or establishing a planned and prioritized manner
pilot projects for the development of Khadi
and village industries.  Promote cultural and heritage value of
the country to generate livelihoods in the
 The KVIC is authorized to establish and identified regions
maintain separate organisations for the
purpose of carrying out any or all of the  Enhancing the tourist attractiveness
above matters besides carrying out any in a sustainable manner by developing
other matters incidental to its activities. world class infrastructure in the circuit /
destinations
 The broad objectives that the KVIC has set
before it are  Follow community based development and
pro-poor tourism approach
! The social objective of providing
employment.  Creating awareness among the local
communities about the importance of
! The economic objective of producing tourism for them in terms of increased
saleable articles.
sources of income, improved living
! The wider objective of creating self- standards and overall development of the
reliance amongst the poor and building area.
TARGET PT 2020 145

 To create employment through active  The test flights had successfully


involvement of local communities; demonstrated their aerodynamics,
 Harness tourism potential for its effects propulsion, structural performance and
in employment generation and economic high maneuvering capabilities.
development  Features
 To make full use of the potential and
! It uses solid fuel propellant and
advantages in terms of available
infrastructure, national culture and has a strike range of 25-30 km with
characteristic strong points of each and capability of hitting multiple targets.
every region throughout the country by ! Developed by DRDO
development of theme based circuits.
! It is capable of hitting the low flying
 Development of tourist facilitation services
objects.
to enhance visitor experience/satisfaction.
! The missile is an all-weather,
25. Correct Answer (d) all-terrain surface-to-air
missile equipped with electronic
Explanation: statements 1 and 2nd are
counter measures against jamming by
correct. Only third statement is incorrect.
aircraft radars
Quick Reaction Surface to Air Missile
(QRSAM) system will be commissioned to ! The missile can be mounted on a truck
Indian Army. and is stored in a canister.

Supplementary Notes ! The missile is equipped with a


midcourse inertial navigation
 DRDO successfully test-fired its Quick
system with a two-way data link and
Reaction Surface to Air Missile (QRSAM)
system, likely to be inducted into the armed a DRDO-developed terminal active
forces by 2021, from a base off Odisha seeker. The system has the capability
coast. to search and track targets while
moving.
 The missile, developed by the Defence
Research and Development Organisation ! QRSAM is a compact weapon system
(DRDO) for the Indian Army, was flight- and is mobile. It has a fully automated
tested from the Integrated Test Range Command and Control System. The
(ITR) at Chandipur missile system comprises of two four-
 It has been developed to replace the ‘Akash’ walled radars both of which encompass
missile defence system, and has 360-degree a 360-degree coverage, namely, the
coverage. Active Array Battery Surveillance
 The first test firing of the missile took place Radar and the Active Array Battery
on 4 June 2017. This was followed by the Multifunction Radar, apart from the
second successful test on 3 July 2017. launcher.

**********
146 TARGET PT 2020

TEST
DAY - 41

Time Allowed: 30 mins Maximum Marks: 50

1. Consider the following statements 3. It is legally binding on the Parties.


regarding the Convention on the
Conservation of Migratory Species of Select the correct answer using the code
Wild Animals: given below:
1. It is the only global convention (a) 2 only
specializing in the conservation of (b) 1 only
migratory species.
(c) 2 and 3 only
2. It is also known as the Bonn
Convention. (d) None of the above.
3. It is a legally binding treaty. 4. Which of the following is/are correct
Which of the above statements is/are regarding Biodiversity Financial
correct? Initiative (BIOFIN)?
1. It is a global initiative of the United
(a) 3 only
Nations Development Programme for
(b) 1 and 2 only funding biodiversity projects.
(c) 2 only 2. It was launched during the 2010
Convention on Biodiversity (Conference
(d) 1, 2, and 3
of Parties) 10 in Nagoya.
2. Which of the following is the National 3. National Biodiversity Authority is the
Implementing Entity (NIE) for Green nodal agency for its implementation in
Climate Fund in India? India.
(a) Small Industries Development Bank of Select the correct answer using the code
India given below:
(b) Ministry of Environment, Forests and (a) 1, 2 and 3
Climate Change
(b) 1 and 2 only
(c) National Bank for Agriculture and
Rural Development (c) 2 and 3 only

(d) Reserve Bank of India (d) 1 and 3 only

3. Which of the following statements 5. Which of the following pairs regarding


regarding the Convention on conventions and their protocols/
International Trade in Endangered Agreement are correctly matched?
Species of Wild Fauna & Flora (CITES) 1. Convention on Biological Diversity:
is/are incorrect? Cartagena Protocol
1. It is an international agreement 2. United Nations Framework Convention
between governments on Climate Change: Nagoya Protocol
2. It is also known as the Washington 3. Vienna Convention for the Protection of
Convention. the Ozone Layer: Kigali Amendment
TARGET PT 2020 147

Select the correct answer using the code 3. It publishes both the Living Planet
given below: Report and the Living Planet Index.
(a) 1 and 3 only Which of the above statements are correct?
(b) 1 and 2 only (a) 1 and 2 only
(c) 2 and 3 only (b) 1 and 3 only
(d) 1, 2 and 3 (c) 2 and 3 only
(d) 1, 2, and 3
6. Which of the following statements is/
are correct? 9. Which of the following statements
1. The Stockholm Convention is a global regarding the Intergovernmental Panel
treaty to protect human health and the on Climate Change is/are correct?
environment from persistent organic 1. Intergovernmental Panel on Climate
pollutants (POPs). Change was established by the World
2. The Green Climate Fund is the Meteorological Organization and UN
designated interim financial mechanism Environment.
for the Stockholm Convention. 2. It has its headquarter in Nairobi.

Select the correct option using the code Select the correct option using the code
given below: given below:

(a) 1 only (a) 1 only

(b) 2 only (b) 2 only

(c) Both 1 and 2 (c) Both 1 and 2


(d) Neither 1 nor 2
(d) Neither 1 nor 2
10. Which of the following pairs are
7. Consider the following statements
correctly matched?
with respect to an international
organization: Organizations: Headquarters
1. It was established by the first 1. International Union Gland
United Nations conference on the for Conservation of
Nature:
environment.
2. Global Green California
2. Maurice Strong was its first director.
Growth Institute:
3. “Earthwatch” is coordinated by this
3. 350.org: Seoul
organization.
Select the correct option using the code
Which of the following organization has given below:
been described above?
(a) 1 only
(a) International Union for Conservation
of Nature (b) 2 and 3 only
(c) 1 and 2 only
(b) Birdlife International
(d) 1 and 3 only
(c) United Nations Environment
Programme 11. Which of the following statements is/
(d) World Wide Fund for Nature are correct?
1. The Global Environment Facility
8. Consider the following statements was established during the Rio Earth
regarding WWF? Summit, 1992.
1. It is the world’s largest conservation 2. The Green Climate Fund was
organization. established during the Rio+5 Summit.
2. It is funded primarily by the World 3. Adaptation Fund (AF) was established
Bank. under the Kyoto Protocol in 2001.
148 TARGET PT 2020

Select the correct option using the code 3. Ministry of Earth Sciences, Indian
given below: Space Research Organization Ministry
of Environment and Department
(a) 1 only of Science and Technology are
(b) 2 and 3 only implementing agencies of this
programme.
(c) 1 and 2 only
Which of the above statements is/are correct?
(d) 1 and 3 only
(a) (a) 1 and 2 only
12. Which of the following statements (b) (b) 1 only
regarding TRAFFIC is/are correct?
(c) (c) 2 and 3 only
1. It is a joint program World Wide Fund
for Nature and World Bank. (d) (d) 1, 2 and 3

2. Its headquarters is located in 15. Consider the following statements


Cambridge, United Kingdom. regarding the International Whaling
Commission (IWC):
Select the correct option using the code 1. India is a member state.
given below:
2. International Convention for the
(a) 1 only Regulation of Whaling is the legal
framework that established the IWC
(b) 2 only
in 1946.
(c) Both 1 and 2 3. Indian Ocean Whale Sanctuary is one
(d) Neither 1 nor 2 of the sanctuaries designated by IWC.

Which of the above statements are correct?


13. With reference to the Global
Environment Outlook, consider the (a) 1 and 2 only
following statements: (b) 2 and 3 only
1. It is a flagship environmental (c) 1 and 3 only
assessment of the Intergovernmental
(d) 1, 2 and 3
Panel on Climate Change (IPCC).
2. The theme of the sixth Global 16. Which one of the following magazine is
Environment Outlook is “Healthy published by Birdlife International?
Planet, Healthy People”. (a) World Birdwatch

Which of the above statements is/are (b) Birding world


correct? (c) Birders world
(a) 1 only (d) Wild bird
(b) 2 only 17. Consider the following statements
(c) Both 1 and 2 with respect to the United Nations
Environmental Programme (UNEP):
(d) Neither 1 nor 2 1. It was founded as a result of the
Stockholm conference in 1972.
14. With reference to the Black Carbon
Research Initiative, consider the 2. It coordinates the UN’s environmental
activities, assisting developing countries
following statements:
in implementing environmentally
1. It is an Indian initiative launched as sound policies and practices.
a part of the National Carbonaceous
3. UNEP facilitates the transfer of
Aerosols Program (NCAP). knowledge and technology for
2. It is to monitor the black carbon sustainable development.
aerosols and assess its impacts on the 4. It is headquartered in Geneva,
microclimates. Switzerland.
TARGET PT 2020 149

Which of the above statements are correct? 2. It supports the implementation, follow-
up, and review of the Sendai Framework
(a) 2 and 4 only
for Disaster Risk Reduction.
(b) 1, 2 and 3 only
3. It publishes the Global Assessment
(c) 1 and 3 only Report on Disaster Risk Reduction in
(d) 1, 2, 3 and 4 collaboration with the World Bank.

Which of the statements given above are


18. With reference to the Environment
correct
Pollution Control Authority (EPCA),
consider the following statements: (a) 1 and 2 only
1. EPCA is a supreme court mandated (b) 2 and 3 only
non-statutory body.
(c) 1 and 3 only
2. The Minister of Environment & Forests
(MoEF) holds the chairmanship of (d) 1, 2 and 3
EPCA.
21. “Nagpur Resolution”, which was seen
3. It has the power to take suo-moto recently in news, refers to?
cognizance of matters related to
environmental pollution. (a) It is United Nations Security Council
resolution that extends the sanctions
Which of the above statements are against the Central African Republic.
incorrect?
(b) The Convention establishes rules of
(a) 1 and 2 only airspace, aircraft registration and
safety, security, and sustainability,
(b) 2 and 3 only
and details the rights of the signatories
(c) 1 and 3 only in relation to air travel.
(d) 1, 2 and 3 (c) It is a convention that plays a role
in the implementation of several
19. Consider the following statements international conventions on nature
regarding the United Nations conservation and biodiversity.
Framework Convention on Climate
Change (UNFCCC): (d) It is to empower the citizens by policy
interventions for better service delivery
1. It is an international treaty drawn at
through timely updation of citizen’s
the UN Conference on Environment
charters, implementation of enactments
and Development, Rio de Janeiro,
1992. and benchmarking standards for
continuous improvement.
2. Kyoto Protocol implemented the
objectives of UNFCCC to fight global 22. With reference to Rohtang Tunnel,
warming by reducing the concentration consider the following statements
of greenhouse gases.
1. It has been recently named after VD
Which of the above statements is/are savarkar
correct? 2. Upon completion, it will be the world’s
(a) 1 only longest highway tunnel at an altitude
of above 10,000 feet.
(b) 2 only
3. It will reduce the distance between
(c) Both 1 and 2 North Jammu and North Kashmir.
(d) Neither 1 nor 2 Choose the correct option from the
following
20. With reference to ‘The United Nations
Office for Disaster Risk Reduction ‘, (a) 1 and 2 only
consider the following statements: (b) 2 only
1. Its functions span the social, economic,
(c) 3 only
environmental as well as humanitarian
fields. (d) 1, 2 and 3
150 TARGET PT 2020

23. Consider the following statements (c) Punchhi Commission


regarding Joint Comprehensive Plan
(d) Sarkaria Commission
of Action (JCPOA)
1. It is also known as the “Iran deal 25. Consider the following statements
2. The agreement is made between Iran, regarding Chief of Defence Staff,
the P5+1and the European Union. 1. It will be in the rank of four star
general.
Choose the correct option from the
following 2. The Chief of Defence Staff will head
the Department of Military Affairs
(a) 1 only
(DMA).
(b) 2 only
3. One of the tasks of CDS will be
(c) Both 1 and 2 Promoting use of indigenous equipment
(d) Neither 1 nor 2 by the Services.

24. Which of the following committee Which of the following statements is/are
recommended the institution of correct?
collegium system for the appointment (a) 1 and 2 only
of chief election commissioner and
election commissioners? (b) 2 only
(a) Dinesh Goswami Committee (c) 1 and 3 only
(b) Administrative Reforms Committee (d) 1, 2 and 3

**********
TARGET PT 2020 151

ANSWER HINTS
DAY - 41

1. Correct Option: (b) range from legally binding treaties


(called Agreements) to less formal
Explanation: instruments, such as Memoranda of
Understanding, and can be adapted
Conservation of Migratory Species of Wild
to the requirements of particular
Animals
regions. The development of models
 As an environmental treaty of the United tailored according to the conservation
Nations, CMS provides a global platform needs throughout the migratory range is a
for the conservation and sustainable use unique capacity to CMS.
of migratory animals and their habitats.
CMS brings together the States through 2. Correct Option: (c)
which migratory animals pass, the Range Explanation:
States, and lays the legal foundation for
internationally coordinated conservation Green Climate Fund
measures throughout a migratory range.
 The GCF helps developing countries limit
 It is also known as the Bonn Convention, or reduce their greenhouse gas (GHG)
as it was signed there in 1979. emissions and adapt to climate change.
It seeks to promote a paradigm shift
 As the only global convention
to low-emission and climate-resilient
specializing in the conservation of
development, taking into account the needs
migratory species, their habitats and
of nations that are particularly vulnerable
migration routes, CMS complements
to climate change impacts. It was formally
and co-operates with a number of other
established by a UNFCCC decision in
international organizations, NGOs and Durban, South Africa in December 2011.
partners in the media as well as in the
corporate sector.  India has moved forward in this regard
by selecting the Ministry of Environment,
 Migratory species threatened with Forests and Climate Change as India’s
extinction are listed on Appendix I of Nationally Designated Authority (NDA)
the Convention. CMS Parties strive for the GCF, which will recommend to the
towards strictly protecting these animals, Board of the GCF funding proposals in the
conserving or restoring the places where context of national climate strategies.
they live, mitigating obstacles to migration
and controlling other factors that might  Further NABARD has been accredited
endanger them. Besides establishing by Green Climate Fund (GCF) Board
obligations for each State joining the as one of the National Implementing
Entity (NIE) for GCF in India. NABARD
Convention, CMS promotes concerted
will be responsible for management and
action among the Range States of many of
oversight of project implementation, which
these species.
includes the origination and preparation
 Migratory species that need or would of a funding proposal, the subsequent
significantly benefit from international management of the necessary stages of the
co-operation are listed in Appendix II implementation process until its conclusion
of the Convention. For this reason, the (project management) on behalf of GCF,
Convention encourages the Range States and reporting obligations.
to conclude global or regional agreements.
3. Correct Option: (d)
 In this respect, CMS acts as a framework
Convention. The agreements may Explanation:
152 TARGET PT 2020

Convention on International Trade in  BIOFIN in India is led by the Ministry


Endangered Species of Wild Fauna & Flora of Environment, Forest and Climate
(CITES) Change (MoEFCC). UNDP India
 It is an international agreement manages the program under the guidance
(multilateral treaty) between of MoEFCC. The initiative is hosted by
governments. the National Biodiversity Authority
(NBA), working with four relevant State
 Also known as the Washington Biodiversity Boards, with technical
Convention it was drafted as a result of a assistance from Wildlife Institute of
resolution adopted by members of IUCN in India (WII) and National Institute of
1963 and entered into force in 1975. Public Finance and Policy (NIPFP).
 Its aim is to ensure that international trade
in specimens of wild animals and plants 5. Correct Option: (d)
does not threaten their survival.
Explanation:
 CITES is legally binding on the
Parties. However, it does not take the Conventions and protocols
place of national laws. It only provides a
framework to be respected by each Party. Convention on Biological Diversity (CBD)
The parties need to adopt their own  The Convention on Biological Diversity
domestic legislation to ensure that CITES (CBD) entered into force on 29 December
is implemented at the national level. 1993. It has 3 main objectives:
 CITES accords varying degrees of protection ! The conservation of biological
to more than 35,000 species of animals and
diversity
plants. The species covered by CITES are
listed in three Appendices, according to the ! The sustainable use of the components
degree of protection they need. of biological diversity
! The fair and equitable sharing of the
4. Correct Option: (d)
benefits arising out of the utilization
Explanation: of genetic resources

Biodiversity Financial Initiative (BIOFIN)  Both the Cartagena Protocol and


Nagoya Protocol are related to the
 UNDP in October 2012, launched the CBD.
Biodiversity Finance Initiative –
BIOFIN, as a new global partnership  The Cartagena Protocol on Biosafety to
seeking to address the biodiversity the Convention on Biological Diversity is
finance challenge in a comprehensive an international agreement which aims
manner – building a sound business case for to ensure the safe handling, transport and
increased investment in the management use of living modified organisms (LMOs)
of ecosystems and biodiversity. resulting from modern biotechnology that
may have adverse effects on biological
 BIOFIN was initiated in response to
diversity, taking also into account risks
the urgent global need to divert more
to human health. It was adopted on 29
finance from all possible sources towards
global and national biodiversity goals, as January 2000 and entered into force on 11
highlighted during the 2010 Convention September 2003.
on Biodiversity (Conference of Parties) 10  The Nagoya Protocol on Access to Genetic
in Nagoya. Subsequently, it was launched Resources and the Fair and Equitable
during CBD COP 11 in 2012. Sharing of Benefits Arising from their
 The overarching objective of BIOFIN Utilization to the Convention on Biological
is to deliver a new methodological Diversity is an international agreement
framework, facilitating the which aims at sharing the benefits arising
identification, development, and from the utilization of genetic resources in
implementation of optimal and a fair and equitable way. It entered into
evidence-based finance plans and force on 12 October 2014, 90 days after the
solutions. date of deposit of the fiftieth instrument of
ratification.
 BIOFIN is coordinated by UNDP through
a global team developing and updating United Nations Framework Convention on
the BIOFIN methodology, supporting
Climate Change
its implementation in the countries, and
developing capacities at the national and  The United Nations Framework
global levels on biodiversity finance. Convention on Climate Change (UNFCCC)
TARGET PT 2020 153

is an international environmental treaty Protocol. Although they do not deplete the


adopted on 9 May 1992 and opened for ozone layer, they are known to be powerful
signature at the Earth Summit in Rio de greenhouse gases and, thus, contributors
Janeiro from 3 to 14 June 1992. It then to climate change.
entered into force on 21 March 1994, after a
sufficient number of countries had ratified 6. Correct Option: (a)
it. The UNFCCC objective is to “stabilize
greenhouse gas concentrations in the Explanation:
atmosphere at a level that would prevent
dangerous anthropogenic interference with Stockholm Convention on POPs
the climate system”.  The Stockholm Convention is a global
 The Framework Convention specifies the treaty to protect human health and the
aim of Annex I Parties stabilizing their environment from persistent organic
greenhouse gas emissions (carbon dioxide pollutants (POPs). POPs are chemicals
and other anthropogenic greenhouse gases that remain intact in the environment for
not regulated under the Montreal Protocol) long periods, become widely distributed
at 1990 levels, by the year 2000. geographically, accumulate in the
fatty tissue of living organisms and
 Kyoto Protocol and Paris Agreement are are toxic to humans and wildlife. POPs
the treaties under the Convention. circulate globally and can cause damage
 Vienna Convention for the Protection of wherever they travel. In implementing
the Ozone Layer the Convention, Governments will take
measures to eliminate or reduce the release
 The Vienna Convention for the Protection
of POPs into the environment. Over 152
of the Ozone Layer is a multilateral
countries ratified the Convention and it
environmental agreement signed in
1985 that provided frameworks for entered into force, on 17 May 2004.
international reductions in the production  The Global Environmental Facility
of chlorofluorocarbons due to their (GEF) is the designated interim financial
contribution to the destruction of the ozone mechanism for the Stockholm Convention.
layer.
 United Nations Industrial Development
 It was the first convention of any kind to Organization is also responsible for
be signed by every country involved, taking supporting developing countries and
effect in 1988 and reaching universal countries with economies in transition to
ratification in 2009. This speaks to the implement the Stockholm Convention.
enormity of ozone depletion at the time
and the willingness of countries around
7. Correct Option: (c)
the world to work together to solve it. The
Convention aimed to promote cooperation Explanation:
among nations by exchanging information
on the effects of human activities on the United Nations Environment Programme
ozone layer.
 The United Nations Environment
 The Montreal Protocol on Substances Programme is an agency of the United
that Deplete the Ozone Layer is a global Nations that coordinates its environmental
agreement to protect the Earth’s ozone activities, assisting developing countries
layer by phasing out the chemicals that in implementing environmentally sound
deplete it. This phase-out plan includes policies and practices.
both the production and consumption of
ozone-depleting substances. The landmark  It’s headquartered at Nairobi, Kenya.
agreement was signed in 1987 and entered  It was founded by Maurice Strong, its first
into force in 1989. director, as a result of the United Nations
 The parties to the Protocol meet once a Conference on the Human Environment
year to make decisions aimed at ensuring (Stockholm Conference) in June 1972, which
the successful implementation of the was the first United Nations conference on
agreement. the environment.

 The most recent amendment, the Kigali  It has overall responsibility for
Amendment, called for the phase-down environmental problems among United
of hydrofluorocarbons (HFCs) in 2016. Nations agencies but international talks on
These HFCs were used as replacements specialized issues, such as addressing global
for a batch of ozone-depleting substances warming or combating desertification, are
eliminated by the original Montreal overseen by other UN organizations.
154 TARGET PT 2020

 One of UNEP’s most widely recognized that contribute to the work of the United
activities is Earthwatch, an international Nations Framework Convention on Climate
monitoring system designed to facilitate Change.
the exchange of environmental information  United Nations Environment Programme
among governments. or just, UN Environment is a program of
the United Nations that coordinates the
8. Correct Option: (b) organization’s environmental activities
Explanation: and assists developing countries in
implementing environmentally sound
World Wide Fund for Nature (WWF) policies and practices. Its headquarter is in
Nairobi, Kenya.
 WWF is an international non-governmental
organization founded in 1961. It was 10. Correct Option: (a)
formerly named the World Wildlife Fund,
which remains its official name in Canada Explanation:
and the United States.
International environmental
 It is working in the field of wilderness organizations
preservation, reduce the most pressing
threats to the diversity of life on Earth and  International Union for Conservation
the reduction of humanity’s footprint on of Nature is an international organization
(NGO), headquartered in Gland,
the environment.
Switzerland, working in the field of
 It works in partnership with foundations, nature conservation and sustainable use
governments, businesses, communities, of natural resources. It is involved in
individuals and more than six million data gathering and analysis, research,
members, to conserve many of the world’s field projects, advocacy, lobbying, and
most ecologically important regions. education. The organization is best known
for compiling and publishing the IUCN
 It is the world’s largest conservation
Red List.
organization with over five million
supporters worldwide, working in more  Global Green Growth Institute is a
than 100 countries, supporting around treaty-based international organization
1,300 conservation and environmental headquartered in Seoul, South Korea,
projects. aiming to promote green growth, a growth
paradigm that is characterized by a balance
 WWF is a foundation, with 55% of funding of economic growth and environmental
from individuals and bequests, 19% sustainability.
from government sources (such as the
World Bank, DFID, USAID) and 8% from  350.org is an international environmental
corporations in 2014. organization addressing the climate crisis.
Its goal is to end the use of fossil fuels
 The living planet report is published and transition to renewable energy by
every two years by WWF since 1998, building a global, grassroots movement.
it is based on living planet index and The 350 in the name stands for 350
ecological footprint calculation. ppm (parts per million) of carbon
 WWF publishes the Living Planet Index in dioxide which has reached 415 ppm as of
collaboration with the Zoological Society 2019. It’s headquartered in Oakland,
California.
of London. Along with ecological footprint
calculations, the Index is used to produce
a bi-yearly Living Planet Report giving an
11. Correct Option: (d)
overview of the impact of human activity Explanation:
on the world.
Climate Financing
9. Correct Option: (a)  The Global Environment Facility (GEF)
Explanation: was established on the eve of the 1992 Rio
Earth Summit to help tackle our planet’s
IPCC most pressing environmental problems.
 Intergovernmental Panel on Climate  The Green Climate Fund was GCF
Change (IPCC) was established by the was established under the Cancun
World Meteorological Organization and UN Agreements in 2010 as a dedicated
Environment in 1988, with headquarter financing vehicle for developing countries
in Geneva. The IPCC produces reports within the global climate architecture,
TARGET PT 2020 155

serving the Financial Mechanism of the Black Carbon Research Initiative


UNFCCC and the Paris Agreement.
 ISRO-GBP (Indian Space Research
 Adaptation Fund (AF) was established Organization’s Geosphere-Biosphere
under the Kyoto Protocol in 2001. Programme) recognized the importance
of Black Carbon aerosols on the climate
12. Correct Option: (b) system and decided to pursue studies of
Black Carbon in subsequent years.
Explanation:
 In view of this, a multi-institutional and
TRAFFIC multi-agency Science Plan has been
 Wildlife Trade Monitoring Network launched in the Ministry of Environment
(TRAFFIC), a joint program of the in association with the Ministry of
World Wide Fund for Nature (WWF) Earth Sciences, Indian Space Research
and IUCN, Organization, Ministry of Science
and Technology and other associated
 It is a non-governmental organization agencies, to monitor aerosols and
working globally on trade in wild assess its impacts through various
animals and plants in the context of both modeling techniques.
biodiversity conservation and sustainable
development.  It is an Indian initiative launched as a
part of the National Carbonaceous
 Its headquarters is located in
Aerosols Program (NCAP) under the
Cambridge, United Kingdom.
aegis of the Indian Network for Climate
Change Assessment, INCCA.
13. Correct Option: (b)
Explanation: 15. Correct Option: (d)

Global Environment Outlook (GEO) Explanation:


 GEO is the UN Environment The International Whaling Commission
Programme’s flagship environmental (IWC)
assessment.
 The International Whaling Commission
 It was first published in 1997 and was (IWC) is the global body charged with
originally requested by the Member the conservation of whales and the
States. management of whaling. It was set up
 GEO is a consultative and participatory to provide for the proper conservation of
process to prepare an independent whale stocks and thus make possible the
assessment of the state of the environment, orderly development of the whaling
the effectiveness of the policy response to industry. Uncertainty over whale numbers
address these environmental challenges and led to the introduction of a ‘moratorium’
the possible pathways to achieve various on commercial whaling in 1986. This
internationally agreed environmental remains in place although the Commission
goals. continues to set catch limits for aboriginal
subsistence whaling.
Global Environment Outlook-6
 The main duty of the IWC is to keep under
 The sixth edition of the Global review and revise as necessary the
Environmental Outlook (GEO-6) report measures laid down in the Schedule
was released by the UN Environment to the Convention which govern the
Programme’s (UNEP). conduct of whaling throughout the
 The theme of the sixth Global Environment world. These measures, among other
Outlook (GEO6), is a healthy planet, things, provide for the complete protection
healthy people. of certain species; designate specified areas
as whale sanctuaries; set limits on the
 GEO-6 builds on previous GEO reports. It, numbers and size of whales which may be
however, differs from previous GEO reports taken; prescribe open and closed seasons
in its emphasis on Sustainable Development and areas for whaling, and prohibit the
Goals and in providing possible means of capture of suckling calves and female
accelerating achievement of these goals. whales accompanied by calves.

14. Correct Option: (d)  The IWC currently has 89 member


governments from countries all
Explanation: over the world. India is a member
156 TARGET PT 2020

state. All members are signatories 17. Correct Option: (b)


to the International Convention for the
Regulation of Whaling. Explanation:

 International Convention for the United Nations Environment Programme


Regulation of Whaling is an international (UNEP)
environmental agreement that governs the  The United Nations Environment
commercial, scientific, and aboriginal Programme is an agency of the
subsistence whaling practices. This United Nations that coordinates its
Convention is the legal framework environmental activities, assisting
that established the IWC in 1946. developing countries in implementing
environmentally sound policies and
 Today, the Commission also works to
practices. The working area of UNEP
understand and address a wide range is divided into seven broad thematic
of non-whaling threats to cetaceans areas: climate change, disasters and
including entanglement, ship strike, conflicts, ecosystem management,
marine debris, climate change, and environmental governance, chemicals
other environmental concerns. and waste, resource efficiency, and the
 In 1994, it created the Southern Ocean environment under review maintaining the
overarching commitment to sustainability
Whale Sanctuary surrounding the
in all its work.
continent of Antarctica. Here, the
IWC has banned all types of commercial  It’s headquartered at Nairobi, Kenya.
whaling.  It was founded by Maurice Strong, its
 Only two such sanctuaries have been first director, as a result of the United
designated by IWC to date. Another is Nations Conference on the Human
Indian Ocean Whale Sanctuary by the Environment (Stockholm Conference)
tiny island nation of Seychelles. in June 1972 and has overall responsibility
for environmental problems among United
Nations agencies but international talks on
16. Correct Option: (a)
specialized issues, such as addressing global
Explanation: warming or combating desertification, are
overseen by other UN organizations.
Birdlife International
 UN Environment has aided in the
 Birdlife International is a global formulation of guidelines and
partnership of conservation treaties on issues such as the
organizations that strives to conserve international trade in potentially harmful
birds, their habitats and global chemicals, transboundary air pollution, and
biodiversity, working with people contamination of international waterways.
towards sustainability in the use of natural Relevant documents, including
scientific papers, are available via the
resources. Each BirdLife Partner is an
UNEP Document Repository.
independent environmental not-for-profit,
non-governmental organisation or NGO.  UNEP work encompasses
Most Partners are best known outside of ! Assessing global, regional and national
the Partnership by their organization’s environmental conditions and trend
name. This allows each partner to maintain
its individual national identity within the ! Developing international and national
global Partnership environmental instruments

 It was earlier known as International ! Strengthening institutions for the


wise management of the environment
Committee for Bird Preservation.
! Facilitating the transfer of
 It is the official Red List Authority for
knowledge and technology for
birds, for the International Union for
sustainable development
Conservation of Nature.
 It publishes a quarterly magazine, 18. Correct option: (a)
World Birdwatch, which contains recent
Explanation:
news and authoritative articles about
birds, their habitats, and their conservation Environment Pollution Control Authority
around the world. (EPCA)
 It publishes the scientific journal, Bird  EPCA is a Supreme Court-empowered
Conservation International. body which is tasked with taking various
TARGET PT 2020 157

measures to tackle air pollution  The largest share of historical and current
in the National Capital Region. It global emissions of greenhouse gases has
is constituted with the objective of originated in developed countries, that per
protecting and improving the quality capita emissions in developing countries
of the environment and preventing are still relatively low and that the share of
and controlling the environmental global emissions originating in developing
pollution in the National Capital countries will grow to meet their social and
Region and also assists the apex development needs,
court in various environment-related  Kyoto Protocol implemented the objective
matters in the region. of the UNFCCC to fight global warming by
 It was notified in 1988 by Environment reducing greenhouse gas concentrations in
ministryunder Environment the atmosphere to a level that would prevent
Protection act, 1986. dangerous anthropogenic interferences
with the climate system.
 Besides the chairman, the EPCA has 20
members. Former secretary BhureLal is 20. Correct Option: (a)
the current chair of EPCA.
Explanation:
 The authority has the power suo-moto,
or on the basis of complaints made The United Nations Office for Disaster
by any individual, association, company, Risk Reduction (UNISDR)
public undertaking or local body carrying
 UNISDRwas created in December 1999.
on any industry, operation or process. The successor to the secretariat of the
 Functions include: International Decade for Natural Disaster
Reduction was established to ensure
! To protect and improve quality the implementation of the International
of environment and prevent and Strategy for Disaster Reduction.
control environmental pollution
in National Capital Region.  UNISDR is part of the United Nations
Secretariat and its functions span the
! To enforce graded Response social, economic, environmental as
Action Plan (GRAP) in NCR as per well as humanitarian fields.
the pollution levels.
 UNISDR supports the implementation,
19. Correct Option: (c) follow-up, and review of the Sendai
Framework for Disaster Risk
Explanation: Reduction adopted by the Third UN World
Conference on Disaster Risk Reduction on
United Nations Framework Convention on 18 March 2015 in Sendai, Japan. The
Climate Change (UNFCCC) Sendai Framework is a 15-year voluntary,
non-binding agreement that maps
 In 1992, countries joined an international out a broad, people-centered approach to
treaty, the United Nations Framework disaster risk reduction, succeeding the
Convention on Climate Change (UNFCCC), 2005-2015 Hyogo Framework for Action.
as a framework for international
cooperation to combat climate change  UNISDR’s vision is anchored on the
four priorities for action set out in
by limiting average global temperature
the Sendai Framework: understanding
increases and the resulting climate change,
disaster risk, strengthening disaster
and coping with impacts that were, by then, risk governance to manage disaster
inevitable. risk, investing in disaster risk reduction
 The Parties to this Convention for resilience, and enhancing disaster
acknowledges that Change in the Earth’s preparedness for effective response
climate and its adverse effects are a and to “Build Back Better” in recovery,
rehabilitation, and reconstruction.
common concern for humankind
 UNISDR informs and connects people
 Human activities have been substantially
by providing practical services and
increasing the atmospheric concentrations tools such as the risk reduction
of greenhouse gases, that these increases website- PreventionWeb, terminology,
enhance the natural greenhouse effect, publications on good practices, country
and that this will result on average in an profiles and the Global Assessment Report
additional warming of the Earth’s surface on Disaster Risk Reduction which is an
and atmosphere and may adversely affect authoritative biennial analysis of global
natural ecosystems and humankind disaster risks and trends.
158 TARGET PT 2020

21. Correct option: (d)  To adopt a holistic approach of systemic


public grievance reforms through improved
Explanation
mapping, formulation of monitoring matrix,
Nagpur resolution is to empower the data collection and evaluation in quality of
citizens by policy interventions for better grievance redressal;
service delivery through timely updation  To provide an enabling environment for
of citizen’s charters, implementation of States and Ministries/ Departments of
enactments and benchmarking standards the Government of India for creating web
for continuous improvement. portals and to adopt a holistic approach for
improved service delivery through digital
Supplementary notes
platforms;
‘Nagpur Resolution: A Holistic approach  To focus on dynamic policy making
for empowering citizens’ and strategic decisions, monitoring of
 The conference was organised by the implementation, appointment of key
Department of Administrative Reforms personnel, coordination and evaluation;
and Public Grievances (DARPG),
Government of India, in collaboration with  To achieve a sense of common identity by
the Government of Maharashtra and the exchange of technical expertise in the areas
Maharashtra State Commission for Right of Improved Service Delivery between
to Public Services. the paired States under the Ek Bharat –
Shresht Bharat Program;
 Positive approach, transparency,
corruption-free system, fast track decision  To work towards long-term engagements
making and social sensitivity are essential in the areas of Improved Service Delivery
to good governance. for Empowering Citizens through greater
 Performance audit of public servants cooperation between the DARPG and the
should be done at regular intervals. participating States and,

 The Conference also resolved to adopt  To ensure timely publication of Good


a holistic approach of systemic public Governance Index to identify the quality
grievance reforms through improved of governance in 10 sectors especially those
mapping, formulation of monitoring pertaining to welfare and infrastructure at
matrix, data collection and evaluation the Union, State and District levels.
in quality of grievance redressal, and
to provide an enabling environment for 22. Correct option: (b)
States and Ministries/Departments of
the Government of India for creating web Explanation
portals and to adopt a holistic approach for
improved service delivery through digital Statement 1 is incorrect: Prime Minister
platforms. honoured the contribution of former Prime
 The Resolution focuses on dynamic policy Minister AtalBihari Vajpayee by naming
making and strategic decisions, monitoring the Strategic Tunnel under Rohtang Pass
of implementation, appointment of key after him.
personnel, coordination and evaluation,
and achieving a sense of common identity Statement 3 is incorrect: It will reduce
by exchange of technical expertise in
the distance between Manali and Leh by
the areas of Improved Service Delivery
between paired States under the Ek Bharat 46 kilometres and save crores of rupees in
Shreshtha Bharat Program. transport costs.
 To empower the citizens by policy Supplementary notes
interventions for better service delivery
through timely updation of citizens Rohtang Tunnel
charters, implementation of enactments
 The historic decision to construct a strategic
and benchmarking standards for continuous
improvement; tunnel below the Rohtang Pass was taken
on June 03, 2000, when late AtalBihari
 To empower citizens by adopting a Vajpayee was the Prime Minister.
bottom-up approach to bring massive
improvements in quality of grievance  The foundation stone for the Access Road
redressal and reduction in timelines of to the South Portal of the tunnel was laid
grievance redressal; on May 26, 2002.
TARGET PT 2020 159

How long is the tunnel, and what is special ! It is an agreement on Iran’s nuclear
about it? program made between Iran, the
P5+1 (the five permanent members
 Upon completion, the 8.8 km-long tunnel
of the United Nations Security
will be the world’s longest highway tunnel
at an altitude of above 10,000 feet (3,000 Council—China, France, Russia,
metres). United Kingdom, United States—plus
Germany) and the European Union.
 It is a 10.5 m-wide single tube, a bi-lane
tunnel with a fireproof emergency tunnel ! JCPOA limits Iran’s uranium
built into the main tunnel itself. The enrichment programme until 2030 and
10.5-m width includes a 1-metre footpath contains monitoring and transparency
on both sides. measures that will remain in place
long after that date.
 Vehicles will travel at a maximum speed
of 80 km per hour inside the tunnel. Up to ! A few days after the JCPOA was
3,000 cars and 1,500 trucks are expected to agreed, it was endorsed by the United
use the tunnel every day. Nations Security Council (UNSC).
 It will reduce the distance between Manali  However, in May 2018, United States
and Leh by 46 kilometres and save crores announced its withdrawal from JCPOA.
of rupees in transport costs.
Why did US withdraw from JCPOA?
 It will also provide all-weather connectivity
to remote border areas of Himachal Pradesh  United States cited major flaw in JCPOA’s
and Ladakh, which otherwise remained cut temporary nature and its lack of controls
off from the rest of the country for about six on Iran’s ballistic missile programme.
months.
 Mr. Trump was also highly critical of
 The project has significant strategic Iran’s actions in Syria and elsewhere in
implications for the military. Once the the region, which he characterizes as Iran’s
tunnel is operational, the forces will have ‘malign behaviour’.
access beyond the Rohtang Pass even in
peak winter. Concern due to US’s withdrawal
 The tunnel is now nearing completion  Supporters: American conservatives in the
and is a step in the direction of providing United States, Israel, Saudi Arabia and
all weather connectivity to remote border allies have supported the move.
areas of Himachal Pradesh and Ladakh
which otherwise remained cut off from the  IAEA concern: By all accounts, Iran was
rest of the country for about six months honouring its provisions. Leading up to
during winters. United States’ withdrawal, the IAEA
asserted that its inspectors had verified
 While Rohtang Pass is at a height of that Iran had implemented its nuclear-
13,050 feet, the pass on the road to Leh related commitments since the agreement.
is Baralacha La at 16,040 feet. A 13.2-km
long tunnel would be required to bypass  Politically motivated: Rather than an
this pass. evidence-based technical objection to the
agreement or its implementation, the US
 An alternative road link to Ladakh has
decision to withdraw from JCPOA seems to
also been developed by BRO on the Darcha-
be a political measure aimed against Iran.
Padam-Nimu axis, but here again; a 4.15
km-long tunnel at Sinka La Pass (16,703  Impact on Iran: The withdrawal caused
feet) would be required for all-weather concerns in Iran due to its impact on Iran’s
access. economy caused by US sanctions.

23. Correct option: (c)  Undermines multilateral diplomacy:It


undermines the value of multilateral
Explanation diplomacy and raises questions about the
sanctity and sustainability of interstate
Supplementary notes
agreements. Furthermore, it challenges the
JCPOA authority of UNSC which had unanimously
passed a resolution endorsing the JCPOA,
 Joint Comprehensive Plan of Action and had called on all UN member states
(JCPOA): JCPOA is also known as the
as well as regional and international
“Iran deal” or “Iran nuclear deal”.
organizations to take action to support the
! The deal was made in July 2015. agreement’s implementation.
160 TARGET PT 2020

! US withdrawal from JCPOA  Recently, the Supreme Court has agreed


risks seriously weakening trust to hear a public interest litigation seeking
and confidence in international that the chief election commissioner and
institutions and arrangements that election commissioners be appointed by a
are essential parts of the global three-member collegium.
security architecture.
 The collegium will comprise the Prime
 Undermines nuclear non-proliferation:
Minister, the leader of opposition in Lok
US action undermines the global effort for
Sabha and the Chief Justice of India.
nuclear non-proliferation by sabotaging an
important and effective anti-proliferation A bench comprising Chief Justice S. A.
agreement. Bobde and Justices B.R. Gavai and Surya
Kant took note of submissions that the
! By exiting JCPOA, the US has plea needed an urgent hearing. Advocate
heightened risks stemming from
AshwiniUpadhyay filed the PIL seeking
Iran.
to ensure more autonomy for the chief
! Due to a multi-pronged effect on other election commissioner’s office and election
parties, this can trigger a preventive commissioners. The plea has also sought
race to acquire nuclear weapons. an independent secretariat for the Election
 Europe’s stand: European Commission Commission of India and that it should also
announced its intention to implement the be given the power to make rules.
blocking statute of 1996 to declare the US
 The Election Commission of India
sanctions against Iran illegal in Europe
and ban European citizens and companies (ECI) is an autonomous constitutional
from complying with them. authority responsible for administering
Union and State election processes in
 The commission also instructed the India.
European Investment Bank to facilitate
European companies’ investment in Iran.  The body administers elections to the Lok
Sabha, Rajya Sabha, State Legislative
24. Correct Answer: (A) Assemblies, and the offices of the President
Explaination: Option (A) is correct and Vice President in the country.
 Structure of the Commission
Supplementary Notes
 The demand of collegium system was first ! Originally the commission had
raised in 1990 when the Dinesh Goswami only one election commissioner but
Committee suggested the need for a after the Election Commissioner
selection committee or a panel to appoint Amendment Act 1989, it has been
the CEC (at that time ECI was a single- made a multi-member body.
member body).
! The commission presently consists
 Based on this of one Chief Election Commissioner
recommendation, 70th Constitutional (CEC) and two Election Commissioners
Amendment Bill, 1990 was introduced (ECs).
in the Parliament which demanded for
the selection committee comprising of ! The secretariat of the commission is
the Presiding Officers of both Houses and located in New Delhi.
Leader of the Opposition in Lok Sabha.
 However, due to lack of political will, it was
25. Correct option: (d)
not passed and was withdrawn in 1993. Till Explanation
date, no such bill was formed.
 The issue was debated in the Constituent All the statements are correct: The Union
Assembly which finally left over to the Cabinet chaired by Prime Minister has
government to decide the appointments. approved to create the post of Chief of
The constituent Assembly suggested that Defence Staff in the rank of a four-star
election commission should be appointed
General with salary and perquisites
after a proposal of the same is supported
by two-third majority of the parliament. equivalent to a Service Chief. The Chief of
Defence Staff will also head the Department
 Global Practice: In the USA, South Africa,
Canada, etc. there is a mechanism for of Military Affairs (DMA), to be created
outside consultation with expert body for within the Ministry of Defence and function
making suitable appointments. as its Secretary.
TARGET PT 2020 161

Chief of Defence Staff resources by bringing about jointness


in operations, including through
 The Chief of Defence Staff, apart from
establishment of joint/theatre
being the head of the Department
commands.
of Military Affairs, will also be the
Permanent Chairman of the Chiefs of Staff ! Promoting use of indigenous
Committee. equipment by the Services
 He will act as the Principal Military  As the Permanent Chairman of Chiefs of
Adviser to RakshaMantri on all tri-Services Staff Committee, CDS will perform the
matters. The three Chiefs will continue following functions:
to advise RM on matters exclusively
concerning their respective Services. ! CDS will administer tri-services
organisations. Tri-service agencies/
 CDS will not exercise any military organisations/commands related to
command, including over the three Service Cyber and Space will be under the
Chiefs, so as to be able to provide impartial command of the CDS.
advice to the political leadership.
! CDS will be member of Defence
 The following areas will be dealt by the Acquisition Council chaired by
Department of Military Affairs headed by RakshaMantri and Defence Planning
CDS:
Committee chaired by NSA.
! The Armed Forces of the Union, ! Function as the Military Adviser to
namely, the Army, the Navy and the
the Nuclear Command Authority.
Air Force.
! Bring about jointness in operation,
! Integrated Headquarters of the
logistics, transport, training, support
Ministry of Defence comprising Army
services, communications, repairs and
Headquarters, Naval Headquarters,
maintenance, etc of the three Services,
Air Headquarters and Defence Staff
within three years of the first CDS
Headquarters.
assuming office.
! The Territorial Army.
! Ensure optimal utilisation of
! Works relating to the Army, the Navy infrastructure and rationalise it
and the Air Force. through jointness among the services.
! Procurement exclusive to the Services ! Implement Five-Year Defence Capital
except capital acquisitions, as per Acquisition Plan (DCAP), and Two-
prevalent rules and procedures. Year roll-on Annual Acquisition Plans
 Apart from the above, the mandate of the (AAP), as a follow up of Integrated
Department of Military Affairs will include Capability Development Plan (ICDP).
the following areas: ! Assign inter-Services prioritisation to
! Promoting jointness in procurement, capital acquisition proposals based on
training and staffing for the Services the anticipated budget.
through joint planning and integration
 Bring about reforms in the functioning
of their requirements.
of three Services aimed at augmenting
! Facilitation of restructuring of Military combat capabilities of the Armed Forces by
Commands for optimal utilisation of reducing wasteful expenditure.

**********
162 TARGET PT 2020

TEST
DAY - 42

Time Allowed: 30 mins Maximum Marks: 50

1. Recently, India became the first 1. It has been initiated by Greenpeace


country that launched a program to International.
replace LPG as cooking fuel. Which of
2. The prime focus of this project is to
the following statements is/are correct
clean up the harmful algal boom across
in this regard?
the oceans.
1. The LPG will be replaced by Methanol
as the fuel. Select the correct option using the codes
2. Sikkim became the first state to launch given below:
a program to replace LPG as cooking (a) 1 only
fuel.
(b) 2 only
Select the correct option using the codes
(c) Both 1 and 2
given below:
(d) Neither 1 nor 2
(a) 1 only
(b) 2 only 4. Consider the following statements:
(c) Both 1 and 2 1. Abdul Ghani has planted close to 50
lakh trees as part of the GreenKalam
(d) Neither 1 nor 2
initiative.
2. Which of the following statements is 2. He is known as the Green Man of
correct regarding the “sovereign blue India.
bond”? 3. Recently, he has started the Tree
1. It has been developed by the support Ambulance program in Chennai.
of the World Bank and Global
Environment Facility Select the correct option using the codes
2. The blue bond is inspired by the green given below:
bond concept. (a) 2 and 3 only
3. Recently, India became the first nation (b) 3 only
to launch such a bond.
(c) 1 and 2 only
Select the correct option using the codes
(d) 1, 2 and 3
given below:
(a) 2 only 5. Consider the following statements
regarding the International Conference
(b) 1 and 2 only
on Status and Protection of Coral Reefs
(c) 2 and 3 only
(STAPCOR – 2018):
(d) 1, 2 and 3 1. It was the first STAPCOR conference
in India.
3. Which of the following statements
regarding the Ocean Clean Up Project 2. It was organized at the Bangaram coral
is/are correct? Island of Lakshadweep.
TARGET PT 2020 163

Which of the above statements is/are correct? Select the correct option using the codes
given below:
(a) 1 only
(a) 2 and 3 only
(b) 2 only
(b) 3 only
(c) Both 1 and 2
(c) 1 and 2 only
(d) Neither 1 nor 2
(d) 1, 2 and 3
6. Which of the following species will be
forwarded by India to include in the 9. Which of the following are related to
global conservation list during the the United Nations Office for Disaster
upcoming Conference of Parties (COP) Risk Reduction?
of the Convention on the Conservation 1. Sendai Framework
of Migratory Species of Wild Animals? 2. Hyogo Framework
1. Amur Falcon 3. Sasakawa Award
2. Asian Elephant
Select the correct option using the codes
3. Gibi given below:
4. Dugong (a) 1, 2 and 3
Select the correct option using the codes (b) 2 and 3 only
given below: (c) 3 only
(a) 1 and 4 only (d) 1 and 2 only
(b) 2 and 3 only
10. Consider the following statements:
(c) 1 and 3 only 1. This species is endemic to the Palghat
(d) 1, 2, 3 and 4 gap in Kerala.
2. It is believed to be called ‘living fossil’.
7. Consider the following statements
regarding Solar Charkha Mission: 3. Recently, it has been declared as
Kerala’s state amphibian.
1. It is an initiative to boost the rural
economy and help in arresting Which of the following species is being
migration from rural to urban areas. described by the above statements?
2. It will be implemented by the National (a) Cricket frogs
Bank for Agriculture and Rural
(b) Purple frog
Development.
(c) Red frogs
Which of the above statements is/are
(d) Rice frogs
correct?
(a) 1 only 11. Climate Change Performance Index
is published jointly by which of the
(b) the only following institutes?
(c) Both 1 and 2 1. NewClimate Institute
(d) Neither 1 nor 2 2. Climate Action Network
3. Germanwatch
8. Which of the following statements is/
are correct? 4. UN Environment
1. The Keeling Curve is used for the Select the correct option using the codes
measurement of Carbon dioxide only. given below:
2. It is measured at the only observatory (a) 1 and 2 only
in Mauna Loa, Hawaii.
(b) 3 and 4 only
3. There is a continuous diurnal variation
(c) 1, 2 and 3 only
in the concentration of Carbon dioxide
as recorded by the Graph. (d) 1, 2, 3 and 4
164 TARGET PT 2020

12. Which of the following statements 3. As per the report, land degradation
regarding the Global Climate Risk is both the cause and consequence of
Index 2020 are correct? climate change.
1. It has been published by Climate Action
Network International. Which of the above statements is/are
correct?
2. According to this report, India has
recorded the highest number of (a) 1 only
fatalities due to climate change in
(b) 1 and 2 only
2018.
3. As per the report, Japan is the worst (c) 2 and 3 only
affected country. (d) 1, 2 and 3
Select the correct option using the codes
15. India recently launched Sustainable
given below:
Livelihoods and Adaptation to
(a) 1 only Climate Change (SLAAC) to improve
(b) 2 and 3 only the adaptive capacity of rural poor to
climatic vulnerability. In this regard,
(c) 1 and 3 only
consider the following statements
(d) 1, 2 and 3 about SLAAC:

13. A Joint Conference of Parties (CoP) 1. It is a first of its kind initiative


to the BRS (Basel, Rotterdam, and aiming to create a cadre of climate-
Stockholm) Conventions was recently smart community resource persons in
held in Geneva. Consider the following villages.
statements in this regard: 2. The project has been launched as a
1. Parties to the Basel Convention part of the National Rural Livelihood
included plastic wastes in the Prior Mission (NRLM).
Informed Consent (PIC) procedure.
3. SLAAC is funded by the Green Climate
2. New chemicals were listed under Fund.
both the Rotterdam and Stockholm
Conventions. Which of the above statements is/are
3. India had major reservations regarding correct?
the adoption of technical guidelines on
(a) 1 only
environmentally sound management
of electrical and electronic wastes. (b) 2 only

Which of the above statements is/are (c) 1 and 2 only


correct? (d) 2 and 3 only
(a) 1 only
16. The Hindu Kush Himalaya (HKH)
(b) 1 and 2 only
region Assessment report reveals that
(c) 2 and 3 only more than 35% of the glaciers in the
(d) 1, 2 and 3 region could retreat by 2100. In light
of this report, consider the following
14. With reference to a Special Report statements:
on Climate Change and Land as seen
1. This report has been released by the
in the news recently, consider the
Indian Mountaineering Foundation.
following statements:
1. It is published by the Intergovernmental 2. The warming of this region is a positive
Panel on Climate Change (IPCC). change for agriculture.
2. The report claims that global food 3. The number of glaciers in the
wastage is also a contributor to climate Himalayan area has increased due to
change. global warming.
TARGET PT 2020 165

Which of the above statements is/are (c) It is being implemented by utilising the
incorrect? vast networks and expertise of ENVIS
Hubs.
(a) 1 only
(d) None of the above
(b) 1 and 3 only
(c) 1 and 2 only 20. Consider the following statements
with respect to the Green Muffler:
(d) 2 and 3 only
1. Green Mufflers are barriers grown
17. With reference to the SAMOA Pathway, near noisy places to reduce the impact
consider the following statements: of noise.

1. It articulates the sustainable 2. Under the Green Muffler scheme,


development pathways and aspirations Asoka and Neem plants are planted
for Small Island Developing States near the house or resident localities to
reduce noise pollution.
(SIDS).
2. Social Development and Women’s Which of the above statements is/are
Empowerment is one of the priority correct?
areas under it. (a) 1 only
3. SIDS is characterized by a narrow (b) 2 only
export base.
(c) Both 1 and 2
Which of the above statements is/are
(d) Neither 1 nor 2
correct?
(a) 1 only 21. With reference to Sugamya Bharat
Abhiyan, consider the following
(b) 1 and 2 only statements
(c) 2 and 3 only 1. The campaign aims at providing equal
(d) 1, 2 and 3 opportunity to persons with disabilities
to participate in all the aspects of life
18. Which among the following is correct and live independently.
regarding the ‘Real-Time National 2. India is not a signatory to the UN
Ambient Noise Monitoring Network’? Convention on the Rights of Persons
(a) It has established by Central Pollution with Disabilities (UNCRPD).
Control Board in association with state Which of the following statements is/are
pollution control boards. correct?
(b) In the first phase, it will cover all the (a) 1 only
metropolitan cities.
(b) 2 and 3 only
(c) In the second phase, it will cover
(c) 3 only
Kanpur, Surat, and Bhopal.
(d) 1, 2 and 3
(d) None of the above
22. Consider the following statements
19. Which of the following is incorrect
regarding recently unveiled India’s
regarding the Green Skill Development
First CNG Bus which can run 1000
Programme?
Kms in one fill,
(a) It aims to train over 5.5 lakh workers 1. The project has been executed by
in the environment and forest sectors Indraprastha Gas Limited (IGL).
in the country through 30 courses by
2. It has been has been achieved through
2024.
pioneering design of Type IV Composite
(b) Botanical Survey of India and Zoological Cylinders in buses, replacing
Survey of India were the nodal Centres traditional very heavy Type-I Carbon
for the pilot programme. Steel cylinders.
166 TARGET PT 2020

Which of the following statement is/are 3. The system is compact with minimum
correct? number of vehicles for a firing unit.
(a) 1 only
Which of the following statements is/are
(b) 2 only correct?
(c) Both 1 and 2 (a) 1 and 2 only
(d) Neither 1 nor 2 (b) 2 only

23. Consider the following statements (c) 1 and 3 only


regarding AtalBhujalYojana (ATAL (d) 1, 2 and 3
JAL),
1. It is a Central Sector Scheme. 25. Consider the following statements
2. It will be implemented over a period of regarding NPR - National Population
5 years (2020-21 to 2024-25) Register
3. The scheme aims to improve ground 1. The database will have demographic
water management. details.
2. NPR is to create a comprehensive
Which of the following statements is/are
correct? identity database of every “usual
resident” of the country.
(a) 1 and 2 only
3. The process of updating NPR will
(b) 2 only be carried out under the aegis of the
(c) 1 and 3 only Registrar General and ex-Officio
Census Commissioner.
(d) 1, 2 and 3
Choose the correct option from the
24. Consider the following statements
following
regarding Quick Reaction Surface to
Air Missile (QRSAM) system, (a) 1 and 2 only
1. It is developed by Defence Research and (b) 2 and 3 only
Development Organization (DRDO).
(c) 1 and 3 only
2. It failed flight test when conducted from
Integrated Test Range, Chandipur. (d) 1, 2 and 3

**********
TARGET PT 2020 167

ANSWER HINTS
DAY - 42

1. Correct Option: (a)  The main beneficiaries are Seychellois whose


livelihoods depend on marine resources
Explanation: and the ocean. This includes artisanal
“Methanol Cooking Fuel Program” of and semi-industrial fishers, operators in
India tourism and seafood value chains, including
aquaculture; national and local institutions
 NITI Ayog on October 5, 2018, launched engaged in the management of marine
this program in Assam. Thus, Assam resources, including fishers’ associations
became the first state in India to do and government entities. Ultimately,
this. the general population will benefit from
 Although China and African nations use a healthier marine environment and
methanol as a cooking fuel, India is the increased food security.
first country where the focus is on replacing
LPG. 3. Correct Option: (d)
 The target is to feed methanol to the Explanation:
Northeast first, and then to the rest of
India. Ocean Clean Up Project

 It will save money for households and  This project has been initiated by
reduce oil imports. Ocean Cleanup which is a non-
governmental organization, based in
2. Correct Option: (b) the Netherlands.

Explanation:  The prime focus is to develop and


implement advanced technologies to
Sovereign blue bond get rid of the plastic pollution in the
oceans.
 The blue bond is a debt instrument issued by
governments, development banks or others  It has been started in the Pacific Ocean to
to raise capital from impact investors to extract Plastic Pollution.
finance marine and ocean-based projects.
 The blue bond is inspired by the green bond 4. Correct Option: (d)
concept. Explanation:
 The sovereign blue bond was issued with  K Abdul Ghani has planted close to 50 lakh
a ceiling value of US$15 million, with a trees by himself and many more lakh’s as
maturity of 10 years. The blue bond, as well part of the GreenKalam initiative. Recently,
as the program of marine and ocean-related he has started the Tree Ambulance program
activities it will support, was prepared in Chennai.
with assistance from the World Bank and
the Global Environment Facility. This  For this, he is known as the Green Man
support includes a partial World Bank of India.
guarantee ($5 million) and a concessional  Salim Ali was an Indian ornithologist
loan from the Global Environment Facility and naturalist. He was the first Indian
($5 million), which will partially subsidize
to conduct systematic bird surveys across
payment of the bond coupons.
India and wrote several bird books that
 The first such bond has been launched popularized ornithology in India. He is
by Seychelles. known as the birdman of India.
168 TARGET PT 2020

 Rajendra Singh is an Indian water of CMS COP-13, Kolam art form is used to
conservationist and environmentalist, depict key migratory species in India like
known as the waterman of India. Amur falcon, humpback whale and marine
turtles.
5. Correct Option: (c)  India will be moving to include
Explanation: the Asian Elephant and the Great
Indian Bustard (Gibi) in the global
STAPCOR-2018 conservation list- the list of species
that merit heightened conservation
 The International Conference on Status
measures.
and Protection of Coral Reefs (STAPCOR
– 2018) with the theme “Reef for Life”  Having the elephant and the Great Indian
was inaugurated by the Union Minister Bustard in the list — more formally known
of Environment, Forest and Climate as Appendix 1 — would coax countries
Change on 22nd October at Bangaram neighboring India, where wild animals
coral Island of Union Territory of such as tigers and elephant foray into,
Lakshadweep. to direct more resources and attention to
protecting them. There are now 173 species
 It was the 3rd such conference after
in Appendix 1.
1998, and 2008, but first in India.
 The mascot for CMS COP13 is “Gibi - The
 The Department of Environment and
Great Indian Bustard”.
Forest, Union Territory of Lakshadweep
Administration organized this mega
conference with the technical support
of Zoological Survey of India and in
association with Ministry of Environment,
Forest and Climate Change, IUCN, ENVIS
in consonance with declaration of the year
2018 as 3rd decadal International year of
Reefs.
 Very soon, an International Atoll
Research Centre will be established in 7. Correct Option: (a)
Lakshadweep.
Explanation:
6. Correct Option: (b)  It is a Ministry of Micro Small & Medium
Enterprises (MSME) initiative launched
Explanation: during June 2018.
13th COP to the CMS  The Khadi and Village Industries
Commission (KVIC) would implement
 The 13th Conference of Parties (COP)
the scheme.
of the Convention on the Conservation
of Migratory Species of Wild Animals  The scheme envisages setting up of Solar
(CMS), an environmental treaty under the Charkha Clusters which would mean a
aegis of the United Nations Environment focal village and other surrounding villages
Programme, is going to be hosted by India in a radius of 8 to 10 kilometers. Further,
during 17th to 22nd February 2020 at such a cluster will have 200 to 2042
Gandhinagar in Gujarat. India has been beneficiaries (spinners, weavers, stitches
designated the President of the COP for and other skilled artisans).
the next three years.
 Each spinner will be given two charkhas
 The Government of India has been taking of 10 spindles each. On average, it is
necessary actions to protect and conserve considered that such a cluster will have
migratory marine species. about 1000 charkhas. A cluster with full
capacity will provide direct employment to
 Seven species that include Dugong, Whale
2042 artisans.
Shark, Marine Turtle (two species), have
been identified for the preparation of the  The objectives of the Scheme are as
Conservation and Recovery Action Plan. follows-
 The theme of CMS COP13 in India is, ! To ensure inclusive growth by the
“Migratory species connect the planet and generation of employment, especially
we welcome them home. “The CMS COP for women and youth, and sustainable
13 logo is inspired by ‘Kolam’, a traditional development through solar charkha
art form from southern India. In the logo clusters in rural areas.
TARGET PT 2020 169

! To boost the rural economy and for Action (HFA) 2005-2015: Building the
help in arresting migration from Resilience of Nations and Communities to
rural to urban areas. Disasters.
! To leverage low-cost, innovative  The United Nations Sasakawa Award
technologies and processes for for Disaster Reduction is one of three
sustenance. prestigious prizes established in 1986
by founding Chairman of the Nippon
8. Correct Option: (d) Foundation, Mr. Ryoichi Sasakawa. It is
the most prestigious international award
Explanation:
in the area of Disaster Risk Management.
Keeling Curve
10. Correct Option: (b)
 The Keeling Curve is a graph of the
accumulation of carbon dioxide in Explanation:
the Earth’s atmosphere based on
continuous measurements taken at Purple frogs
the Mauna Loa Observatory in Hawaii  It is also known as Maveli frog or Pignose
from 1958 to the present day. Frog.
 The curve is named for the scientist Charles  It is endemic to the Western Ghats around
David Keeling, who started the monitoring the Palghat gap in Kerala.
program and supervised it until his death
in 2005.  It can be called ‘living fossil’ as it is
believed that they have co-existed
 The graph is about the continuous with dinosaurs almost 70 million years
diurnal variation in the concentration ago.
of Carbon dioxide largely due to
localized respiration/photosynthesis  It spends most of its lifecycle under the
from plants. ground.

11. Correct Option: (c)


Explanation:
Climate Change Performance Index
 The Climate Change Performance
Index published by Germanwatch,
Climate Action Network International
and the NewClimate Institute.
 Published annually since 2005, the Climate
Change Performance Index (CCPI) is an
9. Correct Option: (a) independent monitoring tool for tracking
countries’ climate protection performance.
Explanation: It aims to enhance transparency in
international climate politics and enables a
United Nations Office for Disaster Risk comparison of climate protection efforts and
Reduction (UNDDR) progress made by individual countries.
 UNDRR (formerly UNISDR) is the United  On the basis of standardized criteria, the
Nations focal point for disaster risk reduction. CCPI currently evaluates and compares
UNDRR oversees the implementation the climate protection performance of 57
of the Sendai Framework for Disaster countries and of the European Union (EU),
Risk Reduction 2015-2030, supporting which are together responsible for more
countries in its implementation, monitoring than 90% of global greenhouse gas (GHG)
and sharing what works in reducing emissions. This year(2020) for the first time
existing risk and preventing the creation of
Chile, as the country holding the COP25
new risk.
presidency, is added to the CCPI.
 The Sendai Framework for Disaster
 India, for the first, time ranks among the
Risk Reduction 2015-2030 was adopted
top ten in this year’s CCPI. The current
at the Third UN World Conference in
levels of per capita emissions and energy
Sendai, Japan, on March 18, 2015.
use are still comparatively low and, along
 The Sendai Framework is the successor with ambitious 2030 targets, result in high
instrument to the Hyogo Framework ratings for the GHG Emissions and Energy
170 TARGET PT 2020

Use categories. While the country receives  India has also recorded the highest
an overall medium rating in the Renewable number of fatalities due to climate
Energy category, India’s 2030 renewable change and the second-highest monetary
energy target is rated very high for its losses from its impact in 2018.
well-below-2°C compatibility. National
experts commend the government for 13. Correct Option: (d)
strong policies to support the expansion of
Explanation:
renewable energy, which is needed to meet
the ambitious targets as recent renewable Joint Conference on BRS Conventions
energy capacity additions are below the
level required. Despite an overall high  The joint meetings of three conventions
rating for its Climate Policy performance, on chemicals and waste that is the 14th
experts point out that the government has meeting of the Conference of the Parties
yet to develop a roadmap for the phase- (COP) to Basel Convention on the Control
out of fossil fuel subsidies that would of Trans-boundary Movement of Hazardous
consequently reduce the country’s high Wastes and their Disposal (COP 14) was
dependence on coal. held along with the 9th meeting of the COP
to Rotterdam Convention on the Prior
12. Correct Option: (b) Informed Consent Procedure for Certain
Hazardous Chemicals and Pesticides in
Explanation: International Trade and the 9th meeting of
the COP to Stockholm Convention on
Global Climate Risk Index 2020 Persistent Organic Pollutants.
 The annually published by the  The theme of the meetings this year
Germanwatch, Global Climate Risk Index was “Clean Planet, Healthy People:
analyses to what extent countries have been Sound Management of Chemicals and
affected by the impacts of weather-related Waste”.
loss events (storms, floods, heatwaves,
 The outcomes of the conference
etc).
included:
 The GermanwatchGlobal Climate Risk
! The establishment of a compliance
Index is an analysis based on one of the
mechanism under the Rotterdam
most reliable data sets available on the
Convention
impacts of extreme weather events and
associated socio-economic data. ! The listing of dicofol and
perfluorooctanoic acid (PFOA),
 This is the 15th edition of this annual
its salts, and PFOA-related
analysis. Its aim is to contextualize
compounds under the Stockholm
ongoing climate policy debates – especially Convention
the international climate negotiations –
looking at real-world impacts over the last ! Parties to the Basel Convention
year and the last 20 years. also adopted technical guidelines
on environmentally sound
 Japan, the Philippines, and Germany management of electrical and
were the most affected countries in 2018 electronic wastes (e-wastes) and
followed by Madagascar, India (5th) and also included plastic wastes in the
Sri Lanka. PIC procedure.
 The yearly monsoon season, lasting from ! Under the Rotterdam Convention, two
June to September, severely affected new chemicals (Phorate and HBCD)
India in 2018. The state of Kerala was were added in the list for mandatory
especially impacted – 324 people died PIC procedure in international trade.
because of drowning or being buried in
the landslides set off by the flooding, the ! India in Joint Conference
worst in one hundred years. Over 220 000 ! The draft technical guidelines
people had to leave their homes, 20 000 stipulated the conditions when used
houses and 80 dams were destroyed. The electrical and electronic equipment
damage amounted to EUR 2.4 billion (US$ destined for direct reuse, repair,
2.8 billion). Furthermore, India’s east coast refurbishment or failure analysis
was hit by the cyclones Titli and Gaja in should be considered as non-waste.
October and November 2018. With wind India had major reservations
speeds of up to 150 kilometers per hour, regarding these provisions as
cyclone Titli killed at least eight people and in the name of re-use, repair,
left around 450 000 without electricity. refurbishment, and failure
TARGET PT 2020 171

analysis there was a possibility  The report shows that sustainable land
of dumping from the developed management including sustainable
world to the developing countries forest management can help reduce
including India in view of land degradation and also tackle climate
the growing consumption of change.
electronic equipment and waste
 Coordinated efforts to tackle climate
across the world. The Indian
change will also help improve land, food
delegation strongly objected to the
security, and nutrition, etc.
proposed decision on these guidelines
during plenary and did not allow it  Reducing over-consumption and waste of
to be passed by the conference of the food, eliminating the clearing and burning
parties (COP). of forests, preventing over-harvesting of
fuelwood, and reducing greenhouse gas
! On the final day of the COP, a modified
emissions will help to address land-related
decision was adopted in which all
climate change issues.
the concerns raised by India were
incorporated like recognition that  Land-Climate Link:
the interim guideline has issues and
! Land use and changes in land use
further work is required especially on
have always been an integral part of
the provision on distinguishing waste
the conversation on climate change.
from non-waste and the guidelines
That is because land acts as both the
were adopted on an interim basis
source as well as a sink of carbon.
only.
! Land Degradation is both the
14. Correct Option: (d) cause and consequence of climate
change.
Explanation:
! Climate change causes the land to
Climate Change and Land degrade through both gradual changes
in temperature and rainfall patterns,
 The IPCC approved and accepted
as well as changes in the distribution
Climate Change and Land: an IPCC
and intensity of extreme events.
special report on climate change,
desertification, land degradation, ! Degraded land is less productive which
sustainable land management, food reduces its ability to absorb carbon
security, and greenhouse gas fluxes thus exacerbating climate change.
in terrestrial ecosystems at its 50th
Session held on 2 – 7 August 2019. 15. Correct Option: (c)
 This is the first time that IPCC has Explanation:
solely focused on the land sector.
Sustainable Livelihoods and Adaptation to
 The current report talks about the
Climate Change (SLACC)
contribution of land-related activities to
global warming — how the different uses  The project is jointly initiated by the
of land, like agriculture, industry, forestry, Union Ministry of Rural Development
cattle-rearing, and urbanization, was and the World Bank under National
affecting emissions of greenhouse gases. Rural Livelihood Mission (NRLM) to
improve the adaptive capacity of the rural
 An important part of the report talks about
poor, to climate variability and change
the manner in which even existential
affecting farm-based livelihoods, through
activities like food production contributes
community-based interventions.
to global warming and is also affected by
it.  SLACC targets to create a cadre of over
200 certified ‘climate-smart’ community
 The report says that if pre-production
resource persons in villages, who will
activities like cattle rearing and post-
help the rural population, cope with
production activities like transport, energy
the impact of climate change.
and food processing, is taken into account,
then food production could contribute as  The training programme of SLACC has
much as 37 percent of all greenhouse gas been launched to help Rural Poor Farm-
emissions every year. based Households adapt to Climate Change
and sustain their livelihoods.
 It points out that nearly 25 percent
of all food produced is either lost or  The course throws light on climate change,
wasted. And even the decomposition variability, interventions that can help in
of the waste releases emissions. improving crop productivity, information on
172 TARGET PT 2020

weather advisory services and alternative  In the last 60 years, extreme cold events
livelihood activities for climate resilience. have become lesser while extreme warm
 This course is being offered by the Centre weather events have become more
for Natural Resource Management of pronounced. Both minimum and maximum
National Institute of Rural Development temperatures are also changing: they are
and Panchayati Raj (NIRDPR), which is moving north, indicating overall warming.
also the Lead Technical Support Agency  Every decade HKH loses one cold night and
for the SLACC project.
half a cold day. While warm nights have
 The SLACC project is funded by the increased by 1.7 per decade, the region gets
Special Climate Change Fund, which 1.2 warm days every decade.
was set up under the United Nations
Framework Convention on Climate  Alarmingly, changes in surface temperature
Change to finance global projects (relative to 1976-2005) in this Himalayan
relating to adaptation, technology region are higher than the global average,
transfer, and capacity building, among and even the South Asian region.
other areas.  The projected changes in the surface mean
 The SLACC project is being temperature over the HKH region is larger
implemented in convergence with the compared to the global mean change by the
MahilaKisanSashaktikaranPariyojana, end of the 21st century.
Mahatma Gandhi National Rural
 The number of glaciers in the
Employment Guarantee Scheme
(MGNREGS) and other Centrally- Himalayan areahas increased in the
sponsored schemes. last five decades and this is an indicator
of how severe glacier melting has been
 A total of 638 drought and flood-prone due to global warming.
villages of Madhya Pradesh and Bihar
are being covered on a pilot basis  The increase in the number of
under this first of its kind initiative. glaciers is primarily due to glacier
It will be scaled up nationwide in due fragmentation — that big ones are
course. splitting into smaller ones. And this is
happening due to consistent loss in areas
16. Correct Option: (a) the glaciers occupy.
Explanation:  Smaller glaciers are shrinking faster than
larger ones, although the smaller glaciers
Hindu Kush Himalaya (HKH) Assessment
of Ladakh show a lower rate of a retreat
Report
than other Himalayan glaciers. However,
 International Centre for Integrated the assessment makes clear that despite
Mountain Development (ICIMOD), a the surety of glaciers in the Hindu Kush
regional intergovernmental body has Mountains losing length since 1973, no
released the first-ever assessment of studies have been done to examine area
climate change impacts on the Hindu change in this region.
Kush Himalayan (HKH) region.
 In 1998-2014, when global warming slowed
 The ICIMOD is pursuing 8 countries,
down, this region continued to warm.
including India, to set up an inter-
governmental body to protect the Hindu  In the 20th century, the HKH region
Kush Himalayan region, known as the oscillated between warming and cooling
water tower of Asia. phases. In the first 40 years, it reported
warming to be followed by a cooling phase
Main findings of the report
in 1940-1970.
 HKH region is warming faster than the
 However, since 1970 it has been warming,
global average. It would continue to warm
through this century even if the world is and as assessed, it would continue to be
able to limit global warming at the agreed through the current century.
1.5 degrees Celsius.  Warming may be good news for
 The per capita fossil fuel carbon dioxide agriculture as the length of the
emission from the HKH countries is one- growing season has increased by 4.25
sixth of the global average though it is days per decade — a positive change
disproportionately impacted. for agriculture.
TARGET PT 2020 173

17. Correct Option: (d)  The United Nations Conference on


Sustainable Development (also known
Explanation: as Rio+20) that took place in Rio de
Janeiro, Brazil in June 2012 recognize
SAMOA Pathway
these vulnerabilities and characteristics
 The Third International Conference on as - their small size, remoteness, narrow
Small Island Developing States was held resource and export base, and exposure
from 1-4 September 2014 in Apia, Samoa. to global environmental challenges and
external economic shocks, including a
 The Conference resulted in the adoption large range of impacts from climate change
of the Small Island Developing States and potentially more frequent and intense
Accelerated Modalities of Action (or natural disasters.
SAMOA) Pathway and the announcement
of 300 multi-stakeholder partnerships in 18. Correct Option: (a)
support of SIDS.
Explanation:
 SAMOA Pathway articulates the
sustainable development pathways Real-Time Ambient Noise Monitoring
and aspirations for SIDS over the next Network
10 years.
 National Ambient Noise Monitoring
 There are 16 priority areas under the Network Programme has been prepared
SAMOA Pathway. Some of them are: and circulated to state pollution control
boards. Central Pollution Control
! Sustained and sustainable, inclusive Board in association with the state
and equitable economic growth with pollution control board established
decent work for all Real-Time National Ambient Noise
! Climate Change Monitoring Network.

! Sustainable Energy  It will cover seven metros in phase-I


viz. Delhi, Hyderabad, Kolkata,
! Disaster Risk Reduction Mumbai, Bangalore, and Chennai have
! Gender Equality and Women’s been selected and in each Metro five Remote
Empowerment Monitoring Terminals have been installed
in different noise zones for continuous
! Biodiversity monitoring.
! Social Development  In Phase-II, another 35 Noise Monitoring
 It also established a unique Stations will be installed in the same seven
cities
intergovernmental SIDS Partnership
Framework, designed to monitor the  In Phase III, Real-Time Noise Network
progress of existing and stimulate the launch will be expanded by 90 stations to cover
of new, genuine and durable partnerships 18 other major cities by 2012. Phase-III
for the sustainable development of SIDS. cities include Kanpur, Pune, Surat,
Ahmedabad, Nagpur, Jaipur, Indore,
 On 27 September 2019, the General
Bhopal, Ludhiana, Guwahati, Dehradun,
Assembly will hold a one-day high-level Thiruvananthpuram, Bhubaneswar,
review of the progress made in addressing Patna, Gandhinagar, Ranchi, Amritsar
the priorities of Small Island Developing and Raipur.
States (SIDS) through the implementation
of the SAMOA Pathway.  The network in the major Metros will also
be augmented by establishing more noise
 The General Assembly has decided that the monitoring stations.
high-level review will result in “a concise
action-oriented and inter-governmentally 19. Correct Option: (a)
agreed political declaration”.
Explanation:
Small Island Developing States (SIDS)
Green Skill Development Programme
 SIDS is a group of small island countries
that are afflicted by economic difficulties  It was launched in line with the Skill India
and confronted by development imperatives programme for skilling India‘s youth in the
environment and forest sector.
similar to those of developing countries
generally but have their own peculiar  It endeavours to develop green skilled
vulnerabilities and characteristics. workers having technical knowledge and
174 TARGET PT 2020

commitment to sustainable development, 21. Correct option: (b)


which will help in the attainment of the
Nationally Determined Contributions Explanation
(NDCs), SDGs, National Biodiversity
Statement 1 is incorrect: Breast Milk Banks
Targets (NBTs), as well as Waste
are established under “National Guidelines
Management Rules (2016).
on Establishment of Lactation Management
 It aims to train over 5.5 lakh workers Centres in Public Health Facilities” by the
in the environment and forest sectors Ministry of Health and Family Welfare.
in the country through 30 courses by
2021. Supplementary notes
 Botanical Survey of India and Accessible India Campaign (Sugamya
Zoological Survey of India were Bharat Abhiyan)
the nodal Centres for the pilot
 Accessible India Campaign (Sugamya
programme.
Bharat Abhiyan) is a nation-wide Campaign
 It provides gainful employment to launched by Department of Empowerment
candidates completing the course in the of Persons with Disabilities (DEPwD) of
zoos/wildlife sanctuaries/national parks Ministry of Social Justice & Empowerment
etc. to provide universal accessibility to persons
with disabilities.
 It is being implemented by utilising
the vast networks and expertise of  The campaign aims at providing equal
ENVIS Hubs. opportunity to persons with disabilities
to participate in all the aspects of life and
20. Correct Option: (c) live independently. The Sugamya Bharat
Abhiyan focuses on developing accessible
Explanation: physical environment, transportation
system and Information & communication
Green Muffler Scheme ecosystem.
 Green Muffler is a technique of reducing  The Government of India with firm
noise pollution by planting 4-6 rows commitment towards socio-economic
around the populated areas or noisy transformation of the persons with
places like along roadsides, industrial disabilities is making efforts to create mass
areas, societies near highways, etc. so that awareness for universal accessibility.
dense trees filter out the noise and obstruct  India is a signatory to the UN Convention
reaching the citizens. It is also a device for on the Rights of Persons with Disabilities
decreasing the amount of noise emitted (UNCRPD).
by the exhaust of an internal combustion
engine. Components of Accessible India
Campaign
 Under the Green Muffler scheme, Asoka
and Neem plants are planted near the Accessible India Campaign (Sugamya
house or resident localities to reduce noise Bharat Abhiyan) has the following three
pollution.
important components
 Trees are known as noise buffers  Built Environment Accessibility
as they control noise pollution by
absorbing high-frequency noise. Even  Transportation System Accessibility
urban noises are muffled by trees just like  Information and Communication Eco-
stone walls. Evergreen shrubs that too with System Accessibility
broader leaves provide year-round noise
protection so, they are best to plant. Trees United Nations Convention on the Rights
absorb sound waves with their branches of Persons with Disabilities (UNCRPD)
and foliage. Plant trees with no space or  To develop, promulgate and monitor the
less space are better efficient to reduce implementation of minimum standards
noise pollution. and guidelines for the accessibility of
 According to USDA National facilities and services open or provided to
Agroforestry Center a properly designed the public;
buffer of trees and shrubs can reduce  To ensure that private entities that offer
noise by about 10 decibels or about 50% as facilities and services which are open or
perceived by the human ear. provided to the public take into account
TARGET PT 2020 175

all aspects of accessibility for persons with can carry 225-275 Kg of CNG with new
disabilities; composite cylinders.
 To provide training for stakeholders on  More storage of CNG means more KM
accessibility issues facing persons with range of buses. Buses fitted with Type IV
disabilities; Composite Cylinders have a running range
of approx. 800- 1000 KM per fill of CNG.
 To provide in buildings and other facilities
open to the public signage in Braille and in  IGL has procured 5 number of Mahindra’s
easy to read and understand forms; Type IV buses. The Buses would be given
to Uttarakhand Transport Corporation
 To provide forms of live assistance and
(UTC) on lease basis after the launch.
intermediaries, including guidelines,
readers and professional sign language  These will ply on Inter-city routes from
interpreters, to facilitate accessibility to Delhi to Dehradun and will be the first
buildings and other facilities open to the CNG Buses in Uttarakhand. Now with this
public; launch, DTC and other State Transport
Corporations can again start their long
 To promote other appropriate forms of
haul operations through CNG Buses.
assistance and support to persons with
disabilities to ensure their access to  In addition, with more capacity of CNG in
information; one vehicle, it is likely that there shall be
reduction in queues at the CNG Stations as
 To promote access for persons with
these buses will not have to come frequently
disabilities to new information and
for fueling.
communications technologies and systems,
including the Internet.  Apart from purchasing new OEM
manufactured Type IV buses, Fleet owners
22. Correct option: (c) and State Transport Corporations also have
the option of retrofitting their existing CNG
Explanation buses having Type I (Steel Cylinders) with
lightweight Type IV Cylinders. Retrofitting
Statement 1 and 2 are correct: In a major with Type IV CNG cylinders is also possible
step towards making India a gas-based in buses running on other fuels.
economy and making CNG as the eco-
friendly option for long distance transport 23. Correct option: (d)
in the country, Minister of Petroleum &
Natural Gas and Steel, unveiled India’s Explanation
first long distance CNG bus fitted with All the statements are correct: The Union
composite CNG cylinders, which can travel Cabinet chaired by the Prime Minister has
around 1000 kms in a single fill. given its approval for the implementation
The project has been executed by of the AtalBhujalYojana (ATAL JAL), a
Indraprastha Gas Limited (IGL) and has Central Sector Scheme with a total outlay
been achieved through pioneering design of Rs.6000 crore to be implemented over a
of Type IV Composite Cylinders in buses, period of 5 years (2020-21 to 2024-25).
replacing traditional very heavy Type-I Supplementary notes
Carbon Steel cylinders.
 Ground water contributes to nearly 65%
Supplementary notes of total irrigated area of the country and
nearly 85% of the rural drinking water
 Mahindra &Mahindra,& Agility Fuel supply. The limited ground water resources
Solutions of USA have partnered with in the country are under threat due to the
IGL for this project, involving introducing increasing demands of growing population,
the new concept of light weight Type urbanization and industrialization.
IV composite cylinders in buses. These
cylinders are 70% lighter than the Type - I  Intensive, and unregulated ground water
(all steel) cylinders which are being used in pumping in many areas has caused
India currently. rapid and widespread decline in ground
water levels as well as reduction in the
 The main advantage of these Cylinders is sustainability of ground water abstraction
that due to its lighter weight, the number structures.
of cylinders can be increased in the vehicle
thus creating more storage capacity on-  The problem of reduction in ground water
board. The buses which used to carry only availability is further compounded by
80-100 Kg of CNG with steel cylinders deteriorating ground water quality in
176 TARGET PT 2020

some parts of the country. The increasing  Implementation of Water Security Plans
stress on ground water due to over- through convergence of various ongoing/
exploitation, contamination and associated new schemes of the Government of India and
environmental impacts threaten to State Governments to facilitate judicious
endanger the food security of the nation, and effective utilization of funds for
unless necessary preventive / remedial sustainable ground water management.
measures are taken on priority.
 Efficient use of available ground water
 The Department of Water Resources, River resources with emphasis on demand side
Development & Ganga Rejuvenation, measures such as micro-irrigation, crop
Ministry of Jal Shakti has taken a diversification, electricity feeder separation
pioneering initiative for ensuring long term etc.
sustainability of ground water resources in
the country through the AtalBhujalYojana Impact:
(ATAL JAL) by adopting a mix of ‘top down’  Source sustainability for JalJeevan Mission
and ‘bottom up’ approaches in identified in the project area with active participation
ground water stressed blocks in seven of local communities.
states, representing a range of geomorphic,
climatic and hydrogeologic and cultural  Will contribute towards the goal of doubling
settings. the farmers’ income.
 ATAL JAL has been designed with the  Will promote participatory ground water
principal objective of strengthening the management.
institutional framework for participatory
 Improved water use efficiency on a mass
ground water management and bringing
scale and improved cropping pattern;
about behavioral changes at the community
level for sustainable ground water resource  Promotion of efficient and equitable use of
management. ground water resources and behavioural
change at the community level.
 The scheme envisages undertaking this
through various interventions, including
24. Correct option: (C)
awareness programmes, capacity building,
convergence of ongoing/new schemes and Explanation
improved agricultural practices etc.
Statement 2 is incorrect: Quick Reaction
ATAL JAL has two major components: Surface to Air Missile (QRSAM) system
 Institutional Strengthening and Capacity developed by Defence Research and
Building Component for strengthening Development Organization (DRDO) was
institutional arrangements for sustainable successfully flight-tested from Integrated
ground water management in the States Test Range, Chandipur off the Odisha coast
including improving monitoring networks,
at 1145 hrs on December 23, 2019.
capacity building, strengthening of Water
User Associations, etc. Supplementary notes
 Incentive Component for incentivising
Quick Reaction Surface to Air Missile
the States for achievements in improved
(QRSAM) system
groundwater management practices
namely, data dissemination, preparation  They are developed by Defence Research
of water security plans, implementation and Development Organisation (DRDO).
of management interventions through
 The missile has been flight-tested with
convergence of ongoing schemes, adopting
full configuration in deployment mode
demand side management practices etc
intercepting the target mid-air, meeting
ATAL JAL will result in: the mission objectives.

 Institutional strengthening for improving  The system operates on the move, comprises
ground water monitoring networks and of fully automated Command and Control
capacity building of stakeholders at System, Active Array Battery Surveillance
different levels which will enhance ground Radar, Active Array Battery Multifunction
water data storage, exchange, analysis and Radar and Launcher.
dissemination.  Both radars are four-walled having 360-
 Improved and realistic water budgeting degree coverage with search on move and
based on an improved database and track on move capability.
preparation of community-led Water  The system is compact with minimum
Security Plans at Panchayat level number of vehicles for a firing unit. Single
TARGET PT 2020 177

stage solid propelled missile has midcourse of Citizens (NRC).


inertial navigation system with two-
way data link and terminal active seeker  Though an NPR doesn’t necessarily mean
developed indigenously by DRDO. it’s guaranteed there will be an NRC, it
clears the path for a nationwide citizens’
 The weapon system is expected to be ready
for induction by 2021. list.

 This is seen as one of the reasons why some


25. Correct option: (d)
states like West Bengal and Kerala, which
Explanation are opposed to the NRC, have stopped work
on the NPR.
All the above statements are correct
 NPR was first done in 2010 and was later
Supplementary notes
updated in 2015 when it was linked with
NPR - National Population Register the Aadhar.
 The Census Commission has said the  The NPR is a register of the usual residents
objective of the NPR is to create a
of the country.
comprehensive identity database of every
“usual resident” of the country.  It contains information collected at the
 The database will have demographic local (village/sub-town), subdistrict,
details. district, state and national level under
 A “usual resident”, for the NPR, is a person provisions of the Citizenship Act, 1955 and
who has lived in an area for at least six the Citizenship (Registration of Citizens
months or more, or a person who intends and Issue of National Identity Cards)
to live in an area for the next six months Rules, 2003.
or more. It is mandatory for every “usual
resident” of India to register in the NPR.  The process of updating NPR will be carried
 The NPR, since it is linked to the Census, is out under the aegis of the Registrar General
seen as the first step towards a nationwide and ex-Officio Census Commissioner,
exercise to implement the National Register India.

**********
178 TARGET PT 2020

TEST
DAY - 43

Time Allowed: 30 mins Maximum Marks: 50

1. Which of the following statements is Which of the above statements are correct?
correct regarding the Brown to Green
(a) 1 only
Report 2019?
(b) 2 only
(a) It is the comprehensive report of
Greenfield and Brownfield projects. (c) Both 1 and 2

(b) It is a review of the G20 climate (d) Neither 1 nor 2


action.
4. Which of the following Indian
(c) It is a comprehensive study of the red companies is a part of ‘Alliance to End
tide. Plastic Waste’?
(d) It is a comprehensive report by (a) Tata Group
MoEFCC about the nature and impacts (b) Reliance Industries
of brown carbon.
(c) Pepsico
2. Which of the following statements (d) All of the above
regarding the Global Coalition for
Disaster-Resilient Infrastructure is/ 5. The Turtle Rehab Centre, which is the
are correct? first of its kind, has been established
1. It is a non-governmental coalition to in___
prevent the loss of infrastructure. (a) Odisha
2. The formation of the Coalition is (b) Bihar
the result of efforts by India and the
United Nations Office for Disaster Risk (c) West Bengal
Reduction. (d) Andhra Pradesh

Select the correct option using the codes 6. Consider the following statements?
given below: 1. Hydroponics is a method of growing
(a) 1 only plants without soil.
(b) 2 only 2. Aquaponics combines aquaculture and
hydroponics.
(c) Both 1 and 2
3. In Aeroponics, very little water is
(d) Neither 1 nor 2 needed.

3. Consider the following statements Which of the above statements are correct?
regarding Atal Bhujal Yojana:
(a) 1 and 3 only
1. The Central Sector Scheme will be
(b) 2 and 3 only
applied across all the states of India.
(c) 1 and 2 only
2. It will be supported by the Asian
Development Bank. (d) 1, 2, and 3
TARGET PT 2020 179

7. Which of the following statements (a) 1 and 4 only


regarding the Vertical Farming are
(b) 2 and 3 only
correct?
1. It is a type of urban farming. (c) 1, 2 and 3 only

2. It uses the same amount of water but (d) 1, 2, 3 and 4


the other raw materials consumed by
10. Which of the following statements
vertical farming is less.
regarding the Global Solar Council are
3. Vertical farm production is no longer correct?
dependent on using fossil resources.
1. It has been established under the aegis
Select the correct option using the codes of the International Solar Alliance.
given below: 2. It was launched during the COP 21
(a) 1 and 2 only Paris conference.

(b) 2 and 3 only 3. India is one of the principal members.

(c) 1 and 3 only Select the correct option using the codes
given below:
(d) 1, 2, and 3
(a) 1, 2 and 3
8. Consider the following statements
regarding the Taj Trapezium Zone (b) 1 and 3 only
(TTZ): (c) 2 and 3 only
1. It has been established in 2019. (d) 1 and 2 only
2. It has been formed to protect the three
World Heritage Sites of Agra. 11. Around 73 countries have joined the
Climate Ambition Alliance (CAA).
3. The zone falls in Uttar Pradesh and
Regarding this, consider the following
Rajasthan only.
statements:
4. The construction, industrial activities
1. It was launched at the 25th Conference
and felling of trees are completely
of the Parties to the United Nations
banned in the TTZ.
Framework Convention on Climate
Which of the above statements are correct? Change.

(a) 2 and 3 only 2. It has set the target to achieve the net
zero carbon dioxide emissions by 2050.
(b) 3 and 4 only
3. It will focus on strong actions to
(c) 1, 3, and 4 only improve the management of water and
(d) 2, 3, and 4 only the sustainability of cities.

9. Which of the following statements Which of the above statements are correct?
regarding ‘hydrochlorofluorocarbon (a) 1, 2, and 3
(HCFC)-141 b’ are correct?
(b) 1 and 3 only
1. It is used in foam manufacturing.
(c) 1 and 2 only
2. It does not contribute to ozone
depletion, but, has a high global (d) 2 and 3 only
warming potential.
12. India is one of the first countries in
3. The Kigali Amendment to the Montreal the world to develop a comprehensive
Protocol has planned to phase down Cooling Action plan. Regarding this,
the substance worldwide. consider the following statements:
4. India has achieved a complete phase- 1. It will help in reducing both direct and
out of it. indirect emissions
Select the correct option using the codes 2. It will help in doubling the farmers
given below: income.
180 TARGET PT 2020

Select the correct option using the codes of Environment and Department
given below: of Science and Technology are
(a) 1 only implementing agencies of this
programme.
(b) 2 only
Which of the above statements is/are
(c) Both 1 and 2
correct?
(d) Neither 1 nor 2
(a) 1 and 2 only
13. Globally Important Agricultural (b) 1 only
Heritage Systems (GIAHS) was
launched to create public awareness & (c) 2 and 3 only
safeguard world agricultural heritage (d) 1, 2 and 3
sites. Which of the following sites finds
its place in the GIAHS list from India? 16. Consider the following statements with
1. Traditional Agriculture Systems reference to Solar Geo-Engineering
Koraput, Odisha. frequently seen in news:
2. Kuttanad below Sea Level Farming 1. It is a process through which the
System in Kerala. reflectivity of the Earth’s atmosphere
3. Saffron Heritage of Pampore, Jammu or surface is decreased to offset climate
& Kashmir. change.

Select the correct answer using the code 2. Methods include marine cloud
given below: brightening and using mirrors in
space.
(a) 1 and 2 only
(b) 1 and 3 only Which of the above statements is/are
correct?
(c) 2 and 3 only
(a) 1 only
(d) 1, 2 and 3
(b) 2 only
14. Brazzaville declaration is in relation
to: (c) Both 1 and 2

(a) BRICS Summit (d) Neither 1 nor 2

(b) Conserve terrestrial, marine and avian 17. India has launched Fulbright - Kalam
migratory species throughout their Climate Fellowship in association with
range. which of the following nation?
(c) Protect the benefits provided by (a) Russia
peatland ecosystems.
(b) United States of America
(d) Removal of non-tariff barriers for
developing countries. (c) France
(d) Germany
15. With reference to the Black Carbon
Research Initiative, consider the
18. Consider the following statements
following statements:
regarding the National Biofuel Policy:
1. It is an Indian initiative launched as
1. Rajasthan has become the first state to
a part of the National Carbonaceous
Aerosols Program (NCAP). implement it.

2. It is to monitor the black carbon 2. India is the first country to have


aerosols and assess its impacts on the operated biofuel-powered flights.
microclimates. 3. The Spicejet biofuel flight was
3. Ministry of Earth Sciences, Indian developed using Jatropha seeds, a
Space Research Organization Ministry third-generation biofuel.
TARGET PT 2020 181

Which of the above statements are 21. Consider the following statements
incorrect? regarding World Trade Organization:
(a) 1 only 1. It came into existence as a result of
(b) 2 and 3 only Paraguay rounds of Negotiations held
under General Agreement on Trade
(c) 1 and 3 only
and Tariffs.
(d) 1, 2 and 3
2. All decisions in WTO Ministerial are
19. The Supreme Court recently ordered made by consensus
the eviction of STs/OTFDs whose 3. WTO provides detailed criteria for
claims as forest dwellers have been the classification of member countries
rejected under the Forest Rights Act, into Developing, Developed and Least
2006. In this regard, consider the
Developed nations.
following statements:
1. The symbiotic relationship between Which of the statements given above is/are
forests and forest-dwelling communities correct?
was first recognized under FRA, 2006.
(a) 1 and 3 only
2. As per the Forest Rights Act, 2006,
Gram Panchayat determines the (b) 2 only
nature and extent of individual and
(c) 2 and 3 only
community forest rights.
3. The Act provides for a right to (d) 1, 2 and 3
intellectual property over traditional
22. Consider the following statements
knowledge related to biodiversity.
regarding Gallantary Awards in
Which of the above statements is/are India:
correct? 1. Only Indian Navy, Army and Air Force
(a) 1 and 2 only personnel are eligible to receive these
(b) 2 only awards.

(c) 3 only 2. Mahavir Chakra is the highest level


of Gallantary award instituted by the
(d) 2 and 3 only
Government of India.
20. With reference to the SAMOA Pathway, 3. These awards announced twice in a
consider the following statements: year.
1. It articulates the sustainable
development pathways and aspirations Which of the statements given above is/are
for Small Island Developing States correct?
(SIDS). (a) 1 only
2. Social Development and Women’s
(b) 3 only
Empowerment is one of the priority
areas under it. (c) 1 and 3 only
3. SIDS is characterized by a narrow (d) 1 and 2 only
export base.
23. Kajin Sara Lake is located in which
Which of the above statements is/are
correct? neighbouring country of India?

(a) 1 only (a) Nepal

(b) 1 and 2 only (b) Bangladesh

(c) 2 and 3 only (c) Sri Lanka


(d) 1, 2 and 3 (d) Myanmar
182 TARGET PT 2020

24. Recently, a creature known as 1. Gangotri


Tardigrade was discovered on an 2. Yamunotri
Israeli spacecraft. What is the unique
characteristic of this creature? 3. Kedarnath

(a) It changes its colour very frequently. 4. Badrinath

(b) It can survive in vacuum conditions Select the correct answer using the
code given below:
(c) It can kill almost all drug resistant
bacteria (a) 2-3-1-4
(d) None of the above (b) 3-2-4-1
(c) 1-2-3-4
25. Arrange the following shrines of the
Char Dham in the west to east order: (d) 4-3-2-1

**********
TARGET PT 2020 183

ANSWER HINTS
DAY - 43

1. Correct Option: (b) and the Paris Climate Agreement.

Explanation: 3. Correct Option: (d)


G20 Brown to Green Report 2019 Explanation:
 The Brown to Green Report 2019 is the
Atal Bhujal Yojana
world’s most comprehensive review of
G20 climate action.  The World Bank-supported, Atal Bhujal
Yojana (ABHY) is an Rs.6000 Crore
 It provides concise and comparable
scheme, for sustainable management of
information on G20 country mitigation
action, finance, and adaptation. groundwater with community participation.
It will be implemented by the Ministry of
 Developed by experts from 14 research Jal Shakti.
organizations and NGOs from the majority
of the G20 countries, the report covers 80  The government of India has identified
indicators. It informs policymakers and over-exploited (OE) and water-stressed
stimulates national debates. areas for the implementation of the scheme
fall in the seven states viz. Gujarat,
2. Correct Optio: (b) Haryana, Karnataka, Madhya Pradesh,
Maharashtra, Rajasthan and Uttar
Explanation: Pradesh including the nine blocks of
poverty-ridden Bundelkhand region.
Global Coalition for Disaster-Resilient
Infrastructure  The funding pattern is 50:50 between
Government of India and the World
 CDRI’s mission is to rapidly expand the Bank.
development of resilient infrastructure and
retrofit existing infrastructure for resilience
4. Correct Option: (b)
and to enable a measurable reduction in
infrastructure losses. Explanation:
 The formation of the Coalition is Alliance to End Plastic Waste
the result of efforts by India and the
United Nations Office for Disaster Risk  It is a not-for-profit organization, partnering
Reduction, responding to India’s Prime with the finance community, government
Minister’s call at the Asian Ministerial and civil society, including environmental
Conference on DRR for action to reduce and economic development NGOs working
damage to critical infrastructure. to make the dream of a world without
plastic waste a reality.
 Developed through consultations with more
than 35 countries, CDRI envisions enabling  The only Indian company as a partner to
measurable reduction in infrastructure this Alliance is the Reliance Industries
losses from disasters, including extreme
climate events. CDRI thus aims to enable 5. Correct Option: (b)
the achievement of objectives of expanding
universal access to basic services and Explanation:
enabling prosperity as enshrined in the
Freshwater turtles rehab center
Sustainable Development Goals, while also
working at the intersection of the Sendai  A first-of-its-kind rehabilitation center
Framework for Disaster Risk Reduction for freshwater turtles will be inaugurated
184 TARGET PT 2020

in Bihar’s Bhagalpur forest division in  A Vertical Farm:


January 2020.
! drastically reduces agricultural land
 The rehab center, spread over half a use.
hectare, will be able to shelter 500 turtles
! saves up to 95% water.
at a time.
 The need to build such a center was felt
! makes cultivation possible,
independent of weather conditions &
after several turtles were found severely
season.
wounded and sick when rescued from
smuggles by rescue teams. ! delivers each harvest with continuous
quality.
6. Correct Option: (d)
! brings maximum freshness into the
Explanation: city.

New mode of agriculture ! saves on transport to the consumer.


 Hydroponics is a predominant system ! limits CO2 and puts less strain on the
of growth that involves the growth of climate.
plants in solutions of nutrients that ! brings agriculture back into urban
are essentially free of soil. In this everyday life.
vertical farming innovation, the roots of
the plants are submerged in a solution of  The primary focus of all functions
nutrients. This is frequently circulated and in and around vertical farming is on
monitored in order to ensure that there is optimal plant growth while maximizing
the maintenance of the correct chemical the use of natural resources, such
composition in the nutrient solution. as the sunlight. With adaptation of
nano cost climate control, low-carbon
 Aquaponics is a symbiotic integration of electricity, high-efficiency lighting,
two mature disciplines: aquaculture vertical farming achieves a low
and hydroponics. It is much like the carbon footprint easily. This is why a
Hydroponics System but is only better. It vertical farm production is no longer
aims to combine the fish and plants in the dependent on using fossil resources.
same ecosystem.
 Aeroponics is defined as “growing plants in 8. Correct Option: (a)
an air/mist environment with no soil and
very little water.
Explanation:
Taj Trapezium Zone (TTZ)
7. Correct Option: (c)
 Taj Trapezium Zone (TTZ) is a defined area
Explanation: of 10,400 sq km around the Taj Mahal to
protect the monument from pollution.
Vertical Farming
 The TTZ was established on December
 Vertical Farming or vertical agriculture
30, 1996, after the ruling of the Supreme
facilitates viable agricultural production
Court.
inside buildings, in the metropolitan
areas of our cities. Vertical Farming  The Court banned the use of coal/ coke
is, therefore, a form of urban in industries located in the TTZ with a
agriculture. mandate for switching over from coal/
coke to natural gas, and relocating them
outside the TTZ or shutting down.
 Recently, in December 2019, the Supreme
Court lifted its earlier interim order
imposing a complete ban on construction,
industrial activities and felling of trees in
the Taj Trapezium Zone (TTZ). But, the
heavy industries will continue to be banned
in the same way.
 The TTZ comprises monuments
including three World Heritage Sites
the Taj Mahal, Agra Fort, and Fatehpur
Sikri. TTZ is so named since it is located
TARGET PT 2020 185

around the Taj Mahal and is shaped like a  It strives to convey 3 Key Messages about
trapezoid. solar power viz. Solar power is already one
of the cheapest forms of electricity globally;
 The geographical limits of the Taj
Solar power is everywhere, and alleviates
Trapezium Zone is defined in the shape
poverty, especially in rural areas; and o
of trapezoid between 26° 45N & 77 ° 15E avoid a greater than 20C increase in global
to 27 ° 45N & 77° 15E in the West of the temperatures, we must strongly accelerate
Taj Mahal and in the East of Taj Mahal the deployment of solar power.
between 27° 00N & 78° 30E to 27° 30E,
lying in the Agra Division of the State  The principal members of the Global
of Uttar Pradesh and in the Bharatpur Solar Council include Australia,
Division of the State of Rajasthan. China, Europe, India, and other Asian
countries, Middle East, South America,
 Responsibility areas: and the US.
! Protecting Taj Mahal from damage
via pollution 11. Correct Option: (d)
! Control Pollution Explanation:
! Keep a record of the Taj Mahal Climate Ambition Alliance
pollution statics
 The alliance is led by Chile and was
! Study city pollution launched at the Climate Action Summit,
New York in 2019.
9. Correct Option: (a)
 At the COP25 to the UNFCCC, it was
Explanation: presented and around 73 countries
have joined the Alliance.
Hydrochlorofluorocarbon (HCFC)-141 b
 Both Chile and the UK will work together
 India has successfully achieved to mobilize additional actors to join the
the complete phase-out of alliance. They will also work towards
Hydrochlorofluorocarbon (HCFC)- COP26 that is to be held in November 2020
141 b, which is a chemical used by foam at Glasgow, UK.
manufacturing enterprises and one
of the most potent ozone-depleting  The main purpose of the alliance is to
upscale 2020 targets of members of the
chemicals after Chlorofluorocarbons
UN and to achieve Net Zero Carbon
(CFCs).
Emissions by 2050.
 (HCFC)-141 b is used mainly as a
 For mitigation, it will focus on the
blowing agent in the production of rigid
submission of enhanced Nationally
polyurethane (PU) foams.
Determined Contributions; reaching new
Hydrofluorocarbons commitments to achieve Net Zero by 2050;
and the implementation of measures to
 Hydrofluorocarbons (HFCs) have been used strengthen the protection of forests and
since the early 1990s as an alternative to oceans.
CFCs and HCFCs.
 For adaptation, it will focus on strong
 HFCs do not contribute to ozone actions to improve the management of
depletion, but they are potent water, resilience in infrastructure and
greenhouse gases. the sustainability of cities.
 The Kigali Amendment to the Montreal
Protocol has planned to phase down 12. Correct Option: (c)
these substances worldwide. Explanation:
10. Correct Option: (c) India Cooling Action Plan
Explanation:  India is one of the first countries in the
world to develop a comprehensive Cooling
Global Solar Council Action plan which has a long term vision
to address the cooling requirement across
 The International Solar Energy
sectors and lists out actions that can help
Society (ISES) is a founding member
reduce the cooling demand.
of the Global Solar Council, launched
at the COP 21 climate change meeting  The India Cooling Action seeks to (i) reduce
in Paris. cooling demand across sectors by 20% to
186 TARGET PT 2020

25% by 2037-38, (ii) reduce refrigerant ! To conserve and promote this


demand by 25% to 30% by 2037-38, (iii) knowledge at a global scale to
Reduce cooling energy requirements by promote food security, sustainable
25% to 40% by 2037-38, (iv) recognize development.
“cooling and related areas” as a thrust
area of research under national S&T
! Providing incentives for the local
population by measures like eco-
Programme, (v) training and certification
of 100,000 servicing sector technicians labeling, eco-tourism.
by 2022-23, synergizing with Skill India  GIAHS Sites in India: The following sites
Mission. These actions will have significant have been recognized.
climate benefits.
! Koraput, Odisha State:
 The overarching goal of ICAP is to provide
sustainable cooling and thermal comfort for  This region has rich biodiversity,
all while securing environmental and socio- growing several varieties of
economic benefits for the society. This will paddy, millets, pulses, oilseeds,
also help in reducing both direct and vegetables.
indirect emissions.  The region primarily a tribal
 The following benefits would accrue to the district inhabited by khonds, bonda
society over and above the environmental tribes practicing poddhu (shifting)
benefits: (i) Thermal comfort for all – cultivation.
provision for cooling for EWS and LIG  Shifting cultivation – loss of forest
housing, (ii) Sustainable cooling – low cover = hurting the biodiversity.
GHG emissions related to cooling, (iii)
Doubling Farmers Income – better cold  Soil erosion, Soil degradation,
chain infrastructure – better value habitat loss.
of produce to farmers, less wastage  Illiteracy, large family, small farm
of produce, (iv) Skilled workforce for holding size.
better livelihoods and environmental
protection, (v) Make in India – domestic  The socio-economic indicators are
manufacturing of air-conditioning and very poor here nearly 84% living in
related cooling equipment’s, (vi) Robust abject poverty.
R&D on alternative cooling technologies
 Kuttanad: Kuttanad is a delta region of
– to provide push to innovation in cooling
about 900 sq. km situated on the west coast
sector.
of Kerala State, India.
13. Correct Option: (d)  Unique feature: Below sea level
rice cultivation site, only such a
Explanation: system in India.
GIAHS  Farmers of Kuttanad have
 GIAHS are the Globally Important developed and mastered the
Agricultural Heritage Systems (GIAHS) spectacular technique of below sea
created to enhance public awareness, level cultivation over 150 years ago.
safeguard world agricultural heritage  They made this system unique as
sites. it contributes remarkably well to
 It was started in 2002 by the FAO (Food the conservation of biodiversity
and Agricultural Organization). and ecosystem services including
several livelihood services for local
 FAO an intergovernmental organization, communities.
headquartered in Rome with 191 member
nations. Aims at helping the world  Kashmir Valley, Pampore region:
population get ensured food security,  Saffron Heritage Site of Kashmir
eliminate hunger, poverty. in India
 Objectives of GIAHS  Grains such as maize, rice, rajmah/
! To understand and appreciate the lentils, fruit and vegetable crops,
nature-friendly agricultural practices and pulses.
of local and tribal populations around
 A set of unique low-tillage
the world.
traditional agricultural practices
! To document indigenous knowledge. are carried
TARGET PT 2020 187

 During the fallow period, the  They are a globally important carbon
growth of fruit, fodder and store. The unregulated exploitation of
mulberry trees along the farm peatlands can potentially be detrimental to
boundaries (Agro-forestry) is the environment and to climate, as it could
practiced, thereby maintaining release carbon emissions that have been
traditional agro-biodiversity. locked in for millennia.
 What will GIAHS do in Indian Sites?
15. Correct Option: (d)
! Contribute to awareness-raising
Explanation:
! Promote the use of modern technologies
to conserve heritage systems. Black Carbon Research Initiative
! Documenting traditional knowledge.  ISRO-GBP (Indian Space Research
! Provide capacity building training for Organization’s Geosphere-Biosphere
farmers to increase productivity and Programme) recognized the importance
marketing practices. of Black Carbon aerosols on the climate
system and decided to pursue studies of
! Modernization, commercialization Black Carbon in subsequent years.
strategies establishing standards,
eco-labeling.  In view of this, a multi-institutional and
multi-agency Science Plan has been
! Establishing sustainable practices launched in the Ministry of Environment
amongst the tribals utilizing their in association with the Ministry of
knowledge, modern technologies. Earth Sciences, Indian Space Research
Organization, Ministry of Science
14. Correct Option: (c) and Technology and other associated
Explanation: agencies, to monitor aerosols and
assess its impacts through various
Brazzaville Declaration modeling techniques.
 Brazzaville Declaration was signed to  It is an Indian initiative launched as a
promote better management and part of the National Carbonaceous
conservation of Cuvette Central Aerosols Program (NCAP) under the
Region in the Congo Basin. It aims aegis of the Indian Network for Climate
to implement coordination and Change Assessment, INCCA.
cooperation between different
government sectors to protect the 16. Correct Option: (b)
benefits provided by peatland
ecosystems. It also recognizes the Explanation:
importance of the scientific breakthrough
of mapping the world’s largest tropical Solar Geo-Engineering/Solar Radiation
peatlands area i.e. carbon stock and to Management (SRM)
prevent it from being emitted.  It is a process through which the reflectivity
 The declaration has been signed by the (albedo) of the Earth’s atmosphere or
Democratic Republic of Congo, the surface is increased, in an attempt to
Republic of Congo and Indonesia in the offset some of the effects of GHG-induced
backdrop of the 3rd Conference of Partners climate change through anthropogenic
of the Global Peatlands Initiatives (GPI), intervention. The technique mimics
taking place in Brazzaville, Republic of big volcanic eruptions that can cool the
Congo. Earth by masking the sun with a veil of ash
or similar other things.
 GPI is an initiative by leading experts and
institutions to save peatlands as the  Solar Radiation Management Governance
world’s largest terrestrial organ. Initiative (SRMGI) is an international,
NGO-driven project, financed by Dustin
 Peatlands are wetlands that contain Moskovitz (co-founder of Facebook)
a mixture of decomposed organic for expanding the discussion of
material, partially submerged in a SRM climate engineering research
layer of water, lacking oxygen. The
governance to developing countries.
complex biodiversity of peatlands means
they are home to a variety of species.  The Royal Society, The academy of
Their high carbon content makes them sciences for the developing world) and
uniquely vulnerable to incineration if they Environmental Defence Fund (EDF) are
are drained. its partners.
188 TARGET PT 2020

 The methods include: of interest and will help build long-


term collaborative relationships
! Space-Based Options/Space
with U.S. faculty and institutions.
Sunshades e.g. using mirrors in
These fellowships are for eight to
space, placing vast satellites at
12 months.
Lagrange Point 1, space parasol, etc.
 Affiliation: The selected candidate
! Stratosphere-Based Options such
will have affiliation with one U.S.
as the injection of sulphate aerosols
host institution during the grant.
into the stratosphere.
USIEF strongly recommends all
! Cloud-Based Options/Cloud applicants to identify institutions
Seeding e.g. Marine Cloud for affiliation and correspond
Brightening (by spraying a fine in advance with potential host
seawater spray in the air), seeding of institutions. If you have a letter of
high cirrus clouds with heterogeneous invitation from a U.S. institution,
ice nuclei. please include it as a part of your
online application. The letter of
! Surface-Based Options e.g.
invitation should indicate the
whitening roofs, growing more
duration of your visit, preferably
reflective crops, etc.
with dates.
17. Correct Option: (b)  Grant Benefits: These fellowships
provide J-1 visa support, round-
Explanation: trip economy class air travel
between India and the U.S., a
Kalam Climate Fellowship
monthly stipend, Accident and
 India and the US have launched the Sickness Program for Exchanges
Fulbright - Kalam Climate Fellowship. The per U.S. Government guidelines, a
fellowship programme will enable Indian modest settling-in allowance, and
research scholars to work with American a professional allowance.
institutions in the field of climate change.
 In the case of postdoctoral research
! Under the fellowship will 6 Indian grantees, and subject to availability
Ph.D. students and post-doctoral of funds, a dependent allowance
researchers will be sponsored to work and international travel may be
with US institutions for a period of up provided for one accompanying
to one year. eligible dependent provided the
dependent is with the grantee in
! The United States-India Educational the U.S. for at least 80% of the
Foundation (USIEF) administers the
grant period.
Fulbright-Kalam Climate Fellowship
on behalf of both the governments.
18. Correct option: (a)
! Fulbright-Kalam Climate
Explanation:
Fellowships are offered for:
 Doctoral Research: These National Biofuel Policy
fellowships are designed  In May 2018, the Union Government
for Indian scholars who are of India approved the National Biofuel
registered for a Ph.D. at an Policy to encourage sustainable and
alternative fuels. Further, with the flight
Indian institution. These of aircraft of Spice Jet from Dehradun to
fellowships are for six to nine Delhi, developed by the Indian Institute of
months. Petroleum, Dehradun using Jatropha
plant seeds in August 2018, India joined
 Postdoctoral Research: These
the elite group of countries (others
fellowships are designed for Indian
being US, Canada and Australia) that
faculty and researchers who are in
operate biofuel-powered flights.
the early stages of their research
careers in India. Fulbright-Kalam  Jatropha is a second-generation
Climate Fellowships will provide advanced biofuel manufactured
opportunities to talented faculty by chemical and physical treatments
and researchers to strengthen their of biomass. Other examples of second-
research capacities. Postdoctoral generation biofuel are feedstock like
fellows will have access to some of Jatropha, Soyabeans, Rapeseed, animal
the finest resources in their areas fats, etc are used.
TARGET PT 2020 189

 Salient features of the policy: belonging to the Scheduled Tribes


(STs) and Other Traditional Forest
! The Policy categorises biofuels as
Dwellers (OTFDs) categories across 16
“Basic Biofuels” viz. First Generation
States, whose claim as forest-dwellers
(1G) bioethanol & biodiesel and
“Advanced Biofuels” - Second has been rejected under the Forest
Generation (2G) ethanol, Municipal Rights Act.
Solid Waste (MSW) to drop-in fuels,  SC ordered the Forest Survey of India
Third Generation (3G) biofuels, bio- (FSI) to make a satellite survey and
CNG, etc. to enable the extension place on record the “encroachment
of appropriate financial and fiscal positions.”
incentives under each category.
 The order is based on affidavits filed by
! The Policy expands the scope of raw states which do not make it clear whether
material for ethanol production by the due process of law was observed before
allowing the use of Sugarcane Juice, the claims were rejected.
Sugar containing materials like
Sugar Beet, Sweet Sorghum, Starch  The Centre argued that the rejection of
containing materials like Corn, claims is particularly high in the States
Cassava, Damaged food grains like hit by Left-Wing Extremism, where
wheat, broken rice, Rotten Potatoes, tribal population is high. Being poor and
unfit for human consumption for illiterate, living in remote areas, they do
ethanol production. not know the appropriate procedure for
filing claims and the gram sabhas, which
! Farmers are at risk of not getting the initiate the verification of their claims,
appropriate price for their produce
are low on awareness of how to deal with
during the surplus production phase.
them. The rejection orders are not even
Taking this into account, the Policy
communicated to these communities.
allows the use of surplus food grains
for the production of ethanol for Forest Rights Act, 2006
blending with petrol with the approval
of the National Biofuel Coordination  The tribal and forest-dwelling communities,
Committee. who had been living within the forests
in harmony with the environment and
! With a thrust on Advanced Biofuels, the ecosystem, continued to live inside
the Policy indicates a viability gap
the forests in tenurial insecurity (as the
funding scheme for 2G ethanol Bio
procedure for settlement of rights under
refineries of Rs.5000 crore in 6 years in
Indian Forest Act, 1927 was hardly
addition to additional tax incentives,
followed), a situation which continued
higher purchase price as compared to
even after independence as they were
1G biofuels.
marginalized.
! The Policy encourages setting up of
 The symbiotic relationship
supply chain mechanisms for biodiesel
between forests and forest-dwelling
production from non-edible oilseeds,
Used Cooking Oil, short gestation communities found recognition in the
crops. National Forest Policy, 1988 which
called for the need to associate tribal
! Roles and responsibilities of all the people in the protection, regeneration,
concerned Ministries/Departments and development of forests.
with respect to biofuels have been
captured in the Policy document to  The Scheduled Tribes and Other
synergise efforts. Traditional Forest Dwellers (Recognition
of Forest Rights) Act, 2006, was enacted
 The National Policy on Biofuels of India not to protect the marginalized socio-economic
only help farmers dispose off surplus stock class of citizens and balance the right to
in an economic manner, but also reduce the environment with their right to life and
India’s oil imports. Rajasthan has become livelihood.
the first state to implement this.
 This Act is crucial to the rights of millions
19. Correct Option: (c) of tribal and other forest dwellers in
different parts of our country as it provides
Explanation: for the restitution of deprived forest rights
across India, including both individual
Encroachment on Tribal Land
rights to cultivated land in forestland and
 The Supreme Court in February 2019 community rights over common property
ordered the eviction of lakhs of people resources.
190 TARGET PT 2020

 The Gram Sabha is the authority to ! Gender Equality and Women’s


initiate the process for determining Empowerment
the nature and extent of Individual
! Biodiversity
Forest Rights (IFR) or Community
Forest Rights (CFR) or both that may ! Social Development
be given to FDST and OTFD.
 It also established a unique
 FRA recognizes and secure: intergovernmental SIDS Partnership
Framework, designed to monitor the
! Community Rights or rights over
progress of existing and stimulate the launch
common property resources of the
of new, genuine and durable partnerships
communities in addition to their
for the sustainable development of SIDS.
individual rights
 On 27 September 2019, the General
! Rights in and over disputed land
Assembly will hold a one-day high-level
Rights of settlement and conversion of
review of the progress made in addressing
all forest villages, old habitation, un-
the priorities of Small Island Developing
surveyed villages and other villages
States (SIDS) through the implementation
in forests into revenue villages (Relief
of the SAMOA Pathway.
Rights)
 The General Assembly has decided that the
! Right to protect, regenerate or
high-level review will result in “a concise
conserve or manage any community
action-oriented and inter-governmentally
forest resource which the communities
agreed political declaration”.
have been traditionally protecting and
conserving for sustainable use (Forest Small Island Developing States (SIDS)
Management Rights)
 SIDS is a group of small island countries
! Right to intellectual property and that are afflicted by economic difficulties
traditional knowledge related and confronted by development imperatives
to biodiversity and cultural similar to those of developing countries
diversity generally but have their own peculiar
! Rights of displaced communities vulnerabilities and characteristics.

! Rights over developmental activities  The United Nations Conference on


Sustainable Development (also known
20. Correct Option: (d) as Rio+20) that took place in Rio de
Janeiro, Brazil in June 2012 recognize
Explanation: these vulnerabilities and characteristics
as - their small size, remoteness, narrow
SAMOA Pathway resource and export base, and exposure
 The Third International Conference on to global environmental challenges and
Small Island Developing States was held external economic shocks, including a
from 1-4 September 2014 in Apia, Samoa. large range of impacts from climate change
and potentially more frequent and intense
 The Conference resulted in the adoption natural disasters.
of the Small Island Developing States
Accelerated Modalities of Action (or 21. Correct Option: (b)
SAMOA) Pathway and the announcement
of 300 multi-stakeholder partnerships in Explanation:
support of SIDS.
 Statement 1 is incorrect: WTO came
 SAMOA Pathway articulates the into existence in Uruguay Rounds of
sustainable development pathways 1986-1994 negotiations being held under
and aspirations for SIDS over the next the General Agreement on Trade and
10 years. Tariffs (GATT).
 There are 16 priority areas under the  Statement 3 is incorrect: There are no WTO
SAMOA Pathway. Some of them are: definitions of “developed” and “developing”
countries. Members announce for
! Sustained and sustainable, inclusive
themselves whether they are “developed”
and equitable economic growth with
or “developing” countries.
decent work for all
! Climate Change Supplementary notes:

! Sustainable Energy World Trade Organization (WTO)


! Disaster Risk Reduction  It is a successor to the General Agreement
TARGET PT 2020 191

on Tariffs and Trade (GATT) established in Supplementary notes:


the wake of the Second World War.
Gallantary Awards
 Several rounds of negotiations were held
under GATT.  Post-independence, first three gallantry
awards namely the Param Vir Chakra, the
 WTO came into existence in Uruguay Maha Vir Chakra and the Vir Chakra were
Rounds of 1986-1994 through Marrakesh instituted by the Government of India on
Agreement in 1995. 26th January, 1950 which were deemed to
have effect from the 15th August, 1947.
 Members – 164 and together, they
constitute 98% of the global trade  The Ashoka Chakra Class-I, the Ashoka
Chakra Class-II and the Ashoka Chakra
 In 2000, 4th WTO Ministerial led to a new
Class-III were instituted by the Government
round of negotiation, Doha Development
of India on 4th January, 1952, which were
Agenda, which is still in progress.
deemed to have effect from the 15th August,
 Mandate – to open trade for the benefit for 1947. These awards were renamed as the
all Ashoka Chakra, the Kirti Chakra and the
Shaurya Chakra respectively in January,
 All decisions are made by consensus. 1967.
 WTO Headquarters is in Geneva,  These gallantry awards are announced
Switzerland. It has no other branch office. twice in a year - first on the occasion of the
 Ministerial Conference is WTO’s top level Republic Day and then on the occasion of
decision making body. It meets once in two the Independence Day.
years.  Order of precedence of these awards is:
 WTO Agreements: ! Param Vir Chakra
! For Goods – Marrakesh Agreement ! Ashoka Chakra
(1995) and Trade Facilitation
! Mahavir Chakra
Agreement ( 2017)
! Kirti Chakra
! For Services – General Agreement on
Trade in Services ! Vir Chakra
! For Intellectual Property - The ! Shaurya Chakra
Agreement on Trade-Related Aspects  SELECTION PROCESS
of Intellectual Property Rights
(TRIPS) ! Ministry of Defence invites
recommendations twice in a year from
 Functions the Armed Forces and Union Ministry
! Administering WTO Trade of Home Affairs for gallantry awards.
Agreement ! Recommendations in respect of civilian
citizens (other than Defence personnel)
! Act as forum for trade negotiation
are received from the Union Ministry
! Handling trade disputes of Home Affairs (MHA).
! Monitoring trade policies ! Recommendations received from
the Armed Forces and MHA are
! Cooperation with other international considered by the Central Honours
organization & Awards Committee (CH&AC)
comprising of Raksha Mantri, three
22. Correct Option: (b) Service Chiefs & Defence Secretary.
Home Secretary is also member for the
Explanation:
cases recommended by the Ministry of
 Statement 1 is incorrect: Along with Air Home Affairs.
Force, Navy and Army, even para-military, ! Thereafter, recommendations of the
state police and civilians are also eligible CH&AC are submitted for approval of
to receive the Gallantary awards for their the Prime Minister and the President.
act of bravery. After approval of the President,
awards are announced on the occasion
 Statement 2 is incorrect: Param Vir Chakra
of the Republic Day and Independence
is the highest level of Gallantary award
Day.
instituted by the Government of India.
192 TARGET PT 2020

23. Correct Option: (a)  Tardigrade can survive even in the


vacuum of space.
Explanation:
 It is a recently discovered lake in Manang Supplementary notes:
district of Nepal
Tardigrades
Supplementary notes:  They often called water bears - are
 Kajin Sara Lake creatures under a millimetre long that
can survive being heated to 150C and
 It is a recently discovered lake in Manang
frozen to almost absolute zero.
district of Nepal
 They can survive even in the vacuum
 It is likely to earn the distinction of the
of space.
world’s highest lake, which is currently
held by Tilicho lake (also in Manang)  They were discovered aboard an
 It is located at an altitude of 5,200 metres Israeli spacecraft that crashed on the
(yet to be confirmed) moon’s surface.

 The lake, called Singar locally, have  Theoretically be possible for the
formed out of the water melted from the tardigrades to be collected, brought
Himalayas back to earth, reanimated, and studied
to see the effects of being on the moon.
 In News – Kajin Sara Lake is a recently
discovered lake in Manang district of 25. Correct Option: (a)
Nepal
Explanation:
24. Correct Option: (b)
 Correct order is Yamunotri, Kedarnath,
Explanation: Gangotri and Badrinath.

**********
TARGET PT 2020 193

TEST
DAY - 44

Time Allowed: 30 mins Maximum Marks: 50

1. Which of the following statements Which of the above statements is/are


regarding forest conservation and correct?
development is/are correct? (a) 1 and 2 only
1. While the National Afforestation
(b) 3 only
Programme is implemented on
degraded forest lands, Green India (c) 1 and 3 only
Mission aims at improving the quality (d) 1, 2, and 3
of the forest.
3. Consider the following statements:
2. Green India Mission was launched 1. Both Cartagena and Nagoya Protocols
under the National Action Plan on are supplementary agreements to the
Climate Change (NAPCC) in 2008. United Nations Framework Convention
on Climate Change.
Select the correct option using the codes
2. Nagoya Protocol aims to ensure the safe
given below: handling, transport, and use of living
(a) 1 only modified organisms (LMOs) resulting
from modern biotechnology.
(b) 2 only
3. Cartagena Protocol aims for the fair
(c) Both 1 and 2 and equitable sharing of benefits
arising out of the utilization of genetic
(d) Neither 1 nor 2 resources.

2. Consider the following statements Which of the above statements is/are


regarding ‘National Afforestation incorrect?
Programme’: (a) 1 only
1. Funds for the scheme are routed (b) 2 and 3 only
through the State Governments which
(c) 1 and 3 only
causes delays.
(d) 1, 2, and 3
2. The scheme is implemented by a three-
tier institutional set up through the 4. Which of the following agencies
State Forest Development Agency at publishes the Emissions Gap Report
annually?
the state level, Forest Development
Agency at the district level and Joint (a) UN Environment Programme
Forest Management Committees at (b) International Energy Agency
the forest division level. (c) Intergovernmental Panel on Climate
3. The Scheme is demand-driven and the Change
afforestation area is sanctioned on the (d) International Union for Conservation
basis of past performances. of Nature
194 TARGET PT 2020

5. Consider the following pairs regarding 8. Singalila National Park is located in


the goals of the Aichi Biodiversity which of the following states?
Targets:
(a) Meghalaya
1. Goal A: Biodiversity benefits to all
(b) West Bengal
2. Goal B: Reduce the direct pressure on
biodiversity (c) Assam
3. Goal C: Safeguard ecosystems, species, (d) Manipur
and genetic diversity
9. Consider the following statements:
Which of the above pairs are correctly
1. It is the second-largest mangrove
matched?
ecosystem in India.
(a) 1 only
2. It harbors the largest number of
(b) 2 and 3 only saltwater crocodile population in the
(c) 1 and 3 only Indian sub-continent

(d) 1, 2, and 3 3. The area has also been designated as


the second Ramsar site after Chilka
6. Consider the following statements lake.
regarding an organization:
Which of the following national parks/
1. It was established in 1948. wildlife sanctuaries has been described
2. It was involved in establishing the above?
World Wide Fund for Nature
(a) Campbell Bay National Park
3. Its headquarter is in Gland.
(b) Gahirmatha Marine Sanctuary
Which of the following organizations has
(c) Marine National Park
been described above?
(d) Bhitarkanika National Park
(a) International Union for Conservation
of Nature
10. Coral Reef Recovery Project is the
(b) Intergovernmental Panel on Climate joint venture of which of the following
Change agencies?
(c) United Nations Environment 1. Wildlife Trust of India
Programme 2. Wildlife Institute of India
(d) Bird Life International 3. Gujarat Forest Department
7. Consider the following statements 4. Maharashtra Forest Department
regarding a species:
Select the correct option using the codes
1. It is one of the five marine turtles that given below:
inhabit Indian coastal waters and
islands. (a) 1 and 2 only
2. It does not nest along the Indian (b) 1 and 3 only
coasts.
(c) 1, 2, and 3 only
3. The largest Indian Ocean nesting site
of the species is in Oman. (d) 2 and 4 only

Which of the following species has been 11. Which of the following is the smallest
described above? national park in India?
(a) Olive Ridley (a) Campbell Bay NP
(b) Leatherback (b) North Button Island NP
(c) Loggerhead (c) Middle Button Island NP
(d) Hawksbill (d) South Button Island NP
TARGET PT 2020 195

12. Consider the following statements (c) 2 and 3 only


regarding a species:
(d) 1, 2 and 3
1. It is the state animal of Jammu and
Kashmir. 15. Consider the following description:
2. it is also found in Himachal Pradesh. “These species have the ability to capture
3. In Kashmir, it is found in the Dachigam the imagination of the public and
National Park. include people to support conservation
action and/or to donate funds.”
Which of the following species has been
described above? The above description is regarding
which of the following species?
(a) Musk deer
(a) Flagship Species
(b) Kashmir gray langur
(c) Hangul (b) Critical Link Species

(d) Snow Leopard (c) Foundation Species


(d) Umbrella Species
13. With reference to the ecological
community, consider the following
16. With reference to the Coniferous
statements:
forests, consider the following
1. Major communities depend only on
statements:
the sun’s energy from outside and are
independent of the inputs and outputs 1. They are present in the regions of a
from adjacent communities. cold environment with high rainfall.
2. Minor communities are dependent on 2. These soils are high in mineral content
neighboring communities and are often
and organic material.
called societies.
Which of the above statements is/are
Which of the above statements is/are
correct? incorrect?

(a) 1 only (a) 1 only


(b) 2 only (b) 2 only
(c) Both 1 and 2 (c) Both 1 and 2
(d) Neither 1 nor 2 (d) Neither 1 nor 2
14. Consider the following statements
17. What is the difference between Photic
regarding the types of Food Chains in
an Ecosystem: and Aphotic zones?
1. The detritus food chain starts with 1. Unlike the photic zone, both respiration
decomposers, which are heterotrophic and photosynthesis take place in the
organisms. aphotic zone.
2. In a terrestrial ecosystem, a much 2. The photic zone is the lowest layer and
larger fraction of energy flows through the aphotic zone is the upper layer of
the grazing food chain in comparison
the aquatic ecosystems.
with the detritus food chain.
3. In an aquatic ecosystem, the detritus Select the correct answer using the code
food chain is the major conduit for given below:
energy flow.
(a) 1 only
Which of the above statements is/are
correct? (b) 2 only

(a) 1 and 2 only (c) Both 1 and 2


(b) 1 only (d) Neither 1 nor 2
196 TARGET PT 2020

18. Identify the incorrect statement with (b) It is a traditional performing artform
respect to Insectivorous Plants: in the state of Kerala
(a) These are plants that are specialized (c) It is a type of puppetry in Odisha
in trapping insects. state.
(b) Brilliant colors, sweet secretions are (d) It is a folk dance drama found across
few attractions used by these plants to south India.
lure their victims.
23. Kondapalli toys are cultural icons of
(c) Some of them even prey upon large
which of the following states?
animals as well.
(a) Odisha
(d) Leaf traps and Pitfall mechanisms are
associated with Insectivorous plants. (b) Andhra Pradesh
(c) Kerala
19. ‘A Realm is a continent or sub-
continent-sized area with unifying (d) Tamil Nadu
features of geography and fauna &
flora’. Which of the following is not 24. Consider the following statements
one of eight recognized biogeographic regarding Ebola Virus Disease (EVD):
realms? 1. People can get EVD through direct
(a) Nearctic realm contact with an infected animal or a
sick or even dead person infected with
(b) Palearctic realm Ebola virus.
(c) Australian realm 2. A new vaccine has been developed to
(d) Icelandic realm cure this disease.

Which of the above statements is/are


20. The Nile Perch introduced in Lake
correct?
Victoria (East Africa) caused the
extinction of more than 200 species of (a) 1 only
native fish, cichlid fish in the lake is an (b) 2 only
example of the loss of biodiversity due
to: (c) Both 1 and 2
(a) Alien species invasions (d) Neither 1 nor 2
(b) Co-extinction 25. Consider the following statements
(c) Habitat loss and fragmentation regarding Jal Jeevan Mission:
(d) Over-exploitation 1. The Har Ghar Nal Se Jal programme,
under the Jal Jeevan Mission, is
21. Magadha Burrowing Frog has been envisioned to provide safe and adequate
confirmed in which one of the following drinking water to all households in
states of India? rural India only.
(a) Madhya Pradesh 2. The Jal Jeevan Mission will adopt a
community based approach.
(b) Jharkhand
(c) Odisha Which of the above statements is/are
correct?
(d) Andhra Pradesh
(a) 1 only
22. Which one of the following statements (b) 2 only
about Kolam is correct?
(c) Both 1 and 2
(a) It is a traditional drawing found across
south India. (d) Neither 1 nor 2

**********
TARGET PT 2020 197

ANSWER HINTS
DAY - 44

1. Correct Option: (a) and development functions to the Joint


Forest Management Committees (JFMCs)
Explanation: at the village level, which are registered
societies.
Forest Conservation schemes
 The scheme is implemented by a
 The conservation and development of
three-tier institutional set up through
forest primarily involves three strategies
the State Forest Development Agency
– afforestation through natural/artificial
(SFDA) at the state level, Forest
regeneration, protection and management.
Development Agency (FDA) at the
The ministry is implementing three major
forest division level and Joint Forest
schemes for the development of forest areas
Management Committees(JFMCs) at
i.e. National Afforestation Programme
the village level.
(NAP) scheme, National Mission for a Green
India (GIM) and Forest Fire Prevention &  The Scheme is demand-driven and the
Management Scheme (FFPM). afforestation area is sanctioned on the
basis of past performance, potential
 While NAP is being implemented for
degraded forest land available for eco-
afforestation of degraded forest lands,
restoration and availability of budget. The
GIM aims at improving the quality
Annual Plan of Operation (APO) of SFDAs
of forest and increase in forest cover
is approved as per the Guidelines of NAP.
besides cross-sectoral activities on a
landscape basis.  NAP is a centrally sponsored scheme which
is implemented with the fund sharing
 The FFPM takes care of forest fire
pattern of 60: 40 percent between Centre
prevention and management measures.
and States wherein the sharing pattern for
 GIM is one of the eight missions launched Northeastern and hilly States is 90:10. The
under the National Action Plan on Climate central share of funds is released through
Change (NAPCC). State Government and state government
transfers the funds to SFDA along with
 GIM’s launch was supposed to coincide
its state share which sometimes causes
with the starting of the 12th five-year plan
a delay in fund availability to SFDAs for
in 2012. But, owing to financial delays the
implementation of NAP causing a delay in
mission was finally launched in 2015.
submission of mandatory documents for
subsequent release of funds.
2. Correct Option: (c)
 Since the inception of the NAP (2000-2002),
Explanation: an amount of Rs.3874.02 crores has been
released till 2018-19 to various States and
National Afforestation Programme
is being utilized for treatment/afforestation
 The overall objective of the National over 21 lakh hectare sanctioned area.
Afforestation Programme (NAP) scheme
is the ecological restoration of degraded 3. Correct Option: (d)
forests and to develop the forest resources
with peoples’ participation, with a focus Explanation:
on improvement in livelihoods of the
Cartagena and Nagoya Protocols to the
forest-fringe communities, especially the
CBD
poor. NAP aims to support and accelerate
the on-going process of devolving forest  The Cartagena Protocol on Biosafety to the
conservation, protection, management Convention on Biological Diversity is
198 TARGET PT 2020

an international agreement which aims to 5. Correct Option: (b)


ensure the safe handling, transport, and
use of living modified organisms (LMOs) Explanation:
resulting from modern biotechnology that Aichi Biodiversity Targets
may have adverse effects on biological
diversity, taking also into account risks  In the COP-10 meeting, the parties agreed
to human health. It was adopted on 29 that previous biodiversity protection
January 2000 and entered into force on 11 targets are not achieved, So we need to do
September 2003. come up with new plans and targets.

 The Nagoya Protocol on Access to Genetic  They came up with Mid/Long term(fulfilled
by 2050) and Short term plans(to be
Resources and the Fair and Equitable
fulfilled by 2020).
Sharing of Benefits Arising from their
Utilization (ABS) to the Convention on  The short term plan is officially known
Biological Diversity is a supplementary as the “Strategic Plan for Biodiversity
agreement to the Convention on Biological 2011-2020”. It is a ten-year framework for
Diversity. It provides a transparent legal action by all countries to save biodiversity
framework for the effective implementation which provides a set of 20 ambitious yet
of one of the three objectives of the CBD: the achievable targets (divided into 5 goals),
fair and equitable sharing of benefits collectively known as the Aichi Targets.
arising out of the utilization of genetic Strategic Goal A: Address the causes of
resources. The Nagoya Protocol on ABS biodiversity loss
was adopted on 29 October 2010 in Nagoya,
Japan and entered into force on 12 October  Make people aware of the values of
2014. biodiversity
 Integrated biodiversity values in
4. Correct Option: (a) development + poverty reduction plan
Explanation:  Subsidies which are harmful to
biodiversity= and eliminate them, phase
Emission Gap Report them out or reform them
 Recently, the tenth edition of the Emissions  Sustainable production and consumption.
Gap Report 2019 has been published.
Strategic Goal B: Reduce the direct
 It is an annual science-based assessment pressure on biodiversity and promote
of the gap between countries pledge on sustainable use
greenhouse gas emissions reductions and
 Reduce the rate of natural habitat loss and
the reduction required to deliver global
forest loss by at least 50%
temperature increase below 2 degrees
Celsius by the end of this century.  Reduce overfishing
 It is launched by the UN Environment  Agriculture, aquaculture, and forestry in a
Programme (UNEP). sustainable manner

 The report also identifies key opportunities  Reduce pollution and excessive use of
for each country to increase the pace of fertilizer
emission reduction necessary to close the  Prevent invasive alien species (non-native)
gap.
 Minimize the choral reflow destruction,
 The Emissions Gap Report measures and ocean acidification
projects three key trendlines:
Strategic Goal C: Safeguard ecosystems,
! The amount of greenhouse gas species, and genetic diversity
emissions every year up to 2030.
 Conserve terrestrial and inland water,
! The commitments countries are coastal – marine areas
making to reduce their emissions
 Prevent the extinction of threatened
and the impact these commitments
species
are likely to have an overall emission
reduction.  Maintain genetic diversity of agro-plants,
domesticated animals and minimizing
! The pace at which emissions must be genetic erosion
reduced to reach an emission low that
would limit the temperature increase Strategic Goal D: Biodiversity benefits to
to 1.5oC, affordably. all
TARGET PT 2020 199

 Safeguard ecosystems for women, tribals,  Five species of sea turtles are known
and poor. to inhabit Indian coastal waters and
islands.
 Combat desertification and restore the
degraded ecosystem  These are the Olive Ridley
(Lepidochelys olivacea), Green
 Operationalize the Nagoya protocol on (Chelonia mydas), Hawksbill
genetic resources, via national legislations (Eretmochelys imbricata), Loggerhead
(Caretta caretta) and the Leatherback
Strategic Goal E: Participatory planning, (Dermochelys coriacea) turtles.
capacity building
 Except the Loggerhead, the remaining
 National biodiversity strategy and action four species nest along the Indian
plans – update for participation coast.
 Integrate the knowledge of tribal  Loggerhead turtles are named for their
communities large heads that support powerful jaw
muscles, allowing them to crush hard-
 Scientific and technological knowledge
shelled prey like clams and sea urchins.
sharing application
 In the Indian Ocean, loggerheads feed
 Financial resources mobilization. along the coastlines of Africa, the Arabian
Peninsula, and in the Arabian Sea. Along
6. Correct Option: (a) the African coastline, loggerheads nest
Explanation: from Mozambique’s Bazaruto Archipelago
to South Africa’s St Lucia estuary. The
IUCN largest Indian Ocean nesting site is
Oman, on the Arabian Peninsula, which
 IUCN is a membership Union composed hosts around 15,000 nests, giving it the
of both government and civil society second-largest nesting population of
organizations. loggerheads in the world.
 It harnesses the experience, resources
8. Correct Option: (b)
and reach of its more than 1,300 Member
organizations and the input of more than Explanation:
15,000 experts.
Singalila National Park
 This diversity and vast expertise make
IUCN the global authority on the status of  Singalila National Park is located on the
the natural world and the measures needed Singalila Ridge at an altitude of more than
to safeguard it. 7000 feet above sea level, in the Darjeeling
district of West Bengal.
 IUCN was established in 1948. It was
 It was initially a wildlife sanctuary and
previously called the International Union
later made into a national park in 1992.
for the Protection of Nature (1948–1956)
The whole of the Singalila range and the
and the World Conservation Union (1990– national park has long been part of the
2008). trekking route to Sandakphu and Phalut.
 It was involved in establishing the
World Wide Fund for Nature, CITES, 9. Correct Option: (d)
and the World Conservation Monitoring Explanation:
Centre, etc.
Bhitarkanika National Park
 Its headquarter is in Gland.
 In April 1975, ex-zamindari forests of
7. Correct Option: (c) Kanika Raj were constituted and declared
as Bhitarkanika Wildlife Sanctuary by
Explanation: State Government of Odisha. Later in
1998, it was declared as a National Park
Loggerhead turtle because of its ecological, faunal, floral,
 India has a coastline of more than 8000 km geomorphological and zoological association
which is rich in biodiversity. Apart from and importance and for the purpose of
sustaining fishing grounds, India’s coastal protection.
waters and beaches provide foraging and  In August 2002, it was designated as
nesting sites for a variety of marine species, the second Ramsar site, after Chilka
including sea turtles. lake.
200 TARGET PT 2020

 It is said to house 70% of the country’s  It is the state animal of Jammu and
saltwater crocodiles. Kashmir.
 The sanctuary is the second largest  Earlier believed to be a subspecies of
mangrove ecosystem in India. red deer (Cervus elaphus), a number of
mitochondrial DNA genetic studies have
10. Correct Option: (b) revealed that the hangul is part of the
Asian clade of the elk.
Explanation:
 The International Union for Conservation
Coral Reef Recovery Project of Nature (IUCN) has declared it as a
 Launched in 2008, the Coral Reef Recovery Critically Endangered species.
Project is a joint venture of Wildlife Trust of
India and the Gujarat Forest Department, 13. Correct Option: (c)
supported by Tata Chemicals Limited
Explanation:
(TCL).
 It seeks to develop and implement Community
appropriate strategies for the conservation  Individuals of any one species depend on
of the Mithapur Reef, situated 12 kilometers individuals of different species for their
south of the Gulf of Kachchh in Gujarat. survival.
 The project, with initial support, received  For example, Animals require plants for
from the World Land Trust, is also working food and trees for shelter. Plants require
in the recovery of coral reefs in Gujarat’s animals for pollination, seed dispersal, and
Marine National Park. soil microorganism to facilitate nutrient
supply.
11. Correct Option: (d)
Types of Communities
Explanation:
South Button Island NP On the basis of the size and degree of
relative independence, communities may
 South Button Island NP with an area be divided into two types:
of 0.03 km2 is the smallest national
park in India.  Major community:

 It is the part of the Rani Jhansi Marine ! These are large-sized, well organized
National Park and is located some 24 and relatively independent. They
km south-west of Havelock Island, South depend only on the sun’s energy from
Andaman. outside and are independent of the
inputs and outputs from adjacent
 It was established in the year of 1987. communities.
 The Park is full of shallow-water coral ! Examples: Tropical evergreen forest
reefs with high visibility. The depth of in the north-east.
these Tropical coral reefs is 6 ft (1.8 m)
as shallow water. Scuba Diving is a very  Minor communities:
popular sport here. ! These are dependent on neighboring
communities and are often called
12. Correct Option: (c) societies.
Explanation: ! They are secondary aggregations
within a major community and are
Hangul
not therefore completely independent
 The Kashmir stag, also called hangul, is a units as far as energy and nutrient
subspecies of elk native to India. dynamics are concerned.
 It is found in dense riverine forests in the ! Examples: A mat of lichen on a cow
high valleys and mountains of the dung pad.
Kashmir Valley and northern Chamba
district in Himachal Pradesh. Structure of a community
 In Kashmir, it’s found in the Dachigam  In a community, the number of species
National Park where it receives and size of their population vary greatly.
protection but elsewhere it is more at A community may have one or several
risk. species.
TARGET PT 2020 201

 The environmental factors determine the decomposers.


characteristics of the community as well as
 The detrivores are eaten by predators.
the pattern of organisation of the members
in the community. Grazing Food Chain (GFC) V/S Detritus
 The characteristic pattern of the Food Chain (DFC)
community is termed as a structure that  In an aquatic ecosystem, the grazing
is reflected in the roles played by various food chain is the major conduit for
populations, their range and the type of energy flow.
area they inhabit, the diversity of species
in the community and the spectrum of  On the other hand, in a terrestrial
interactions between them. ecosystem, a much larger fraction of
energy flows through the detritus food
14. Correct Option: (b) chain (DFC) than through the grazing
food chain(GFC).
Explanation:
 In a terrestrial ecosystem, a much
Food Chain larger fraction of energy flows through
the detritus food chain than through
 Transfer of food energy from green the grazing food chain.
plants (producers) through a series
Basis Grazing Food Detritus Food
of organisms with repeated eating Chain Chain
and being eaten link is called a
Definition The grazing The detritus
food chain. food chain food chain
 E.g. Grasses → Grasshopper → Frog → begins in the starts from the
autotrophs detritivores.
Snake → Hawk/Eagle.
(green plants).
 Each step in the food chain is called the
trophic level. Energy In the grazing In detritus food
Supply food chain, chain, the main
 A food chain starts with producers and the energy is energy source
ends with top carnivores. taken from the is dead organic
sunlight as matter.
 The trophic level of an organism is the green plants
position it occupies in a food chain. prepare food in
its presence.
 It illustrates the order in which a chain of
organisms feeds upon each other. Organisms In grazing In detritus food
food chain chain subsoil
Types of Food Chains macroscopic organisms
organisms are are involved,
included. that could be
Grazing Food Chain macroscopic or
 The consumers who start the food chain, microscopic.
utilising the plant or plant part as their
Number of Releases a Releases a
food, constitute the grazing food chain.
Energy lesser quantity greater quantity
 For example, in a terrestrial ecosystem, of energy into of energy to the
the grass is eaten by a caterpillar, which the air. air.
is eaten by lizard and lizard is eaten by a
snake. 15. Correct Option: (a)
 In Aquatic ecosystem phytoplankton Explanation:
(primary producers) are eaten by
zooplanktons which are eaten by fishes and Flagship Species
fishes are eaten by pelicans (water bird).
 Flagship species are species that have
the ability to capture the imagination
Detritus Food Chain of the public and induce people to
 This type of food chain starts from the support conservation action and/or to
organic matter of dead and decaying donate funds.
animals and plant bodies from the grazing
 These are popular charismatic species
food chain.
that serve as symbols and rallying points
 Dead organic matter or detritus feeding to stimulate conservation awareness and
organisms are called detrivores or action.
202 TARGET PT 2020

 Flagship species can represent an Aphotic zone


environmental feature, cause, organisation
 The lower layers of the aquatic
or geographic region.
ecosystems, where light penetration
and plant growth are restricted forms the
16. Correct Option: (b)
aphotic zone.
Explanation:  Only respiration activity takes place.
Coniferous forest (Boreal Forest)  The aphotic zone is positioned below the
littoral and photic zones to the bottom of
 Cold regions with high rainfall, strong
the lake where light levels are too low for
seasonal climates with long winters
photosynthesis. Respiration occurs at all
and short summers are characterized
depths so the aphotic zone is a region of
by boreal coniferous forest.
oxygen consumption. This deep, unlit region
 This is characterized by evergreen plant is also known as the profundal zone.
species such as Spruce, fir and pine trees,
etc and by animals such as the lynx, wolf, 18. Correct Answer: (c)
bear, red fox, porcupine, squirrel, and
amphibians like Hyla, Rana, etc.
Explanation:

 Boreal forest soils are characterized by Insectivorous Plants


thin podzols and are rather poor. Both  These are plants that specialize in
because the weathering of rocks proceeds trapping insects.
slowly in cold environments and because
the litter derived from conifer needle (leaf )  They differ from normal plants in their
is decomposed very slowly and is not rich mode of nutrition.
in nutrients.  However, these plants never prey
 These soils are acidic and are mineral upon humans or large animals.
deficient. This is due to the movement  Insectivorous plants can broadly be divided
of large amounts of water through the into two categories based on their method
soil, without a significant counter-upward of trapping their prey:
movement of evaporation, essential soluble
nutrients like calcium, nitrogen, and ! Active Plants: These can close their
potassium which are leached sometimes leaf traps the moment insects land
beyond the reach of roots. This process on them.
leaves no alkaline oriented cations ! Passive Plants: These have a ‘pitfall’
to encounter the organic acids of the mechanism, having some kind of jar
accumulating litter. or pitcher-like structure into which
 The productivity and community stability the insect slips and falls, to eventually
of the boreal forest are lower than those of be digested.
any other forest ecosystem.  The insectivorous plants often have several
attractions such as brilliant colors,
17. Correct Option: (d) sweet secretions and other curios to lure
their victims.
Explanation:
Photic zone 19. Correct Option: (d)
 It is the upper layer of the aquatic Explanation:
ecosystems, up to which light penetrates
and within which photosynthetic activity is Bio Geographic Realm
confined.  Bio geographic realms are large spatial
 The depth of this zone depends on the regions within which ecosystems share a
transparency of water. broadly similar biota.
 Scholars have recognized eight Terrestrial
 Both photosynthesis and respiration
biogeographic realms:
activity takes place.
 The photic (or “euphotic”) zone is the
! Nearctic realm
lighted and usually well-mixed portion ! Neotropical realm
that extends from the lake surface down to
! Palearctic realm
where the light level is 1% of that at the
surface. ! Indo-Malayan/Oriental realm
TARGET PT 2020 203

Records of the Zoological Survey of


India.
 The frog is endemic to agricultural
areas in Nawadih and Joungi village of
Jharkhand’s Koderma district.
 It has been named as Spahaerotheca
Magadha and will be known by the common
name of ‘Magadha Burrowing Frog’.
 The discovery points to the fact that new
species of frogs can still be found even in
the ‘Central Indian Landscape’ other than
! Afrotropical/Ethiopian realm the Western Ghats and the Northeast,
! Oceania realm where most discoveries are currently
taking place.
! Australian realm
! Antarctic realm 22. Correct Option: (a)
Explanation:
20. Correct Option: (a)
 Option (a) is correct: Kolam is a
Explanation: traditional drawing found across south
Alien Species invasions India.

 Accidental or intentional introduction of Supplementary notes:


non-native species into a habitat has led to  Kolam is a traditional drawing found
the declination or extinction f indigenous across south India.
species.
 It is drawn by using rice flour, chalk,
 Alien species cause the decline or extinction chalk powder or rock powder, often using
of indigenous species. E.g. naturally or synthetically colored powders.
! The Nile Perch introduced in Lake  It is a geometrical line drawing composed of
Victoria (East Africa) caused the curved loops, drawn around a grid pattern
extinction of more than 200 of dots.
! species of native fish, cichlid fish in  Kolams are thought to bring prosperity to
the lake. homes. Every morning, millions of women
! Invasive weed species like carrot grass draw kolams on the ground with white rice
(Parthenium), Lantana and water flour.
hyacinth  Through the day, the drawings get walked
! (Eicchornia) caused damage to our on, washed out in the rain, or blown around
native species. in the wind; new ones are made the next
day
! The illegal introduction of the African
Catfish (Clarias gariepinus) for
23. Correct Option: (b)
aquaculture is
! posing a threat to the indigenous Explanation:
catfishes in our rivers.  Option (b) is correct: Kondapalli toys -
cultural icons of Andhra Pradesh - are one
21. Correct Option: (b) of the most sold handicrafts in India and
abroad, across online, wholesale, and retail
Explanation: platforms.
 Option (b) is correct: A new species of
burrowing frog has been confirmed in Supplementary notes:
Jharkhand’s Chhota Nagpur Plateau.
Kondapalli Toys
Supplementary notes:  Kondapalli Toys are the toys made of wood
in Kondapalli of Krishna district, a village
Magadha Burrowing Frog nearby Vijayawada in the Indian state of
 The frog was first discovered in 2015. Andhra Pradesh.
 Its existence was verified and confirmed  It was registered as one of the geographical
recently and published in the journal indication handicraft from Andhra Pradesh
204 TARGET PT 2020

as per Geographical Indications of Goods  The two promising drugs are made from
(Registration and Protection) Act, 1999. Ebola antibodies – a protein produced
by the immune system to defend against
 These toys were one of the variety of infection. Regeneron’s product is a cocktail
toys assembled in the houses during the of three Ebola antibodies, while mAb114 is
festivals of Sankranti and Navratri and is a single antibody developed by scientists at
referred as Bommala Koluvu. NIAID.
 The Kondapalli toys are made from soft
wood known as Tella Poniki which is 25. Correct Option: (c)
found in nearby Kondapalli Hills. Explanation:
 The wood is first carved out and then the  Both the statements are correct
edges are smooth finished.
Supplementary notes:
 The later step involves coloring with either
oil or water-colours or vegetable dyes and Jal Jeevan Mission
enamel paints are applied based on the
 Prime Minister flagged the growing water
type of the toys.
crisis in the country and said around 3.5
trillion will be spent under the ambitious
24. Correct Option: (c)
Jal Jeevan Mission aimed at providing
Explanation: potable water.

 Both the statements are correct  Providing safe drinking water to


all parts of the country remains a
Supplementary notes: priority.
 Ebola virus disease (EVD), also known as  Under the Jal Jeevan Mission, the
Ebola hemorrhagic fever (EHF), is a viral government will focus on rainwater
hemorrhagic fever of humans and other harvesting and water conservation in 256
primates caused by ebola viruses. districts in the first phase and carry out
other initiatives, including renovation of
 It is a rare and deadly disease in people traditional water bodies and tanks, reuse of
and nonhuman primates. water and recharge structures, watershed
development and intensive afforestation.
 The viruses that cause EVD are
located mainly in sub-Saharan Africa.  The NDA government has formed a
People can get EVD through direct new ministry to address all water
contact with an infected animal (bat or issues that looks at the management
nonhuman primate) or a sick or dead of water resources and drinking water
person infected with Ebola virus. supply in a holistic manner. The Jal
Shakti Ministry, formed by integrating
 Signs and symptoms typically start the Water Resources and Drinking
between two days and three weeks after Water and Sanitation Ministries,
contracting the virus with a fever, sore aims to work with state governments
throat, muscular pain, and headaches. to ensure Har Ghar Jal to all rural
Vomiting, diarrhea and rash usually follow, households by 2024.
along with decreased function of the liver
 This assumes importance given that water
and kidneys.
crisis has presented itself as a clear and
 Two experimental drugs showed present danger to India. Policy think-tank
survival rates of as much as 90% in a Niti Aayog said in a report that nearly
clinical trial in Congo. 600 million Indians already face “high to
extreme water stress.
 Two experimental drugs – an antibody
cocktail called REGN-EB3 developed by  Cities, including Delhi, Bengaluru,
Regeneron and a monoclonal antibody Chennai and Hyderabad, will run out of
called mAb114 – will now be offered to all groundwater by 2020, affecting 100 million
people.
patients infected with the viral disease in
an ongoing outbreak in the Democratic  Matters are only likely to worsen with the
Republic of Congo (DRC). country’s water demand likely to double by
2030, indicating there will be a 6% loss in
 The drugs showed “clearly better” results,
gross domestic product by 2050.
according to U.S. National Institute of
Allergy and Infectious Diseases (NIAID).  India’s most water-stressed blocks are in
Tamil Nadu (541), followed by Rajasthan
TARGET PT 2020 205

(218), Uttar Pradesh (139) and Telangana Scheme and Nehru Yuva Kendra
(137), with several states reeling from Sangathan will be involved along with
drought-like conditions. non-government organisations (NGOs)
 To encourage local participation, and students from local engineering
National Cadet Corps, National Service colleges.

**********
206 TARGET PT 2020

TEST
DAY - 45

Time Allowed: 30 mins Maximum Marks: 50

1. Which of the following statements is/ (b) 2 only


are correct?
(c) Both 1 and 2
1. Adaptation is permanent, while
acclimation is temporary. (d) Neither 1 nor 2

2. Acclimation is a natural process that 4. Which of the following statements is/


occurs for every type of organism. are incorrect?
Select the correct option using the codes 1. Symbiosis is a close relationship
given below: between two species in which one
species can be harmed.
(a) 1 only
2. Both mutualism and parasitism are a
(b) 2 only symbiotic relationship.
(c) Both 1 and 2 Select the correct option using the codes
(d) Neither 1 nor 2 given below:
(a) 1 only
2. In India, Littoral and swamp forests
are found in which regions? (b) 2 only
1. Andaman and Nicobar Islands (c) Both 1 and 2
2. Lakshadweep (d) Neither 1 nor 2
3. Delta of Ganga
5. Blue Flag Certification is provided to
Select the correct option using the codes the beaches by which of the following
given below: organizations?
(a) 3 only (a) Ocean CleanUp
(b) 2 and 3 only (b) International Maritime Organization
(c) 1 and 3 only (c) Foundation for Environmental
Education
(d) 1 only
(d) UN Environment
3. Which of the following statements is/
are correct? 6. Consider the following statements
1. Succession occurs faster in the regarding ‘Great Indian Bustard’:
peripheral area of a large continent. 1. It is the state bird of Rajasthan because
it is found only in Rajasthan.
2. Heterotrophic succession occurs in
an area where the green plants are 2. It has been categorized as Critically
completely absent. Endangered by the IUCN.
3. It is the mascot for the 13th Conference
Select the correct option using the codes
of Parties of the Convention on the
given below:
Conservation of Migratory Species of
(a) 1 only Wild Animals.
TARGET PT 2020 207

Which of the above statements is/are 2. To conserve the biodiversity


correct? 3. Sustainable use of the components of
(a) 1 only biodiversity
(b) 2 and 3 only Select the correct option using the codes
(c) 1, 2, and 3 given below:

(d) 1 and 2 only (a) 2 only


(b) 3 only
7. Which of the following is/are the
result(s) of competition in an (c) 2 and 3 only
ecosystem? (d) 1, 2, and 3
1. Extinction
11. The largest national park of the
2. Migration Andaman & Nicobar Islands is____
3. Resource Partition (a) Campbell Bay NP
Select the correct option using the codes (b) Mahatama Gandhi Marine NP
given below:
(c) Rani Jhansi Marine NP
(a) 1 only
(d) Mount Harriett NP
(b) 2 and 3 only
12. Which of the following statements
(c) 1 and 2 only
regarding the Appendices of the
(d) 1, 2, and 3 Convention on the Conservation of
Migratory Species of Wild Animals
8. One Trillion Tree Initiative has been (CMS) is/are correct?
launched by which of the following 1. There are four appendices of the
organizations? Convention.
(a) Wildlife Conservation Society 2. Appendix I includes birds whereas
(b) World Economic Forum Appendix II includes the animals.
(c) BirdlLife International Select the correct option using the codes
(d) International Union for Conservation given below:
of Nature (a) 1 only

9. Consider the following statements: (b) 2 only


1. Wetlands International is the only (c) Both 1 and 2
global organization dedicated to (d) Neither 1 nor 2
the conservation and restoration of
wetlands. 13. Ex-situ Conservation means conserving
2. Its headquarter is in Ramsar, Iran. biodiversity outside the areas where
they naturally occur. Which of the
Which of the above statements is/are following is a prominent method of Ex-
correct? situ conservation?
(a) 1 only (a) National Parks
(b) 2 only (b) Wildlife Sanctuaries
(c) Both 1 and 2 (c) Reserved Forests
(d) Neither 1 nor 2 (d) Botanical Garden

10. Which of the following are the 14. Mangroves exhibit a Viviparity mode
objectives of the Convention on of reproduction. This implies:
Biological Diversity? (a) Seeds germinate in the tree itself before
1. To mitigate the climate change falling to the ground
208 TARGET PT 2020

(b) The new plant grows from an outgrowth 19. Exposure to which of the following
in the plant body e-waste metal is known to cause
damage to our DNA?
(c) The new plant grows from the
fragments of the parent plant after (a) Cadmium
they fall on the ground (b) Hexavalent Chromium
(d) Seeds germinate after falling to the (c) Beryllium
ground.
(d) Barium
15. Identify an incorrect pair regarding the
various medicinal plants and their 20. Identify the animal listed in Schedule
sources: 1 of the WPA, 1972:
(a) King Cobra
(a) Quinine: cinchona tree
(b) Flying Squirrel
(b) Digitalis: foxglove plant
(c) Bengal Porcupine
(c) Taxol: Vinca rosea
(d) Great Indian Bustard
(d) Morphine: Opium plant
21. A new species of marmoset has been
16. Consider the following statements
recently discovered in which part of
about the primary pollutants:
the world?
1. These persist in the form in which they
(a) Brazil
are added to the environment.
(b) Argentina
2. DDT, plastic and peroxyacetyl nitrate
(PAN) are examples of primary (c) Chile
pollutants. (d) Peru
Which of the above statements is/are
22. Sandbox Model recently seen in news
correct?
is related to which of the following
(a) 1 only organization?
(b) 2 only (a) RBI
(c) Both 1 and 2 (b) DRDO
(d) Neither 1 nor 2 (c) IMD
(d) SEBI
17. Which inorganic pollutants are
produced by the Soap and Detergent 23. Which of the following provisions are
Industry? included in the recently approved
(a) Sulphides and bleaching liquors ‘Extension of benefits of Child Care
Leave (CCL)’?
(b) Tertiary ammonium compounds
1. It is now granted to women employees
alkalies
only.
(c) Various acids and alkalies and silica
2. It has put the age limit on the child to
(d) Fluorine, nitrates of metals and be cared to 22 years.
phosphorus 3. It has set the minimum period in each
spell to 5 days.
18. Exposure to which of the following
e-waste metal is known to cause Which of the above statements is/are
damage to our DNA? correct?
(a) Cadmium (a) 1 only
(b) Hexavalent Chromium (b) 1 and 2only
(c) Beryllium (c) 3 only
(d) Barium (d) 1, 2 and 3
TARGET PT 2020 209

24. Consider the following statements (c) 3 only


about United Nations Security Council
(UNSC): (d) 2 and 3 only
1. It consists of 15 members with
presidency rotating yearly among its 25. Strait of Gibraltar connects which two
members. water bodies?
2. It meets only once in a year.
(a) Mediterranean Sea and Black Sea
3. India currently is not a member of
UNSC. (b) Atlantic Ocean and Mediterranean
Which of the above statements is/are Sea
correct?
(c) Black Sea and Caspian Sea
(a) 1 only
(b) 1 and 2 only (d) North Sea and Baltic Sea

**********
210 TARGET PT 2020

ANSWER HINTS
DAY - 45

1. Correct Option: (a)  The most pronounced and the densest is


found in the Sunderban in the Ganga delta
Explanation: where the predominant species is Sundri
Acclimation versus Adaptation (Heriteera).

 Adaptation is a change in both the physical  It consists mainly of whistling pines,


and chemical composition of an organism mangrove dates, palms, and bullet wood.
brought about by habitat changes, while  They have roots that consist of soft tissue
acclimation is a physical reaction made in so that the plant can breathe in the water
order to adjust to said changes.
 Adaptation takes a long time to achieve and 3. Correct Option: (d)
usually affects the whole group to which it
Explanation:
belongs. It is part of the evolution process,
which all living things must undergo Succession
in order to cope with the ever-changing
planet. One good example of adaptation is  Succession would occur faster in
the camel and its ability to survive for long areas existing in the middle of the
periods of time in the desert with very little large continent. This is because here all
water. propagules or seeds of plants belonging
 Acclimation is a form of adaptation that an to the different seres would reach much
organism undergoes when transferred to a faster, establish and ultimately result in
different habitat. It doesn’t take as long as the climax community.
evolutionary adaptation and it doesn’t affect  Succession in which, initially the green
the body composition of the entire species. plants are much greater is quantity is
Adjustment is made by modifying physical known as autotrophic succession; and the
reactions to environmental changes, like ones in which the heterotrophs are
shivering when exposed to cold weather. It greater in quantity (but not completely
only occurs in the lifespan of the organism
absent) is known as heterotrophic
and doesn’t affect the evolution patterns of
succession.
its species.
 Adaptation is permanent, while acclimation 4. Correct Option: (d)
is temporary.
Explanation:
 Adaptation is a natural process that occurs
for every type of organism. This is to ensure Symbiotic interaction
the continuity and survival of species.
 Symbiosis is a general term for
interspecific interactions in which
2. Correct Option: (c)
two species live together in a long-
Explanation: term, intimate association.

Littoral and swamp forests  There are three basic types of symbiosis
viz. mutualism, commensalism, and
 Littoral and swamp forests are found parasitism.
along the Andaman and Nicobar
Islands and the delta areas of rivers  Mutualism is a symbiotic relationship
such as Ganga, Brahmaputra, Krishna, in which both species benefit. Example:
etc. Lichens.
TARGET PT 2020 211

 Commensalism is a symbiotic relationship  The theme of CMS COP13 in India is,


in which one species benefits while the other “Migratory species connect the planet and
species is not affected. Example: Shark we welcome them home” and the mascot is
and Remora. It’s worth noting that many “Gibi - The Great Indian Bustard”.
apparent commensalism relationships
 They are found in Rajasthan, Punjab,
actually turn out to be slightly mutualistic
Gujarat, Maharashtra, Andhra
or slightly parasitic when we look at them
Pradesh, Karnataka, Madhya Pradesh,
more closely.
the greater part of Central India,
 Parasitism is a symbiotic relationship in Central Provinces and the Deccan,
which one species (the parasite) benefits and parts of Thar desert.
while the other species (the host) is harmed.
 They inhabit dry and semi-dry grasslands
Example: Covid-19.
with dispersed bushes and patches of scrub.
Breeding tends to occur in undisturbed or
5. Correct Option: (c)
less degraded grassland sites.
Explanation:
 It is listed in Schedule I of the Indian
Blue Flag program Wildlife (Protection)Act, 1972, in the CMS
Convention and in Appendix I of CITES,
 A Blue Flag beach, marina or boating as Critically Endangered on the
operator is not only a place to give a sense IUCN Red List and the National Wildlife
of pride to the community and to attract Action Plan (2002-2016). It has also been
tourism, but it also promotes environmental identified as one of the species for the
issues and awareness. recovery program under the Integrated
 A world-renowned eco-label trusted by Development of Wildlife Habitats of the
millions around the globe, the Blue Flag Ministry of Environment and Forests,
program is operated under the auspices Government of India.
of the Foundation for Environmental
Education and is headquartered in 7. Correct Option: (d)
Copenhagen, Denmark.
Explanation:
 In order to qualify for this prestigious
award, a series of stringent environmental, Consequences of competition
educational, safety-related and access-  When two species depend on the same
related criteria must be met and resource, they are called to be in
maintained. competition.
 Central to the ideals of the Blue Flag  The results of the Competition can be
program is the aim of connecting the Resource partition (temporary measures),
public with their surroundings and Migration, extinction (if prolonged
encouraging them to learn more about their interaction)
environment.
 The mission of Blue Flag is to promote 8. Correct Option: (b)
sustainability in the tourism sector, through
Explanation:
environmental education, environmental
protection, and other sustainable One Trillion Tree Initiative
development practices. Thanks to Blue
Flag and its partnerships, more than 4,500  The World Economic Forum (WEF) at
beaches, marinas, and eco-tourism boats are its Davos meeting has launched a global
concretely contributing to the sustainable initiative to grow, restore and conserve 1
development goals. Blue Flag also trillion trees around the world - in a bid to
campaigns against disparity, inequality, restore biodiversity and help fight climate
unemployment, health threats, depletion of change.
natural resources, environmental threats,  1t.org is a World Economic Forum initiative,
pollution, and general environmental designed to support the UN Decade on
degradation. Ecosystem Restoration 2021-2030, led by
UNEP and FAO.
6. Correct Option: (b)
 It offers a platform for leading governments,
Explanation: businesses, civil society and ecopreneurs
committed to serving the global trillion
Great Indian Bustard trees community.
212 TARGET PT 2020

9. Correct Option: (a) 12. Correct Option: (d)


Explanation: Explanation:

Wetlands International Appendix I & II of CMS

 Wetlands International is the only  CMS has two Appendices. These


appendices list migratory species to which
global not-for-profit organization
the Convention applies. The text of the
dedicated to the conservation and Convention defines the basic obligations
restoration of wetlands. Its vision is a of the Contracting Parties towards species
world where wetlands are treasured and listed in Appendix I and Appendix II. These
nurtured for their beauty, the life they obligations are quite distinct for the two
support and the resources they provide. Appendices, and a migratory species can be
listed in both Appendices at the same time
 Its headquarter is in Wageningen, if the circumstances so warrant.
Netherlands.
 Appendix I comprises migratory
species that have been assessed
10. Correct Option: (c)
as being in danger of extinction
Explanation: throughout all or a significant portion
of their range. The Conference of the
Convention on Biological Diversity Parties has further interpreted the term
“endangered” as meaning “facing a very
 The Agreed Text of the Convention on high risk of extinction in the wild in the
Biological Diversity was adopted on 22 near future” Parties that are a Range State
May 1992 in the Nairobi Conference. to a migratory species listed in Appendix
I shall endeavor to strictly protect them
 The Convention entered into force on 29 by prohibiting the taking of such species,
December 1993. with very restricted scope for exceptions;
 It has 3 main objectives: conservation of conserving and where appropriate restoring
their habitats; preventing, removing or
biological diversity, sustainable use of
mitigating obstacles to their migration
the components of biological diversity,
and controlling other factors that might
fair and equitable sharing of the endanger them.
benefits arising out of the utilization
of genetic resources.  Appendix II covers migratory species
that have an unfavorable conservation
status and that require international
11. Correct Option: (a) agreements for their conservation
Explanation: and management, as well as those
that have a conservation status that
National parks of the Andaman & Nicobar would significantly benefit from the
Islands international cooperation that could
be achieved by an international
S.No. Name of State/ Protected Area agreement. The Convention encourages
Area (km2) the Range States to species listed in
Appendix II to conclude global or regional
1 Campbell Bay NP 426.23 agreements for the conservation and
management of individual species or
2 Galathea Bay NP 110 groups of related species.

3 Mahatama Gandhi Marine 281.5  In this respect, CMS acts as a framework


(Wandoor) NP convention from which separate
instruments evolve. The Agreements may
4 Middle Button Island NP 0.44 vary from legally binding treaties (the
Agreements in the proper sense) to less
5 Mount Harriett NP 46.62 formal instruments, such as Memoranda
of Understanding, Action Plans or
6 North Button Island NP 0.44 Species Initiatives, covering to the extent
possible the entire migratory range of
7 Rani Jhansi Marine NP 256.14 the species concerned. The development
of instruments tailored according to the
8 Saddle Peak NP 32.54
conservation needs of species and adapted
9 South Button Island NP 0.03 to the requirements of particular regions is
a unique capacity of CMS.
TARGET PT 2020 213

13. Correct Option: (d) to at least $40 billion worldwide sales


annually.
Explanation:
 For example quinine is used to treat
Ex-situ Conservation malaria (from the cinchona tree);
Digitalis is used to treat chronic heart
 In Ex-situ conservation, animals are reared
trouble (from the foxglove plant,
or plants are cultivated outside the areas
Cinchona officinalis); and morphine
where they naturally occur.
and cocaine are used to reduce pain;
 Even reintroduction of an animal or drug for leukemia from Vinca rosea,
plant into the habitat from where it has taxol from Taxus brevifolia etc; and
become extinct is another form of ex situ hundreds of life saving antibiotics.
conservation.
 In recent years, more than 5000 species
 Example of this another form of ex situ of flowering plants have been analysed
conservation includes the reintroduction by scientists for the presence of valuable
of Gangetic gharial in the rivers of Uttar drugs.
Pradesh, Madhya Pradesh and Rajasthan
where it had become extinct. 16. Correct Option: (a)
 Some prominent methods of Ex-situ Explanation:
conservation include seed banks, botanical
gardens, horticultural and recreational Pollutants
gardens.  Pollutants are the materials or factors,
which cause an adverse effect on the
14. Correct Option: (a) natural quality of any component of the
Explanation: environment.
 For example, smoke from industries and
Characteristics of Mangroves automobiles, chemicals from factories,
 Mangroves exhibit Viviparity mode radioactive substances from nuclear plants,
of reproduction i.e. seeds germinate sewage of houses and discarded household
in the tree itself (before falling to the articles are common pollutants.
ground).
Classifications
 This is an adaptive mechanism to overcome
the problem of germination in saline According to the form in which they persist
water. after release into the environment.
 Primary Pollutants: These persist in
15. Correct Option: (c) the form in which they are added to
Explanation: the environment e.g. DDT, plastic.
 Secondary Pollutants: These are
Medicinal plants
formed by interaction among the primary
 About 40% of all the drugs used throughout pollutants. For example, peroxyacetyl
the world have active ingredients extracted nitrate (PAN) is formed by the
from plants and animals. Drugs that are interaction of nitrogen oxides and
derived from natural compounds amount hydrocarbons.

17. Correct Option: (b)


Explanation:
Sources of Water Pollution
Type of Inorganic pollutants Organic pollutants
Industry
Mining Mine Wastes: Chlorides, various metals, ferrous sulphate,
sulphuric acid, hydrogen sulphide, ferric hydroxide, surface
wash offs, suspended solids, chlorides and heavy metals.
Iron and Steel Suspended solids, iron cyanide, thiocyanate, sulphides, oxides Oil, phenol and
of copper, chromium, cadmium, and mercury. neptha
214 TARGET PT 2020

Chemical Plants Various acids and alkalies, chlorides, sulphates, nitrates of Aromatic compounds
metals, phosphorus, fluorine, silica and suspended particles. solvents, organic
acids, nitro compound
dyes, etc.
Pharmaceutical - Proteins,
carbohydrates, organic
solvent intermediate
products, drugs and
antibiotics
Soap and Tertiary ammonium compounds alkalies Flats and fatty
Detergent acids, glycerol,
polyphosphates,
sulphonated
hydrocarbons.
Food processing - Highly putrescible
organic matter and
pathogens
Paper and Pulp Sulphides, bleaching liquors. Cellulose fibres, bark,
woods sugars organic
acids.

18. Correct Option: (b) 20. Correct Option: (d)


Explanation: Explanation:

Negative effects of E-Waste exposure Wildlife Protection Act (WPA), 1972


 Cadmium: Toxic cadmium compounds  Animals listed in schedule 1 and part II
accumulate in the human body and harm of schedule 2 has absolute protection -
internal organs especially the kidneys. offenses under these are prescribed the
highest penalties.
 Barium: Short-term exposure to barium
causes brain swelling, muscle weakness,  Animals listed in schedule 1 include Lion-
damage to the heart, liver, and spleen. tailed macaque, Rhinoceros, Great Indian
bustard, Narcondam hornbill, Nicobar
 Beryllium: Exposure to beryllium can megapode, Blackbuck, etc.
cause lung cancer. Beryllium also causes a
 Animals listed in schedule 2 include Rhesus
skin disease that is characterised by poor
macaque, Dhole, Bengal Porcupine, King
wound healing and wartlike bumps
cobra, Flying squirrel, Himalayan brown
 Hexavalent Chromium: It can cause bear, etc.
damage to DNA and is extremely toxic
in the environment. 21. Correct Option: (a)
Explanation:
19. Correct Option: (b)
 Option (a) is correct: A new species of
Explanation: marmoset has been discovered in the south-
west of Pará State in Brazil in an area of
Negative effects of E-Waste exposure the Amazon that has suffered extensive
 Cadmium: Toxic cadmium compounds illegal logging and agricultural incursion
accumulate in the human body and harm
Supplementary notes:
internal organs especially the kidneys.
 A new species of marmoset (Monkey) has
 Barium: Short-term exposure to barium
been discovered in the south-west of Pará
causes brain swelling, muscle weakness, State in Brazil in an area of the Amazon
damage to the heart, liver, and spleen. that has suffered extensive illegal logging
 Beryllium: Exposure to beryllium can and agricultural incursion.
cause lung cancer. Beryllium also causes a  The new species of marmoset was found at
skin disease that is characterised by poor Tapajos River.
wound healing and wartlike bumps
 The region where the marmoset was
 Hexavalent Chromium: It can cause discovered is “one of the main fronts of forest
damage to DNA and is extremely toxic destruction within the ‘arc of deforestation’,
in the environment. a region infamously characterised by
TARGET PT 2020 215

fast, intense and disordered conversion of  The entity should either be a company
forests to pastoral and agricultural land incorporated and registered in the country
and human settlements. or banks licensed to operate in India.
 The new species has been named Mico  While money transfer services, digital
munduruku after the Munduruku know-your customer, financial inclusion
Amerindians who live in the area where and cyber security products are included,
the monkey was discovered. crypto currency, credit registry and credit
information have been left out.
 Mico munduruku occurs only within an
area in south-western Pará State, Brazil.
23. Correct option: (c)
 The marmosets’ markings and pigmentation
were the first characteristics distinguishing Explanation:
them from others in their genus. Aside  Statement 1 is incorrect: Besides the
from having a white tail, the individuals women employees, the benefits of CCL are
displayed a beige-yellowish back, white being extended to single male service
hands, white feet, and white forearms with personnel also.
a beige-yellowish patch on their elbow.
 Statement 2 is incorrect: The limit on
 Their DNA was the second difference. the child to be cared is kept unchanged
Genomes show they are closely related but at 18 years.
separate from other known Amazonian
marmosets. Supplementary notes:
 Until now, CCL was being granted to
22. Correct Option: (a) woman officers in defence forces. But from
Explanation: now, DoPT has made certain amendments
for grant of CCL to civilian employees,
 Option (a) is correct: The Reserve Bank whereby the CCL granted to woman
of India (RBI) issued the final framework employees till now has been extended to
for regulatory sandbox in order to enable single male government servants also.
innovations in the financial technology
space.  Single male service personnel and women
officers of defence forces will also be able
Supplementary notes: to avail CCL in respect of child with 40%
disability without any restriction of age
Regulatory Sand Box limit of the child.
 The Reserve Bank of India (RBI) issued  The age limit of 22 years prescribed earlier
the final framework for regulatory in the case of a child with 40% disability has
sandbox in order to enable innovations been removed for the purpose of availing
in the financial technology space. CCL. For the normal child the limit is kept
 A regulatory sandbox usually refers to unchanged at 18 years.
live testing of new products or services in  The minimum period of CCL that can be
a controlled/test regulatory environment availed in each spell has been reduced to
for which regulators may permit certain five days from the earlier limit of 15 days.
regulatory relaxations for the limited
purpose of the testing. 24. Correct Option: (c)
 RBI said the objective of the sandbox was Explanation:
to foster responsible innovation in financial
services, promote efficiency and bring  Statement 1 is incorrect: The Security
benefit to consumers. Council consists of fifteen members. The
body’s presidency rotates monthly among
 The proposed FinTech solution should its members.
highlight an existing gap in the financial
ecosystem and the proposal should  Statement 2 is incorrect: The Security
demonstrate how it would address the Council meets year-round, unlike the
problem, and bring benefits to consumers General Assembly.
or the industry and/or perform the same
Supplementary notes:
work more efficiently.
 It is one of the six principal organs of the
 RBI will launch the sandbox for entities
United Nations (UN).
that meet the criteria of minimum net
worth of 25 lakh as per their latest  Like the UN as a whole, it was created
audited balance sheet. following World War II to address the
216 TARGET PT 2020

failings of a previous international member states and funded independently


organization, the League of Nations, in of the main UN budget.
maintaining world peace.
 Unlike the General Assembly, the Security
 The council held its first session in 1946. Council meets year-round. Each Security
Council member must have a representative
 It is the only body of the UN with the
authority to issue binding resolutions to available at UN Headquarters at all times
member states. in case an emergency meeting becomes
necessary.
 The Security Council consists of fifteen
members: 25. Correct option: (b)
! The great powers that were the victors Explanation:
of World War II – the Soviet Union
(now represented by Russia), the  Option (b) is correct: Strait of Gibraltar
United Kingdom, France, Republic connects Atlantic Ocean and Mediterranean
of China (now represented by the Sea.
People’s Republic of China), and the
United States – serve as the body’s Supplementary notes:
five permanent members.  Gibraltar is a British Overseas Territory
! These can veto any substantive located at the bottom of Spain on the
resolution, including those on the narrow gap between Europe and Africa.
admission of new member states or  Known as the Gib or the Rock, it is a small
nominees for the office of Secretary- 2.5-mile-squared area with a population
General. of just 30,000 - but it has huge strategic
! In addition, the council has 10 non- importance.
permanent members, elected on a  This is because from this spot a navy can
regional basis to serve a term of two potentially control shipping in and out of
years. the Mediterranean - much of it coming
! The body’s presidency rotates monthly from Asia having travelled through the
among its members. Suez Canal.
 Resolutions of the Security Council are  The UK, a key member of NATO, has a
typically enforced by UN peacekeepers, naval and military base there for this
military forces voluntarily provided by reason.

**********

You might also like